Download as pdf or txt
Download as pdf or txt
You are on page 1of 393

tr

Topic-wise Practice Tests

ish
Also Helpful for GAIL, BARC, HPCL, BHEL, ONGC, SAIL, DRDO & Other PSU’s

GATE

na
’s
Civil Engineering
This book would act as a one-stop assessment solution for GATE Aspirants. It consists of
both topic-wise tests and full length mock tests for thorough practice. Out of the 7 mock

Topic-wise Practice Tests


tests, 5 mock tests are provided at end of the book and 2 are AIMGATEs– online mock
tests simulated as per GATE online exam. As a result, this book would serve as an effective

Practice Tests
tool for GATE aspirants to crack the examination and familiarize themselves with the GATE

GATE

Topic-wise
online exam environment.

G AT E TEST SERIES
E
FRMGEATEs

GATE Civil Engineering


Civil Engineering 2 AI

Cover Image: Bannafarsai_Stock.shutterstock.com

Cover Image: Nonwarit. 123rf.com


H IGH LIGH TS
 1800+ Problems for practice
 51 topic-wise tests based on latest
GATE pattern
in.pearson.com  Detailed solutions given for each test
trishna’s

 Tests on General Aptitude and


Engineering Mathematics
 5 Full-length Mock Tests
About Pearson
Pearson is the world’s learning company, with presence across 70 countries
worldwide. Our unique insights and world-class expertise comes from a long
history of working closely with renowned teachers, authors and thought
leaders, as a result of which, we have emerged as the preferred choice for
millions of teachers and learners across the world.
We believe learning opens up opportunities, creates fulfilling careers and
hence better lives. We hence collaborate with the best of minds to deliver you
class-leading products, spread across the Higher Education and K12 spectrum.
Superior learning experience and improved outcomes are at the heart of
everything we do. This product is the result of one such effort.
Your feedback plays a critical role in the evolution of our products and you
can contact us at reachus@pearson.com. We look forward to it.
This page is intentionally left blank
Topic-wise Test

GATE
(Graduate Aptitude Test in Engineering)

Civil Engineering

Trishna Knowledge Systems


Copyright © 2019 Pearson India Education Services Pvt. Ltd

Published by Pearson India Education Services Pvt. Ltd, CIN: U72200TN2005PTC057128.

No part of this eBook may be used or reproduced in any manner whatsoever without the publisher’s
prior written consent.

This eBook may or may not include all assets that were part of the print version. The publisher
reserves the right to remove any material in this eBook at any time.

ISBN 978-93-530-6653-6
eISBN:

Head Office: 15th Floor, Tower-B, World Trade Tower, Plot No. 1, Block-C, Sector-16,
Noida 201 301,Uttar Pradesh, India.
Registered Office: 4th Floor, Software Block, Elnet Software City, TS-140, Block 2 & 9,
Rajiv Gandhi Salai, Taramani, Chennai 600 113, Tamil Nadu, India.
Fax: 080-30461003, Phone: 080-30461060
www.in.pearson.com, Email: companysecretary.india@pearson.com
Contents
Preface  vii Part III CIVIL
ENGINEERING3.1
Part I  General Aptitude 1.1
Unit I Engineering
Part A  Verbal Ability Test
Mechanics3.3
Verbal Ability Test 1 1.5
Verbal Ability Test 2 1.9 Engineering Mechanics Test 1 3.5

Verbal Ability Test 3 1.12 Engineering Mechanics Test 2 3.9

Verbal Ability Test 4 1.17


Unit II Solid Mechanics 3.19
Verbal Ability Test 5 1.30
Solid Mechanics Test 1  3.21
Part B  Numerical Ability Solid Mechanics Test 2  3.27

Unit I Quantitative Unit III Structural


Aptitude1.33 Analysis3.33
Quantitative Ability Test 1 1.35 Structural Analysis Test 1 3.35
Quantitative Ability Test 2 1.40 Structural Analysis Test 2 3.44
Quantitative Ability Test 3 1.48
Quantitative Ability Test 4 1.55 Unit IV Construction
Quantitative Ability Test 5 1.62 Materials and
Quantitative Aptitude Test 6 1.68 Management3.51
Quantitative Aptitude Test 7 1.74 Construction Materials and
Management Test 1 3.53
Unit II Logical Reasoning Construction Materials and
Test1.81 Management Test 2 3.56

Logical Ability Test 1 1.83


Logical Ability Test 2 1.90 Unit V Concrete
Structures3.61
Part II Engineering Concrete Structures Test 1 3.63
Mathematics2.1 Concrete Structures Test 2 3.68
Engineering Mathematics Test 1 2.3
Engineering Mathematics Test 2 2.9 Unit VI Steel
Engineering Mathematics Test 3 2.16 Structures3.75
Engineering Mathematics Test 4 2.23 Steel Structures Test 1 3.77
Engineering Mathematics Test 5 2.29 Steel Structures Test 2 3.83
vi | Contents

Unit VII Geotechnical Solid Waste Management and


Pollution Test 3 3.154
Engineering3.89
Environmental Engineering Test 4 3.157
Soil Mechanics Test 1  3.91
Soil Mechanics Test 2 3.94
Geotechnical Engineering Test 3 3.97 UNIT XI Transportation
Foundation Engineering Test 4 3.103 Engineering3.165
Transportation Infrastructure Test 1 3.167
Traffic Engineering Test 2 3.170
Unit VIII Fluid Mechanics and
Transportation Engineering Test 3 3.174
Hydraulics3.107
Highway Engineering Test 4 3.179
Fluid Mechanics Test 1 3.109
Fluid Mechanics Test 2 3.114
Hydraulics Test 3 3.121 Unit XII Geomatics
Engineering3.183
Geomatics Engineering Test 1 3.185
Unit IX Water Resources
Geomatics Engineering Test 2 3.188
Engineering3.127
Hydrology Test 1 3.129
Irrigation Test 2 3.133 Part IV  Mock Tests 4.1
Water Resource Engineering Test 3 3.138 Mock Test 1 4.3
Mock Test 2 4.16
Mock Test 3  4.29
Unit X Environmental
Mock Test 4 4.42
Engineering3.143
Mock Test 5 4.53
Water Supply Engineering Test 1 3.145
Waste Water Engineering Test 2 3.149
Preface
Graduate Aptitude Test in Engineering (GATE) is one of the primarily tests for various undergraduate subjects—
Engineering/Technology/Architecture and postgraduate level for Science. The GATE examination pattern has undergone
several changes over the years—sometimes apparent and sometimes subtle. It is bound to continue to do so with changing
technological environment. Apart from giving the aspirant a chance to pursue M.Tech. from institutions like the IITs /NITs,
a good GATE score can be highly instrumental in landing the candidate a plush public sector job since many PSUs are
recruiting graduate engineers on the basis of their performance in GATE.
  Topic-wise Practice Tests GATE Civil Engineering acts as a one-stop assessment tool for all GATE aspirants. The
book consists of topic-wise tests on (1) General Aptitude, (2) Engineering Mathematics, and (3) Civil Engineering. Five
full-length mock tests based on latest GATE pattern are provide at the end of the book, which will help students to check
their level of preparation for GATE exam. As a result, this book would serve as an effective tool for GATE aspirant to crack
the examination.

Highlights
•• Includes (2000+) problems for practice
•• Includes (51) topic-wise tests based on latest GATE pattern.
•• Detailed solutions given for each test
•• Includes tests on General Aptitude and Engineering Mathematics
•• Includes 5 full-length mock tests based on latest GATE pattern
Despite of our best efforts, some errors may have inadvertently crept into the book. Constructive comments and suggestions
to further improve the book are welcome and shall be acknowledged gratefully.
This page is intentionally left blank
PART I  GENERAL APTITUDE

Part A
Verbal Ability Test

Part B
Numerical Ability Test
This page is intentionally left blank
Part A
Verbal Ability Test

Verbal Ability Test 1���������������������������������������������������������������������������������������������������������������������� 1.5


Verbal Ability Test 2���������������������������������������������������������������������������������������������������������������������� 1.9
Verbal Ability Test 3�������������������������������������������������������������������������������������������������������������������� 1.12
Verbal Ability Test 4�������������������������������������������������������������������������������������������������������������������� 1.17
Verbal Ability Test 5�������������������������������������������������������������������������������������������������������������������� 1.30
This page is intentionally left blank
Verbal Ability Test 1
Number of Questions: 40 Time: 30 min

Directions for questions 1 to 5: Each of the given sentences 10. Injustice and discrimination / can never be /
has four underlined parts. One of them has a mistake. Mark        (A)       (B)
the number of the wrong part as answer. tolerated by / no one.
1. In the Sub-Saharan countries incidences of   (C)   (D)
              (A) 11. Alas! / How lovely / and fragrant /
Sunstrokes correlates positively withthe level of (A)   (B)     (C)
    (B)        (C) these flowers are!
Solar radiation.    (D)
  (D) 12. Your story is / so ridiculous that / it could not/
2. Either you transfer the data which was demanded   (A)     (B)     (C)
   (A)      (B) be believed.
nor file a report explaining why you did not   (D)
       (C) 13. Government schools / has too many students /
submit the overall annual figures.      (A)       (B)
      (D) in a class / for a teacher to control.
3. Neither the judge nor I am ready toannounce  (C)      (D)
   (A)    (B)     (C) 14. Either of / the methods / lead to the /
who the winner is.   (A)   (B)    (C)
   (D) same result.
4. He went about the bad phase in his career with     (D)
  (A)     (B)     (C) 15. The doctor has / advised him / to avoid the sugar /
philosophical clam   (A)     (B)      (C)
   (D) in his milk.
5. A score of apple is purchased by him   (D)
   (A)     (B)  (C) Directions for questions 16 to 20: Each sentence given
for his consumption. below is divided into four parts. One of them has an error.
   (D) Mark the number of the incorrect part as your answer.
Directions for questions 6 to 10: Read each sentence to find 16. The church accepts this popular sentiment /
out whether there is any error in it. The error, if any, will be         (A)
in one part of the sentence. Identify the error and mark the gives it a religious significance /and crystallizes /
number of the erroneous part as your answer.      (B)         (C)
6. All the members / of the club / was present / in a system
  (A)      (B)    (C)   (D)
at the special meeting. 17. The government of the Tudors /
    (D)       (A)
7. Over the course of the twentieth century/ were masters in the art of disguising /
        (A)         (B)
the internal combustion engine / common place, and sometimes sordid, motives /
      (B)         (C)
has replaced the horse / to the basic means of transport. beneath a glittering façade of imposing principles.
     (C)         (D)         (D)
8. We must / never give up with /trying to enhance/ 18. The increasing reluctance of the sun to rise,/
 (A)     (B)      (C)          (A)
the quality of life. the extra nip in the breeze /
    (D)     (B)
9. It will profit a man nothing / if he was / the patten of shed leaves dropping - all the evidences
      (A)      (B) of fall /
to gain the world / and lose his own soul.            (C)
   (C)       (D) drifting in winter were clearer each day.
       (D)
1.6 | Verbal Ability Test 1

19. Because of it’s hardness / this steel is / used principally/ 25. We cannot achieve our goals. We lack confidence in our
     (A)     (B)     (C) abilities.
for making razors. (A) When we lack confidence ………..
   (D) (B) However we can achieve our goals if we lack …..
(C) Whenever we achieve our goals ………
20. Laying aside all hindrance /
(A) A and B (B) A only
     (A)
(C) B only (D) C only
thrusting away all private aims /
(E) B and C
      (B)
devote yourself unswerving and unflinchingly / Directions for questions 26 to 30: Each sentence has a miss-
         (C) ing part. Choose the best option from those given below the
to the vigorous and successful prosecution of this war statement to make up the missing part.
          (D) 26. Given the long and porous border between the two
countries and, more important, the links _______.
Directions for questions 21 to 25: In each question below, (A) among Nepalese and Indian rebel groups, New Delhi
two sentences are given. These two sentences are to be com- cannot afford ignoring the Maoists threat of Nepal
bined into a single sentence without changing their mean- (B) between Nepalese and Indian rebel groups, New
ing. Three probable starters of the combined sentence are Delhi cannot afford the ignorance of the Maoists
given which are denoted by (A), (B) and (C). Any one or threat in Nepal
more or none of them may be correct. Find out the correct (C) among Nepalese and Indian rebel groups, New Delhi
starters(s) and accordingly select your answer from among cannot afford to ignore the Maoists threat of Nepal
the given five answer choices. (D) between Nepalese and Indian rebel groups, New
Delhi cannot afford to ignore the Maoists threat in
21. I am a layman. I do not know how a computer
Nepal
works.
(A) Being a layman, I ……….. 27. At a time when Beijing’s officially scripted anti-Japa-
(B) As I am a layman, I ………. nese protests are bound to prompt a rethink in Japan
(C) While I am a layman, I…….. about the advisability of continued investment in
(A) A and B (B) B and C China, India should be __________.
(C) A and C (D) B only (A) persuading aggressively Japanese business to
(E) A only shifting at least some of their mammoth invest-
ments to its secure location
22. He always listened to good advice. He rose to a good (B) aggressively persuading Japanese business to
position in life. shifting at least some of their mammoth invest-
(A) Though he listened to good advice …….. ments to its secure location
(B) As he rose to good position in life …….. (C) aggressively persuading Japanese business to shift
(C) Since he listened to good advice ……. at least some of their mammoth investments to its
(A) A and B (B) B and C secure location
(C) A only (D) B only (D) persuading aggressively Japanese business to shift
(E) C only at least some of their mammoth investments to its
23. The soldiers saw the camp of the enemy. They started secure location
attacking the enemy. 28. ________, that they could compete successfully, even
(A) While the soldiers saw the enemy camp …… with the higher techniques of production, which were
(B) When the soldiers saw the camp ……. being established in England.
(C) Because the soldiers saw the camp ……. (A) So efficient and highly organized were Indian
(A) B and C (B) A and C methods of production, and such was the skill of
(C) B only (D) A only India’s artisans and craftsmen
(E) C only (B) So efficiently and highly organized were Indian
methods of production, and such were the skill of
24. The weather is warm. I like to go for swimming now. India’s artisans and craftsmen
(A) If the weather is ………… (C) So efficient and highly organized were Indian
(B) Whenever the weather is ……… methods of production, and such was the skill of
(C) As the weather is ……. India’s artisan and craftsman
(A) A and B (B) A only (D) So efficiently and highly organized were Indian
(C) B only (D) C only methods of production, and such were the skill of
(E) None of these India’s artisan and craftsman
Verbal Ability Test 1 | 1.7

Directions for questions 29 to 33: In the following ques- 35. (A) If you want to play well, you must practise.
tions, two sentences are given. There may be an error in the (B) If you want to play well, one must practise.
sentence(s). Mark as your answer (C) If one want to play well, you must practise.
(A) if there is an error only in the first sentence; (D) If one wants to play well, he must practise.
(B) if there is an error only in the second sentence; 36. (A) Einstein was more cleverer than any other scientist.
(C) if there are errors in both the sentences and (B) Einstein was more cleverer than any scientist.
(D) if there is no error in either of the two sentences. (C) Einstein was cleverer than any other scientist.
(D) Einstein was cleverer than any scientist.
29. I. He said that he will come, but he didn’t.
II. I admit, ‘No news is good news now - a -days’. 37. (A) An argument developed among his sister and him.
(B) An argument developed between his sister and he.
30. I. I and my friend like to play tennis in grass court. (C) An argument developed between his sister and his.
II. Each of the mistakes have to be corrected before (D) An argument developed between his sister and him.
printing.
38. (A) The youngster will benefit from the experience.
31. I. I cannot see anything wrong with the plan. (B) The youngster will benefit by the experience.
II. You may read the book if you have enough time. (C) The youngster will benefit of the experience.
32. I. There is a little truth in what we have heard. (D) The youngster will benefit out of the experience.
II. You are not going to the theatre, isn’t it? 39. (A) T here were lesser children in the class than
expected.
33. I. He informed me before he had posted the letter
(B) There were fewer children in the class than
yesterday.
expected.
II. No one is as happy as he.
(C) There were a little children in the class than
Directions for questions 34 to 40: A sentence is given in expected.
four different forms. Only one of them is correct grammati- (D) There were a small children in the class than
cally. Mark the number of the correct one as the answer. expected.
34. (A) My sister likes painting, dancing and to cooking. 40. (A) The child can’t hardly wait till its birthday.
(B) My sister likes painting, dancing and to cook. (B) The child can wait till it’s birthday.
(C) My sister like painting, dancing and cooking. (C) The child can hardly wait till its birthday.
(D) My sister likes painting, dancing and cooking. (D) The child can wait hardly till its birthday.

Answer Keys
1. B 2. C 3. D 4. A 5. A 6. C 7. D 8. B 9. B 10. D
11. A 12. C 13. B 14. C 15. C 16. C 17. B 18. D 19. A 20. C
21. A 22. D 23. C 24. D 25. B 26. D 27. C 28. A 29. A 30. C
31. D 32. B 33. A 34. D 35. A 36. C 37. D 38. A 39. B 40. C

Hints and Explanations


1. Indences correlate. Choice (B) 10. Choice (D)
2. ‘Nor’ must be replaced by ‘or’. Choice (C) 11. Choice (A)
3. ‘Who the winner is’ is redundeant. Use only ‘the win- 12. Choice (C)
ner’. Choice (D) 13. Choice (B)
4. ‘Went about’ is the wrong phrase in the context of the 14. Choice (C)
given sentence. Choice (A)
15. Choice (C)
5. “A score of apples’ is the correct phrase, as ‘score’
16. The third part of the sentence should be ‘and crystal-
means twenty or a set of twenty. Choice (A)
lizes it’. Crystallizes is a verb which takes an object.
6. Choice (C)
 Choice (C)
7. Choice (D)
17. Part 2 of the sentence is faulty as the preposition used
8. Choice (B) should be ‘of’ and not ‘in’. One is the ‘master of the art’
9. Choice (B) not in the art. Choice (B)
1.8 | Verbal Ability Test 1

18. The fourth part of the sentence is faulty, as ‘drifting 27. The adverb ‘aggressively’ qualifies ‘persuading’ and
into winter’ is correct - it indicates motion. ‘In’ does hence must precede it (rules out choices 1 and 4).
not indicate motion. Choice (D) Choice 2 is wrong because ‘to shifting’ is incorrect.
19. In the first part of the sentence ‘its’ is in the genitive  Choice (C)
case, so there need not be an apostrophe, ‘Because of 28. Since the verb at the beginning of the sentence is ‘were’
its hardness’ is correct. Choice (A) (plural) it must be ‘efficient and highly organized’. If
20. The third part of the sentence should read ‘devote your- it were ‘efficiently’ then both (efficiently and highly)
self unswervingly and unflinchingly’ to maintain paral- qualify ‘organized’ and the verb would be was (rules
lelism in construction. Choice (C) out 2 and 4). We are talking of artisans and craftsmen
21. The two sentences can be combined into a simple sen- (plural again not singular) Choice (A)
tence by changing the verb ‘am’ into ‘v + ing’ (i.e.) 29. The past tense should be used. The first sentence should
‘being’. Hence ‘A’ is possible. The conjunction ‘as’ also be “He said that he would come, but he didn’t”.
can be used, as it shows reason. But ‘C’ cannot be used  Choice (A)
as the sentence cannot be combined with the conjunc- 30. The second person must come first.
tion ‘while’.  Choice (A) Statement 1 should be “My friend and I like to play ten-
22. The sentence donates a positive meaning. Hence it nis on grass court.”
cannot begin with ‘though’. The conjunction ‘as’ and Statement 2 Each of the mistakes has to be corrected
‘since’ can begin the sentence. But here ‘B’ begins with before printing. Choice (C)
the second sentence which changes the meaning of the 31. No error in both the sentences. Choice (D)
given sentence. Choice (E)
32. You are not going to the theatre, are you? Choice (B)
23. The sentences cannot give the same meaning if the con-
33. He informed me before he posted the letter yesterday.
junctions ‘while’ or ‘because’ are used. It shows a par-
ticular time and hence ‘when’ would be the right way to  Choice (A)
begin the sentence. Choice (C) 34. The gerund form should be maintained throughout a
24. ‘If’ or ‘whenever’ can also begin the sentence. But here sentence. Choice (D)
it is not a general statement. It specifies that particular 35. ‘You’ must be followed by ‘you’. ‘One’ must be fol-
time as ‘now’ is used. Hence ‘as’ is the most appropri- lowed ‘one’. Therefore, Choice (A) is right and the
ate way to begin this sentence. Choice (D) other choices are incorrect. Choice (A)
25. The sentence is about ‘not achieving’. ‘B’ and ‘C’ talk 36. ……. Cleverer than any other……… Choice (C)
about ‘achieving’ and hence cannot be appropriate. 37. ‘Between’ should be followed by the objective case
 Choice (B) ‘him’ and not the subjective case ‘he’. Choice (D)
Solutions for questions 26 to 28: 38. You benefit from something. Correct preposition.
26. ‘Between’ is better than ‘among’ since only two – the  Choice (A)
Indian and Nepalese – rebel groups are mentioned. 39. In case of numbers we use ‘fewer’. ‘Less/Lesser’ are
(Between is used for two or more, among for three used in the case of weight and ‘little/small’ in the case
or more). We are talking of the threat in Nepal not of of size. Choice (B)
Nepal. New Delhi cannot ignore (disregard intention- 40. ‘Hardly’ indicates ‘cannot’. So, ‘the child cannot wait
ally) not ignorance (lacking knowledge). Choice (D) for its birthday’ is being intended. Choice (C)
Verbal Ability Test 2
Number of Questions: 40 Time: 30 min
Directions for questions 1 to 5: In each of the following 14. REPEAL
questions, 3 words are related in some way. Find the “odd (A) continue (B) prolong
man” out. (C) promote (D) reject
1. (A) Expert (B) Professional (E) abrogate
(C) Civilized (D) Maestro 15. ABSCOND
2. (A) Blue (B) Crimson (A) run away (B) give away
(C) Ruby (D) Scarlet (C) move away (D) forbid
3. (A) Commentary (B) Critique (E) waste away
(C) Authority (D) Review
Directions for questions 16 to 25: In each of the questions
4. (A) Decahedron (B) Decade given below, identify the word which is opposite (antonym)
(C) Decagon (D) Decibel in meaning to the question word.
5. (A) Grave (B) Coffin
(C) Tomb (D) Monument 16. FUSION
Directions for questions 6 to 15: In each question given (A) union (B) participation
below, identify the word which is similar in meaning (syno- (C) isolation (D) marriage
nym) to the question word. (E) gloom
6. ABATE 17. SOMNOLENT
(A) improve (B) decrease (A) drowsy (B) lively
(C) subside (D) sharpen (C) cheerful (D) joyous
(E) sweep (E) active
7. RENOUNCE 18. MELLIFLUOUS
(A) relinquish (B) withdraw (A) harmonious (B) pitchable
(C) forgive (D) punish (C) discordant (D) internal
(E) accept (E) external
8. ABDUCT 19. PRODIGAL
(A) ransack (B) surround (A) generous (B) revisable
(C) induce (D) destroy (C) frugal (D) pauper
(E) kidnap (E) rich
9. DETEST 20. DISASTER
(A) hate (B) rebel (A) puzzle (B) success
(C) neglect (D) pretend (C) omen (D) festival
(E) captivate (E) fiasco
10. CLEANSE 21. ANIMATE
(A) polish (B) flow (A) truthful (B) false
(C) absolve (D) reveal (C) active (D) dull
(E) revolve (E) lazy
11. ABODE 22. WRETCHED
(A) sanctuary (B) residence (A) filthy (B) tidy
(C) reformatory (D) dwelling (C) neat (D) clear
(E) shelter (E) scenic
12. RESCIND 23. SQUANDER
(A) withhold (B) countermand (A) waste (B) liberate
(C) hamper (D) suppress (C) presume (D) donate
(E) encroach (E) economize
13. EPITOMIZE 24. PERTURBED
(A) disappoint (B) distend (A) servable (B) controllable
(C) exemplify (D) generate (C) composed (D) decided
(E) lengthen (E) resolved
1.10 | Verbal Ability Test 2

25. SLUGGISH (C) questionable dealings.


(A) lethargic (B) indolent (D) constant quarrel
(C) apathetic (D) intelligent
31. We were kept on tenterhooks while the judges were
(E) exuberant
deciding the winners.
Directions for questions 26 to 29: In each of the follow- (A) in anxiety (B) in trouble
ing questions, an idiomatic expression and its five possible (C) on the bridge (D) in the witness box
meanings are given. Pick out the correct meaning of the idi- 32. The dispute among the students came to a head and the
omatic expression and mark the number of that meaning as principal declared a holiday.
your answer. (A) came to one man’s decision
26. To be on the fiddle (B) reached a crisis
(A) to work on something important (C) did not stop
(B) to be doing something dishonest to get money (D) started all over again
(C) to constantly find fault with others 33. After getting a job he had no difficulty in keeping the
(D) to have a less important position than somebody or wolf out of the door.
something else alive (A) being physically safe
(E) to always keep oneself busy (B) getting the door clear
27. As the crow flies (C) avoiding starvation
(A) in a very swift manner (D) keeping the job intact
(B) in a straight line 34. He asked me not to thrust my nose into his affairs.
(C) in a very precise manner (A) smell anything wrong
(D) very short distance away (B) meddle officiously
(E) in a clumsy way (C) forget
28. In a melting pot (D) combine
(A) in a very difficult situation
(B) to take an important decision Directions for questions 35 to 40: In each of the following
(C) to be prone to bad influences questions four numbered choices are given. Three of them
(D) in a helpless situation belong to the same category. Mark the number of the ‘odd
(E) in the process of changing man’ as your answer.

29. To open somebody’s eyes 35. (A) Parents (B) Love


(A) to keep a watch on someone (C) Guidance (D) Punishment
(B) to be more observant and quick to notice things 36. (A) Periphery (B) Perfunctory
(C) to refuse to listen to others (C) Cursory (D) Superficial
(D) to become close or friendly with someone
(E) to make someone realize or understand something 37. (A) Prune (B) Abridge
(C) Trim (D) Spruce
Directions for questions 30 to 34: In each question, a sentence
38. (A) Overfly (B) Overeat
is given with an idiom (underlined). Four possible meanings
(C) Overdressed (D) Overdose
of the idiom are also given. Identify the correct meaning and
mark the number of the correct choice as answer. 39. (A) Serendipity (B) Fortuity
(C) Guess (D) Coincidence
30. The officer was guilty of sharp practices and so he was
dismissed. 40. (A) Vague (B) Abstract
(A) angry behaviour (C) Intangible (D) Empirical
(B) disobeying

Answer Keys
1. C 2. A 3. C 4. D 5. D 6. B 7. A 8. D 9. A 10. C
11. D 12. B 13. C 14. D 15. A 16. C 17. D 18. C 19. C 20. B
21. D 22. B 23. D 24. C 25. D 26. B 27. B 28. D 29. A 30. C
31. A 32. B 33. C 34. B 35. A 36. A 37. D 38. A 39. C 40. D
Verbal Ability Test 2 | 1.11

Hints and Explanations


1. Expert, professional and maestro talk about the skill 18. The word mellifluous means pleasingly smooth or
or knowledge of a person; ‘civilised’ talks about the musical to hear. The word discordant (cacophonous,
behaviour of a person. Choice (C) harsh) is its antonym. Choice (C)
2. Crimson, ruby and scarlet are shades of red. 19. Prodigal (wasteful, extravagant) and frugal (thrifty,
 Choice (A) economical) are antonyms. Choice (C)
3. Commentary, critique, review refer to the expression of 20. Success is an antonym of the word disaster which also
opinion. Choice (C) means failure. Choice (B)
4. Choices 1, 2 and 3 are related to ‘ten’ (ie) ‘deca’. In 21. Animate (alive) and dull are antonyms. Choice (D)
‘decibel’, ‘deci’ means one-tenth. Choice (D)
22. The words wretched (miserable) and tidy are antonyms.
5. Grave, tomb and coffin are related to death.  Choice (B)
 Choice (D)
23. Squander means to waste (money, time, etc) in a reck-
6. The words abate and decrease are synonymous. less or foolish way. Economize is its antonym.
 Choice (B)  Choice (E)
7. The word relinquish is synonymous with renounce 24. Composed (calm) is an antonym of the word perturbed
(give up, discard). Choice (A) (alarmed). Choice (C)
8. Abduct and kidnap are synonymous. Choice (E) 25. Exuberant (lively, cheerful) and sluggish (inactive) are
9. Detest means to loathe or hate. Choice (A) antonyms. Choice (E)
10. Cleanse (rid of something unpleasant or unwanted) 26. “To be on the fiddle” means doing something dishonest
is synonymous with absolve (declare free from guilt, to get money. Choice (B)
blame or sin). Choice (C) 27. The idiom, ‘as the crow flies’ means ‘in a straight line’.
11. The word abode which is a formal or literary term means  Choice (B)
a house or home, ‘dwelling’ is its closest synonym. The 28. The idiom, ‘in a melting pot’ means ‘in the process of
word ‘residence’ can be ruled out because it refers only changing’. Choice (E)
to a person’s home, whereas the word ‘abode’ has a wider 29. ‘To open somebody’s eyes’ is to cause or make some-
connotation. For eg. the abode of animals, the abode of one realize or understand something. Choice (E)
god but it is absurd to say the residence of god or the
residence of animals. Choice (D) 30. Choice (C)

12. Rescind means to cancel (a law, order or agreement) 31. Choice (A)
the word countermand, which also the means the same, 32. Choice (B)
is its synonym. Choice (B)
33. Choice (C)
13. The word epitomize means to be a perfect example of.
34. Choice (B)
The word ‘exemplify’ is its closest synonym.
 Choice (C) 35. Parents are not concepts as the other three are.
 Choice (A)
14. Repeal means to officially cancel (a law of act of parlia-
ment) the word ‘abrogate’ also means the same. 36. Periphery. The other three are associated with casual
 Choice (E) attitude. Choice (A)

15. Abscond means to leave quickly and secretly to escape 37. The first three refer to cutting short. Choice (D)
from custody or avoid arrest. Choice (A) 38. Overfly is to fly above a place. In all the other three over
16. Fusion (the process of joining two or more things to has the meaning of excessive. Choice (A)
form a whole) and isolation are antonymous. 39. Guess. The others refers to a favourable chance.
 Choice (C)  Choice (C)
17. Active (alert) is an antonym of somnolent (sleepy; 40. Empirical is verifiable the others are not.
drowsy). Choice (E)  Choice (D)
Verbal Ability Test 3
Number of Questions: 40 Time: 30 min

Directions for questions 1 to 10: In each question the D. “Don’t be led by others, be your own master,”
word at the top is used in four different ways. Select the Sampath said severely.
option in which the usage of the word is INCORRECT or
6. CLOSE
INAPPROPRIATE.
A. The soldiers advanced in a close formation.
1. COMMUNICATE
B. Over the next few months we have to keep a close
A. The deaf and dumb communicate by means of sign
eye on sales.
language.
C. Alind closed down in the nineties.
B. The excitement was palpable and communicated
itself to the crowd. D. The police often close ranks when one of their offic-
ers is accused.
C. People living in the suburbs have
to communicate a long distance every day. 7. BACKGROUND
D. A contagious disease is communicated A. The name ‘TIME’ is written in red on a white
through physical contact. background.

2. OPPOSITE B. The film has good background music by Rehman.

A. Being an adolescent, Mrinal felt shy talking to mem- C. The elections in Sri Lanka took place on a back-
bers of the opposite sex. ground of violence.

B. I expected the bride to be shy and quiet, but she was D. The Director asked for more background on the
just the opposite. company’s financial position.

C. The Raos live further down, on the opposite side of 8. DEFENCE


the road.
A. Whenever Roja was criticized, her brother leapt to
D. The bank is opposite to the supermarket. her defence.
B. Why don’t you give a chance for the body’s natural
3. RAW
defence mechanism to protect it?
A. Dostovsky’s novels often portray life in the raw.
C. No cost is too high when it is for the defence of the
B. Women labourers often get raw deal from country.
contractors.
D. News of an imminent attack forced the troops onto
C. Her own experiences provided the raw material for the defence.
her first novel.
9. CHANGE
D. Being marooned on the island forced them to eat
raw meat. A. The property changed hands several times in the last
decade.
4. PICTURE B. Expecting a change of heart from that stubborn mule
A. There have been a number of changes recently - let is useless.
me put you in picture.
C. Some of my old dresses will have to be changed to
B. Atticus always told the old lady that she looked a fit me now.
picture.
D. Marriage has changed Sania for better.
C. Ever since he went into pictures the couple have
been drifting apart. 10. EARTH
D. From the reports, the picture for the service sector A. Yasho was the happiest person on earth when she
is encouraging. won the gold medal.
B. Aditya flung his bike on the earth and rushed inside
5. MASTER when he saw smoke emanating form the house.
A. Vivek realised that he was expected to master
C. The good earth always gives back several times
Japanese before leaving for Japan.
what you put in.
B. Tilak has a masters in Business Administration.
D. Be sure to earth household electrical gadgets so that
C. The master bedroom was spacious and comfortable. they are safe to handle.
Verbal Ability Test 3 | 1.13

Directions for questions 11 to 20: In each of the following (B) It plans to recruit 500 research scientists.
questions, a paragraph with a ‘blank’ is given. From the four (C) This will help develop GE’s global business.
choices, select the sentence, which can go into the blank to (D) The project will be over by December, 2000.
make the paragraph logically coherent. 16. One major change in careers is that one can work from
11. India has the distinction of becoming a country with home. (_____.) So far, only work relating to Information
a billion people. Thus, it becomes the second largest Technology has been thus affected. It is expected that
populated country in the world. (____.) many other careers will afford this flexibility in the future.
(A) The global population has almost touched the six (A) Therefore one should develop a confident, outgo-
billion mark. ing personality.
(B) Statistically speaking, every sixth person in the (B) There is no such thing as a permanent job.
world is an Indian. (C) New technologies ensure that geographical dis-
(C) Forced population control is not desirable. tance is not a hindrance to one’s work.
(D) It is difficult to judge how many forests have been (D) While it is true that people will switch jobs faster
encroached upon. than ever before, one must be loyal to one’s
12. In the armed forces, before independence, Muslims organisation.
constituted around 35 percent of the total strength. 17. (____.) There are several cave paintings, stone engrav-
(____.) Why so few Muslims? ings and carved figures which bear this out. The
(A) Most of them were recruited from Punjab and Neanderthal man attempted this too, but his drawings
nearby areas. of the tools he used show that they were rather crude.
(B) Today, it has gone down to just a mere two percent (A) Prehistoric man used sophisticated tools for draw-
out of a total close to a million. ing and carving figures.
(C) This is basically due to a lack of lobby. (B) The Cro-Magnon man, who was the forerunner
(D) This can be attributed to a change in human of modern man, earned his daily bread through
paintings.
behaviour.
(C) The Cro-Magnon man, who was the forerunner of
13. It would be a Herculean task to remodel our settlements modern man, was the first fine artist in the history
to keep them clean through proper waste disposal sys- of man’s evolution.
tems. As such we have already realized that, recycling (D) Prehistoric man pursued painting and carving fig-
of waste liquids and solids would be a saner approach. ures as a hobby.
(____.) 18. The natural atmosphere which man has inherited from
(A) Now we are very used to human interference with the past, has been deteriorating under the impact of
nature. industrialization. Factories pump millions of tons of
(B) We may not be able to survive utilising the avail- dust into the air, vehicles spread fumes and sprays are
able resources. used to kill agricultural pests – all combine to change
(C) But organising such measures will involve consid- the ideal picture. (____.) The situation near big cities
erable time, effort, management and education. and heavily industrialized areas has become particu-
(D) Many of our organisation lack this foresight. larly bad, and the air is not fit for breathing.
14. When a bird hits an aircraft, it can cause potentially (A) Movement of vehicular traffic on the roads should
catastrophic damage. (____.) So a team at Britain’s be restricted.
Defence Evaluation and Research Agency plans to (B) The pollution of air has become a matter of great
use crystals that glow when fractured to warn of such concern because it continues to increase as civili-
unseen damage. zation spreads.
(A) This makes visual inspection of damage unreliable. (C) Society will have to move towards stricter pollu-
(B) This is one of the greatest dangers of information tion control.
technology. (D) The atmosphere should be protected as it is a great
(C) This is the ease with which communication goes and irreplaceable resource for living.
on these days. 19. In recent times, the number of working women has
(D) But in planes made of carbon composites, such increased considerably in urban areas. With more and
damage may be impossible to spot. more women opting for career-oriented courses, offices
15. The General Electric Company is setting up India’s first and business establishments are flooded with applica-
multi-disciplinary research centre. (____.) It will con- tions from qualified women. (____.) Women are work-
tribute to the development of multi-disciplinary engi- ing side by side with men in all walks of life.
neering capabilities in India. (A) A working woman’s life is not a bed of roses.
(A) Named the GE India Technology centre, it is also (B) In fact, there are very few workplaces today which
the largest of its kind. do not have single women.
1.14 | Verbal Ability Test 3

(C) It is possible to maintain a good standard of liv- 27. (A) imminent (B) preliminary
ing only if the woman contributes to the family (C) precursory (D) terrible
income. 28. (A) authorized (B) recommended
(D) Even in small towns and villages, most women are (C) sanctioned (D) banned
employed.
29. (A) defended (B) ignored
20. For several thousands of years, the moon has been the (C) cherished (D) maintained
only satellite of the earth. Today, however, the earth
30. (A) departed (B) depleted
has many other satellites – all made by man. (____.)
(C) withdrawn (D) vanished
However, some of them will still be going around the
earth thousands of years from now. Directions for questions 31 to 40: In each of the fol-
lowing questions a pair of words in capitals is given fol-
(A) Artificial satellites do not fall because they are not lowed by four numbered pairs of words. Select from the
affected by earth’s gravity. choices the pair which exhibits the same relationship
(B) They travel in an orbit around the earth. as the capitalised pair of words and mark the number as
(C) As they speed along, they tend to go straight off your answer.
into space. 31. COGENT : CONVINCING
(D) These artificial satellites are very much smaller (A) Insane : Distinguished
than the moon. (B) Laconic : Pithy
Directions for questions 21 to 30: In the following passage (C) Illogical : Reasonable
there are blanks, each of which has been numbered. These (D) Jovial : Abstruse
numbers are printed below the passage and against each, 32. RETROSPECTION : PAST
five words are suggested, one of which fits the blank appro- (A) Syllogism : Logic
priately. Find the appropriate word in each case and mark its (B) Idiosyncrasy : Coherence
number as your answer. (C) Prognostication : Future
In most developed and developing nations, the illicit (D) Transience : Rigidity
trade in live wild animals is (21) . Each year millions of
(22) are wrenched from their natural habitats by people 33. EULOGISE : LAMBAST
(23) to make quick money, then routed through a ragtag (A) Mystify : Narrate
chain of middlemen and international dealers to meet (B) Dissemble : Besmirch
the (24) demand of private collectors in Saudi Arabia, (C) Invigorate : Debilitate
pet shops in Germany, Japan and the U.S.; zoos and (D) Malinger : Adhere
circuses in Eastern Europe and folk healers in Asia. Ac- 34. LION : PRIDE
cording to a wildlife expert, it is the third biggest (25)
(A) Rabbit : Burrow (B) Pup : Litter
business, after drugs and arms.
(C) Whale : Consort (D) Sow : Sty
Though many exotic species can be purchased (26)
trade in animals and birds that are in (27) danger of 35. PENURIOUS : AFFLUENCE
extinction is (28) under the United Nations Convention (A) Interrogation : Accusation
on International Trade in Endangered Species (CITES), (B) Garnishment : Command
which has been signed by 120 nations. The treaty also (C) Taciturn : Verbosity
regulates trade in other species that are seriously threat-
(D) Condemnation : Mischief
ened, but its provisions are widely (29) , even in signa-
tory countries. Tigers have all but (30) from China and 36. MACHIAVELLIAN : DECEIT
are fast disappearing from India and Siberia. (A) Amphibious : Plants
21. (A) diminishing (B) unfavourable (B) Acquisition : Assumption
(C) miserable (D) flourishing (C) Acquittal : Suit
(D) Naïve : Gullibility
22. (A) creatures (B) categories
(C) people (D) characters 37. BENEVOLENT : GRASPING
(A) Repulsive : Pushing
23. (A) hopeless (B) agreeable
(B) Euphonious : Discordant
(C) desperate (D) susceptible
(C) Churlish : Impolite
24. (A) fulfilled (B) imperative (D) Rebellious : Disorderly
(C) unavoidable (D) insatiable
38. FRIGHTEN : PETRIFY
25. (A) licensed (B) virtual (A) Enamour : Protect
(C) authentic (D) illegal (B) Sneer : Appreciate
26. (A) inequitably (B) immorally (C) Abbreviate : Interest
(C) legally (D) profitably (D) Humiliate : Mortify
Verbal Ability Test 3 | 1.15

39. MUNIFICENT : STINGY 40. EUPHORIC : ECSTASY


(A) Inclement : Merciless (A) Modified : Version
(B) Incorrigible : Recalcitrant (B) Redundant : Relevant
(C) Articulate : Obscure (C) Licentious : Sentiment
(D) Egregious : Outstanding (D) Cryptic : Enigma

Answer Keys
1. C 2. D 3. B 4. A 5. B 6.  A 7. C 8. D 9.  C 10. B
11.  B 12.  B 13. C 14. D 15. A 16. 
C 17.  A 18. 
B 19. D 20. D
21.  D 22. A 23. C 24. D 25. D 26. 
C 27.  A 28. 
D 29. B 30. D
31.  B 32. 
C 33. C 34. B 35. C 36. 
D 37.  B 38. 
D 39. C 40. D

Hints and Explanations


1. In sentence 3 the intended word is ‘commute’ (travel walk on, we use ‘ground’ for outside and ‘floor’ for
regularly by bus, train etc between your place of work inside. In sentence 2 it should be ‘ground’ not earth.
and home) and not “communicate” (to exchange infor-  Choice (B)
mation, ideas etc). Choice (C)
1 1. The topic is about India. Global population, popula-
2. In sentence 4 opposite is a preposition meaning “on tion, forests are irrelevant. Hence, option B is the only
the other side of a particular area from something” and relevant choice. Choice (B)
does not require ‘to’. Choice (D)
12. Option B is the only statement that leads to the ques-
3. In sentence 2 it should be ‘a raw deal’. The idiom tion, “Why so few Muslims?” Choice (B)
means ‘the fact of somebody being treated unfairly’.
13. The first line has the words “Herculean task” indicating
 Choice (B) that the task is not easy. Option C brings out the aspect
4. Sentence 1 should read … in the picture. The idiomatic that such measures will require a lot in terms of time,
expression ‘to put somebody in the picture’ means to effort, management and education. Option C is the right
give somebody the information they need in order to answer. Choice (C)
understand a situation. In sentence 2 ‘to look a picture’ 14. Choice D is the only relevant statement. Since, the
means to look very beautiful or special. Choice (A) damage could be impossible to spot, an agency is com-
5. When ‘masters’ refers to a university degree it ing up with a new alternative (as suggested by the last
takes the apostrophe, hence ‘Master’s in Business line). Choice (D)
Administration’. Choice (B) 15. Stylistically, the flow of the sentences is as follows: GE
6. In sentence 1 ‘close’ means ‘without space’. Hence it is going to set up a research center. Named . . ., it is .
is ‘close formation’ not ‘a close formation’. To “close . . largest of its kind. It will (this research center will)
ranks” (sentence 4) means to work closely together to contribute to . . . Hence, option A is the right answer, as
defend themselves. Choice (A) it fits the blank perfectly. Choice (A)
7. In sentence 3, background refers to the past. It should 16. Sentence 3 is the ideal choice because it supports the
be ‘. . . against a background of violence’ but not ‘on’. previous sentence and explains the subsequent sen-
 Choice (C) tences. Choice (C)
8. Sentence 4 should read ‘ . . . onto the defensive’ - an 17. The second line states, “There were . . . Figures which
idiom that means acting in a way that shows that you bear this out. He attempted to create his daily life
expect to be attacked or criticized. Choice (D) through this. The Neanderthal man attempted this too,
but his drawings of the tools . . . So, we need a state-
9. ‘Change’ is a very general term that is used to describe
ment in the blank which is a general statement on the
any act of making something different. In sentence 3
pre-historic man and drawing/painting. Our choice nar-
the right word is ‘alter’ not ‘change’. You ‘alter’ some-
rows down to (A) and (D). Out of these two choices,
thing by making a difference in its appearance, charac-
option A is better because it contrasts the sophisticated
ter or use. You cannot use ‘change’ here. Choice (C)
tools used by pre historic man with crude ones used by
10. To talk about our planet we use ‘earth’. Earth is also Neanderthal man. Choice (A)
used to refer to the soil. As a verb it means making an
18. Sentence 2 explains how air pollution has increased
electrical equipment safe by connecting it to the ground
with the spread of civilization. This sentence explains
by a wire. But when we refer to the hard surface we
the last sentence of the paragraph. Choice (B)
1.16 | Verbal Ability Test 3

19. Choice (D) best supports the argument put forth in the 28. It is considered illegal because it has been ‘banned’ by
paragraph. Choice (D) the CITES, a world organisation established in order to
20. Choice (D) speaks about artificial satellites, which protect endangered species of birds and animals glob-
though smaller than the moon will revolve around the ally. Choice (D)
earth even after several years. The sentence emphasizes 29. The treaty not only ‘bans’ the trade of certain endan-
the power of artificial satellites. gered species but also regulates trade in other species
 Choice (D) that are threatened. But still the illegal trade of all these
animals continues to flourish even in the 120 countries
21. The illicit trade in wild animals is prospering or grow-
that have signed the treaty. This means that the treaty
ing or ‘flourishing’ in most developed and developing
and its directives are ‘ignored’ even by these countries.
nations. The passage goes on to explain how it is ‘flour-
All the other choices are positive and conveys an oppo-
ishing’. All the other choices are negative terms which
site meaning and hence are incorrect. Choice (B)
are inappropriate in this context.
30. The last sentence says that tigers are fast disappear-
 Choice (D)
ing from India and Siberia. Preceding this, it says
22. Animals live in their natural habitat. These ‘creatures’ tigers have already disappeared from China. Hence
are wrenched or harshly pulled away from their natural ‘vanished’ is the most appropriate choice. ‘Deplete’ or
habitat. Choice (A) ‘withdrawn’ or ‘depart’ shows that they are decreasing.
23. Who are the people who wrench these creatures away But the sentence suggests that tigers have already dis-
from their natural habitat? They are those who are appeared from China. Hence choice D is most appro-
‘desperately’ looking for quick money. The ‘desperate’ priate. Choice (D)
desire to make quick money drives these people to get 31. Cogent and convincing are synonyms as are laconic
themselves involved in this illicit trade. Choice (C) and pithy. Choice (B)
24. The paragraph goes on to give a list of people who are 32. Retrospection is analysis of something related to the
interested in this trade – from private collectors to folk past. Prognostication means prediction, which is related
healers. Hence there is always a heavy demand or an to the future. Choice (C)
‘insatiable’ (impossible to satisfy) demand for these
33. Eulogise means praise, whereas lambast indicates criti-
animals. Choice (D)
cism. Hence, option (C) is the answer as this choice
25. But this trade in live wild animals is an ‘illegal’ busi- also has a pair of antonyms. ‘Invigorate’ means ‘to
ness and is rated next to drugs and arms business at energise’ and ‘debilitate’ means to ‘weaken’.
the global level. It cannot be ‘authentic’ or ‘licensed’ or
 Choice (C)
‘legal’ as the second para goes on to explain the direc-
tives of the CITES treaty made by various nations in 34. A group of lions is referred to as a ‘pride’, whereas a
order to protect the endangered species from extinc- group of pups (puppies) is called ‘litter’. Choice (B)
tion. Choice (D) 35. Antonyms Choice (C)
26. Some of these exotic species can be purchased. The use 36. A machiavellian person indulges in deceit. A naïve per-
of ‘though’ indicates that this purchase is considered son shows gullibility. Choice (D)
‘legal’ with reference to certain species of animals and 37. Antonymous relationship Choice (B)
birds which are available in plenty. Further the sentence
38. Relationship of degree. Petrify indicates extreme fright.
goes on to say that trade in some species of birds and
Mortify indicates extreme humiliation. Choice (D)
animals that are near extinction is not allowed under the
UN convention. Hence ‘legally’ is the most appropriate 39. A munificent person is generous (not stingy) Likewise,
word. Choice (C) any argument which is articulate is very clear (not
obscure). Choice (C)
27. There are certain species of animals and birds that
are in ‘imminent’ (impending) danger of extinction. 40. Cryptic and enigma are also synonymous.
Trading of such animals and birds are considered In both the first is an adjective and the second a noun.
illegal. Choice (A)  Choice (D)
Verbal Ability Test 4
Number of Questions: 40 Time: 30 min

Directions for questions 1 to 8: Four alternative summaries and dissimilarities to make it shine out of the
are given below each text. Choose the option that best cap- group.
tures the essence of the text. (B) Aristotle suggests that man as an object has to be
1. An experiment is an observation that can be repeated, defined first as an animal and then shown to be dif-
isolated and varied. The more frequently you can repeat ferent from other animals in his rationality. Such
an observation, the more likely are you to see clearly an interpretation helps in revealing the unique
what is there and to describe accurately what you have characteristic of an object.
seen. The more strictly you can isolate an observation, (C) In order to define an object, Aristotle first assigns
the easier does your task of observation become, and it to its class and then points out its dissimilarities.
the less danger is there of being led astray by irrelevant Such a comparison makes its individuality more
circumstances, or of placing emphasis on the wrong conspicuous.
point. The more widely you can vary an observation, (D) According to Aristotle, any good definition of an
the more clearly will the uniformity of experience object has two parts, one that identifies it with its
stand out and the better is your chance of discovering group and the other that shows its uniqueness.
laws. A definition that includes both these will help in
(A) It is essential that scientific experiments be identifying the individuality of the object.
repeated, isolated and varied because repetition 3. Sociologists are often criticized for their use of jargon,
increases accuracy, isolation facilitates observa- their apparent predilection to develop new words while
tion and variety increases the chance of discover- at the same time giving new and often strange meanings
ing laws. to old and familiar terms. The charges are often justi-
(B) Isolation and repetition of experiments help in fied. Equally often they go beyond reason. Systematic
reducing the possibility of error in observation. discussion is impossible if one does not work with more
The lesser the margin of error, the better is the or less precisely defined terms. Without a technical
chance of discovering new laws. language, scientific communication becomes cumber-
(C) Scientific experiments should be isolated, repeated some and inefficient. In the humanities too, the desire
and varied. Repetition helps to see clearly and iso- to be more precise in analysis leads to the elaboration
lation reduces the chance of being led astray. But of technical terms.
it is uniformity in the variety that gives a better (A) Sociologists are often criticized for the use of
chance of discovering laws. strange jargon and for giving extended meaning to
(D) An experiment should be isolated, repeated and familiar terms. But they are justified in doing so as
varied. They help in observing things clearly and such usage makes their analysis more precise and
accurately. They also increase the chance of dis- systematic.
covering new laws. (B) Critics are justified in their charge against the
2. Aristotle in his little treatise on ‘Definitions’ suggests sociologists for their use of strange jargon or for
that every good definition has two parts, stands on two giving extended meaning to familiar terms. But
solid feet: first it assigns the object in question to a the sociologists give the example of the scientists
class or group whose general characteristics are also to justify their stand.
its own - so man is, first of all, an animal : and sec- (C) In a desire to be more precise in their analysis
ondly, it indicates wherein the object differs from all sociologists develop new words or give strange
the other members in its class - so man, in Aristotelian meaning to old terms. Their justification is that
system, is a rational animal, his ‘specific difference’ only such extension of meaning can help them in
is that unlike all other animals he is rational. Aristotle systematic discussions.
drops an object into the ocean of its class, then takes it (D) While critics are justified in accusing sociologists
out all dripping with generic meaning with the marks of using strange jargons and for giving extended
of its kind and group; while its individuality and differ- meaning to familiar terms, sociologist use such
ence shine out all the more clearly for this juxtaposition terms in their desire to be more precise and sys-
with other objects that resemble it so much and are so tematic in their discussions and analysis.
different. 4. How big is your footprint? We’re not talking about
(A) A good definition is like a man standing on two whether you walk around in dainty Cinderalla’s slippers
solid feet. It not only helps in assigning an object or size 18 Wellington boots, but rather the size of the
to a generic group but also shows the differences damaging dent that you leave in the Earth’s environment
1.18 | Verbal Ability Test 4

and its resources while going about your daily life. One (C) Protected areas are threatened by hunting and log-
way of finding out if your impact on the world is like tee- ging as also by subtle actions like poaching on
tering on tiptoes or stomping about in steel-capped boots its land for cultivation. The main reason for the
is to measure your personal ecological and carbon foot- threat are population pressure on land, poverty
prints. A carbon foot print is a measurement of the effect and corruption.
you have on the climate in terms of the total amount of (D) Poaching by coffee growers of protected land is as
greenhouse gases that your actions cause to be produced, much a threat as hunting and logging in Asia. The
while an ecological footprint is a measure of the amount villain of the piece is, however, corruption and
of productive land required to support your resource poor law enforcement.
demands and to absorb the waste you produce. 6. Two decades after the event, and the word ‘Chernobyl’
(A) Is your footprint on the environment like walking still carries a lot of baggage but then, we’re still 25,000
on tiptoe or stomping around in steel boots? That years away from being clean. The meltdown at Reactor
is what is measured by ecological footprint - the 4 of the Soviet nuclear power station was caused by
land you require for resources to meet your needs, an unnecessary low-power test, an experiment to deter-
the waste you dump and the greenhouse gases that mine whether the reactor could restart itself with all
you cause. external power shut off. The result was a black fire-
(B) How much we impact our environment is mea- ball that blew the reactor’s roof off and spread radia-
sured by carbon footprint and ecological footprint tion across much of the Northern Hemisphere. Equally
- the former a measure of the greenhouse gases we toxic was the Soviet government’s initial response :
are responsible for and the latter the land we need a three-day silence as to the dangers of the situation.
for resources and waste. Protective foam was sprayed around the nearby towns,
(C) If one walks around daintily like Cinderella, one Chernobyl and Pripyat, whose combined population
harms the environment less than if one tramps was 135,000, but otherwise, life proceeded as normal
around in boots. The greenhouse gases one emits, - children played in the foam, marched in the May Day
the waste one generates and the resources one parade and breathed in the contaminated air. While the
needs, all measure the damage one inflicts on official death toll remains at 41, who knows how many
one’s surrounding. fatalities resulted from the delay in evacuating these
(D) The effect our actions have on our environment is towns?
measured by carbon footprint, the amount of land (A) Twenty years after Chernobyl, caused by an
we need to sustain ourselves, the waste and green- experiment, we are still emotionally involved and
house gases generated by us are noted to arrive at haven’t been able to wipe out the radiation that
this figure. spread over the Northern Hemisphere. No one
5. It’s clear that in many countries protected areas are knows how many deaths have resulted from the
seriously compromised. The dramatic rise of hunting Soviet government’s refusal to evacuate the people
for bushmeat in west and central Africa over the past in the region.
30 years has been well documented, as has the explo- (B) Chernobyl was caused by an unnecessary test that
sion in the past decade of illegal logging in southeast affected millions. The Soviet government’s low-
Asia. But elsewhere, protected areas face more insidi- keyed response led to many more deaths though
ous threats. The Dong Hua Sao National Biodiversity officially it is only 41. We are still emotionally
Conservation Area in Laos, for example, has suffered a swayed by the memory and are far from cleaning
gradual erosion of its boundaries at the hands of coffee up the mess.
growers keen to take advantage of its rich volcanic soil. (C) The Soviet government’s reaction to Chernobyl
The reasons for such pressures are varied and complex, was as poisonous as the gas it emitted. The gov-
but they often include population growth, land pres- ernment’s apathy led to millions of avertable
sure, poverty, corruption and poor law enforcement. deaths and we still do not know how to stop the
(A) While hunting and logging are generally consid- radiation from continuing and spreading.
ered the main threat to protected areas, a greater (D) Chernobyl spread radiation across the Northern
evil is the gradual occupation of land by people Hemisphere and affected so many that two decades
living around who are attracted by its fertile land. later we are still emotionally affected by the mem-
(B) Population growth, poverty, and corruption have ory. The Soviet government put up a brave front
led people in Laos to gradually occupy the land by not evacuating the people and spraying foam to
under conservation for biodiversity. Thus it is contain the radiation.
a greater challenge than the rise in hunting for 7. Down syndrome babies are generally born to older
bushmeat in west and central Africa or logging in mothers. The probability of having a Down syndrome
southeast Asia. baby grows rapidly and exponentially as the age of the
Verbal Ability Test 4 | 1.19

mother increases, from 1 in 2,300 at the age of twenty but this was used relatively sensibly to bring about
to 1 in 100 at forty. It is for this reason alone that Down a more-than-proportionate growth in the size of its
embryos are the principal victims or their mothers the economy.
principal users, of genetic screening. In most countries (A) Though Jefferson warned against public debt, the
amniocentesis is now offered to - perhaps even imposed USA accumulated enormous debt as it considered
on - all older mothers to check whether the foetus car- it to be an essential prerequisite for an economy to
ries an extra chromosome. If it does, the mother is grow.
offered or cajoled into an abortion. The reason given is (B) As the USA amassed an enormous debt, Jefferson
that despite the happy demeanor of these children, most felt public debt to be an evil that should be avoided.
people would rather not be parent of a Down child. If But it is essential for an economy to grow.
you are of one opinion, you see this as a manifestation (C) Though Jefferson called public debt ‘the greatest
of benign science, miraculously preventing the birth of of dangers’ yet the example of the USA reaffirms
cruelly incapacitated people at no suffering. If you are it to be a proven method of bringing about growth
of another opinion you see the officially encouraged in the economy.
murder of a sacred human life in the dubious name of (D) Jefferson said public debt would lead to the weak-
human perfection and to the disrespect of disability. ening of an economy but his own country showed
You see, in effect, eugenics still in action, more than that it is one of the means of achieving a more than
fifty years after it was grotesquely discredited by Nazi proportionate progress.
atrocities. Directions for questions 9 to 18: A number of sentences
(A) Since Down syndrome babies are born to women are given below which, when properly sequenced, form a
over forty, they are often forced to have amnio- coherent paragraph. Each sentence is labelled with a letter.
centesis. This has been controversial. Some see Choose the most logical order of sentences from among the
the prevention of the birth of a disabled child as four given choices to construct a coherent paragraph.
kindness while others think the state is indulging
9. (a) Here is the world’s newest temple of modern art,
in eugenics as did the Nazis.
an artistic triumph in itself, and combinig gran-
(B) The probability of having a child with Down syn-
deur, originality and stunning power.
drome increases as the age of the mother increases.
(b) It can be compared to a cathedral, in that it occu-
So older women take the amniocentesis test and
pies such a vast space as inspires awe.
abort a Down embryo even though this amounts
(c) One of the unique elements of this edifice is a
to a murder. The killing of the disabled is similar
monstrous steel spider that must surely have been
to the practices of the Nazis.
inspired by a horror movie.
(C) The amniocentesis has been a controversial test
(d) The place is a huge physical entity, vast, and
since it is used to identify and do away with a
stupendous.
Down embryo in older women. While some think
(A) abdc (B) cdba
it is kindness to prevent the birth and unnecessary
(C) adbc (D) cadb
suffering that a Down child suffers, others think
they are murdering the disabled to selectively 10. (a) In the South, freshly grated coconut is the usual
breed a healthy generation. gamish.
(D) Mothers over forty are more likely to have a (b) In other parts, fresh coriander leaves provide the
Down child. They are persuaded or forced to have mandatory finishing touch.
amniocentesis. The abortion of Down embryo can (c) Besides the tempering exercise, the fat-laden
be seen as an expression of benevolent science or coconut helps to release fat-soluble carotence,
as officially sanctioned murder of disabled, that is from a carrot for example.
eugenics fifty years after the Nazis. (d) Traditional gamishes are another characteristic
feature of our cooking.
8. “I place economy among the first and most important
(A) cabd (B) dacb
of Republican virtues and public debt as the greatest of
(C) acdb (D) cbda
the dangers to be feared”. So wrote Thomas Jefferson
in 1816 in a letter to William Plumer, the governor of 11. (a) Every ceramic object is a sculpture in miniature,
New Hampshire. However, contrary to the wisdom of and constitutes a study in several kinds of paradox.
the great man, national debt is by no means an inher- (b) The fragility of material culture is nowhere more
ently bad thing. Borrowing money is a historically manifest than it is in the art of pottery.
tried-and-tested method of expanding the produc- (c) At the formal level, the solidity of the ceramic
tive capacity of an economy, if not a pre-requisite. object conveys a sense of permanence and endur-
The USA would go on to amass an enormous debt, ing value.
1.20 | Verbal Ability Test 4

(d) At the same time, the baked earth, from which it (b) The land to accommodate huge population was
is made, renders it fragile, frangible, a hostage to obtained by cutting down trees.
chance and threat of damage. (c) Area cleared by chopping down forests was used
(A) bacd (B) abcd to set up industries, thermal plants etc.
(C) bcda (D) acdb (d) Acre after acre of forest is cleared in one day leav-
12. (a) It is more than 200 years since Tipu Sultan’s mys- ing the atmosphere susceptible to pollution and
terious death occurred while defending his fort in the earth poorer.
Srirangapattana near Mysore in India. (e) The speed of cutting down forest has increased
(b) As a warrior, he was a formidable and implacable ever since.
enemy. (A) cabde (B) ecdba
(c) His reputation in the annals of British history puts (C) bcaed (D) dcbae
him in the same class as Chenghis Khan, Attila the 17. (a) No place in the house seemed secure.
Hun and Emperor Napoleon. (b) I remembered the agonies of my own childhood
(d) But his legend in India and in the West, is still when my sister discovered I was writing poems
going stron. and began to tease me by chanting them in public.
(A) abcd (B) adcb (c) When my daughter began to write her memories,
(C) acbd (D) adbc at the age of four, I decided that she must have a
13. (a) Puranic literature describes the progression of the place to keep them, if only a section of a bureau
cosmos which passes through various yugas, each drawer or as it happened, a box with a key.
of which is measured in precise numerical terms. (d) It seems to me that we can’t learn too early in life
(b) But in the cosmic scheme of Time, it does not to respect the privacy of the individual.
amount to much. (e) I tried desperately to hide the notebook of poems.
(c) Ten centuries may be a substantial time-span in (A) bcdae (B) dcbea
human reckoning. (C) cedba (D) aecbd
(d) In that calculation, our millennium, just over, does
18. (a) My father and mother were the complements
not form even a small fraction of the Kaliyuga, the
of each other.
last of the four cosmic cycles.
(b) Her face was responsive, my father’s impassive.
(A) cdba (B) abcd
(c) My mother was fragile, my father robust.
(C) adbc (D) cbad
(d) My mother was not intellectual her natural propen-
14. (a) My boss asked me to call on one of the Ministers sity was intuitive. Her face illustrated that saying
he knew there. “Appearances are deceptive: for it did not show
(b) I was leaving for Delhi on work as usual. the immense strength of her moral convictions.
(c) This meeting was supposed to sort out the mess (e) My mother’s face rippled to emotions as waters to
he was in, created by a new amendment to excise the wind.
duty provisions on export of cotton. (A) acbed (B) cabde
(d) At Delhi I sought an appointment with the Minister (C) ecbad (D) dbcea
concerned.
(A) dabc (B) bdca Directions for questions 19 to 23: In each question below,
(C) bacd (D) cdab a paragraph or a statement is given followed by four state-
15. (a) Since then sponsorship and endorsements started ments. Classify each of the four statements as per the fol-
playing a big role, with the profit motive becoming all lowing categories and from among the answer choices,
pervasive. select the one that gives the sequence of letters that matches
(b) Recent reports trace the corruption of Olympic with your categorization. Categorise the statement as
movement to the total commercialisation of games (A) if it is a CONCLUDING ASSERTION
in 1984. (B) if it is a SUPPORTING REASON
(c) Sordid behind the scenes going on in the IOC have (C) if it is a STATEMENT OF ARGUMENT
been finally exposed. (D) if it is a COUNTER ARGUMENT
(d) Salt Lake city which is bidding for Winter Olympics 19. With globalisation making its presence felt, a number
bent all the norms and even bribed IOC members. of foreign influences are gradually seeping into our cul-
(A) cdba (B) cabd ture. And these are felt in food as well. Moreover, with
(C) cdab (D) dcba the break-up of the joint family system and more and
16. (a) Which pump out an enormous quantity of smoke, more working couples setting up homes on their own,
poisonous gases and other wastes in the surround- people prefer eating out to returning home to cook after
ing environs. a hectic day’s work. Capitalising on this new trend,
Verbal Ability Test 4 | 1.21

established as well as upcoming restaurateurs feel that (b) Countries which had adopted similar procedures
constant improvisation and innovation is the key to a have now started reaping the benefits.
successful restaurant. (c) The media should be banned from pronouncing
(a) Most restaurateurs feel that sticking to the cui- the winner before the polls close so as to prevent
sine of the 60’s evokes feelings of nostalgia and any sort of malpractice from taking place.
improves sales. (d) Different voting procedures in various states and
(b) Customers want to experiment and wish to tryout the massive media involvement ensures free and
cuisine from other countries. fair polls and thus are invaluable.
(c) Similar changes have been noticed in the enter- (A) abbb (B) abcd
tainment industry where viewers now give pref- (C) abdd (D) abbd
erence to comedies and action parked adventure 22. Memory may have been found to be affected by genes,
movies. but no gene has been found to have even a small
(d) Restaurants which improvise and innovate will causative relationship with normal ‘intelligence’.
definitely be successful. Intelligence has never been successfully defined or
(A) cbbd (B) bcda measured, unless one refers to highly specialized,
(C) cbca (D) bbcb arbitrary, narrowly defined and largely learned skills
20. Organization of the Petroleum Exporting Countries measured by standard IQ tests. Why should genetic
(Opec) has used output curbs to maintain average oil intelligence have evolved along the lines of IQ tests
prices over the past three years in its $ 22 to $ 28 tar- when better measures could have been evolved is some-
get range. Opec was worried that rising supplies from thing inexplicable.
rival non-Opec nations and another year of modest (a) Genes that influence ‘intelligence’ are masked by
demand growth could cause a downward price spiral. chemicals produced in the brain and are thus dif-
It is particularly vulnerable to a price fall during the ficult to detect.
second quarter when demand eases. Forecasts from the (b) IQ tests are the most accurate way of measuring
Paris based International Energy Agency backed this intelligence exhibited by humans and no other
outlook. It is estimated that if Opec keeps pumping measure is required.
unchecked it will overwhelm world demand next year (c) IQ tests are not the best measure of intelligence
by 1.8 million BPb causing a huge stock build-up. as these tests can be taken quite successfully by
(a) The existence of a similar cartel in the global coal people who have practiced hard enough.
market would have definitely pushed up the price (d) IQ levels in a human being are function of the
of coal. nutrition that one receives in childhood.
(b) A price range of $ 22 - $ 28 ensures that the pro- (A) ddbc (B) ddda
ducers make the minimum possible profit out of (C) dbaa (D) dbad
their operations required to keep their economy in 23. After Rwanda and Kosovo, another dark page of
shape. modern history is being written in East Timor by the
(c) Any excess production by OPEC would be mopped Indonesian army. It’s a shame the international com-
up in an energy hungry global market and thus munity did not react faster in condemning Indonesia
OPEC need not worry about excess production. and preventing the genocide, which has been on-going
(d) OPEC countries should concentrate on developing since the mid 70’s in East Timor. Above all political and
their non oil based economy so as to be able to economic considerations, the international community
absorb any adverse shocks as a result of decreas- should never forget that the brave people of East Timor
ing oil reverses. are struggling and dying for something that is so dear
(A) cbdc (B) bcca to all of us - freedom.
(C) abcd (D) bcda (a) The international community has not found any
21. The recent elections have thrown up so many critical evidence of wrong doing by the Indonesian army
weaknesses in the American electoral system - incon- and hence has been averse to condemning it.
sistent voting procedures from state to state, early dec- (b) The East Timorians are waging a battle not for
laration of the winner by the media before the polls freedom but because of the fact that most of them
closed and partisanship of breathtaking proportions are being forced to convert to Christianity.
that intensified after the balloting. Surely a set of statu- (c) Genocide in any place under any pretext is abomi-
tory rules about the counting and recounting of votes nable and it is the responsibility of the interna-
could have saved the US all this embarrassment. tional forum to put a stop to it.
(a) The adoption of a set of rules to be uniformly (d) History has proved that freedom is a concept for
applied all over the country would most certainly which people have sacrificed their lives and hold
eradicate the weakness in the electoral system. it in high esteem.
1.22 | Verbal Ability Test 4

(A) dcbb (B) dcaa (B) When the thinking part of the brain is otherwise
(C) dcab (D) abcc engaged, we are left with gut-level reaction and
Directions for questions 24 to 28: Each of the following instinctively condemn bad behaviour.
questions has a paragraph from which a sentence has been (C) Scientists have long wondered whether hypocrisy
deleted. From the given options, choose the sentence that is driven by emotion or reason.
completes the paragraph in the most appropriate way. (D) When we judge our own transgressions less
harshly than we judge the same transgressions in
24. This is a rare moment in the history of economic
others it may be because we have this instinct to
globalisation. Policy making across the developed
preserve our self-image.
and developing world has never been so uniform and
homogenised. Whether it is G-7 or G-20 economies the 27. Celebrity has become the primary commodity of pop-
only buzz word among policy makers is ‘inject more ular culture. Fans used to fall for a specific album or
liquidity”. President George Bush himself made the film, but now the public tends to base its consumption
unusal gesture of walking into the meeting of G-20 on the aura of celebrity attached to any given product.
countries, seeking their active support in mitigating the Singers can act in films and actors can record albums,
impact of the financial meltdown. _____________ not thanks to any special talent but because their brand
(A) At a macro level, the attempt by the G-7 govern- is big enough to transcend categories. _____________
ments is to ensure that the big asset bubble built (A) Witness the birth of the celebrity luxury fashion
over the past six years must not be allowed to brand.
burst. (B) Fashion magazines have all but abandoned the
(B) There are clear pointers that the global financial practice of putting models on the cover of their
crises will result in loss of demand across the magazines.
board. (C) Celebrities have wised up to their incredibly pow-
(C) The thirst for more money seems unending. erful market potential, moving from endorsing
(D) Never before have the heads of state met only someone else’s high end products to producing
to discuss how to put in extra cash in the global their own.
financial system. (D) The most successful start-ups have been those by
celebrities with iconic personal style.
25. At the heart of the wellness concept is the most roman-
tic notion of all: the suggestion that we might postpone, 28. The growth stories of China and India have always been
halt and even reverse the aging process itself. While different–China is well known for being the world’s
some scientists are doing research on ageing and on factory, while India’s new wealth has been built on ser-
human mortality, many wellness buffs-and even some vices. But the result is the same. Over the next twenty
reasonably accredited scientists-are promising that we years 213 million Chinese households and 123 million
may soon penetrate the secret of the ageing process and Indian ones will begin to have discretionary income.
with this knowledge, actually extend Homo Sapiens’ ______________
normal life span. __________________ (A) If both countries continue roughly on their current
(A) While some may search for a magic elixir, others growth paths we will witness the creation of mas-
stress on diet and exercise as the keys. sive new consumer markets.
(B) The death barrier may actually be broken. (B) The speed of the change will rival Japan’s eco-
(C) Wellness is a kind of superstate of mental and nomic miracle of the 1950s.
physical well being. (C) That will lead to an Asian shopping spree of his-
(D) Those who remain impervious to these romantic toric proportions.
hopes have other incentives or coercions. (D) The projection is that incomes will grow eight
fold cutting China’s poverty rate to just 16 percent.
26. To test the role of cognition in hypocrisy, scientists
had volunteers assign themselves an easy task and a Directions for questions 29 to 40: Select the correct alter-
stranger an onerous one. But before judging the fair- native from the given choices.
ness of their actions they had to memorize seven num- 29. Manufacturers of a food drink claim that their product
bers. This play keeps the brain’s thinking regions too is a complete planned food containing all the nutrients
tied up to think about anything else, and it worked: necessary for the health and growth of a child. However,
hypocrisy vanished. People judged their own behav- it is better that children are given a well-balanced diet
iour as harshly as they did others’-strong evidence that consisting of a variety of foods for health and growth.
moral hypocrisy requires a high-order cognitive pro- Which of the following, if true would best support the
cess. _____________ position above?
(A) When ‘people like us’ torture, it is justified; when (A) The flavour of the food drink does not appeal to
people unlike us do, it is an atrocity. many children.
Verbal Ability Test 4 | 1.23

(B) Children who are used to taking the food drink do (D) The service provided should take care of the ulti-
not take natural foods rich in these nutrients. mate beneficiary of the services.
(C) The nutrients contained in a well-balanced diet 33. A recent study indicates that Venadial, a new medi-
are more easily absorbed by the body for effec- cine currently produced in the country X only, actu-
tive use. ally reduces cholesterol levels. Venadial, derived
(D) The contention of the manufacturers that the food from the resin of pine trees, works by activating a
drinks are complete planned foods is yet to be metabolic response that is not yet well understood.
proved right. However, cholesterol levels fell fifteen to twenty
30. A recent study conducted with school children aged ten per cent among participants in the study who con-
to fifteen showed that a plan where these children were sumed Venadial daily and reduced the risk of heart
provided with a proper meal at school ensured that attack by one-third. Therefore company A in country
they were less likely to remain absent from the school Y should obtain the exclusive right to sell Venadial in
than other children. Thus providing a proper meal at Y. The profits are sure to increase within a short span
school plays a role in reducing student absenteeism. So of time.
such a plan must be introduced in schools to reduce Which of the following, if possible must first be done
absenteeism. in order to translate the above optimism into reality?
Under which of the following conditions will the break- (A) A publicity campaign informing the public of cho-
fast plan work best? lesterol and its permissible levels.
(A) In a locality where student absenteeism is (B) A publicity campaign advertising the role of
significant. Venadial in reducing heart attacks.
(B) In a school where there are more boys, (who are (C) Conducting research on the metabolic response to
more likely to remain absent), than girls. Venadial which is not yet well understood.
(C) In schools, where a majority of students have both (D) Convince the government to allow import without
parents working. restraint.
(D) In schools where the students are wards of those 34. Software companies have grown by leaps and bounds.
who cannot afford a square meal a day. The employees required to man these companies far
31. It is generally said that radiations emitted by cell outnumber the available candidates with academic
phones cause immediate damage to the membrane in skills relevant to the jobs that they are required to
the ear and results in hearing impairment. Hence one is perform.
advised to avoid using a cell phone to prevent total loss Which of the following plans, if possible, can allow such
of hearing. companies to meet their manpower requirements?
Which of the following, if true, seriously weakens the (A) The institutions can enhance the pay of the exist-
above argument? ing employees to encourage them to take up addi-
(A) Use of electronic gadgets such as computers also tional work.
impairs hearing besides affecting eyesight. (B) The institutions can scale down their operations to
(B) Electronic gadgets such as i pods do not damage the extent where the requirements can be met with
hearing in users. the number of available persons with the needed
(C) A majority of those who use hearing-aids are academic skills.
found to have never used cell phones in their lives. (C) The institutions can recruit even those who do not
(D) It is found in a survey that ninety per cent of have the necessary skills and give them training to
employees working as telephone operators have enable them to perform their jobs.
been regular users of cell phones for years. (D) The institutions can enter into agreements with
32. Last year, Alpha University implemented a procedure the academic institutions like colleges and univer-
that encouraged students to evaluate the teaching effec- sities to absorb their students in toto.
tiveness of their professors. 35. Smoking is a serious health concern as it affects not
Which of the following principles can the above plan only those who smoke, but also others who don’t, as
be taken to be based upon? they inhale the polluted air. The Government is con-
(A) The effectiveness of any service depends on how cerned about this. So it has decided to rope in film
useful it actually is to the receiver of the service. celebrities for its campaign against smoking, through
(B) The effectiveness of any service is measured by the documentary films. Which of the following, if true,
perception of the receiver of the service regarding most strengthens the plan of the Government in its
its utility. campaign against smoking?
(C) The receiver of a service should have the freedom (A) The general public abides by the advice of film
to choose the mode of delivery. celebrities on the screen.
1.24 | Verbal Ability Test 4

(B) The film celebrities are role models for the com- at substantially very low rates compared to the plastic
mon man. goods. Which of the following, if true, would under-
(C) Film celebrities earn huge sums of money and it mine the purpose for which paper - made goods are
is in the fitness of things that they contribute to a sold at very low rates by the administration?
social cause. (A) The shop keepers were found to sell the plas-
(D) It is easier to produce such films with film person- tic goods at a price higher than that fixed by the
alities as they are already well versed in acting. administration.
36. It is true that there are international laws against drug (B) The tourists were found to possess a higher value
traffic. But if the Government takes stern action to for aestheticism (which they find in plastic goods
plug its entry in one place, drugs will enter the country compared to the paper goods) than for costs.
through other sources. (C) The tourists themselves were well aware of the
Which of the following is most like the argument above desirability of using biodegradable goods as
in its logical structure? against non biodegradable ones.
(A) It is true that the officials of the police department (D) A similar strategy adopted in another hill sta-
are expected to be incorruptible. But if they are tion in the same country did not produce desired
paid poorly, they may resort to graft to sustain a results immediately.
decent living. 39. Company “Trendy” is engaged in the manufacture
(B) It is true that most of the terrorists are religious of products which face stiff competition from oth-
fundamentalists. But there are several persons who ers in the market. Constant change in the products,
are deeply religious but they are not terrorists. their design, packaging, etc is the key to success in
(C) It is true that there are severe laws against the industry. Trendy has been allocating a significant
violation of copy rights. But some company portion of the resources over the years to research
would have done it if the defendant had not done and development which takes care of devolvement
it first. of products and innovations in designs etc. However,
(D) It is true that adoption of third degree methods the allocation has fallen from a whopping ten percent
against prisoners to extract confessions is against to a mere 6 percent of the total resources, in the cur-
law. But it cannot be helped in certain cases where rent year, because of which it is feared that the sales
the prisoner does not reveal all the truth. might drop significantly this year. Which of the fol-
37. Balance of trade refers to the difference between the lowing is an assumption made in the conclusion drawn
exports and imports of a country. It is said to be posi- above?
tive when exports are more than imports and negative if (A) Ten per cent of the total resources is enough allo-
imports are more than exports. The exports of a country cation for research development activities, to for
‘C’ increased over the previous years’ exports. However achieve the desired levels of sales.
the balance of trade has become negative though it was (B) The allocation of resources to research and devel-
positive last year. opment efforts determines the extent of innovation
Which of the following cannot be true if the above in products and product designs.
statements are true? (C) Sales of the company will also fall from ten per
(A) The imports during this year are more than last cent to six per cent.
year’s. (D) The production of goods by the company would
(B) Exports during the previous year were more than also drop significantly in the current year.
imports. 40. Which of the following, if the dictum ‘more the
(C) Last year’s imports were more than this year’s resources for R&D, more will be the innovations’ be
exports. true, would most weaken the conclusion drawn in ques-
(D) The imports during this year are more than the tion 5 above?
exports. (A) The total resources allocated by the company to
38. The town administration of a hill station where tour- all the activities put together were double those of
ists far outnumber the local residents, found that the the previous year.
tourists were using non biodegradable plastic cups, (B) The sales of the same products by the other com-
plates and polyethylene bags in the place of substitutes panies also drop in the year.
made out of paper which are environmentally prefer- (C) The market share of the company for the products
able. In order to reduce the use of non biodegradable it manufactures increases during the current year.
goods but at the same time not to annoy the tourists the (D) The resources allocated to marketing during this
administration encouraged the sale of both plastic and year increased substantially over the previous
paper made goods, but the paper-made goods were sold year.
Verbal Ability Test 4 | 1.25

Answer Keys
1. A 2. C 3. D 4. B 5. C 6. B 7. D 8. C 9. C 10. B
11. D 12. D 13. D 14. C 15. A 16. C 17. B 18. A 19. C 20. A
21. B 22. A 23. C 24. D 25. B 26. B 27. C 28. C 29. D 30. D
31. D 32. B 33. B 34. C 35. A 36. C 37. C 38. B 39. B 40. A

Hints and Explanations


1. The main points are: Choice B is the right answer.
(1) An experiment is repeated to increase accuracy. The first sentence in choice C makes an absurd
(2) It is isolated to make the task of observation easier. suggestion.
(3) Variety in experiments gives a better chance of Choice D mixes up the concept of ecological foot-
discovering laws. print and carbon footprint. Choice (B)
Choice A covers all the important points and hence can
5. The main points in the text are:
be called its summary. Choice B does not say anything
(1) Hunting and logging are a threat to protected
about ‘variety’. Choice C is not as clear or comprehen-
areas.
sible as choice A. Choice D does not specify what helps
(2) A more subtle threat is illegal occupation of forest
in what aspect. Choice (A)
land by cultivators.
2. The main points are: (3) The reason for all the above is pressure of popu-
(1) Aristotle says any good definition of an object has lation on land, poverty, corruption and poor law
two parts. enforcement.
(2) The first is to assign the object to its class or genre Choice A is wrong because it calls poaching ‘a greater
with which it has common features. evil’, not stated in the passage.
(3) The second is to indicate where and how it is Choice B becomes too specific by mentioning Laos,
unique. west and central Africa and southeast Asia. Choice D is
(4) Juxtaposing it with objects of the same kind helps again specific as it talks of coffee growers. Choice C is
in identifying its individuality. appropriate. Choice (C)
Choice A does not mention ‘Aristotle’ and Choice B
focuses on the definition of ‘man’ in particular and then 6. The main points in the text are:
says ‘object’. Hence A and B can be ruled out. Choice (1) Chernobyl still evokes poignant memories after 20
C summarises the passage. The idea given in the last years and we are far from cleaning up the mess.
sentence of choice D is incorrect.  Choice (C) (2) It was caused by an unnecessary experiment.
(3) The Soviet government did not act immediately
3. The main points are:
and we do not know how many deaths were caused
(1) Sociologists use strange jargon or give extended
by the failure of the government to evacuate the
meaning to familiar terms.
people.
(2) They are criticized for doing this.
The original para does not mention that the Soviet
(3) But they have their reasons.
government refused to evacuate people. It was
(4) They do it in a desire to be more precise and sys-
reluctant and delayed the process. So, choice A
tematic in their analysis.
is a distortion. Choice C distorts in calling the
Choice A is incorrect as it says’ . . . They are justified
radiation ‘a gas’, and it leaves out many impor-
to do so . . .’ which is not stated in the passage. It is
tant points. Choice D distorts in saying ‘the Soviet
the critics ‘who are justified’. The second sentence in
government put up a brave front’ – not stated in
choice B makes it incorrect. ‘The charges/criticism . . .’
the text. Choice B is concise and correct.
is not mentioned in choice C. Choice D precisely sum-
 Choice (B)
marizes the passage. Choice (D)
4. The main points in the text are: 7. The main points in the text are:
(1) The damage we inflict on our surrounding is mea- (1) Older women, those over forty, are more likely to
sured by carbon footprint and ecological footprint. have a child with Down syndrome.
(2) Carbon footprint measures the greenhouse gases (2) As such they are offered, or forced to have,
that is emitted because of us. amniocentesis.
(3) Ecological footprint measures the land we need (3) The abortion of a Down embryo can be seen as a
for resources and for dumping waste. benign aspect of science or as officially encour-
Choice A misses carbon footprint. aged murder.
1.26 | Verbal Ability Test 4

(4) It is eugenics, fifty years after the Nazis. 14. B is the opening sentence (it makes a generalized state-
Choice A is not the best summary because it is ment). A follows - it explains what the work (mentioned
not amniocentesis but abortion that is controver- in B) is. C follows A – ‘This meeting’ in C linking it ‘to
sial. Choice B is not apt since it says older women call on….’ In A. D concludes with what he did.
take the test and abort, as if the initiative is theirs.  Choice (C)
Choice C makes the test controversial. Choice D
15. C is a better opening sentence than D, as it is a gen-
is apt.  Choice (D)
eralized statement. D follows C – it explains what
8. The main points in the text are : the ‘sordid’ going on mentioned in C are. A follows
(1) Jefferson warned against public debt. B – ‘sponsorship and endorsements’ in A linking it to
(2) But it is a tried-and-tested method. The USA ‘commercialization’ in B. Choice (A)
being an example.
16. B is the opening sentence. C follows B – ‘Areas
Choice A distorts the meaning when it says the
cleared’ in C linking it to ‘land..... obtained by cut-
USA considered it to be a prerequisite. Choice B
ting down trees’ in B. A follows C – ‘set up industries
distorts in saying that because the USA amassed
….. which pump out …..’. E and D follow – The speed
debt Jefferson warned against it. Choice C is an
of cutting down has increased and its consequences.
apt summary. Choice D appears to be right but
 Choice (C)
Jefferson never said (as per the passage) that pub-
lic debt would weaken economy. Choice (C) 17. D is the opening sentence – ‘privacy’ is the central idea
of the para. C follow D giving an example to elabo-
9. A and C are our possible starting parts. ‘One of the’ in
rate the need for privacy. B follows C – the author’s
C indicates that it is a continuation of ‘a them’ rather
child hood experience compared to his daughter’s. E
than a starting part and hence C is rejected as our start-
and A follow to complete the author’s reminiscence.
ing part which leaves A as the only possible starting
 Choice (B)
part. “The place’ in D relates to ‘the newest temple’ in
A and hence D follows A. ‘Vast space’ in B connects 18. A is the opening sentence – it sets out the basic idea
with ‘huge physical entity’ in D and hence B follows D. of the para. C follows A – explaining (very briefly)
 Choice (C) how his parents were the complements of each other.
B follows C – one more instance of complementing. E
10. As per the choices A, C and D are our possible starting
and D follow elaborating the qualities of his mother.
parts. ‘Besides’ in C rules it out as our starting part
 Choice (A)
and out of A and D, D is a better starting part as it is a
more generalized statement than A. ‘Usal garnish’ in A 19. The passage states that globalisation has made its
follows ‘garnishes’ in D. ‘Fat-laden coconut’ in C links impact on the food tastes of the masses, it goes on to
up with ‘freshly greated cocounut’ in A and hence C state that people prefer innovation in cooking.
follows A. B provides a contrast to A and hence follows A - states a point that runs counter to the argument
C. Choice (B) given in the passage regarding the changing
11. The ‘paradoz’ associated with ceramic sculptures is tastes of the masses hence it is a COUNTER
elaborated in C and D. C says that the ceramic object’s ARGUMENT.
solidity gives a sense of permanence but this is con- B - gives one more reason for the changing states and
trasted by D which states that the baked earth form hence it is a SUPPORTING ARGUMENT.
which this sculpture is made makes it more prone to C - states a point that is not mentioned in the pas-
damage on account of its fragile nature. “fragility in B sage and hence this is a STATEMENT OF
connets with ‘fragile, frangible’ in D and hence B fol- ARGUMENT.
lows D. Choice (D) D - categorically states that restaurants which cater
to the changed tastes will be definitely successful
12. A is our starting part ‘but’ in D offers a contrast by
and hence it is a CONCLUDING ASSERTION.
emphasizing the fact that though 200 years have
 Choice (C)
elapsed since Tipu’s death yet his fame is growing and
hence D follows A. B describes Tipu and hence follows 20. The passage talks about the efforts made by Opec to
D. ‘Reputation’ in C is a logical extension of ‘formi- maintain he level of prices at a certain level and also
dable and implacable’ in B. Choice (D) the scenario that emerges as a result of the efforts that it
13. A and C are our possible starting part of which C is makes to achieve this objective.
better as it is a very general statement. ‘It’ in B refers to A - talks about a hypothetical case of how the coal
the ‘ten centuries’ in C and hence B follows C. ‘In that prices would have benefited from the existence of
calculation’ in D refers to the ‘yugas’ in A and hence D such a cartel. Hence this is a STATEMENT OF
follows A. Choice (D) ARGUMENT.
Verbal Ability Test 4 | 1.27

B - gives a reason as to why the Opec nations would 24. The central idea of the paragraph is the notion that the
want to maintain the oil prices in a specific range meeting of G-20 nations to discuss how to inject more
and hence it is a SUPPORTING REASON. liquidity was a rare or unusual event. This idea reaches
C - This point runs against the view of the passage its natural conclusion in 4. Choice (D)
that excess production by Opec would result in a 25. The paragraph harps on the fact that wellness can
decrease of the general price of oil and hence it is reverse the ageing process. The penultimate sentence
a COUNTER ARGUMENT. refers to ‘extending life span’. This leads to the conclu-
D - talks about a possible remedy to the problem and as sion that death may be conquered which is presented in
such it is a new argument that has been proposed. option B. Choice D begins another idea and so cannot
Hence, it is a STATEMENT OF ARGUMENT. conclude this paragraph. Choice (B)
 Choice (A)
26. The focus of the paragraph is on the role of cognition
21. The passage talks about the weakness by the American
in hypocrisy and the revelation that when the mind is
electoral system and gives a few examples to substanti-
busy thinking about other things morality becomes a
ate this.
gut-reaction. This conclusion is brought out in option
A - states categorically that the adoption of a system of
(B) which summarizes the result of the experiment and
rules across the country would correct the system
is hence a good conclusion. Choice (B)
and stress the root hence, this is an ASSERTION.
B - gives a SUPPORTING REASON as to how other 27. The paragraph focuses on the idea that celebrity sta-
countries have benefited by implementing the rules. tus sells. Awareness of celebrities regarding their brand
C - the role played by the media has been specified in value has led to the stars producing their own products
the passage but the question of banning it has not instead of endorsing those of others. This is the logical
been discussed and as such C which proposes that conclusion found in option C. Option A can continue
the media be banned from announcing results is a the para rather than conclude it. Option D can begin a
STATEMENT OF ARGUMENT. new paragraph. Choice (3)
D - gives a reason in support of the measures being 28. The central idea of the paragraph is the growth track
proposed in the passage to remedy the situation of India and China and the fact that this will lead to
and as such it is a SUPPORTING REASON. greater disposable incomes in both countries. Sentence
 Choice (B) 3 which states the consequence of such incomes is the
22. The passage states the relation between memory and logical conclusion of the idea. Sentence 1 can be ruled
genes and says that intelligence has not been defined out because the ‘if’ in the sentences raises doubts about
clearly and talks about how ineffective IQ tests are in the growth pattern and thus contradicts what has been
measuring intelligence. stated earlier. Choice (3)
A - this contradicts the first line of the passage and
hence this is a COUNTER ARGUMENT. 29. Argument evaluation:
B - also contradicts what is stated in the passage and Situation: Manufacturers claim that their food drinks
hence it is a COUNTER ARGUMENT. contain all the nutrients necessary for a child. But a bal-
C - provides support to the view given in the passage anced diet containing variety of foods is better.
and hence it is a SUPPORTING REASON. Reasoning: A well balanced diet is preferable to a food
D - states a new point not mentioned hitherto in drink because it acts better for the child.
the passage and hence it is a STATEMENT OF (A) ‘Flavour’ is irrelevant to the argument.
ARGUMENT. Choice (A) (B) Children’s preference is not the issue.
(C) Also outside the scope of the argument.
23. The passage laments the apathy shown by the interna- (D) Correct. It properly identifies the statement which
tional community towards the genocide being perpe- supports the argument.
trated in East Timor by the Indonesian army in order to D is correct. Choice (D)
the quell the demand for freedom.
A - contradicts what is stated in the passage and hence 30. Evaluation of plan:
it is a COUNTER ARGUMENT. Situation: Providing a meal to school going children is
B - introduces a new angle to the problem stated and likely to reduce absenteeism.
hence it is a STATEMENT OF ARGUMENT. Reasoning: A proper meal provided to school going
C - firmly states that genocide is an aberration or civil children reduces absenteeism. This plan will best work
society and must be stopped and hence this is an only where the absenteeism has been due to non-avail-
assertion. ability of a good meal to the children.
D - substantiates what is given in the passage and (A) The absenteeism may be due to other reasons and
hence it is a supporting reason. Choice (C) hence the plan to provide as usual may not work.
1.28 | Verbal Ability Test 4

(B) The composition of the students whether they are (C) Research on the metabolic response to Venadial is
boys or girls is beyond the scope of the argument. irrelevant to the issue.
(C) ‘Working parents’ doesn’t provide ground for pro- (D) This is also irrelevant to the argument.
viding breakfast and hence reducing absenteeism. B is correct. Choice (B)
(D) Correct. It properly identifies the condition under 34. Evaluation of a plan:
which the breakfast plan will succeed.Choice (D) Situation: Software companies find the persons
31. Argument evaluation: required outnumbering the availability. The plan is to
Situation: Radiations from cell phones cause hearing meet the requirement.
defects. Hence cell phones must be avoided. Reasoning: The available candidates with necessary
Reasoning: The argument is that cell phones should academic skills is insufficient to meet the requirement.
not be used as they cause heaving defects. Hence the So the only way is to recruit even those without the
statement which tells us that cell phones do not affect necessary academic skills, but impart them training to
the ears weakens the argument. make them fit for the jobs.
(A) The effect of using ‘computers’ is beyond the (A) It is irrelevant to the discussion as ‘pay’ is the
scope of the argument. issue.
(B) Use of a `Ipod’ is not the question. (B) The question is of a plan to meet the enhanced
(C) It is not necessary that all those who are defective demand. So scaling down operations is not the
in hearing must use hearing aids. answer.
(D) Correct. It properly identifies the statement which (C) Correct. It properly identifies the correct plan to
weakens the argument. Since the telephone opera- meet the increased need.
tors’ job involves having, the fact that their hearing (D) As it is already stated that the number of candi-
is not impaired despite using cell phones regularly dates with necessary skills is insufficient, entering
weakens the argument. Choice (D) into agreement to absorb the students in to will not
3 2. Evaluation of a plan: help.
Situation: The students of Alpha University were C is correct. Choice (C)
asked to evaluate the teaching effectiveness of their 3 5. Evaluation of plan:
professors. Situation: The Government wants to involve film ce-
Reasoning: The university asks the students, the recip- lebrities in anti smoking campaigns as it is concerned
ients of the service, to evaluate the effectiveness of about the health hazards that smoking causes to the
teaching of the professors, the providers of services. So public.
the plan is based on the principle that the effectiveness Reasoning: The Government plans to involve film ce-
of a service rendered is measured by how the receiver lebrities in its anti smoking campaign. This plan will be
views it. successful if the general public heeds the film celebri-
(A) The plan doesn’t measure the actual effectiveness ties words on the screen.
of teaching which must have been done by evalu- (A) Correct. It properly identifies the situation when
ating the students not the teachers. the Government’s plans will be successful.
(B) Correct. The statement properly identifies the (B) The film celebrities may be smokers in real life
principle on which the plan is based. and hence if the public emulates their lives, the
(C) It is irrelevant to the argument. plan will not be successful.
(D) This may be the purpose of the plan but not the (C) ‘Why the film celebrities need to be involved in
principle on which it is based. social causes’ is beyond the scope of the argument.
B is correct. Choice (B) (D) It may be helpful in production of the film but it
3 3. Evaluation of a plan: does not strengthens the achievement of the pur-
Situation: Venadial is effective in reducing cholesterol pose of the plan. Choice (A)
levels and the risk of heart attack according to a study. 3 6. Argument constructions:
Company A in country Y should sell the medicine, as Situation: Though laws exist against drug trafficking,
its sole selling agent, to earn profit in a short span of it is bound to exist. Even if the Government plugs one
time. source, as it will enter through others.
Reasoning: To realise the optimism expressed above Reasoning: The situation is one where there is a law
people must be made aware of the effectiveness of Ve- against something. But even if it is curbed at one end, it
nadial in reducing heart attacks. is bound to be committed by someone else.
(A) Knowing the permissible cholesterol level will not (A) It is not similar. It provides a reason for why the
induce the public to go for the medicine. police may be corrupt despite laws.
(B) Correct. It properly identifies the course to be (B) It only says that all religious persons need not be
adopted. terrorists.
Verbal Ability Test 4 | 1.29

(C) Correct. It properly identifies the situation which 39. Argument construction:
is a similar argument to the one given. Situation: Trendy’ is a company engaged in the man-
(D) It gives an explanation as to why third degree ufacture of goods which need constant innovation in
methods become inevitable. Choice (C) products and their designs. It is feared that a drop in
the percentage of allocation of resources of R&D will
37. Argument construction: reduce sales.
Situation: Last year exports were more than imports. Reasoning: Unless the proportion of resources allocat-
This year imports were more than exports. Also exports ed to R&D is going to determine the extent of innova-
during this year were more than last year’s exports. tion, the conclusion cannot be drawn as above.
Reasoning: Last year E, > I, (E – Exports and I – Im- (A) It cannot be an assumption as there is no mention
ports). This year I2 > E2. Also E2 > E1. So I2 > E2 > from which it can be assumed that a particular per-
E1 > I1. Any statement contradicting the cannot be an centage of resources has to be allocated for research
inference. and development to achieve the desired results.
(A) It is correct because I2 > I1 so it is not the answer. (B) Correct. It correctly identifies the assumption that
(B) It is correct as E1 > I1, so it is not the correct unless the proportion of resources allocated to the
choice. research and development to the total resources
(C) Correct. It properly identifies a situation which determines the extent of innovation, it cannot be
cannot be inferred. This statement implies I1 > E2. held that sales will be affected.
This is not possible as E2 >I1. (C) It is not an assumption.
(D) It can be inferred as I2 > E2. (D) It is outside the scope of the argument.
C is correct. Choice (C) B is correct. Choice (B)
38. Evaluation of a plan: 40. Argument evaluation:
Situation: The Administration wants to encourage the Situation:  As above
use of biodegradable paper products by encouraging Reasoning:  Allocation of resources are held to direct-
their sale at very low prices. ly govern the sales. Then the argument will be weak-
Reasoning: The strategy will fail to work if the users ened if there had been no real decrease in allocation of
do not take the bait, i.e. they don’t give importance resources to R&D.
to price differentials. (A) Correct. If the total resources are doubled, then
six percent of it will be more than ten percent of
(A) It should in effect actually help the administration
the pervious years total resources. Hence there has
in realising it s purpose.
been a real increase in allocation to R&D which
(B) Correct. It properly identifies a situation where the
can if anything only increase the sales.
tourists may not give any importance to the price
(B) It is outside the scope of the argument.
differentials.
(C) It may be true but it does not mean that the sales
(C) It should actually strengthen not undermine the have improved, as the sales of other companies
administrations purpose. also might have dropped.
(D) It is said that the results were not produced only in (D) Resources allocated to marketing is not the crux
the short run. the argument.
B is correct. Choice (B) A is correct. Choice (A)
Verbal Ability Test 5
Number of Questions: 20 Time: 30 min

Directions for question 1: The question has a sentence with 9. When the dilapidated structure collapsed, the _____ of
three blanks followed by four pairs of words as choices. the people could be heard from a distance.
From among the choices, select the pair of words that can (A) gossip (B) wails
best complete the given sentence (C) eviction (D) bombardment
1. It is now recognized in many countries that family 10. He does not _____ the ugly aspects of human nature in
and social environment of the children have a ______ his picture of life.
impact on their performance than the quality of teach- (A) eliminate (B) include
ing at school and children from poor families score sig- (C) exclude (D) extricate
nificantly ______ in vocabulary, communication skills, 11. The airline has increased its frequency _____ nine
simple arithmetic and the ability to concentrate com- flights a week to eleven flights a week.
pared to children from ______ income households. (A) between (B) of
(A) meagre...higher...higher (C) from (D) for
(B) lower......lower....greater
Directions for question 12 to 17: The question has a sen-
(C) prime.....faster.....lower
tence with two blanks followed by four pairs of words as
(D) greater....lower.....higher
choices. From among the choices, select the pair of words
Directions for questions 2 to 11: Fill in the blanks in the that can best complete the given sentence.
given sentences so as to make sense. Select the correct 12. I am not a _____, I am a _____.
word from the answer choices and mark its number as the (A) ‘have been’ . . . ‘’will be’
answer. (B) ‘has been’ . . . ‘will be’
2. If the items in a diary are reliably listed and the list (C) ‘was been’ . . . ‘will being’
is ________ reviewed then certainly nothing should be (D) ‘had been’ . . . ‘will be’
forgotten. 13. During his childhood, Williams ______ his parents
(A) perversely (B) phlegmatically ______ teachers.
(C) languidly (D) conscientiously (A) never obeyed . . . . or
3. The child had a _____ birth defect in the form of a cleft palate. (B) never had obeyed . . . . or
(A) hereditary (B) compulsive (C) never obeyed . . . .nor
(C) congenial (D) congenital (D) never had obeyed . . . . nor
4. The people of South Asia had been left behind in eco- 14. The two men were very ______ and rather looked the
nomic development _____ of tensions and conflicts. same but had ______ views on important aspects of
(A) in view of (B) because life.
(C) want (D) full (A) close . . . wide (B) similar . . . diverse
5. Financial difficulties _______ him to discontinue his (C) different . . . complex (D) familiar . . . similar
studies, and take up a job. 15. I am not a ____, I am a ____.
(A) enchanted (B) compelled (A) ‘have been’ . . . ‘will be’
(C) entreated (D) dictated (B) ‘had been’ . . . ‘will be’
6. Sixty houses were totally gutted ______the fire (C) ‘was been’ . . . ‘will being‘
accident. (D) ‘has been’ . . . ‘will be’
(A) at (B) beyond 16. The two thieves _______ arrested on the charge of
(C) in (D) off stealing from _______ house
7. The Kaveri community in Bihar, which has for long (A) was . . . an (B) am . . . a
_______ with poverty and backwardness, is yet to join the (C) were. . . an (D) were. . . a
mainstream because of its nomadic lifestyle. 17. The country has no need to test its nuclear arsenal in
(A) suffered (B) exhausted the absence of an external _____ which is precisely
(C) struggled (D) figured why we have a voluntary _____ on testing.
8. Alzheimer’s disease is not usually _______ until sig- (A) provocation . . . regression
nificant memory loss is evident. (B) collaboration . . . relapse
(A) detected (B) observed (C) wherewithal . . . setback
(C) revealed (D) disclosed (D) aggression . . . moratorium
Verbal Ability Test 5  |  1.31

Directions for question 18 to 20: Read the following para- have responded by taking control of all prominent
graph and answer the question that follows: advertising sites in the city, including those at train sta-
18. People are throwing themselves off the Golden Gate tions and airports, and their use will be limited to offi-
Bridge at the rate of two a month, which makes it the cial sponsors only. Nevertheless, preventing ambushes
most popular place in the world for suicide. Many San is difficult.
Franciscans think that the solution is to put up a sim- Which of the following is closest in meaning to
ple barrier. The Psychiatric Foundation of California, ‘ambush’?
which has proposed to construct a barrier to act as a (A) Deception (B) Integrity
deterrent, has heard several arguments against a bar- (C) Virtue (D) Equity
rier; the most persistent has been that people would 20. Studies of promising young prodigies frequently reveal
simply kill themselves somewhere else, so why bother? that most turned out to be failures, in both their pro-
Here the word ‘deterrent’ is closest in meaning to? fessional and personal lives, in later years. Researchers
(A) Impediment (B) Assistance also found that most of the prodigies were diagnosed
(C) Indemnity (D) Restitution with symptoms of paranoia and schizophrenia at a later
19. In “ambush marketing” campaigns, companies try to stage. The researchers concluded that mental illnesses
promote their brands at sporting events without pay- inhibited the prodigies from realizing their full poten-
ing sponsorship fees. Ambush marketers have replaced tial in later life.
hooligans as the villains of sporting events, because Here, the word ‘prodigies’ is closest in meaning to
they undermine official sponsors, who are the main (A) genius (B) Maestros
source of revenue in some sports. The IPL organizers (C) Adept (D) Apprentice

Answer Keys
1. D 2. D 3. D 4. B 5. B 6. C 7. C 8. A 9. B 10. B
11. C 12. B 13. A 14. B 15. D 16. D 17. D 18. A 19. A 20. A

Hints and Explanations

Explanatory notes for question 1: 5. Compel means to force. Financial difficulties forced
1. The sentence compares the social environment and the him to drop his studies and take up a job. Choice (B)
quality of teaching on the performance of children. 6. ‘In’ is the correct preposition to be used as it expresses
Hence the comparative degree of an adjective has to be a period of time during which an event happens.
used in the first blank. ‘Lower’, and ‘greater’ are com-  Choice (C)
parative degree adjectives and ore apt. Hence choices 7. The sentence is about a community in Bihar which
A and C can be ignored. According to the passage, has not progressed. The reason for this is its poverty
children from low income social background are likely and backwardness. Exhausted means extremely tired.
to score ‘lower’ than their affluent counterparts with This is a wrong choice. ‘Figured’ means to understand.
higher income background. All these words are found ‘Sustained’ means continued for a long time. The word
in choice D making it the right answer.  Choice (D) ‘suffered’ means to experience physical or mental pain.
Explanatory notes for questions 2 to 11: So these words are not suitable when used in this par-
2. The context demands ‘proper listing’ and ‘meticulous ticular context. ‘Struggled’ means to use a lot of effort
review’ of the list. Among all the options only option to defeat someone, prevent something, or achieve
D conveys this meaning. Options B and C refer to something. The community has been struggling for a
‘laziness’ and ‘perversely’ refers to ‘contrary to what long time to join the mainstream. Choice (C)
is accepted’. The correct option is D ‘conscientiously’ 8. Detected, which means to discover or identify the pres-
which refers to ‘being careful and thorough in one’s ence or existence of something, fits the blank appropri-
work’. Choice (D) ately. Choice (A)
3. Birth defects are congenital (present from birth) and 9. Wails are prolonged high-pitched cries of pain or grief
not hereditary (genetic), compulsive (uncontrollable), thus the word is apt here. Choice (B)
or congenial (affable; friendly). Choice (D) 10. The preposition with choices (A), (B), and (D) is ‘from’
4. The reason the people of South Asia have been left (exclude “from”). Only ‘include’ will collocate with the
behind in tensions and conflicts. Choice (B) preposition ‘in’. Choice (B)
1.32 | Verbal Ability Test 5

11. The frequency is increased from something to some- 15. A ‘has been’ is a person or thing considered to be no
thing. Hence ‘from’ is the appropriate preposition in longer of any importance. The speaker say that he/she
the blank. Choice (C) does have it him/her to remain relevant. Choice (D)
16. Since ‘thieves’ is plural, ‘very’ should be ‘were’. Since
Explanatory notes for questions 12 to 17:
the word ‘house’ begins with a consonant, the article
12. The sentence implies that the focus should be on what before it should be ‘a’ Choice (D)
the speaker ‘will be’ in the future and not what he was
17. The word ‘voluntary’ rules out ‘regression’ ‘relapse’
in the past. I has - been is a person considered to be out-
and ‘setback’. Only ‘moratorium’ makes sense.
moded, past his/her prime or no longer of any impor-
 Choice (D)
tance. Choice (B)
Explanatory notes for questions 18 to 20:
13. The correct option is 1. “Never……or” is more apt then
“Never……nor”. Similarly, there are no two actions 18. The word ‘deterrent’ means an obstacle. This is closest
taking place here. Therefore, the sentence should be in in meaning to ‘impediment’. The rest of the options are
simple past. antonyms or inapt in the given concept. Choice (A)
 Choice (A) 19. The word ‘ambush’ means to be in a concealed posi-
14. The words apt in the context are ‘similar’ and diverse; tion, waiting to make a move. Choice (A)
note the use of the conjunction ‘but’ to link these oppo- 20. The meaning of the word ‘prodigy’ is a genius.
site words. Choice (B)  Choice (A)
PART B
NUMERICAL ABILITY

UNIT I
QUANTITATIVE APTITUDE

Quantitative Ability Test 1��������������������������������������������������������������������������������������������������������� 1.35


Quantitative Ability Test 2��������������������������������������������������������������������������������������������������������� 1.40
Quantitative Ability Test 3��������������������������������������������������������������������������������������������������������� 1.48
(Indices, Surds and Logarithms)������������������������������������������������������������������������������������������ 1.48
Quantitative Ability Test 4��������������������������������������������������������������������������������������������������������� 1.55
(Data Interpretation)������������������������������������������������������������������������������������������������������������� 1.55
Quantitative Ability Test 5��������������������������������������������������������������������������������������������������������� 1.62
(Permutations and Combinations)���������������������������������������������������������������������������������������� 1.62
Quantitative Aptitude Test 6������������������������������������������������������������������������������������������������������ 1.68
(ERPV, Numbers)����������������������������������������������������������������������������������������������������������������� 1.68
Quantitative Aptitude Test 7������������������������������������������������������������������������������������������������������ 1.74
(Quadratic equations and Inequalities)��������������������������������������������������������������������������������� 1.74
This page is intentionally left blank
Quantitative Ability Test 1
Number of Questions: 35 Section Marks: 30

Directions for questions 1 to 35: Select the correct alterna‑ commission is increased to 25%, then what will be the
tive from the given choices. profit percentage on the cost of manufacturing for the
1. What is the percentage change in the volume of a cyl‑ company?
inder if its height increases by 20% and radius remains (A) 10% (B) 25%
the same? (C) 30% (D) 20%
(A) No change 10. Chetan started a cable TV service with an investment
(B) 10% increase of `2,00,000. After a few months David joined him
(C) 20% decrease with an investment of `1,50,000. If at the end of the
(D) 20% increase year David’s share was `3,00,000 out of a total profit
2. 30% of a number when subtracted from twice the num‑ of `7,80,000 after how many months did David join
ber equals to 33 less than five times the same number. Chetan?
What is the number? (A) 2 (B) 4
(A) 15 (B) 10 (C) 8 (D) 10
(C) 20 (D) 5 11. A man sells an article at a profit of 25%. Had he bought
3. In the second and the third match of a series Dravid’s it at 25% less and sold for `25 less, he would still have
score increases by 10% and 217/33% respectively over gained 25%. Find the cost of the article.
the previous match. By what percentage did his score (A) `80 (B) `40
increase in the third match as compared to the first (C) `60 (D) `50
match? 12. A man sold a Doberman and a German Shepherd for
(A) 331/3% (B) 317/33% `5,750 each. On the Doberman he made a profit of
(C) 35 /33%
15
(D) 379/33% 25%, and on the German Shepherd he made a profit
4. Two tests are given with maximum marks of 300 and of 15%. Find the approximate profit percentage on the
200 respectively. A student scores an overall percent‑ whole transaction.
age of 60%. If he has scored 40% in the second test, (A) 23% (B) 16%
then how many marks did he score in the first test? (C) 18% (D) 20%
(A) 200 (B) 180 13. A, B, and C invested capitals in the ratio 4 : 5 : 6. At
(C) 220 (D) 240 the end of the year, they received the profits in the ratio
5. If a kerosene dealer sells kerosene at `8 per litre, he 6 : 5 : 4. Find the ratio of time periods for which they
loses `400, but if he sells at `10 per litre he makes a invested their capitals.
profit of `400. Then, how many litres of kerosene did (A) 9 : 6 : 2 (B) 9 : 8 : 6
the dealer sell? (C) 9 : 3 : 4 (D) 9 : 6 : 4
(A) 250 (B) 200 14. A man purchases a certain number of chocolates at 2
(C) 350 (D) 400 per rupee and the same number of pepper‑mints at 5
6. By selling 50 metres of cloth, a merchant gains the cost per rupee. He mixes them together and sells them at
of 10 metres. Find the gain percentage. 3 per rupee. What is his approximate profit or loss
(A) 25% (B) 331/3% percentage?
(C) 20% (D) 30% (A) 5% profit (B) 5% loss
(C) 4% profit (D) 4% loss
7. By selling 60 metres of cloth, a merchant gains the sell‑
ing price of 10 metres. The gain percentage is 15. A shopkeeper bought a table marked at `600 at succes‑
(A) 25% (B) 20% sive discounts of 10% and 20% respectively. He spent
(C) 15% (D) 162/3% `8 on transportation and sold the table for `450. Find
his profit percentage.
8. If 3/5th of B’s income is 25% less than A’s income, then
(A) 27/11% (B) 23/11%
what percentage of B’s income is A’s income?
(C) 2 /11%
5
(D) 28/11%
(A) 60% (B) 80%
(C) 120% (D) 125% 16. A sum of money is invested at a certain rate of sim‑
ple interest. Find the annual rate of interest if the sum
9. If a dealer gets a commission of 10% on the list price
becomes 84% more in 6 years.
from the company, then the profit made by the com‑
(A) 12% (B) 14%
pany is 50% of its manufacturing cost. If the dealer’s
(C) 16% (D) 18%
1.36 | Quantitative Ability Test 1

17. A sum of money becomes 191/125 times itself, when 25. The weighted average of the scores of all the students
invested at compound interest at 20% p.a. Find the of three sections X, Y and Z of a class is 331/3% more
duration of the investment. than the average of the section X. The weighted average
(A) 1 year (B) 2 years of sections Y and Z is 655/11. If section X has a strength
(C) 3 years (D) 4 years of 40 and an average score of 45, what is the combined
18. Two sums of money in the ratio 3:4 are lent for a period strength of the section Y and Z?
of 5 years. The rates of interest on the two sums are in (A) 60 (B) 75
the ratio 1:2. The difference in the simple interest on the (C) 110 (D) 85
two sums is `1000. Find the total simple interest on the 26. Find the expression for the sum of n terms of an arith‑
two sums. metic progression, if the tenth term is 40 and the 12th
(A) `2000 (B) `2200 term is 44.
(C) `2400 (D) `2500 (A) 10n + 25n2 (B) 20n + 20n2
19. Two litres of 20% H2SO4 solution, three litres of water (C) 25n + 15n (D)
2
n2 + 21n
and five litres of 10% H2SO4 are mixed together. How 27. The first term of an arithmetic progression consisting
many litres of the resulting solution must be mixed of 30 terms is 10 and the common difference is 5. Find
with thirty litres of 18% H2SO4 solution so as to get a the ratio of the sum of the 30 terms of the arithmetic
15% H2SO4 solution? progression. to the sum of the last 20 terms of the A.P.
(A) 30 (B) 10 (A) 99 : 13 (B) 96 : 17
(C) 15 (D) 20 (C) 99 : 86 (D) 99 : 68
20. In which of the following ratios by weight should three 28. If the sum of the fifth, thirteenth and eighteenth terms
alloys of gold of purity 18, 20 and 22 carats be mixed of an A.P is zero, find the 12th term of the arithmetic
to form a fourth alloy whose purity is 201/3 carats? progression.
(A) 3 : 4 : 5 (B) 4 : 3 : 5 (A) -2 (B) -1
(C) 4 : 5 : 6 (D) 6 : 5 : 4 (C) 0 (D) 1
21. A family consists of a grandfather, a grandmother, 29. Find the sum of the first 10 terms of the series: 3 (22) +
father, mother and three children. The average age of 4 (32) + 5 (42) + ……..
father, mother and the 3 children is 25 years. The aver‑ (A) 3009 (B) 4860
age age of the three children is 15 years. The average (C) 3408 (D) 3608
age of parents and grandparents is 50 years, then find 30. The sum of the first eight terms of a geometric progres‑
the average age of the grand parents (in years). sion. is 510 and the sum of the first four terms of the
(A) 70 (B) 68 geometric progression. is 30. Find the first term of the
(C) 58 (D) 60 geometric progression, given that it is positive.
22. Groundnuts contain 70% oil by weight. Oil is partially (A) 2 (B) 4
extracted and what is left behind is groundnut cake (C) 6 (D) 8
which contains 17% oil by weight. What is the weight 31. The terms of an arithmetic progression are all posi‑
of the groundnuts which yield 1 kilogram of cake? tive. The square of fourth term equals the sum of the
(A) 3.77 kg (B) 2.67 kg squares of the previous two terms. The sum of the first
(C) 3.58 kg (D) 2.77 kg four terms is 14. Find the common difference.
23. One-third of a bottle full of pure listerene is replaced (A) 1
with water. Find the ratio of water and listerene if the (B) 2
above process is carried out for a total of four times. (C) 2
(A) 16 : 65 (B) 16 : 89 (D) Cannot be determined
(C) 89 : 16 (D) 65 : 16 32. The first, second and third terms of a geometric pro‑
24. The number of matches played by a cricketer in the gression are equal to the first, seventh and twelfth terms
year 2003 is one-fourth of the total number of matches of an arithmetic progression. If the first term and com‑
played by him upto the end of 2002. His average score mon difference have opposite signs, find the 37th term
(i.e., runs scored per match played) upto the end of the of the arithmetic progression.
year 2003 is four-thirds of his average upto the end of (A) 1 (B) 0
the year 2002. What is the ratio of the runs scored in the (C) 1 (D) 2
year 2003 to that of the total scored upto the end of the 33. Find the sum of the terms of the series
year 2002? (1) × (20), (2) × (19), (3) × (18), …(20) × (1).
(A) 2 : 3 (B) 3 : 2 (A) 1750 (B) 1645
(C) 3 : 4 (D) 15 : 16 (C) 1540 (D) 1435
Quantitative Ability Test 1 | 1.37

34. In a geometric progression, each term is the sum of all 35. The sum of five numbers in geometric progression is
the terms following it. The sum to infinity of the terms 31
62. The sum of their reciprocals is . Find the square
is 32. If all the terms are positive, then find the first 32
of the third number.
term.
(A) 100 (B) 121
(A) 16 (B) 16 2
(C) 36 (D) 64
(C) 64 (D) 8 2

Answers Key
1. D 2. B 3. A 4. C 5. D 6. C 7. B 8. B 9. B 10. A
11. A 12. D 13. D 14. B 15. B 16. B 17. C 18. B 19. C 20. A
21. D 22. D 23. D 24. A 25. C 26. D 27. C 28. C 29. B 30. A
31. A 32. B 33. C 34. A 35. D

Hints and Explanations

1. Volume of a cylinder = p r² h \ % Profit = (60 – 50) × 100/50 = 20%. Choice (C)


As p is constant and there is no change in radius, vol‑ 7. Let the cost price of 1m = CP
ume varies only with height. New height = (1.2) h Let the sale price of 1m = SP
∴ New volume = 1.2 p r² h i.e., the volume increases Given 60SP = 60CP + 10SP i.e., 60CP = 50SP
by 20%. Choice (D) \ %Profit = (60 – 50) × 100/50 = 20%. Choice (B)
2. Let the number be = x 8. Let A’s and B’s increase be a and b
According to the problem, 2x – 0.3x = 5x – 33
3b 25a  3
⇒ 3.3x = 33 ⇒x = 10 Choice (B) = a− = a 
5 100  4
3. Let Dravid’s score in the first match be x.
110 11x 4 80
It increases by 10% in the second to x= . \ a = b = b
100 10 5 100
In the third match it increases by a in 80% of b. Choice (B)
7 700 700  4000
% to 100 + 9. Let the list price be = LP
21 % =  % = %
33 33  33  33 If 10% of LP is given as commission to the dealer the
i.e., score in the third match is profit for the company is 50% ⇒ 0.9 LP = 1.5 CP
 1   4000   11x  4 x If 25% commission is given then
    = 0.75 LP = 0.75 × (1.5 CP / 0.9) = 1.25 CP
100   33   10  3
4x x % of profit = 25% Choice (B)
Over the two year it increases by − x = .
3 3 10. Chetan’s total investment = 2,00,000 X 12
x David’s total investment = 1,50,000 X x
1 1
Fractional increase = 3 = = 33 %�  Choice (A) (where x is the period of investment of David)
x 3 3 Chetan’s share in total profit
4. The two tests together had 300 + 200 = 500 marks = 7,80,000 – 3,00,000 = Rs.4,80,000
The student scored an overall percentage of 60% 2, 00, 000 × 12 4,80, 000
Now, = ⇒ x = 10.
\ Total marks scored by the student = 500 × 0.6 = 1,50, 000 × x 3, 00, 000
300
∴ David joined after 2 months. Choice (A)
Marks scored by the student in the second test
= 200 × 0.4 = 80 11. SP = 1.25 CP
Hence marks scored in the first test = 300 – 80 = 220. Also given SP - 25 = 1.25 × 0.75 CP
 Choice (C) Solving the above equations CP = 80 Choice (A)
5. Let the quantity of kerosene sold by the dealer be x 12. Total selling price = 5750 × 2 = `11,500
litres Total cost price =
\ CP – 8x = 400 and 10x – CP = 400 5750 5750
⇒ 8x + 400 = 10x – 400 ⇒ 5x = 400 Choice (D) + = 4600 + 5000 = ` 9600
1�25 1�15
6. Let the cost price of 1m = CP 1900
\Overall profit percentage = × 100 ≅ 20 
Let the sale price of 1m = SP 9600
Given 50SP = 50CP + 10 CP i.e., 60CP = 50SP  Choice (D)
1.38 | Quantitative Ability Test 1

13. Ratio of time is 6/4 : 5/5 : 4/6 20. Let the weight of 18, 20 and 22 carat gold be x, y and z
6 × 60 5 × 60 4 × 60 respectively.
: : = 9 : 6 : 4 Choice (D) 18 x + 20 y + 22 z 61
4 5 6 =
x+ y+ z 3
14. Let the number of éclairs be x
Only option (1) satisfies this condition. Choice (A)
There CP = x/2
CP of peppermint = x/5 21. Average age of children and parents 5 members = 25
x x 7x ∴ Sum of ages = 125 years.
Therefore total CP = + =
2 5 10 Average age of 3 children = 15
2x
Total SP = Sum of ages of 3 children = 45
3 ∴ Sum of ages of parents = 80
2x 7x Sum of the ages of parents and grand parents = 50(4)

= 200.
P% = 3 10 × 100. Therefore loss = 5%Choice (B)
7x Sum of the ages of grand parents = 120
10 Average = 120/2 = 60 Choice (D)
15. CP = 600 (.9) (.8) = 432 22. Since groundnut contains 70% of oil, it means in 1kg
Total CP = 432 + 8 = 440 of groundnut there is 30% of (or 0.3kg) solid material.
SP = 450 From this 1kg of groundnut, 0.3kg is solid which trans‑
10 lates to 83% of cake.
P% = x 100 = 23/11% Choice (B)
440 0�3
∴ The cake in 1kg of groundnut is kg.
16. If a sum of money invested under simple interest at 0�83
R% p.a. becomes N times after t years, R is given by Groundnut cake Groundnut
100 ( N − 1) 0�3
. kg – 1 kg
t 100 (1⋅ 84 − 1) 0�83
As N = 1 × 84 and t = 6 years, R = = 14
6 Choice (B) 1kg – ?
∴ the rate is 14% p.a.
1(1kg)
17. Let the sum of money be ` p and duration of the invest‑ ⇒ = 2.77kg Choice (D)
 0�3 
ment be n years.  
n 0�83 
91 216  20 
1 p= p = p 1 +
125 125  100  23. Let the capacity of the bottle be 1 litre.
3 n
The fraction of listerine left after repeating the process
 6  6 4
 3 − 1  2   16 
4
  =   ⇒n = 3. Choice (C) 4 times =  = =
5 5  3   3   81
18. Let the two sums of money be 3x and 4x and the rates ⇒ For every 81 lt of mixture, listerine is 16 lt and
of interest on the two sums be R% p.a. and 2R% p.a. water is 81 – 16 = 65 lt.
∴ The ratio of water and listerine is 65 : 16.
respectively. Difference in the simple interest on the
 Choice (D)
(4 x ) (2 R) (5) (3 x ) ( R) (5) 24. The number of matches and average score are tabulated
two sums = − = 1000
100 100 below.
xR Upto end In 2003 Upto end
⇒ = 1000 of 2002 of 2003
4
⇒ xR = 4000 Average 3x –– 4x
Total simple interest on the two sums No. of 4n n 5n
(4 x ) (2 R) (5) (3 x ) ( R) (5) 55 xR matches
= + = = ` 2200 .
100 100 100
(∵xR = 4000) Choice (B)
Score upto end of 2002 (3x) (4n) = 12 xn.
19. Concentration of H2SO4 in the resulting solution Score upto end of 2003 (4x) (5n) = 20 xn.
2(0�2) + 3(0) + 5(0�1) Score in 2003 = 8 xn.
= = 9% Required ratio = 8 xn : 12 xn = 2 : 3 Choice (A)
2+3+5
Let the amount of the resulting solution that is being 2 5. Let A (x + y + z) be the average score of the three section
mixed with 30 lt of 0.18 H2SO4 be x. combined. It is given that A (x + y +z)
(0.09)x + 0.18(30) = 0.15 ( x + 30) = ( 1 + 33 1/3%) of Average of x.
0.9 = 0.06x ⇒ x = 15 Choice (C) = (4/3)45 = 60….. (1).
Quantitative Ability Test 1 | 1.39

Let n be the combined strength of sections Y and Z. 510


Average of the two sections Y and Z together = 655/11 = r4 + 1 = = 17.
30
Total strength of X + Total strength of (Y and Z)
= Total score of (X + Y + Z) r4 = 17 - 1= 16
5 r = ± 4 16 = ± 2
⇒ (45) (40) + n(65 ) = (n + 40) (60)
11 510 ( r − 1)
First term of the G. P, a =
720 n 60 n r8 − 1
⇒ 1800 + = 60 n + 2400 ⇒ = 600
11 11 As first term is positive, r = 2 is taken.
⇒ n = 110. 510 (2 − 1) 510 (1)
a= = = 2. Choice (A)
∴ The combined strength of section Y and Z = 110. 28 − 1 255
 Choice (C) 31. Let the first term and the common difference be a and d
26. If the first term of the A.P is a and the common differ‑ respectively.
ence is d, we have the tenth term as (a + 3d)2 = (a + 2d)2 + (a + d)2
a + 9d = 40 ….. (1) a2 + 6ad + 9d2 = 2a2 + 6ad + 5d2 ⇒ a = ± 2d
and the twelfth term as a + 11d = 44 …. (2) As all the terms are positive, a = 2d
Subtracting (1) from (2), we have 2d = 44 - 40, a + a + d + a + 2d + a + 3d = 14 ⇒ d = 1.
d=2 Choice(A)
Substituting the value of d in (1) we get, a = 22. 32. Let the first term of either progression be a.
The sum of n terms of the A.P. Let the common difference of the arithmetic progres‑
n n sion be d.d (a + 36d) = 0
= [2a + (n - 1) d] = [2(22) + (n - 1) 2]
2 2 as a and d have opposite signs, d ≠ 0.
n n a + 36d = 0
= [44 + 2n - 2] = [42 + 2n] = 21n + n2 Choice (D) 37th term = 0 Choice (B)
2 2 30
27. Sum of the first 30 terms of the A.P = [2 (10) + 29 33. The terms of the series are in the form x (21 – x)
2 20
(5)] Required sum = ∑ x (21 - x)
= 15 [20 + 145] = 15 [165] = 2475. x =1
10 (21)(20)(21) 1
Sum of the first 10 terms of the A.P = [2 (10) + 9(5)] = − (20)(21)(41) = 1540 Choice (C)
2 2 6
= 5[20 + 45] = 5 [65] = 325. Ratio of the sum of the first
30 terms of the A.P. to the sum of the last 20 terms of 34. Let the first term and the common ratio be a and r
the A.P = (2475) : (2475 - 325) = 2475 : 2150 respectively
= 5[495] : 5[430] = 495 : 430 = 99 : 86. Choice (C) First term = sum of all the terms following it
a
28. If the first term is a and the common difference is d, a= −a
1− r
we have the sum of the fifth, thirteenth and eighteenth
a (1 – 2r) = 0
terms as
As all the terms are positive, a ≠ 0.
(a + 4d) + (a + 12d) + (a + 17d) ⇒3a + 33d = 0.
1 – 2r = 0
Dividing by 3, we have a + 11d = 0. Hence the 12th term
r = 1/2
of the A.P is 0. Choice (C) a
29. S = 3(2) + 4(3) + 5(4) + …… 10 terms.
2 2 2 = 16 Choice (A)
1− r
= (2 + 1)22 + (3 + 1)32 + (4 + 1)42 + ….. 10 terms a
= (23 + 22) + (33 + 32) + (43 + 42) +….. 10 terms 35. Let the first number and the common ratio be 2 and
r
= (23 + 33 + 43 + ……+ 113) + (22 + 32 + 42……+112) r respectively.
= (13 + 23 + 33 …+ 113) + (12 + 22 + 32 +…+ 112) – 13 – 12 a a
= 4355 + 505 = 4860. Choice (B) + + a + ar + ar2 = 62 …. (1)
r2 r
30. If the first term of the G.P is a and the common ratio is r2 r 1 1 1 31
a( r 8 − 1) + + + + 2 =
r, we have = 510 … (1) a a a ar ar 32
r −1 1  2 a a  31
a( r 4 − 1) ⇒  ar + ar + a + + 2  =
and = 30 …. (2) a2  r r 32
r −1 62
a( r 8 − 1)
(1) ÷ (2) ⇒a2 = 31 = 64 Choice (D)
Dividing (1) by (2) we have r 4− 1
a( r − 1) 32
r −1
Quantitative Ability Test 2
Number of Questions: 35 Section Marks: 30

Directions for questions 1 to 16: Select the correct alterna- in 18 more days. Find the time for which A worked (in
tive from the given choices. days).
1. Two men, two women and six part-time workers take (A) 6 (B) 20
12 days to complete a job. The same job can be com- (C) 12 (D) 24
pleted by 10 men and 18 part-timers in 4 days. If two 9. Six taps working together take 12 minutes to fill a tank.
men and three women take 16 days to complete that Find the time taken (in minutes) by 24 taps working
job, find the time taken by one woman to complete that together to fill a tank twice as big.
job (in days). (A) 6 (B) 8
(A) 96 (B) 60 (C) 12 (D) 24
(C) 75 (D) 100 10. P can do a piece of work in 12 days working 6 hrs a day.
2. A can complete a job in 20 days. B works twice as fast Q can do the same work in 18 days working 5 hrs a day.
as A. They both work together for 5 days. On the 6th day, If P and Q work together 4 hrs a day, then in how many
they complete the job with the help of C. Find the time days can they complete the work?
taken by C alone to complete the job (in days). (A) 10 (B) 11
(A) 5 (B) 6 (C) 12 (D) 14
(C) 10 (D) 12 11. A, B and C can complete a piece of work in 20, 30 and
3. Four men take ten days to complete one-third of a 20 days respectively. They start the work together but
work. How many more men are required to complete A leaves after 5 days. After some more days C leaves.
the remaining work in five days? B completes the remaining work in 5/3 more days. For
(A) 16 (B) 14 how many days does B work?
(C) 15 (D) 12 10
(A) (B) 10
4. A and B working separately can do a piece of work in 5 3
days and 10 days respectively. They work on alternate 14
days starting with B on the first day. In how many days (C) (D) 14
3
will the work be completed?
(A) 6 (B) 7 12. A, B and C take 20, 30 and 60 days to complete a job. A
(C) 8 (D) 9 works along with B on the 1st day and with C on the 2nd
day. If they continue in this manner, then find the time
5. A tank is fitted with three pipes A, B and C. The three
taken (in days) to complete the work.
pipes can be used as inlet or outlet pipes with the same
5 4
flow rates. When one among A, B and C in turns works (A) 12 (B) 18
as outlet pipe and the other two as inlet pipes, it takes 6 5
30, 40 and 24 minutes to fill the tank respectively. Find 1 1
the time taken by A to fill the empty tank (in minutes). (C) 14 13
(D)
4 4
(A) 30 (B) 40
(C) 25 (D) 20 13. Amar can complete a job in 15 days, while Bhavan
can complete it in 10 days. They start working together
6. A and B take respectively 12 days and 27 days more and two days before the work was expected to be com-
time to complete a piece of work, working alone, than pleted, Bhavan left. Find the time taken by Amar to
when they work together. Find the time taken by them complete the remaining work (in days).
to complete the work working together. (A) 4 (B) 5
(A) 15 days (B) 20 days (C) 6 (D) 8
(C) 24 days (D) 18 days
14. A pipe can fill a 1000 litre tank in 10 minutes while
7. A, B and C work together to complete a job. A gets `600 another pipe can empty a 600 litre tank in 8 minutes.
out of the total share of `2400. If A works twice as fast If they work together, then how long will they take (in
as B, find the share of C. minutes) to fill a 500 litre tank?
(A) `1200 (B) `1500 (A) 10 (B) 15
(C) `1000 (D) `1600 (C) 20 (D) 25
8. A, B and C work at the same rate. A starts the job and 15. A and B can complete a job in 25 days and 20 days
after 25% of the work is completed, he leaves. B and respectively, working alone. With the help of C,
C take over and complete the remaining work together they can complete the job in 62/3 days. Find the
Quantitative Ability Test 2 | 1.41

percentage of work completed by the fastest worker 19. What is the plane’s cruising speed (in kmph)?
of the three. (A) 825
1 (B) 900
(A) 20 % (B) 25% (C) 875
3
(D) Cannot be determined
2
(C) 40% (D) 16 % 20. What is the time difference between cities A and Q?
3
(A) 1 hour
16. A tank has three inlet pipes I, II and III fitted to it whose 1
flow rates are in the ratio 2 : 5 : 6. Pipe III takes 1 hour (B) 2 hours
less than pipe II to fill the tank. Find the time (in hours)
2
(C) 2 hours
taken by pipe I to fill the tank.
(D) Cannot be determined
(A) 6 (B) 10
(C) 5 (D) 15 Directions for questions 21 to 35: Select the correct alter-
native from the given choices.
Directions for questions 17 and 18: These questions are
based on the following data. 21. Amar covered the first one-fourth of a certain dis-
In a city there are 5 major traffic junctions – A, B, C, D and tance at 2 km/hr, half of the remaining distance
E. There are no direct roads connecting AC,BE or CE but at 3 km/hr and the remaining distance at 4 km/
for every other pair of junctions, there are direct connecting hr. Find his average speed (in km/hr) for the entire
roads, which all happen to be of equal length. Traffic moves journey.
at recommended uniform speeds on each road – at 20 km/ 7 8
(A) 2 (B) 2
hr on BD and AD, at 30 km/hr on AE, at 40 km/hr on BC 11 11
and CD, and at 60 km /hr on AB and DE (Assume any direct 9 10
connecting road is straight line). (C) 2 (D) 2
11 11
17. A traffic inspector wants to visit any three traffic junc-
tions in the shortest possible time, starting from A. 22. A man starts from P at 8 a.m. and reaches Q by 9 : 30
What are the three points that he can visit (other than a.m. At what time should he start from Q to reach R at
A) in order if he drives at the recommended speed on 11 : 30 a.m., where PQ : QR = 10 : 11?
each road? (A) 10 : 01 a.m. (B) 9 : 59 a.m.
(A) BCD (B) EDB (C) 9 : 50 a.m. (D) 9 : 51 a.m.
(C) BDC (D) BDE 23. A boat covered a certain distance upstream and returned
18. A new direct connecting road is constructed joining A to the starting point. If the speed of the boat in still
and C with its recommended speed fixed at 50 km/hr. water is doubled and the speed of the stream is tripled,
If AB = 10 km, find the time (in minutes) in which the it would have taken the same time for the round trip.
traffic inspector can complete the round trip ABCA. Find the ratio of the speed of the boat in still water to
the speed of the stream.
(A) 12 + 25 3
(A) 5 : 2 (B) 3 : 2
(B) 25 + 12 3
(C) 7: 2 (D) 3 : 2
(C) 24 + 12.5 3
24. A boat started travelling downstream from a point A
(D) Cannot be determined
on a river. After it had travelled 12 km, a log started
Directions for questions 19 and 20: These questions are floating from A. The boat travels for 2 more hours in the
based on the following data. same direction and then turns around and meets the log
Cities P, Q and A are in different time zones. P and Q are at a point 12 km from A. If the speed of the boat in still
located at 4500 km, east and west of A respectively. The water is thrice the speed of the stream, find the speed of
table below describes the schedule of an airline operating the stream (in km/hr).
non-stop flights between A and P, A and Q. All the times (A) 1 km/hr (B) 2 km/hr
indicated are local and on the same day. 3
(C) km/hr (D) 4 km/hr
Departure Arrival 2
City Time City Time
25. In a race, A gives B a start of 25 m and C a start of 50
A 7 : 00 am P 3 : 00 pm m. If B runs 50% faster than C and all the three reach
A 9 : 00 am Q 12 : 00 noon the finishing point simultaneously, then find the ratio of
Planes cruise at the same speed to both the cities but the speeds of A and C.
effective speed is influenced by a steady wind blowing from (A) 2 : 1 (B) 4 : 3
east to west at 75 kmph. (C) 5 : 4 (D) 3 : 1
1.42 | Quantitative Ability Test 2

26. In a 200 m race, A gives B a start of 10 m and beats him (A) 20 (B) 25
by 10 m or 2 seconds. Find the speed of A (in m/s). (C) 30 (D) 35
200 150 31. When the speed of a train is increased by 5 m/s, it
(A) (B)
17 17 would take 40 seconds to cross a 200 m long platform.
125 50 If it crosses a 300 m long platform in 50 seconds, at its
(C) (D) original speed, then find the original speed of the train
17 9
(in m/s).
27. On a 900 m long circular track, A, B and C start run- (A) 35 (B) 15
ning from the same point simultaneously. A runs in the (C) 20 (D) 30
clockwise direction at 2 m/s while B and C run in the
32. Two trains take 80 seconds to cross each other, when
anti-clockwise direction at 3 m/s and 4 m/s respec-
travelling in the same direction. They take 60 seconds
tively. Find the time interval (in seconds) between A
to cross each other, when travelling in opposite direc-
and C meeting for the first time at the starting point and
tions. Find the ratio of the speeds of the faster and the
B and C meeting for the first time.
slower train.
(A) 600 (B) 450
(A) 4 : 1 (B) 7 : 1
(C) 750 (D) 800
(C) 3 : 1 (D) 6 : 1
28. A, B and C run along a circular track with speeds in
33. There are two cars 80 km apart. When they travel in
the ratio 1 : 2 : 4 starting from the same point simulta-
the same direction, they would take twice the time to
neously. If A takes 3 minutes to complete one round of
meet, compared to the time they would take to meet
the track, find the time taken (in minutes) by the three
while travelling towards each other. Find the ratio of
to meet at the starting point for the first time.
their speeds.
3
(A) 6 (B) (A) 3 : 1 (B) 2 : 1
2 (C) 4 : 3 (D) 3 : 2
(C) 3 (D) Cannot be determined 34. By travelling 20% faster than his usual speed, a per-
29. Two men A and B start from two points P and Q simul- son reaches his office from home 10 minutes earlier
taneously towards each other. They meet after two than his usual time. By how many minutes would he be
hours of their starting, B takes 3 hours less to reach P delayed as compared with his usual time, if he travels
than A takes to reach Q. Find the ratio of the speeds of 25% slower than his usual speed?
A and B. (A) 10 (B) 15
(A) 3 : 2 (B) 2 : 1 (C) 20 (D) 25
(C) 1 : 2 (D) 3 : 1 35. A frog spots a snake 30 m behind it. It starts mov-
30. A man travels 51 km in 61 minutes and 30 seconds ing away from it at 12 m/s. After 5 seconds, it sees
with an usual speed of 50 km/hr. There are some speed that the snake has just begun to move towards it
breakers on the road. Each speed breaker reduces his at 20 m/s and increases its speed by 3 m/s. Find
speed to 80% of his usual speed for a distance of 50 m the time taken by the snake (in seconds) to catch
about the speed breaker. Find the number of speed the frog.
breakers that he crossed. (A) 15 (B) 18
(C) 21 (D) 24

Answer Key
1. A 2. C 3. D 4. B 5. A 6. D 7. B 8. C 9. A 10. A
11. B 12. D 13. B 14. C 15. C 16. D 17. A 18. B 19. A 20. C
21. D 22. D 23. C 24. B 25. A 26. D 27. B 28. C 29. C 30. A
31. D 32. B 33. A 34. C 35. B

Hints and Explanations


1. Let the work done by 1 man, 1 woman and part-timer in Work done by 2 men and 3 women in 16 days
a day be m, w and p units respectively. = [[3 w] + 3 w] 16 = 96w
Given (2m + 2w + 6p) 12 = (10m + 18p) 4 Time taken by 1 woman to complete that job
3 96w
24w = 16m ⇒ m = 2 w = = 96 days Choice (A)
w
Quantitative Ability Test 2 | 1.43

1 1 6. Let the time taken by A and B working together to com-


2. A can do th of the job in a day. B can do th of the plete the work be t days.Time taken by A alone and B
20 10
alone to complete the work is (t + 12) days and (t + 27)
9 days respectively.
job in a day. In 6 days they will together complete th
10 Work done by A and B working together in a day
1 1 1 1
of the job. The balance th of the job is done by C on = + which is equal to .
10 t + 12 t + 27 t
6th day. Hence C would take 10 days to complete the 1 1 1
⇒ + =
job independently. Choice (C) t + 12 t + 27 t
3. Four men can do one-third of the work in 10 days. In 5 t + 27 + t + 12 1
⇒ =
days, one-third of the work can be completed by 8 men. (t + 12) (t + 27) t
Two-thirds of the work can be completed by 16 men. As
⇒ 2t2 + 39t = t2 + 39t + 324
there are 4 men, 12 men are required additionally.
⇒ t2 = 324 ⇒ t = 18. Choice (D)
 Choice (D)
1 1
4. A can do the work in 5 days. The part of the work done 7. As A gets 4 th of the total share, he completes 4 th of
1
by A in one day = the total work. B whose rate is half that of A, completes
5
1
B can do the work in 10 days. th of the total work, for which he gets `300.
1 8
The part of the work done by B in one day = \ Share of C = 2400 – 600 – 3000
10
= `1500. Choice (B)
1 1 3
both A and B in two days = + =
5 10 10  3
8. The time taken by B and C together to complete   th
3 9  4
The part of the work done in 6 days = 3 × = of the work is 18 days
10 10
\ The time taken to complete 1 unit of work
9 1  4
The part of the remaining work = 1 – = = 18 ×   = 24 days
10 10  3
1 \ The time taken by each alone to complete the
As B starts the work, th of the work can be done by work = (24) (2) = 48 days.
10
B on 7th day. ⇒ The time taken by A to complete (1/4)th of the
\ In 7 days, the work will be completed.  1
work =   (48) = 12 days Choice (C)
 Choice (B)  4
5. Let the time (in minutes) taken by A, B and C to either 9. Time taken by 24 taps working together to fill the tank
fill or empty the tank be a, b and c respectively. 1
would be th of the time taken by 6 taps working
1 1 1 1 4
+ − =  ….. (1)
b c a 30 together i.e. 3 minutes. To fill a tank twice as big, 24
taps would take 6 minutes working together
1 1 1 1  Choice (A)
+ − =  ….. (2)
c a b 40
10. Time taken by P to complete the work in 12 × 6 i.e., 72
1 1 1 1 man hours.
+ − =  ….. (3)
a b c 24 Time taken by Q to complete the work in 18 × 5 i.e.,
90 man hours
1 1 1 1
Adding the above three equations, + + = The part of the work done by P and Q in
a b c 10
1 1 90 + 72 1
 --- (4) 1 hr = + = = 40
Subtracting equation (1) from equation (4), 72 90 72 (90 )
2 2 \ They complete the work in 40 hours.
we get =
a 30 By working 4 hrs per day, they can complete in
⇒ a = 30. Choice (A) 40
4 i.e., 10 days Choice (A)
1.44 | Quantitative Ability Test 2

11. Let the total work be 1 unit 1000 lit


5 14. Filling rate of first pipe = = 100 lit/min.
x +5+ 10 min
5 x+5 3 =1
+ + 600 lit
20 20 30 Emptying rate of second pipe = = 75lit/min.
⇒ x = 10/3 8 min
5 Working together they can fill 25 lit/min. Time taken by
Time for which B worked = x + 5 + = 10 days .
500
3
them working together to fill a 500 litre tank = or
 Choice (B) 25
20 minutes. Choice (C)
1 1 1
1 2. A and B complete + i.e., th of the work on 1 5. Let us assume that the third person takes c days to com-
20 30 12
plete the work independently. Total work completed by
1 1 1 1 1 1 1
1st day. A and C complete + i.e., th of the + + =
20 60 15 the three in a day, working together =
25 20 c 6 2
3 3
work on the 2nd day. In 2 days, th of the work would
20 1 3 1 1 1 3
9 = − −
be completed. Working in this way, th of the work ⇒ =
10 c 20 20 25 c 50
1
would be completed in 12 days. Of the balance th of As the third person completes most of the work in a
10 day, he is the fastest.
1 Ratio of work completed by the three persons
the work, A and B would complete th of the work
12 1 1 3
= : : = 4:5:6
the next day. A and C would complete the balance 25 20 50
1 1 Percentage of work done by the third person
th of the work in another th of a day. Hence a
60 4 6
= (100%) = 40%. Choice (C)
1 (4 + 5 + 6)
total of 13 days would be taken to complete the
4 16. Let the volume of the tank be V litres. Let the filling
work. rates of pipes I, II and III be 2x, 5x and 6x respectively
(in litres/hour)
Alternate method:
V V
Assume the work (in units) to be the LCM of the indi- = − 1 ⇒ V = 30x
vidual time taken by A, B and C to complete the job i.e., 6 x 5x
60 units. Capacities of A, B and C would be 6 units a 30 x
day. A and B would complete 5 units the first day. A and
Time taken by pipe I to fill the tank = = 15 hours .
2x
C would complete 4 units the second day. Hence 9 units  Choice (D)
would be completed in two days. In 12 days, 54 units
If the 10 possible pairs of points, for 7 pairs, the dis-
would be completed. Of the balance 6 units, 5 units
tances between the points equal. This is possible if of
would be completed the 13th day. The balance 1 unit
the 5 points, 4 are consecutive vertices of a regular hex-
1
would be completed in th of the 14th day. Hence a agon and the 5th is the centre of the circum circle. We
4 can think of the following figure.
total of 131/4 days would be taken to complete the Now we have to final which of these 5 points in A,
work. Choice (D) which is B etc., AC, BE, CE have no direct roads con-
13. Amar and Bhavan would have taken 6 days to complete necting them. In the figure 13, 25 and also 35 have no
the job working together. Bhavan left after 4 days of the direct roads connecting them (35 is connected through
start of the work. In 4 days, Bhavan would have com- 4 not directly)
4 2 \ We get the following possibilities. For either ques-
pleted i.e. th of the work. Amar completes the
10 5 tions that follow it does not matter, which of the
3 two figures we use.
remaining th of the work for which he would have 1 7. The routes and recommended speeds are shows in the
5
figure below.
3
taken (15) i.e., 9 days. Hence Amar completes the The routes and the time taken are tabulated below.
5 We can take
remaining work in 5 days. Choice (B) AB = BC = CD = DB = AD = DE = EA = r km
Quantitative Ability Test 2 | 1.45

Route time Route Time Solutions for questions 21 to 35:


r r r
(1) ABCD + + 21. Let the total distance covered by Amar be d km. Amar
60 40 40 d 3d
covered km at 2 km/hr, km at 3 km/hr and
r r r 4 8
(2) AEDC + +
30 60 40 3d
km at 4 km/hr.
r r r 8
(3) AEDB + +
30 60 20 d 3d 3d
r r r 11d
(4) ABDE + + Total travel time of Amar = 4 + 8 + 8 = hours
60 20 60 2 3 4 32
Multiplying all the time by LCM (60, 40, 30, 20) or Average speed of Amar
120, we get 8r, 12r, 9r, 10r respectively Total distance he travelled
=
\ For ABCD the time is the least. Choice (A) His total travel time
1 8. If AC and BE are also connected, each dist each dis- d 10
= =2 hours
tance would be 3 r. The time for the round trip ABCD 11d 11
11d / 3
is 3
r
+
r
+
3r 10 r + 15r + 12 3r
= =
(
25 + 12 3 (10 ) ) Alternative method:
It can be seen from the normal method, as d cancels
60 40 50 600 600
finally, any value of d can be taken. Taking d = 8,
(∵ r = 10 km) = 25 + 12 3 min. Choice (B) 2 3 3 11
total travel time of Amar = + + = hours
Solutions for questions 19 and 20: 2 3 4 4
Let the speed of the plane be x km 8 10
Average speed of Amar = =2 km/hr .
Let the time difference between A and P be t hours (i.e., P is 11 11
t hours ahead of A). 4
\ The time difference between A and Q is t hours (Q  Choice (D)
is t hours behind A) 11
\ we have the following equations. 22. From Q to R, he has to cover of the distance from
4500 10
(from A to P) =8–t .….. (1) P to Q.
x − 75
11
4500 \ Time taken by him to reach R from Q will be
10
(from A to Q) =3+t …… (2)
x + 75 times the time taken by him to reach R from Q i.e.,
 2x  11
(1) + (2) ⇒ 4500  2 90 minutes × .
2  = 11 10
 x − 75 
\ He needs 99 minutes.
⇒ 11x2 – 11(75)2 = 9000x To reach R by 11 : 30 a.m. he should start from
9000 ± (9000)2 − 4 (11)(−11) 752 Q at 9 : 51 a.m. Choice (D)
\ x =
2 (11) 23. Let the distance travelled in each direction (upstream as
well as downstream) be d km. Let the speed of the boat
9000 ± 150 (61)
9000 ± (150) 60 2 + 121 in still water be x km/hr and the speed of the stream be
= =
22 22 y km/hr. Total travel time of the boat
9000 + 9150 = upstream travel time + downstream travel time
As x is positive, x = = 825 d d
22 +
=
4500 x+ y x− y
From (1), =8–t
825 − 75 Now the speed of the boat in still water and the speed
of the stream are 2x km/hr and 3y km/hr respectively,
⇒ 6 = 8 – t ⇒ t = 2 d d
19. Choice (A) Total travel time = +
2x + 3y 2x − 3y
2 0. Choice (C)
1.46 | Quantitative Ability Test 2

d d d d 3 minutes. Time taken by all the three to meet for the


Given, + = +
x + y x − y 2x + 3y 2x − 3y L L L L
first time = LCM  , ,  = = 3 minutes.
 x 2x 4x  x
2dx 4 dx
= ; dx (2x2 - 7y2) = 0  Choice (C)
x2 − y2 4 x2 − 9 y2
2 9. Let the speeds of A and B be x km/hr and y km/hr
7 respectively. Distance from P and Q to their first meet-
dx = 0 or x = ± y ing point are 2x km and 2y km respectively. Times
2
taken by A and B to reach Q and P from their first meet-
As dx cannot be 0 (∵ d > 0 and x > 0) and x and y are 2y 2x
both positive, ing point are hours and hours respectively.
x y
7 x 7
x= y⇒ = . Choice (C) y x
2 y 2 2 = 2 +3
x y
24. Let the point 12 km from A and the point where the
Substituting the choices in the above equation, only
boat turns back be B and C respectively.
x 1
If the speed of the stream is y km/hr, speed of the boat = , satisfies the condition. Choice (C)
in still water = 3y km/hr. It travels for 2 hrs to cover BC. y 2
\ BC = (3y + y) 2 = 8y km 30. If the man covers the entire distance at the usual speed,
Time taken by the boat to travel from C to B 51 2 (60 )
8y he takes hr or 1.02 hr = 1 hr min = 1 hr / min
= = 4 hr. 50 100
3y − y
12 s.
As the boat takes 6 hours to travel from B to C and But he actually takes 1 hr 1 min 30 s, i.e., 18 s more.
back, For one speed breaker, he takes a certain extra time,
12
= 6 or y = 2  Choice (B)  0.05 0.05  0.05  1  5 (36)
y which is  −  hr =   hr = s
 40 50  10  20  200
25. Let the length of the race be x m. = 0.9s
By the time A finish the race, B and C would have run 18
(x – 25) and (x - 50) m respectively. \ He has to cross or 20 speed breakers.
0.9
x − 25 3  Choice (A)
As B is 50% faster than C, = ⇒ x =100
x − 50 2
3 1. Let the length of the train be L m and speed of the train
Ratio of the speeds of A and C = x : (x - 50) = 2 : 1. be s m/sec. Time taken by the train to cross a 200 m
 Choice (A) L + 200
long platform at increased speed = = 40
26. Speed of B = 10/2 = 5 m/s s+5
Time for which B would have run when A finishes the ⇒ L = 40s
200 − (10 + 10 ) Time taken by the train to cross a 300 m long plat-
race = = 36 seconds. L + 300
5 = 50
form (in seconds) =
So, A takes 36 seconds to run the race. s
200 50 As L = 40s,
\ Speed of A = = m/s. Choice (D)
36 9 40s + 300 = 50s
s = 30. Choice (D)
27. Time taken by A and C to meet for the first time at the
 900 900  32. Let the lengths of the two trains be L1 and L2
starting point = LCM 
 2
,  = 450 sec Let the speeds of the faster and slower trains be S1 and
4 
S2 respectively.
Time taken by B and C to meet for the first time L1 + L2
900 = 80 ⇒ L + L = 80 (S - S )
= = 900 seconds S1 − S2 1 2 1 2
Difference of the speeds of B and C
L1 + L2
Required time interval is 450 seconds. = 60 ⇒ L + L = 60 (S - S )
 Choice (B) S1 − S2 1 2 1 2

2 8. Let the speeds (in m/min) of A, B and C be x, 2x and 4x 80 (S1 - S2) = 60 (S1 + S2)
respectively. Let the length of the track be L m. Given S1 7
that the time taken by A to complete one round = L/X = 20S1 = 140S2 ⇒ = . Choice (B)
S2 1
Quantitative Ability Test 2 | 1.47

33. Let the speeds (in km/hr) of the faster and slower cars ⇒ His usual time = 60 minutes
be x and y respectively.Time taken by the cars to meet If he travels 25% slower than his usual speed, he
when they travel in the same direction and in the 25 3
80 80 would travel at S − S= S
100 4
opposite direction are hours and hours
x− y x+ y
3
respectively. If his speed is ths of his usual speed, he would
4
80  80  4
= 2
x− y  x + y  take
3
rds of the usual time to travel to office. He
80 (x + y) = 160 (x - y) 240y = 80x 1
x 3 would be late by rd of the usual time i.e., 20
= . Choice (A) 3
y 1
minutes. Choice (C)
34. Let the usual speed of the man be S km/hr. If he travels 35. In 5 seconds, the frog would move 60 m. When the
20 6 snake is spotted by the frog, the frog would be 90 m
20% faster, he would travel at S + S = S km/hr.
100 5 ahead of the snake. Time, the snake would take to catch
6 the frog (in seconds)
As his speed is th of his usual speed, he would take 90
5
=
5 DIfference of speeds of snake and frog
th of the usual time to travel to office. He saves one
6 90
= = 18 . Choice (B)
sixth of his usual time = 10 minutes 20 − 15
Quantitative Ability Test 3
(Indices, Surds and Logarithms)
Number of Questions: 35 Section Marks: 30

Directions for questions 1 to 35: Select the correct alterna- 9. If 3x+3 – 3x−3 = 6552, then find x2.
tive from the given choices. (A) 5 (B) 25
1. Find the value of x, if 2x = 8y and 64y = 216x + y − 2. (C) 3 (D) 9
1 1 10. If x, y, z are real numbers such that xyz = 1, then the
(A) 2 (B) 2 1 1 1
4 2 + +
expression
1 1 1 + x + y −1 1 + y + z −1 1 + z + x −1
(C) 3 (D)
3
2 3 is equal to
3
(A) 1 (B)
x+ y+z
2. If x = 3 9 3
9 3 9 3 9................. , what is the value 3 x+ y+z
of x? (C) −1
−1 −1 (D)
x +y +z 3
(A) 3 (B) –3
a b c

(C) 3 (D) 9 (81a ) (81b ) (81c )


11. If = 3.
(6561b ) − c (6561c ) − a (6561a ) − b
3. If 93x – 4 = 6561. 27 x − 2 , then find the value of x. Then a + b + c could be 1
(A) 1 (B) 2 (A) 2 (B)
(C) 3 (D) None of these 3
1 1
1 1 2 (C) – (D) −
4. If p  = q m = r n and + = ,then which of the 2 3
 n m
following is valid? (p > 1, q > 1, r > 1) 1
12. If x = 1
find x.
Ι. p = q 2   ΙΙ. pq =  2   ΙΙΙ. p2 = qr 4+
1
(A) Both Ι and Ι (B) Both ΙΙ and ΙΙΙ 3+
1
(C) Both ΙΙΙ and Ι (D) None of these 4+
4 3 + ....∞
23
3( −1)
5. Find the value of x if (125) = (25)
2x −3
. 3 3
(A) 3+ 3−
(B)
(A) 4.5 (B) 2.5 2 2
(C) 1.5 (D) None of these 1 1
(C) 3 + (D) 3−
2 2 2
6. If 22 x + 3 = 82 x +1 and x is positive, then what is the value
of x? 8 88 88

(A) 2 (B) 3 13. If A = 888 , B = 88 , C = 8888 and D = 88 , which of


(C) 1 (D) 4 the following represents the ascending order of the
values of A, B, C, D?
7. If t1 = 5 , t2 = 5 5 , t3 = 5 5 5 and so on, then (A) CDAB (B) CABD
the product of the first ten terms (t1) (t2) (t3) (t4)…. (t10) (C) CBAD (D) ACBD
is equal to
14. Solve for x: x + x − 1− x = 1 .
512 4609 2048 18431
(A) 5 5
(B) 16
(A) 1 (B)
1024 512 4607
59217 (D) 5 4 25
(C)
(C) 5 (D) 0
8. If x ≠ x, then
2

x 4 b + x 2( a + b) + x 4 a 15. The arithmetic mean of two surds is 5 + 9 2 , and one


is equal to
( x 2a + x a+ b + x 2b )( x 2a − x a+ b + x 2b ) of the surds is 1 + 12 2
What is the square root of the other surd?
xa x 2a
(A) b
(B) 4−3 2
(A) 6 − 21 2 (B)
x x 2b
(C) x
a+b
(D) None of these (C) 3 ( ) 2 2− 3
2 + 1 (D) ( )
Quantitative Ability Test 3 | 1.49

1 1 24. If a2 + 4b2 = 12ab, what is the value of log(a + 2b)?


16. + =
6 + 7 − 13 6 − 7 − 13  a  b
(A) log   + log   + log 2
1  2  2
(A) 6 (B) 1
6 (B) (log a + log b – log 2)
1 2
(C) 6 (D) 1
6 (C) (log a + log b + 4 log 2)
17. Find the square root of 2
1
 1 1 1  (D) (log a – log b + 4 log 2)
1 + 2 + 1 + 3 + 2 + 4 + 3
+ ......... 2
 
 1  log m p.log n p
  25. Simplify .
 324 + 323  log m p + log n p
1 (A) 1 (B) logp (m + n)
(A) 3 2 (B) (C) logp mn (D) logmnp
2
3 −1 26. If a > 1, loga a + log 1 a + log 1 a + …..+ log 1 a =
(C) 2 3 (D) a2 a3 a 20
2 (A) 420 (B) 210
18. If xy = yz = zx and (x, y, z) > 0, then (C) 380 (D) 190
 xy + yz + zx  xy + yz + zx log x log y log z
(A) 3   (B) 27. If = = , then find the value of
 xyz  xyz y−z z−x x− y
x+ y+z xy + yz + zx
(C)
xyz
(D)
x+ y+z ( )(
2 2

)(2 2

)
log  x y + yz + z + z y z + zx + x + z z x + xy + y + z 
2 2


(A) 2 (B) 0
19. If log43, log4 (3m − 2) and log4  3m − 8  are in
  (C) 3 (D) None of these
3
arithmetic progression, then the number of possible 2 8. If abc = 1, then find the value of
1 1 1
values of m are + +
log bc a log ac b log ab c 3
3 3

(A) 1 (B) 2
−1 1
(C) 4 (D) 5 (A) (B) log abc
3 3
20. If logx 162 = m and logx72 = n, then what is the value of
(C) –1 (D) log a + b + c abc
logx 7776 in terms of m and n?
m + 3n 3m − 5n 29. If log 6 27 = t , then find the value of log18 4 in terms
(A) (B)
m + 5n m + 2n of t.
m + 3n 3m − 5n  2 −t  3−t
(C) (D) (A) 3   (B) 2 
2 2  2+t  3+ t 
21. Which of the following is a possible value of x if 6+t 4 −t
(C) (D)
log3x2 − log3x x = 8logx 3? 3+ t 3+ t
1 1 3 0. For a ≥ b, b > 1 the value of the expression
(A) (B)
81 243  a  b
log a   + log b   can never be
(C) 243 (D) 9  b  a
(A) 0 (B) 1
22. If a = a = log 4 31 , then _____ . (C) −2 (D) −0.5
(A) a < 2 (B) 2 < a < 2.5
(C) 2.5 < a < 2.8 (D) 2.8 < a 31. If log 4 ( x 2 + x ) − log 4 ( x + 1) = 2 , then x =
23. If log10 (2x + 3) − 1 = log10x, then find x. (A) 2 (B) 4
(C) 8 (D) 16
2 3
(A) (B) 4 3 2. If log 10 3 = 0.4771, then find the number of digits in
7
7 3 (243)50.
(C) (D) (A) 200 (B) 205
8 8
(C) 120 (D) 210
1.50 | Quantitative Ability Test 3

33. If (log 16) (log 27) = (log x) (log y) and (log 4096) (A) 2.5 (B) 3
[log x – log 27] = [log16 – log y] (log 512), which of the (C) 1.5 (D) 1
following can be the value of (x – y)?
(A) –11 1 1 1
35. If logbca = , logcab = and logabc = , find the
(B) 73 p q r
(C) –73 1 1 1
(D) More than one options value of + +
p +1 q +1 r +1
34. What is the value of? 3
(A) 1 (B)
2
log64 512 512 512 512.................. ∝ . (C) 2 (D) None of these

Answer Keys
1. A 2. A 3. D 4. D 5. B 6. B 7. C 8. D 9. B 10. A
11. C 12. B 13. B 14. B 15. C 16. B 17. A 18. A 19. B 20. C
21. A 22. B 23. D 24. C 25. D 26. B 27. B 28. C 29. B 30. B
31. B 32. C 33. D 34. C 35. A

Hints and Explanations

1. Given 2x = 8y ⇒ 2x = (23)y Hence p  = k ; log p k = 


⇒ x = 3y
log q k = m; log r k = n given that
Also 64y = 216x + y − 2
⇒ 64y = 63 (3y + y − 2) 1 1 2
+ =
⇒ 64y = 612y − 6  n m
⇒ 4y = 12y − 6 1 1 2
3 ⇒ + =
⇒ 6 = 8y ⇒ y = 4 log p k log r k log q k

9 1 ⇒ log k p + log k r = 2 ⋅ log k q


∴ x = 3y = 4 = 2
4 ⇒ log k ( pr ) = log k q 2 pr = q2 Choice (D)
 Choice (A)
34
5. 2 is always even, as 2 raised to any power is even.
34
2. Given x = 3
9 3
9 3 9 3 9................. Hence ( −1)2 = (–1)even number = +1
4
23
3( −1) 1
Hence, (25) = (25)3 = 56.
⇒ x = 9 3 9
3 3
9 3 9 3 9................. = 9x
(125)2x–3 = 56. ⇒ 53(2x–3) = 56. ⇒ 2x – 3 = 2; x = 2.5
⇒ x = 0, 3, –3. Since x is positive, x = 3. Choice (A)  Choice (B)
(x – 2)
3. 93 x – 4 = 38. (27) 2 6. By equating the index of 2 on both sides we get
2x2 + 3 = 6x + 3 ⇒ x = 3 (as x > 0) Choice (B)
( x – 2)
93 x – 4 = 38. (33 )
1 1
2 1−
7. t1 = 5 2 = 5 2

(x – 2) 3 1
3 1−
32( 3 x – 4 ) = 38 ·3 2
t2 = 5 4 = 5 22

3x
6x – = 8+8– 3 7
1−
1
2 t3 = 5 8 = 5 23

(3 x – 6 ) 1−
1
6x – 8 = 8 + t10 = 5 210
2
9x 26  1− 1   1− 12   1− 13 
= 13 ⇒ x =  Choice (D) = (t1 )(t2 ) (t3 ) ..... (t10 ) =  5 2   5 2   5 2 
2 9    
4. Given that p  = q m = r n  1− 110 
…..  5 2 
Let each be equal to k.  
Quantitative Ability Test 3 | 1.51

2 2 2
1 1 1 1 
10 −  + 2 + 3 + .... + 10 
11. 81a + b + c 2 2 2

= 5 2 2 2 2 
[ −2 bc − 2 ca − 2 ab ]
= 81a + b + c + 2 ab + 2 bc + 2 ca
81
1 1 1 1
10 − 1 + + 2 + .... + 9  1
= 81( a + b + c ) = 3 = 814
2

= 5 2 2 2 2 

 1  1
10 −
1−
1  210  ⇒ a + b + c = ±
2 1 2
1−  Choice (C)
=5 2
1
10 −1+
1 1 2. Given x = 1
=5 210
=
1024
59217 4+
1
 Choice (C) 3+
1
2( a + b) 4+
x +x
4b
+x 4a
3 + ......∞
8. Let E = E = .
( x + x + x )( x − x
2a a+ b 2b 2a a+ b
+ x 2b )
⇒ x =
1
x=
3+ x
1 ⇒ 4 (3 + x ) + 1
Den (E) = x ( x + x + 1) . x ( x
2b 2a− 2b a−b 2b 2a− 2b
− x a − b + 1) 4+
3+ x
x (1 + x +x4b
) 2(a - b ) 4( a -b )
3+ x
E= ⇒ x =
x 4 b 1- x (a- b ) + x 2( a-b )  1 + x a-b + x 2( a-b )  4 x + 13
Considering xa-b = t, we get ⇒ 4x2 + 12x − 3 = 0
1+ t 2 + t 4 1+ t 2 + t 4 −12 ± 144 + 48
= =1 ⇒ x =
( )(
1- t + t 2 1 + t + t 2 1 + t + t
2
) 4 8

(1 – t + t ) (1 + t + t2) = (1 + t2– t) (1 + t2 + t)
2
⇒ x =
(
4 −3 ± 12 )
=( (1 + t2)2 – t2) 8
Note: The condition x2 ≠ x means x ≠ 0, x ≠ 1. If x = 0, −3 ± 2 3
⇒ x =
E is not defined. If x = 1, E = 1. 2
∴ This condition need not be imposed. But impos- Since x > 0,
ing the condition does not make the statement −3
(that E = 1) false. Choice (D) x = x = + 3 Choice (B)
2
9. 3 – 3 = 6552
x+3 x−3
8 88 88

 1 13. A = 888 B = 88 C = 8888 D = 88


3x 33 − 3  = 6552 Since the base of all the numbers is 8, the number power
 3 
with highest index is the greatest number. Clearly ‘C’
 728 
3x  = 6552 has the lowest value.
 27  Consider A = 888
8

3x = 243 = 35
and B = 8888 .
⇒ x = 5
Consider the indices is 888
∴ x2 = 52 = 25 Choice (B)
and 888
1 0. Given xyz = 1
1 1 (88)8 and (811)8
⇒ qxy = , =z -- (1) Since 811 > 88
z xy
888 > 888
Given expression, ∴ B > A
1 1 1 Also, among the four powers the greatest power is
−1
+ −1
+
1+ x + y 1+ y + z 1 + z + x −1 8
88 . Hence D is the largest number.

y 1 1 ∴ the ascending order is CABD. Choice (B)
= + +
y + xy + 1 1 + y + xy 1 + + 1 (from (1))
1
14. x + x − 1− x = 1
xy x
Squaring both sides we get, x − 1 − x = 1 + x − 2 x
y 1 xy
= + + Squaring again, we get, 1 – x = 1 + 4x – 4 x
y + xy + 1 1 + y + xy xy + 1 + y
16x = 25x2
y + 1 + xy 16
= = 1 Choice (A) x=  Choice (B)
1 + xy + y 25
1.52 | Quantitative Ability Test 3

15. Let the other surd be “a”.  1x + 1y + 1z  1 1 1


1
a + 1 + 12 2 =
x
log 1k = x+ y+z
kx  
= 10 + 18 2
a=9+6 2 1 1 1 1
Similarly log x xyz = + + and 1 log xyz
( )
2
= 9 + 2 18 = 6+ 3 y x y z z
y

∴ a= ( 6+ 3  ) Choice (C) =
1 1 1
+ +
x y z

16.
1
=
( 6 + 7 + 13 ) (1) Hence the given expression is equal to
6 + 7 − 13 ( 6+ 7+ 13 ) ( 6 + 7 − 13 ) 1 1 1
3 + +  = 3 
 xy + yz + zx 
 
6 + 7 + 13 6 + 7 + 13 x y z xyz
 Choice (A)
= =
( ) − ( 13)
2 2
6+ 7 13 + 2 42 − 13
 8
19. Given log4 3 + log4  3m −  = 2 log 4 (3m − 2)
6 + 7 + 13  3
=
2 42  8
( )
2
⇒ 3  3m −  = 3m − 2
1
=
(
1 6 − 7 + 13 )  3
⇒ 3m + 1 − 8 = 32m + 4 − 4 (3m)
6 − 7 − 13 ( 6 − 7 − 13 )( 6 − 7 + 13 ) ⇒ 32m − 7 (3m) + 12 = 0
6 − 7 + 13 6 − 7 + 13 Let 3m = x
= = ⇒ x2 − 7x + 12 = 0
( ) − ( 13)
2 2
6− 7 13 − 2 42 − 13 x2 − 4x − 3x + 12 = 0
x (x − 4) − 3 (x − 4) = 0

=
− ( 6− 7 + 13 ) ⇒ (x − 4) (x − 3) = 0
⇒ x = 4 or 3
2 42 ⇒ 3m = 4 or 3m = 3
⇒ m = log3 4 (or) m = 1
Required value =
6 + 7 + 13
+
− ( 6 − 7 + 13 ) Hence m can take two values. Choice (B)
2 42 2 42 20. Given logx 162 = m
2 7 1
= = ⇒ log x 34 (2) = m
 2 42 6 Choice (B) ∴ m = 4 log x 3 + log x 2
1 1 Given logx 72 = n
17. The given function is 1 + + +
2 +1 3+ 2 ⇒ log x 32 23 = n
2 −1 3− 2 4 − 3 ∴ n = 2 log x 3 + 3log x 2
= 1+ + + + ....+
Let logx3 = 1 and logx2 = b
2 −1 3−2 4−3
⇒ m = 4a + b --- (1)
(on rationalizing the denominator of each term)
  n = 2a + 3b --- (2)
= 1 + 2 – 1 + 3 – 2 + 4 – 3 + .... + 2 (2) − (1) gives
324 − 323 2n − m
5b = 2n − m ⇒ b =
5
= 324 = 18   (∵ all terms cancel off except 324 )
3m − n
Hence, the square root of the given expression is 18 similarly a =
10
= 3 2 . Choice (A) Now consider logx 7776
18. Let xy = yz = zx = k
1 1 1
= log x 35.25
⇒ x = k , y = k , z = k
y z x
= 5[logx3 + logx2]
1  3m − n 2n − m 
consider log z xyz = 5 +
x  10 5 
1  1. 1 1   m + 3n  m + 3n
= log 1  k y k z .k x 
x kx  
= 5 = 2
  10  Choice (C)
Quantitative Ability Test 3 | 1.53

1 1 1
21. log3 x2 − log3x x 26. log a a + log a a + log a a + …. + log a a
2 3 20
= 8 logx3
x2 log a log a log a log a
log = 8log x 3 = + + + ..... +
⇒ 3 log a 1 1 1
x x log a 2 log a 3 log a 20
⇒ log3 x = 8log x 3 20 × 21
= 1 + 2 + 3 + ….+ 20 = = 210  Choice (B)
1 8 2
⇒ log3 x =
2 log3 x log x log y log z
27. Let = = =k
⇒ (log3 x ) = 16
2 y−z z−x x− y
⇒ log3x = 4 logx = k(y − z)
1 logy = k(z − x)
⇒ x = 34 = 81 or x = 3−4 =

81
Choice (A) logz = k(x − y)
2
+ yz + z 2 + z
1 log x y = (y2 + yz + z2 + z) (log x)
2 2. log 4 31 = log 22 31 = log 2 31
2 = k ( y − z ) ( y 2 + yz + z 2 + z )
2 < 31 < 2
4 5
= k ( y3 − z3 ) + k ( y − z) z  --- (1)
⇒ log 2 24 < log 2 31 < log 2 25
= k ( z − x ) ( z + xz + x + z )
2 2
2 2
+ xz + x + z
log y z
⇒ 4 log 2 2 < log 2 31 < 5 log 2 2
4 1 5 = k ( z 3 − x3 ) + k ( z − x) z  --- (2)
⇒ < log 2 31 <
= k ( x − y ) ( x 2 + xy + y 2 + z )
2 2
+ xy + y + 2
2 2 2 log z x
1 = k(x3 − y3) + k (x y) z  --- (3)
⇒ 2 < log 2 31 < 2.5. Choice (B)
2 Adding (1), (2) and (3),
2 2 z 2 + xz + x 2 + z x 2 + xy + y 2 + z
23. log (2x + 3) − 1 = logx log x y − yz + z + z + log y + log z
= k(y3 – z3) + kz (y – z) + k(z3 – x3) + kz(z – x) + k(x3 – y3)
log (2x + 3) − log10 = logx + kz(x – y)
 2 x + 3 = k(y3 – z3 + z3 – x3 + x3 – y3) + kz(y – z + z – x + x – y)
⇒ log  = log x
 10  = 0 + 0 = 0 Choice (B)
2x + 3 1 1 1
⇒ =x 2 8. + +
10 log bc a log ac b log ab c 3
3 3

⇒ 2x + 3 = 10x 1 1 1
3 = + +
⇒ x = log 1 a3 log 1 b3 log 1 c 3
8 a b c
 Choice (D)
 1 1 1
24. a2 + 4b2 = 12ab; adding 4ab to both sides of the equa- ∵bc = a , ac = b and ab = c 
tion, we get (a + 2b)2 = 16ab = 1 1 1
2 log (a + 2b) = 4 log 2 + log a + log b + +
1 log a-1 a3 log b-1 b3 log c-1 c 3
log (a + 2b) = 1 1 1
2
= −3 + −3 + −3 = −1  Choice (C)
[log a + log b + 4 log 2] Choice (C)

log m p.log n p log10 27 3log10 3


25. 29. t = log6 27 = =
log m p + log n p log10 6 log10 2 + log10 3
1 1 3log 3 3log 2
= = Now 3 – t = 3 − =
log m p + log n p 1 1 log 2 + log 3 log 2 + log 3
+
log m p.log n p log n p log m p 3log 3 3log 2 + 6 log 3
3 + t = 3+ =
1 1 log 2 + log 3 log 2 + log 3
= =
log p n + log p m log p mn (3log 2)
= logmnp Choice (D)
3− t
=
(log 2 + log 3)
3 + t (3log 2 + 6 log 3)
(log 2 + log 3)
1.54 | Quantitative Ability Test 3

log 2 = log 512 (log16 – logy)


= = log18 2
log 2 + 2 log 3 becomes

∴ 2
(3- t ) = 2 log 2 = log18 4
12 log 2 (X – 3log3) = 9log2(4log2 – Y)
18 4(X – 3log3) = 3(4log2 – Y)  –––––––– (1)
 3+ t Choice (B)
and (log 16) (log 27) = (logx) (logy) becomes 12
 a  b log2log3 = XY ––––––– (2)
30. Given: log a   + log b  
 b  a eliminating Y using (1) and (2) we have
= log a a − log a b + log b b − log b a = 2 − (logab +logba) 4(X – 3log3) = 3(4log2 – 12(log2) (log3) ÷ X)
1
But logab + logb a is in the form of x + which is ≥ 2 4X (X – 3log3) = 3(4X log2 – 12 log2.log3)
x
∴ logab + logba ≥ 2 4X2 – 12X log3 = 12X log2 – 36 log2.log3
∴ The given expression can not be positive X2 – 3X log3 – 3X log2 + 9 log2log3 = 0
∴ It can not be 1 Choice (B) (X – 3log3) (X – 3log2) = 0
 ( x 2 + x)  ⇒ X = log27 or X = log8
31. log  =2
 ( x + 1)  i.e., logx = log27 or logx = log8
4

( x2 + x) ⇒ x = 27 or 8
= 42 when x = 27 then y = 16 and x – y = 11 and when
( x + 1)
x = 8, then y = 81 and x – y = – 73
⇒ x2 + x = 16x + 16
 Choice (D)
= x² – 15x – 16 = 0
x² – 16x + x – 16 = 0
x (x – 16) + 1(x – 16) = 0 34. Let 512 512 512 512........... ∝ = x
⇒ (x + 1) (x − 16) = 0
x2 = 512 512 512 512........ ∝
x = –1 or 16
x = 16 = 4 x2 = 512x, ⇒ x = 0 or 512.
 Choice (B)
As x is clearly not zero, x = 512.
32. Let k = (243) = 3
50 250

Hence the required quantity is log 64 512.


Taking log on both sides, we get
log k = 250 log3 3
= log82 8 = = 1.5 
3
Choice (C)
= 250 (0.4771) 2
100 3 5. logbca = 1/p ⇒ p = logabc ⇒ p + 1 = logaabc
= 25(4.771) = (4.771) = 119.4 Similarly logcab = 1/q ⇒ q + 1 = logbabc
4
The characteristic of log k is 119. and logabc = 1/r ⇒ r + 1 = logcabc
Number of digits in (243)50 are 119 + 1 = 120 1 1 1
∴ + + .= logabca + logabcb + logabcc
 Choice (C) p +1 q +1 r +1
33. Given (log16) (log 27) = (log x) (log (y)
= logabcabc = 1 Choice (A)
let logx = X and logy = Y then (log 4096) (logx – log27)
Quantitative Ability Test 4
(Data Interpretation)
Number of Questions: 30 Section Marks: 30

Directions for questions 1 to 4: These questions are based Directions for questions 5 to 8: These questions are based
on the information given below. on the following line graph.
Vinod Melkote, HR Manager of XYZ Ltd. received partial Profit percentages of companies X, Y and Z from
information from his subordinates about the employees for 2009-10 to 2013-14
annual appraisal. 50%
40%
Performance
Total 30%
Below Above
Average
Average Average 20%

Males 96 10%

Females 30 0%
2009-10 2010-11 2011-12 2012-13 2013-14
Total 48 X Y Z

Vinod also received the following information. Note: Profit = Income - Expenditure.; Profit % =
(1) One-third of the total employees are above average Profit
performers. Expenditure × 100
(2) 25% of the males are below average performers.
5. If the profit of X in 2010-11 is `120 lakhs, then what is
(3) The total number of females is twice the number
its income in 2010-11? (in `lakhs)
of average performing males.
(A) 520 (B) 200
1. How many people are above average performers? (C) 320 (D) 140
(A) 48 (B) 60
6. If the income of Y in 2013-14 is thrice that of X in
(C) 66 (D) 54
2009-10, then what is the ratio of profits of X and Y in
2. How many females are there? 2013-14?
(A) 54 (B) 72 (A) 12 : 1 (B) 12 : 13
(C) 90 (D) 84 (C) 1 : 13 (D) None of these
3. What is the difference between the total number of 7. If in 2009-10, the ratio of profits of X, Y and Z is
average performers and that of males? 4 : 2 : 3, then what is the ratio of their expenditures
(A) 18 (B) 12 in that year?
(C) 24 (D) 20 (A) 3 : 6 : 4 (B) 2 : 2 : 1
4. What is the ratio of the number of average performers (C) 3 : 2 : 4 (D) None of these
and the number of females? 8. If both Y and Z had the same profit in 2011-12, then
(A) 6 : 7 (B) 11 : 12 what is the ratio of their incomes in that year?
(C) 3 : 4 (D) 2 : 3 (A) 4 : 3 (B) 1 : 1
(C) 26 : 21 (D) 2 : 3

Directions for questions 9 to 13: These questions are based on the following bar graph.
Trade volumes with India by four countries from 2010 through 2014
2756

(`.00' Crores)
2651
2583

3000
2476
2359
2349

2340
2210

2147

2069
2055

2500
1958

1936
1871

1836
1604

2000
1458
1417

1349
1164

1500
1000
500
0
USA JAPAN NETHERLANDS GERMANY
2010 2011 2012 2013 2014
1.56 | Quantitative Ability Test 4

9. From the year 2010 to 2011, the trade with India by (A) 2010 (B) 2011
which of the following countries showed the highest (C) 2012 (D) 2014
percentage increase? 12. In how many of the given years is the trade volume of
(A) USA (B) Japan Netherlands with India greater than the average trade
(C) U.K (D) Germany volume per year by the Netherlands in India in the
10. In the year 2013, the trade volume by the given coun- given period?
tries constitutes 62.5% of the total trade volume of (A) 1 (B) 2
India. What is the trade volume (in `’00 crores) by all (C) 3 (D) 4
other countries with India in that year? 1 3. For which country is the percentage increase in the
(A) 4590 (B) 5430 trade volume with India in any year when compared to
(C) 6140 (D) 7250 that in the previous year, the greatest?
11. During which year is the total trade volume by the (A) USA (B) Japan
given countries with India the highest? (C) UK (D) Germany

Directions for questions 14 to 18: These questions are based on the following pie-charts which show the percentage
distribution of births in different areas, during the two years 2013 and 2014.
2013 2014
Semi-Urban Urban Urban
Semi-urban 22%
18% 18% 18%

Rural Tribal
22% Metropolitan 10%
Metropolitan
30%
36% Tribal Rural
6% 20%

Total number of births = 3.6 lakh Total number of births = 3.0 lakh

14. What is the difference in the number of births in the Directions for questions 19 to 20: Select the correct alter-
year 2013 and 2014 in rural areas? native from the given Choices.
(A) 16,400 (B) 19,200 19. A total of 600 persons participated in a survey. Each
(C) 18,600 (D) 17,600 respondent was asked whether he/she owned a vehicle.
15. The percentage increase/decrease in the number of Any respondent owning a vehicle was asked to mention
births in semi-urban areas from 2013 to 2014 is _____. whether he/she owned a two-wheeler or four wheeler or
(A) 16.66% decrease both.
(B) 20% increase The results of the survey are tabulated below.
(C) 16.66% increase
men Women
(D) 20% decrease
16. What is the ratio of the number of births in the metro- Number of Only Four-wheeler 80 68
persons
politans in 2013 to that in 2014? Only Two-wheeler 60 40
having own
(A) 36 : 25 (B) 15 : 17 vehicle Both 120 92
(C) 3 : 7 (D) 7 : 3
Number of persons not owning
17. The number of births in tribal areas in the year 2013 is 40 100
a vehicle
what percentage of that in 2014?
(A) 56% (B) 64% What percent of the respondents do not own a four
(C) 72% (D) 84% wheeler?
18. If 18% of the children born in 2013 and 16% of the (A) 60% (B) 40%
children born in 2014 lacked good medical facilities in (C) 30% (D) 80%
the same year, then what is the total number of children 2 0. The monthly rainfall chart for a certain city was pre-
(in lakhs) who doesn’t lacked good medical facilities in pared, based on 40 years of data. The graph below
these two years? shows the x (x percentile means that for x% of the
(A) 6.329 (B) 5.472 40 years, the rainfall was less than the indicated
(C) 4.289 (D) 5.689 value.
Quantitative Ability Test 4  |  1.57

(A) 9000 (B) 9600


700 (C) 9500 (D) 9800
600 22. Which of the following numbers is the least?
(A) 100 cc bikes
500
Rain fall (in mm)

(B) 150 cc bikes


400
(C) 100 cc scooters with gear
300 (D) 150 cc scooters without gear
200 Directions for questions 23 and 24: These questions are
100 based on the following table which gives the percentage by
0
weight of proteins, carbohydrates, minerals and fats in four
Jan Feb Mar Apr May Jun Jul Aug Sep Oct Nov Dec formulations – A, B, C and D.
Ave ra ge 10 pe rce ntile 90 pe rce ntile
Formulation Proteins Carbohydrates Minerals Fats
A 10 20 25 45
Which of following statements can be concluded?
(i) The average rainfall in December is less than that B 25 35 20 20
in June. C 30 10 40 20
(ii) Each year, the amount of rainfall in July is more
D 15 50 30 5
than that in February.
(iii) In May, there is at least 250mm of rainfall each
Proteins Carbohydrates Minerals Fats
year.
(iv) The rainfall in August can be estimated with better Cost (in `)
4 3 2 1
per 10 g
certainity than the rainfall in November.
(A) (i) and (iii) (B) (i) and (iv) 23. What is the cost of a mixture containing 100 g each of
(C) (ii) and (iv) (D) (ii) and (iii) A, B, C and D?
Directions for questions 21 and 22: Study the following (A) `96.5 (B) `98
table and pie chart carefully to the answer the questions. (C) `98.5 (D) `99
Percentage of two wheelers of the total vehicles man- 24. Which of the following would cost the least?
ufactured in a year by a certain company. (A) 400 g of B
(B) 400 g of C
Out of which
Type
Percentage of (C) 200 g of A and 200 g of D
total vehicles 100 cc 150 cc (D) 300 g of A and 150 g of D
Scooters
Directions for question 25: Select the correct alternative
25% 70% 30% from the given choices.
(without gear)

Scooter with
25. The table below shows the test batting averages of 5
35% 40% 60% cricket players from 2010 to 2014. The test batting
gear
average of any batsman in any number of matches is
Bikes 40% 65% 35% his total score in those matches divided by the number
of those matches.
A Year P Q R S T
C 40%
40% 2010 44 46 41 42 19
2011 55 52 44 48 22
2012 50 55 36 52 28

B 2013 46 51 40 46 34
20% 2014 48 52 35 40 39

The two wheelers are manufactured in three models as Who among the five players had the least percent-
shown in the above pie chart. age increase in the test batting average from 2010 to
Note: The above percentages mentioned in the table are 2014?
applicable for all models. The total number of vehicles (A) P (B) Q
produced in that year is 70,000. (C) T (D) S
2 1. What is the total number of 150 cc bikes produced by Directions for questions 26 and 27: These questions are
the company in that year? based on the data given below.
1.58 | Quantitative Ability Test 4

A survey was conducted among 100 students in a hostel all the units produced by the companies in month M.
to find their favourite breakfast dish. Five students liked Percentage of Number of
cutlet and sandwich only. 21 students liked omlette only. Company
accepted units rejected units
25 students liked sandwich and 30 liked cutlet. There are 3 C1 80% 32
students who liked both sandwich and omlette. 48 students
liked exactly one dish among the three. 2 students liked all C2 90% 29
the three dishes. C3 85% 27
26. How many students like at least one dish? C4 81% 38
(A) 69 (B) 100 C5 76% 36
(C) 2 (D) 21
C6 77% 46
27. How many students like cutlet or sandwich but not
both? C7 84% 20
(A) 48 (B) 41 C8 88% 21
(C) 31 (D) 62 C9 93% 28
Directions for question 28: Select the correct alternative
C10 95% 30
from the given choices.
28. Ten companies produce the same tool. Each of those The company which produced the greatest total num-
companies rejects all the defective units of the tool ber of units in month M is
produced. The table below gives the percentage of (A) C10 (B) C9
accepted units and the number of rejected units among (C) C2 (D) C4

Directions for questions 29 and 30: These questions are based on the pie charts given below. Pie chart – 1 represents the
distribution of income of Manohar and pie chart – 2 represents the split up of expenditure on education.
Pie-Chart-1 Pie-Chart-2

College Fee
Food 30%
Education Tution Fee
30%
40% 40%

Savings
5%

Petrol Magazines Books


Rent
15% 10% 5% 25%

Note: Total income of Manohar is `15000 (C) The tuition fee is `2,460.
2 9. Which of the following statement/s is/are true? (D) Both (B) and (C).
(A) The expenditure on food is `2800 more than the 3 0. The expenditure on books is how much more/less than
expenditure towards college fee. the expenditure on food?
(B) The expenditure on education is `2,250 less than (A) `750 more (B) `750 less
the remaining expenditure. (C) `3,000 more (D) `3,000 less

Answer Keys
1. B 2. D 3. C 4. A 5. A 6. D 7. B 8. C 9. D 10. A
11. D 12. C 13. D 4. B 15. A 16. A 17. C 18. B 19. B 20. B
21. D 22. D 23. C 24. C 25. A 26. A 27. B 28. A 29. B 30. D
Quantitative Ability Test 4  |  1.59

Hints and Explanations


1. Given that 25% of the males are below average per- Let the number of male average performers be x. Then,
formers. Hence their number is 25% of 96 = 24. we get the following table.
As the total number of below average performers is 48 Below Above
and 24 of them are males, remaining 24 are females. Average Total
Average Average
Given, number of females = 2 (Number of male aver-
Males 24 x 72 - x 96
age performers)
Let the number of male average performers be x. Then, Females 24 30 2x - 54 2x
we get the following table. Total 48 30 + x x + 18 2x + 96
Below Above 1
Average Total
Average Average Given, above average performers = (total number of
3
Males 24 x 72 - x 96 employees)
1
Females 24 30 2x - 54 2x i.e., x + 18 = (2x + 96)
3
Total 48 30 + x x + 18 2x + 96 ⇒ 3x + 54 = 2x + 96
1 ⇒ x = 42
Given, above average performers = (total number of
3 Total number of average performers = x + 30 i.e., 72
employees) Also as the number of males = 96,
1
i.e., x + 18 = (2x + 96) the required difference is 96 - 72 = 24. Choice (C)
3
4. Given that 25% of the males are below average per-
⇒ 3x + 54 = 2x + 96 ⇒ x = 42
formers. Hence their number is 25% of 96 = 24.
Number of above average performers = x + 18 As the total number of below average performers is 48
i.e., 60. Choice (B) and 24 of them are males, remaining 24 are females.
2. Given that 25% of the males are below average per- Given, number of females = 2 (Number of male aver-
formers. Hence their number is 25% of 96 = 24. age performers)
As the total number of below average performers is 48 Let the number of male average performers be x. Then,
and 24 of them are males, remaining 24 are females. we get the following table.
Given, number of females = 2 (Number of male aver- Below Above
age performers) Average Total
Average Average
Let the number of male average performers be x. Then, Males 24 x 72 - x 96
we get the following table. Females 24 30 2x - 54 2x
Below Above Total 48 30 + x x + 18 2x + 96
Average Total
Average Average
1
Males 24 x 72 - x 96 Given, above average performers = (total number of
3
employees)
Females 24 30 2x - 54 2x
1
Total 48 30 + x x + 18 2x + 96 i.e., x + 18 = (2x + 96)
3
1 ⇒ 3x + 54 = 2x + 96
Given, above average performers = (total number of ⇒ x = 42
3
employees) Required ratio of above average performers : Number
1 of females = x + 30 : 2x i.e., 72 : 84 = 6 : 7.Choice (A)
i.e., x + 18 = (2x + 96)
3 5. Given profit of X in 2010-11 = `120 lakhs
⇒ 3x + 54 = 2x + 96 Profit percentage of X in 2010-11 = 30%
⇒ x = 42 Profit
Number of females = 2x As profit % = × 100
Expenditure
i.e., 2(42) = 84. Choice (D)
120 lakhs
3. Given that 25% of the males are below average per- 30 = × 100 = `400 lakhs.
expenditure
formers. Hence their number is 25% of 96 = 24.
As the total number of below average performers is 48 ∴ Expenditure of X in 2010–11 is `400 lakhs
and 24 of them are males, remaining 24 are females. Hence, its income in 2010–11 = `520 lakhs.Choice (A)
Given, number of females = 2 (Number of male aver- 6. As the income of X in 2013–14 is not known, the
age performers) required ratio cannot be determined. Choice (D)
1.60 | Quantitative Ability Test 4

7. Given, 20% of X; 10% of Y : 30% of Z = 4 : 2 : 3 15. The number of births in semi-urban area in 2013 = 18%
i.e., X : Y : Z = 20 : 20 : 10 = 2 : 2 : 1. Choice (B) of 3.6 lakh = 64,800
8. Let the profit of Y and R in 2010-11 be `x. Then, using The number of births in semi-urban area in 2014 = 18%
x x of 3 lakh = 54,000
the graph, we have = 30 and = 40 Percentage decrease
exp Q exp R
64,800 − 54, 000
Let expQ = 400x and expR = 300x = × 100 = 16.66%. Choice (A)
64,800
∴ 30(expQ) = 40(expR)
⇒ Expenditures of X and Y are in the ratio 4 : 3. 16. The number of births in metropolitan area in 2013 =
Incomes ratio = 520x : 420x = 26 : 21. Choice (C) 36% of 3.6 lakh = 1,29,600
The number of births in metropolitan area in 2014 =
9. The percentage increase in the trade volume from 2010
30% of 3 lakh = 90,000
to 2011
354 Required ratio = 129600 : 90000 = 36 : 25.Choice (A)
for USA = × 100 < 25% 17. The number of births in tribal area in 2013 = 6% of 3.6
1604
711 lakh = 21,600
for Japan = × 100 < 40% The number of births in tribal area in 2014 = 10% of 3
1871
lakh = 30,000
311 21600
for UK = × 100 < 20% ∴ Required percentage = × 100 = 72%.
1836 30000
652  Choice (C)
for Germany = × 100 > 40% 18. The number of children lacking good medical facilities
1417
in 2013 = 82% of 3.6 lakh = 2,95,200
The percentage increase in trade volume is the greatest
The number of children lacking good medical facilities
for Germany. Choice (D)
in 2014 = 84% of 3 lakh = 2,52,000
10. Total trade volume by the given countries in 2013 \ Total number of children who does not lack good
= 2210 + 1936 + 2340 + 1164 = 7650 medical facilities = 2,95,200 + 2,52,000
Given that 62.5% of the total trade volume = 7650 = 5,47,200 = 5.472 lakhs. Choice (B)
37.5
Then, 37.5% of the total trade volume = × 7650 19. A total of 80 + 68 + 120 + 92, viz., 360 persons own a
62.5 four wheeler.
3
= × 7650 = `459,000 Crores. Choice (A) ∴ The remaining 60 + 40 + 40 + 100, viz., 240 of the
5
respondents do not own a four wheeler, i.e., 40%
11. By observation, we find that the total trade volume is
of the respondents do not own a four-wheeler.
the least in 2010 and 2013.
 Choice (B)
By comparing the total trade volume in 2011 and that
in 2012, we find that the trade volume in 2012 is lower. 20. (i) The average rainfall in December is 150mm. The
Now, by comparing the total trade volume in 2011 and average rainfall in June is between 400mm and
that in 2014 we find that the total trade volume in 2014 500mm. ∴ (i) is true.
is the highest. Choice (D) (ii) For each month, we know the average rainfall, the
10 percentile value (i.e., the value below which 10
12. We can observe that the trade volume by the Netherlands
percent of the data falls) and the 90 percentile val-
in India is less than 2000 in one year and more than that
ue. In a particular year, the rainfall in July may be
in four years. By assuming the average to be 2000, we
less than that in February. (ii) cannot be concluded
find the average of the deviations
(iii) This statement does not follow (From the explana-
−164 + 147 + 476 + 340 + 651 1450
= = = 290 tion in (ii) above)
5 5 (iv) We see a narrow gap between any two of the three
The average = 2290. Three values are more than the curves in August and a significant gap between
average. Choice (C) any two of the three curves in November. ∴ (iv)
13. The trade volume from Germany registered more than follows. Only (i) and (iv) follow.
40% growth. By observation, we find that for no other  Choice (B)
country there is more than 40% growth in the trade vol- Solutions for questions 21 and 22:
ume in any two successive years. Choice (D) Total number of vehicles produced = 70,000
14. No. of births in rural area in 2013 = 22% of 3.6 lakh Scooters without gear = 25% of 70,000.
= 79,200 25
No. of births in rural area in 2014 = 20% of 3 lakh = 70,000 × = 17,500
100
= 60,000
Difference = 79,200 − 60,000 = 19,200. Choice (B) 100 cc scooters without gear = 17,500 × 70 = 12,250
100
Quantitative Ability Test 4  |  1.61

150 cc scooters without gear = 17,500 - 12,250 = 5,250 100


Number of scooters with gear = 35% of 70,000
C S
35 5 = 25
i.e., 70,000 × = 24,500 10 17
100 40 30
100 cc scooters with gear = 24500 × = 9,800 2
100 13 1

60 21
150 cc scoters with gear = 24,500 × = 14,700 O
100
40
Number of bikes = 70,000 × = 28,000
100 (i) Only cutlet = 10
65 (ii) Only omlette = 21
100 cc bikes = 28,000 × = 18,200 (iii) Only sandwich = 17
100
(iv) All the three = 2
35
150 cc bikes = 28,000 × = 9,800 (v) Exactly two dishes = 13 + 5 + 1 = 19
100 (vi) At least two dishes = 19 + 2 = 21
21. Number of 150 cc bikes produced in that year = 9,800. (vii) Cutlet = 10 + 5 + 2 + 13 = 30
 Choice (D) (viii) Sandwich = 5 + 2 + 1 + 17 = 25
22. Of the four options the number of 150 cc scooters with- (ix) Omlette = 13 + 2 + 1 + 21 = 37
out gear is the least in number i.e., 5,250. Choice (D) (x) None = 100 - (10 + 17 + 21 + 5 + 13 + 1 + 2) = 31
Solutions for questions 23 and 24: 26. The number of students who like at least one dish =
The cost of 100 g of A = 4 (1) + 3 (2) + (in `) 2 (2.5) + sum of the elements = 69. Choice (A)
(4.5) = 19.5 27. The number of students who like cutlet or sandwich but
100 g of B (in `) = 26.5 not both is given by 10 + 13 + 17 + 1 = 41. Choice (B)
100 g of C (in `) = 25 Solutions for question 28:
100 g of D (in `) = 27.5
28. The number of units produced by C1, C2, C3 …….C10
23. The cost of the required mixture = 19.5 + 26.5 + 25 32 × 100 29 × 100 27 × 100 38 × 100 36 × 100
+ 27.5 = `98.5. Choice (C) Are , , , ,
100 − 80 100 − 90 100 − 85 100 − 81 100 − 76
24. Cost of 400 g of B = `106 46 × 100 20 × 100 21 × 100 28 × 100
, , , ,and
400 g of C = `100 100 − 77 100 − 84 100 − 88 100 − 93
200 g of A and 200 g of D = `94 30 × 100
300 g of A and 150 g of D = `99.75. Choice (C) 100 − 95
25. Among the five players, only P, Q and T had an Respectively i.e.160, 290, 180, 200, 150, 200, 125,
increase in the test batting average from 2010 to 2014. 175, 400 and 600
Percentage increase in the test batting average from ∴ C10 produced the greatest total number of units in
2010 to 2014 of month M. Choice (A)
48 − 44
P= × 100% Solutions for questions 29 and 30:
44
29. Statement A is not true as (30% – 40% of 40%) of
100% 1
= =9 % 15000 ≠ 2800
  11 11
Statement B is true, since the expenditure on
52 − 46
Q= × 100%  40 
education =  (15000) = 6000.
46  100 
300 1 (15 + 10 + 30)
  = % = 13 % The remaining expenditure = (15000) =
23 23 8250. 100
39 − 19 20 ∴ Difference is `2250. Choice (B)
T= × 100% = × 100% which is > 100%.
19 19  40  25 
30. Expenditure on books = 15000  = 1500.
P had the least percentage increase in the test batting  100   100 
average. Choice (A)
 30 
The expenditure on food =  (15000) = 4500.
Solutions for questions 26 and 27:  100 
From the given data we get the following diagram. Expenditure on books is less than the expenditure on
food by 4500 - 1500 = `3000. Choice (D)
Quantitative Ability Test 5
(Permutations and Combinations)
Number of Questions: 35 Section Marks: 30

Directions for questions 1 to 35: Select the correct alterna- 9. In how many arrangements of the word
tive from the given choices. MATHEMATICS, the two A’s are separated?
1. In how many ways can five boys and five girls be 10! 9!
(A) (B)
arranged around a circular table such that on either side 2!2!2! 2!2!2!
of every boy, a girl must sit?
9 × 10 !
(A) 2560 (B) 2880 (C) 9 × 10 ! (D)
(C) 120 (D) 1440 2!2!2!

2. Twelve boys have to be seated in a row such that two 10. Find the number of ways of arranging the letters of the
particular boys occupy the middle two positions. In word CALENDAR in such a way that exactly two let-
how many ways can they be seated? ters are present in between L and D?
12! (A) 2640 (B) 3600
(A) (B) 10! 2! (C) 2600 (D) 7200
2!
10!
(C) 12! (D) 11. There are 12 men and 7 women. In how many ways can
2! a team of six members be formed such that there are
3. How many five letter words can be formed using the atmost two women?
letters of the word “QUESTION” so that the word con- (A) 16,683 (B) 16,386
tains 2 vowels and 3 consonants? (C) 16,863 (D) 16,638
(A) 5670 (B) 120 12. Three bags X, Y and Z contain six, five and four mar-
(C) 2880 (D) 1440 bles respectively. A person has to choose 11 marbles at
4. There are seven pairs of shoes. In how many ways can random. In how many ways can this be done such that
one select 4 shoes from them such that no complete at least 3 marbles are to be chosen from each bag.
pair is included among them? (A) 860 (B) 840
(A) 840 (B) 420 (C) 960 (D) 870
(C) 13440 (D) 560 13. In a factory there are three class I employees, two
5. There are 4 apples, 3 oranges and 6 mangoes in a bas- class II employees, three class III employees and four
ket. In how many ways can one select one or more class IV employees. A team of five members is to be
fruits from the basket? formed. In how many ways can this be done if the team
(A) 139 (B) 140 must have at least one class I and at most two class IV
(C) 71 (D) 72 employees?
6. There are 10 questions in a paper each with four (A) 590 (B) 491
options, of which only one is correct. In how many (C) 600 (D) 591
ways can a student get exactly 7 questions correct given 14. There are two groups X and Y in a colony. X consists of
that he attempted all the questions? five boys and four girls, Y consists of four boys and five
(A) 2400 (B) 2880 girls. They plan an educational tour of four boys and
(C) 3240 (D) 3600 four girls such that exactly four persons are selected
7. A number is formed using the digits 5, 8, 1, 4 and 3. from each of the two groups. In how many ways can
When we arrange the numbers in ascending order find this be done?
the rank of the number 58413. (Each digit occurs at (A) 5626 (B) 5026
most once in each number) (C) 15876 (D) 43758
(A) 228 (B) 290 15. There are 20 players and 6 of them are from Hyderabad.
(C) 299 (D) 300 In how many ways can a team of 12 players be formed
8. A team of 11 is to be selected from two groups A and B, so that exactly three persons of the team are from
which consist of 10 and 8 persons respectively. In how Hyderabad?
many ways can this team be selected such that exactly (A) 4004 (B) 20020
five members are selected from the first eight persons (C) 40040 (D) None of these
of group A? 16. In how many ways can five men and five women be
(A) 11,760 (B) 10,760 seated in a row, so that all men are sitting together and
(C) 12,760 (D) 11,670 all women are sitting together?
Quantitative Ability Test 5 | 1.63

(A) (5!)2 (B) (5!)22! 26. The number of ways of posting seven different letters
into three post boxes so that at least one letter is posted
(5!)2

(C) (D) (5!)24! in each post box is


2! (A) 1806 (B) 1803
17. For a company board meeting, eight directors and a (C) 2184 (D) 2059
chairperson have to be seated around a circle. If two
particular directors are seated on either sides of the 27. A palindrome is a word, which spells the same when
chairperson, in how many ways can they be seated? read from left to right or from right to left. How many
(A) 1440 (B) 1200 palindromes of length 8 can be formed using the opera-
(C) 10080 (D) 1080 tive symbols +, –, ÷ and ×?
(A) 8! (B) 216
18. In how many ways can seven persons be selected from (C) 256 (D) 243
14 persons such that two particular persons are selected
and three other particular persons are not selected? 28. Six identical balls have to be placed in the square cells
(A) 120 (B) 126 of the given figure such that each row contains at least
(C) 300 (D) 240 one ball. In how many ways can this be done? (Given
that each square can take at the most one ball)
19. There are six different consonants and three different
vowels of the English alphabet. The number of words
that can be formed using them such that each word con-
tains two vowels and three consonants is
(A) 3600 (B) 1800
(C) 2400 (D) 7200 (A) 84 (B) 76
20. Find the number of sides of a regular polygon in which (C) 77 (D) 81
the number of diagonals is equal to one and half times 29. The number of two digit codes that can be formed using
the number of its sides. the digits 0 to 9 with ‘0’ not taking the tens place and an
(A) 5 (B) 6 odd number taking the units place is
(C) 8 (D) 10 (A) 40 (B) 50
21. Find the number of ways of arranging the letters of (C) 45 (D) 36
the word RAINBOW such that the vowels occupy odd 30. There are 5 balls of different colours and 5 boxes of
places. colours the same as those of the balls. The number of
(A) 7! (B) 720 ways in which the balls, one in each box can be placed
(C) 576 (D) 120 such that a ball does not go to a box of its own colour is
22. On Sports Day in a school, some competitions are held. (A) 40 (B) 44
Every student has to play exactly one game with every (C) 42 (D) 36
other student. It was found that in 36 games both play- 31. A question paper consists of 5 problems, each problem
ers were girls and in 126 games one player was a boy having 3 internal choices. In how many ways can a can-
and the other was a girl. Find the number of games didate attempt one or more problems?
played in which both players were boys. (A) 63 (B) 511
(A) 56 (B) 72 (C) 1023 (D) 15
(C) 91 (D) 45
32. Six points are marked on a straight line and five points
23. A box contains coins of denominations 50 paisa, `1, marked on another line which is parallel to the first line.
`2 and `5. Given that there are unlimited coins of each How many straight lines, including the first two, can be
type, find the number of ways of selecting the coins so formed with these points?
that any such selection gives a total amount of `10. (A) 29 (B) 33
(A) 49 (B) 12 (C) 55 (D) 32
(C) 48 (D) 46
33. Sixteen guests have to be seated around two circular
24. Find the number of ways in which the letters of the tables, each accommodating 8 members. 3 particular
word THURSDAY can be arranged such that no word guests desire to sit at one particular table and 4 others at
starts with T or ends with Y. the other table. The number of ways of arranging these
(A) 9360 (B) 31680 guests is
(C) 29520 (D) 30960 9!(7!)
(A) 9C5 (B)
25. How many natural numbers are there from 10000 to 4!5!
1000000 for which the sum of the digits is 3? 9!(7!) 2

(A) 16 (B) 36 (C) (D) (7!)2


4!5!
(C) 27 (D) 35
1.64 | Quantitative Ability Test 5

34. In how many ways can one or more of 5 letters be 35. The number of non negative integral solutions to the
posted into 4 mail boxes, if any letter can be posted equation a + b + c = 14 is
into any of the boxes? (A) 78 (B) 45
(A) 54 (B) 45 (C) 120 (D) 110
(C) 55 – 1 (D) 45 – 1

Answer Keys
1. B 2. B 3. C 4. D 5. A 6. C 7. D 8. A 9. D 10. B
11. C 12. D 13. D 14. A 15. C 16. B 17. A 18. B 19. D 20. B
21. C 22. C 23. A 24. D 25. B 26. A 27. C 28. D 29. C 30. B
31. C 32. D 33. C 34. C 35. C

Hints and Explanations

1. First arrange the girls in the circle as shown in the dia- 5. We can choose the apples in 5 ways i.e., either choosing
gram. They can be arranged in (5 – 1)! ways. 0 apples or 1 apple or 2 apples or 3 apples or 4 apples.
G1 Similarly the oranges can be chosen in 4 ways and the
G5 mangoes can be chosen in 7 ways.
G2
Therefore the total number of ways in which one can
select one or more fruits from 4 apples, 3 oranges and
6 mangoes = 5 × 4 × 7 – 1 = 139 ways.
G4 (we subtract 1 for the case where we select 0 apples, 0
G3 oranges and 0 mangoes i.e., no fruit at all).Choice (A)
Then there are 5 gaps and the 5 boys can be seated in 6. In each question, there are 3 wrong options and exactly
these gaps in 5! Ways. one correct option.
∴ The total number of arrangements is 4! × 5! = The number of ways in which a student gets exactly 7
2880. Choice (B) questions correct = 10C7(3)3
10
C7 gives us the number of ways in which the student
2. There are two middle positions (6th and 7th). The two
selects seven questions (in which the student marks
particular boys can occupy the middle two positions in
correct option) from the given 10 questions.
2! ways. The remaining 10 boys can be seated in 10!
33 gives us the number of ways in which the student
ways.
answers the 3 questions which he gets wrong.
∴ The total number of ways = (10!) (2!). Choice (B)
Therefore the required number of ways = 3240.
3. The word QUESTION, contains 4 vowels and 4 conso-  Choice (C)
nants; 2 vowels and 3 consonants can be selected from
7. The number of single digit numbers = 5.
4 vowels and 4 consonants in 4 C2 . 4 C3 ways. = 6 × 4 =
Given digits are {5, 8, 1, 3, 7}.
24
The number of two digit numbers formed with the
Again these 5 letters can be rearranged among them-
digits is 5P2 = 20. The number of three digit numbers
selves in 5! ways.
formed with the digits is 5P2 = 60.
∴ The total number of 5 letter words formed = 5! × 24
The number of four digit numbers formed with digits
= 120 × 24 = 2880. Choice (C)
is 5P4= 120.
4. Since there are 7 pairs of shoes and 4 shoes are to be The number of five digit numbers formed which begin
selected without any complete pair being included in it, with either 1 or 3 or 4 is 4!
the 4 shoes must be from 4 different pairs. The number of five digit numbers formed which begin
This can be done in 7C4 ways (choosing 4 pairs from with 51 or 53 or 54 is 3!
the 7 pairs) The number of 5 digit numbers which begin with 581
Now from each of these 4 pairs, one can select a shoe or 583 is 21.
in 2 ways The next number 58413
(i.e., any one of the two shoes present in each pair) The rank of the number 58413 is 5 + 20 + 60 + 120 + 3
Therefore the total number of ways in which one can (24) + 3 (6) + 2(2) + 1 = 300. Choice (D)
select 4 shoes from 7 pairs of shoes such that no com-
8. In group A, there are 10 persons.
plete pair is included among them = 7C4(2)(2)(2)(2) =
From first 8 persons 5 can be selected in 8C5 ways and
35(16) = 560 ways. Choice (D)
remaining 6 must be selected from the remaining 2
Quantitative Ability Test 5 | 1.65

persons of group A and 8 persons from group B. Class II +


This can be done in 10C6. Class I (3) Class III Class IV Number of ways
∴ The required number of ways = 8C5 10C6 = 11760. (3) (5) (4) of selecting them
 Choice (A) 1 4 0 3
C1 5C4 4C0 = 15
11! 1 3 1 3
C1 5C3 4C1 = 120
9. The total number of words that can be formed is
2!2!2! 1 2 2 3
C1 5C2 4C2 = 180
Number of arrangements in which the 2 A’s are together
2 3 0 3
C2 5C3 4C0 = 30
10!
= 2 2 1 3
C2 5C2 4C1 = 120
2!2!
Total number of arrangements in which the A’s are sep- 2 1 2 3
C2 5C1 4C2 = 90
arated = Total number of words – number of words, in 3 2 0 3
C3 5C2 4C0 = 10
which the two A’s together.
3 1 1 3
C3 5C1 4C1 = 20
10!  11  9 (10!)
 − 1 = 3 0 2 C3 5C0 4C2 = 6
3
=  Choice (D)
2!2! 2 2!2!2!
  Total = 591
10. C A L E N D AR
 Choice (D)
12345678 1 4. The following table shows the number of persons and
There are 5 positions to fix the L and D i.e., (1, 4), (2, the number of ways of selecting them as per the given
5), (3, 6), (4, 7) and (5, 8) and L and D can be inter- conditions.
changed. The remaining 6 letters can be arranged in
Number of ways of select-
6! Boys Girls ing them
ways.
2! X(5) Y(4) X(4) Y(5)
6!
∴ Required number of ways = × 5 × 2 = 360 × 10
2! 0 4 4 0  5
C04C4 4C0 5C0 (or) = 1
= 3600. Choice (B)
1 1. The following table gives the complete possibilities of 1 3 3 1 5
C1 4C3 4C3 5C1 (or) = 400
selecting the team as per the given conditions
2 2 2 2 5
C2 4C2 4C2 5C2 (or) = 3600
Men (12) Women (7) Number of ways
3 1 1 3 5
C3 4C1 4C1 5C3 (or) = 1600
4 2 12
C4 × 7C2
4 0 0 4 5
C4 4C0 4C0 5C1 (or) = 25
5 1 12
C5 × 7C1
      
Total = 5626
6 – 12
C6
 Choice (A)
Total number of ways of selecting the team
15. There are 20 players, of them 6 are from Hyderabad.
= 12C4 7C2 + 12C5 7C1 + 12C6
Exactly 3 players can be selected from 6 players in 6C3
= 10395 + 5544 + 924 = 16863. Choice (C)
ways and the remaining 9 players are to be selected
1 2. The following table will give the number of marbles from the remaining 14 players.
selected from each bag and the number of ways of This can be done in 14C9 ways.
selecting them. ∴ Total number of ways of selecting the team
X(6) Y(5) Z(4) Number of ways of selecting them = 6C3 14C9 = 40040. Choice (C)
3 4 4    6
C3 5C4 4C4 = 100 1 6. There are 5 men and 5 women.
As all men and all women are to sit together, treat all
3 5 5    6
C3 5C5 4C3 = 80
men as one unit and all women as one unit.
4 3 4    6
C4 5C3 4C4 = 150 The two units can be arranged in 2! ways.
4 4 3    6
C4 5C4 4C3 = 300 But five men and five women can arrange among them-
selves in 5! 5! ways.
5 3 3    6
C5 5C3 4C3 = 240
∴ Total number of ways they can sit is given by
Total number of ways = 870 (5!)2 2!.  Choice (B)
 Choice (D) 1 7. As two particular directors are to sit on either sides of
1 3. The following table gives the number of persons of the chairperson we treat these three as one unit. The
each category and the number of ways of selecting remaining six directors and this one unit of three per-
them as per the given conditions sons can sit around a circular table in (7 - 1)! = 6! ways.
1.66 | Quantitative Ability Test 5

But the two directors sitting on either side of the chair- If z = 0, then y = 0, 1, 2, …10; the number of solutions
person can arrange themselves in 2! ways. = 11
\The required number of arrangements possible z = 1, then y = 0, 1, 2, .. 8; the number of solutions = 9
= 6! 2! = 1440 ways. Choice (A) ---------------------------------------
18. Total there are 14 persons, of them two particular per- ---------------------------------------
sons are always selected. z = 5, then y = 0 and x = 0; the number of solutions = 1
We need to select only 5 persons from 14 – 2 = 12 per- ∴ The number of possible selections in this case
sons. As three other particular persons are not to be = 1 + 3 + 5 + 7 + 9 + 11 = 36.
selected, keeping them away there are only nine per- Similarly, we can show that, the number of selections
sons left from which we have to select any five persons. when one 5 rupee coin is selected = 12
This is possible in 9C5 = 126 ways. Choice (B) The number of selections when two 5 rupee coins are
selected = 1
19. There are 6 consonants , of them 3 can be selected in
Hence, the total number of possible selections
6
C3 ways. Similarly, 2 vowels can be selected in 3C2
= 36 + 12 + 1 = 49. Choice (A)
ways.
These 3 consonants and 2 vowels can be arranged in 24. The number of words that start with T and end with
5! Ways. Y = 6!
∴ Total number of words that can be formed Keeping T and Y fixed in the first and the last places, the
= 6C3 × 3C2 × 5! = 7200 Choice (D) remaining 6 letters can be arranged in the remaining 6
places in 6! or 720 ways.
20. The number of diagonals of a regular polygon of n
The number of words starting with T = 7!
n ( n − 3)
sides = Keeping T fixed in the first place, the remaining seven
2 letters can be arranged in the remaining seven places in
n ( n − 3) 3 7! or 5040 ways.
It is given that, = n ⇒ n = 6. Choice (B)
2 2 ∴ (5040 – 720) words start with T but do not end
21. There are seven letters in the word RAINBOW, of them with Y. Similarly (5040 – 720) words end with Y
three are vowels and four are consonants. but do not start with T. The total number of words
There are four odd places, three vowels can occupy that can be formed using all the letters in the given
four places in 4P3 ways and the remaining 4 places can word = 8!
be occupied by the remaining 4 consonants in 4! ways. Therefore, the number of words that do not start with
∴ Total number of arrangements possible in given by T or end with Y = 8! – 4320 – 4320 – 720 = 30960.
4
P3 × 4! = 576 Choice (C)  Choice (D)
22. Let m boys and n girls participate in the competitions. 2 5. It could be five–a digit or a six–digit number.
Given the number of games in which both players are Case I: It is a five digit number.
girls = 36 Digits used
No of values
(in that order)
⇒ nC2 = 36 3, 0 3–––– 1
n( n − 1) 2, 1 2––––
4
C1 = 4
⇒ = 36 ⇒ n = 9 4
C1 = 4
2 1, 2 1––––
1, 1, 1 1––––
4
C2 = 6
∴ Number of girls = n = 9
Total 15
The number of games in which one boy and one girl
participated = 126 Case II: It is a six–digit number
m
C1 nC1 = 126 Digits used No. of values
m(9) = 126 ⇒ m = 14 3, 0 3––––– 1
Number of boys, m = 14 2, 1 2–––––
5
C1 = 5
Number of games, in which both players were boys 1, 2 1–––––
5
C1 = 5
14 (13) 1, 1, 1 1–––––
5
C2 = 10
= mC2 = 14C2= = 91. Choice (C)
1(2) Total 21

23. The number of `5 coins that can be selected is 0, 1 or 2. Therefore the total number of natural numbers from
Let x, y and z represent the number of 50 paise, `1 and 10000 to 1000000 such that the sum of the digits is 3 =
`2 coins respectively to be selected. 15 + 21 = 36. Choice (B)
Case 1 :  When the number of `5 coins selected is zero, 26. The possible combinations, internal arrangements and
then total number of ways in case of each combination are
x listed below.
+ y + 2 z = 10 i.e., x + 2y + 4z = 20
2
Quantitative Ability Test 5 | 1.67

Internal  1 1 1 1
Combinations
arrangements
Number of ways = 5!  − + − 
 2! 3! 4! 5!
= 60 - 20 + 5 - 1 = 44. Choice (B)
3! 7! 31. Given that, the question paper consists of 5 problems.
1, 1, 5 =3 3 × = 126
2! 5! For each problem, one or two or three or none of the
choices can be attempted.
7! ∴ Hence, the required number of ways = 45 - 1.
1, 2, 4 3! = 6 6 × = 630 = 210 - 1= 1024 - 1 = 1023.
4! 2!
 Choice (C)
3! 7! 32. We know that, the number of straight lines that can be
1, 3, 3 =3 3× = 420 formed by the ‘n’ points in which r points are collinear
2! 3! 3!
and no other set of three points, except those that can be
3! 7! selected out of these r points are collinear) is nC2 - rC2 +1.
2, 2, 3 =3 3×3× = 630 ∴ Hence, the required number of straight lines
2! 2! 2! 3!
= 11C2 - 6C2 – 5C2 + 1 + 1
= 55 - 15 - 10 + 2 = 32.
Total 1806
 Choice (D)
 Choice (A) 33. After arranging 3 and 4 particular guests, the remaining
27. The first and eighth places must be filled with the same number of people is 9.
symbol. This can be done in 4 ways. To arrange on first table we require 5 members. They
Similarly the second and seventh place must be filled can be selected in 9C5 ways.
with same symbol. This can be done in 4 ways. To arrange on the second table, we require 4 members.
They can be selected in 4C4 ways.
∴ Hence, required arrangements is = 9C5 (7!) (7!)
1st 2nd 3rd 4th 5th 6th 7th 8th = 9C5 (7!)2.
Similarly, the other entries can be filled.  Choice (C)
∴ The number of ways for forming a palindrome of 34. Let the letters be L1, L2, L3, L4 and L5 and the mail boxes
length 8 symbols is 44 = 256. Choice (C) be B1, B2, B3, B4. Now L1 can be dealt in 5 ways i.e.,
28. There are 9 boxes in total, so the 6 identical balls can be either post it into B1 or B2 or B3 or B4 or do not post it at
placed in these boxes in C6 ways. These include 3 ways
9 all (since one or more letters have to be posted, there is
in which the balls are placed in exactly two rows. a possibility of not posting L1 at all). Similarly each of
∴ Required number of ways = C6 – 3 9 L 2
, L 3
, L 4
and L5 can be dealt in 5 ways, giving us a total
= 84 – 3 = 81. Choice (D) of 5 5
possibilities which includes the case of not posting
any of the letters, which has to be ruled out. Hence the
29. Units place can be filled with 1, 3, 5, 7 or 9 and the tens
required ways are 55 –1.
can be filled with any of the 9 non-zero digits.
∴ The number of two digit codes that can be formed  Choice (C)
= 9 × 5 = 45. Choice (C) 35. We know that, the number of non negative integral
30. ∴ Required number of ways solutions of a1 + a2 + a3+-------+ ar = n is n + r – 1Cr – 1 here
 1 1 1 n = 14 r = 3
1
= n! 1 − + − + − − − − + ( −1) n  ∴ Required answer is (14 + 3 – 1) C3 – 1 = 16C2 = 120.
 1! 2! 3! n !  Choice (C)
= 5! 1 − 1 + 1 − 1 + 1 − 1 
 1! 2! 3! 4! 5!
 
Quantitative Aptitude Test 6
Number of Questions: 35 Section Marks: 30

(ERPV, Numbers)
Directions for questions 1 to 6: Select the correct alterna- calls on a single line, his bill for that line would have
tive from the given choices. been `700.
1. A three-digit number when reversed and subtracted 7. What was the total number of calls on the first line?
from the original number gives the result as 792. If the (A) 375 (B) 300
sum of the digits of the number is 18, find the tens digit (C) 350 (D) 400
of the number. 8. What is the number of free calls per line?
(A) 6 (B) 7 (A) 150 (B) 125
(C) 5 (D) 8 (C) 180 (D) 200
2. A sum of `209 was intended to be divided among A, B Directions for questions 9 to 35: Select the correct alterna-
and C in the ratio 2 : 4 : 5. By mistake, it got divided in tive from the given choices.
the reciprocal of the intended ratio. Find the gain of A
due to this mistake. 9. The cost of 2 pens, 4 erasers and 5 sharpeners is `36.
(A) `48 (B) `72 The cost of 3 pens,7 erasers and 9 sharpeners is `63.
(C) `56 (D) `60 Find the total cost of one pen, one eraser and one
sharpener.
3. In a fraction, the numerator exceeds the denominator
(A) `9 (B) `8
by 7. If twice the numerator exceeds thrice the denom-
(C) `10 (D) `11
inator by 2, find the fraction.
8 19 10. If a + b – c : b + c – a : a + c – b = 3 : 4 : 5, find a : b :
(A) (B) c.
15 12
(A) 1 : 2 : 3 (B) 8 : 7 : 9
20 17 (C) 5 : 7 : 8 (D) 1 : 3 : 2
(C) (D)
3 24
11. For which of the following values of k does the system
4. If a : b = 3 : 4, c : b = 2 : 7 and c : d = 5 : 7, find of equations 2x + 5y = 1 and 6x + 15y = k/2 have infinite
a : b : c : d. solutions?
(A) 105 : 140 : 50 : 70 (B) 105 : 140 : 40 : 49 (A) 6 (B) 3
(C) 105 : 140 : 40 : 56 (D) 105 : 140 : 50 : 49 (C) –6 (D) Any value except 6
5. In a parking place, there are a total of 20 vehicles (bikes 3a2 + 4b 2
and cars) are parked. If the total number of tyres of all 12. If a : b = 3 : 4, find the value of .
4 a2 + 3b 2
these vehicles is 70, find the ratio of the number of
bikes and cars. (No vehicle has spare tyres). 11 12
(A) 3 : 1 (B) 1 : 2 (A) (B)
12 11
(C) 1 : 3 (D) 2 : 1 12 13
6. Ajay, Balu, Chetan and Dinesh have a total of `240 (C) (D)
13 12
with them. The amount with Ajay is half of the total
amount with the others. The amount with Balu is one- 13. The distance a stone falls under free fall varies directly
fourth of the total amount with the others. The amount with the square of the time for which it falls. If a stone
with Chetan is one-fifth of the total amount with the falls at a distance of 35 m in the fourth second, find the
others. Find the amount with Dinesh. total distance it falls in the first 5 seconds.
(A) `30 (B) `72 (A) 100 m (B) 140 m
(C) `40 (D) `48 (C) 135 m (D) 125 m
Directions for questions 7 and 8: These questions are based 14. A sum of `750 is divided among P, Q and R. If `30,
on the following information. `20 and `10 is added to their respective shares then the
Siddharth has two landlines of a particular telephone service ratio of amounts of P, Q and R becomes 10 : 8 : 9. What
provider. For each line,he has to pay a monthly rent of `250. is the share of P?
This entitles him to a certain fixed number of free calls (A) 280 (B) `250
per line. Calls over and above this allowance are charged (C) `260 (D) `270
at the rate of `1 per call. One month he received bills 15. Ajay told Bharat, “I am four times as old as you were
of `450 and `350 for the two lines. Had he made all his when I was as old as you are”. Bharat told Ajay “Ten
Quantitative Aptitude Test 6 | 1.69

years ago, I was nine years younger to you”. Find the 23. What is the least value of x, if the nine-digit number
sum of present ages of Ajay and Bharat. 23x4567y4 is divisible by 44?
(A) 39 years (B) 36 years (A) 1 (B) 0
(C) 42 years (D) 45 years (C) 5 (D) 7
16. The ratio of the incomes of A and B is 3 : 2. The ratio 24. A number when divided by 259 leaves a remainder of
of their expenditures is 5 : 4. If the savings of A is twice 161. If one-seventh of the same number is divided by
that of B, find the ratio of the income and expenditure 37, the remainder will be
of B. (A) 14
(A) 3 : 16 (B) 3 : 4 (B) 23
(C) 3 : 2 (D) 5 : 4 (C) 7
17. A bag has a total of 40 coins in denominations of `1, `2 (D) Cannot be determined
and `5. If the total value of the coins is `130, find the 25. (363 + 362 - 37) is not divisible by, which of the
maximum number of `5 coins. following?
(A) 24 (B) 21 (A) 185
(C) 23 (D) 22 (B) 37
18. The age of Harish is 8 years more than twice the age of (C) 36
Ganesh. After how many years will the age of Harish be (D) More than one of the above
twice the age of Ganesh? 26. Mohit has 290 toffees with him. He distributes all his
(A) 4 toffees among his friends, such that each of his friends
(B) 6 gets a different number of toffees, which is at least 5 and
(C) 8 at most 29. What could be the least number of friends?
(D) Cannot be determined (A) 19 (B) 20
19. Rohan made 13 calls from a public booth. Each was (C) 13 (D) 14
either a local call or STD call or ISD call. The average 27. When the King of Patiala distributed 33,274 gold coins
costs of his local calls, STD calls and ISD calls were equally among his subjects, the Maharaja of Mysore
`6, `11 and `13, respectively. The total amount spent also distributed 30,905 gold coins equally to his sub-
by Ram on the calls was `119. Find the number of ISD jects, each giving the same number of coins to his sub-
calls he made jects. Surprisingly, both the kings were left with the
(A) 3 (B) 4 same number of gold coins.If the number of coins that
(C) 2 (D) 5 each subject received is a 2-digit number, what is the
20. Ram has a certain number of notes of the denomina- difference in the number of subjects of the two kings?
tions `5, `10 and `20. The total amount he has is `540. (A) 309
If he has a total of 24 currency notes in the denomina- (B) 103
tions of `5 and `20, then what is the greatest number of (C) 23
notes of `10 notes he could have? (D) Cannot be determined
(A) 42 (B) 40 28. The units digit of (24n) (67n) + (53n) (79n), (where n is a
(C) 39 (D) 38 natural number) is
21. Three vessels are filled to their capacities with mix- (A) 3
tures of milk and water. The ratioof their capacities is (B) 1
2:3:4. The ratio of the quantities of milk and water in (C) 5
the first,the second and the third vessels is 1:3, 5:1 and (D) Cannot be determined
3:5 respectively. Find the ratio of the total quantity of 29. Three bells X, Y and Z ring at regular intervals and ring
milk in the vessels to the total quantity of water in the simultaneously 24 times in a day. If Y rings less fre-
vessels. quently than X but more frequently than Z, what could
(A) 1 : 1 (B) 2 : 3 be the minimum number of times for which Y rings in a
(C) 3 : 2 (D) 3 : 4 day?
22. a, b, c are positive integers such that a : b = 4 : 3 and (A) 48 (B) 36
b : c = 4 : 3. If the sum of the squares of c and b is less (C) 54 (D) 72
than square of the sum of b and a by 2236, then what 30. If a, b and c are three consecutive positive integers,
is the value of the number which is neither the greatest then which of the following is not necessarily true?
nor the least? I. (4a + 5b + 3c) is an odd number.
(A) 24 (B) 32 II. (2a + 3b + 4c) is an even number.
(C) 18 (D) 36 III. a2 b3 c4 is an even number.
1.70 | Quantitative Aptitude Test 6

IV. (3a + 2b + 4c) is an odd number. 33. The sum and the difference of the LCM and the
(A) Only I (B) Only II and III HCF of two numbers is 784 and 756 respectively.
(C) Only III and IV (D) Only II and IV What is the least possible difference between the two
31. Some students have to be seated in some rows, such numbers?
that equal number of students is seated in each row. If (A) 112 (B) 70
there are 13 rows 3 students will be left out and if there (C) 84 (D) 98
are 21 rows, 11 students will be left out. How many 34. A number when divided by 30 and 36 leaves a remain-
students will be left out if there are 19 rows and less der of 15 and 21 respectively. When such a number is
than 300 students? divided by 35, it leaves a remainder of 10. What is such
(A) 7 (B) 9 least number?
(C) 11 (D) 16 (A) 885 (B) 895
3 2. After the division of a number successively by 4, 5 and (C) 985 (D) 15
3 the remainders obtained are 1, 2 and 2 respectively. 35. What will be the remainder when 223 × 226 × 228 is
What will be the remainder if the least of such numbers divided by 11?
is divided by 37? (A) 10 (B) 7
(A) 12 (B) 14 (C) 6 (D) 1
(C) 17 (D) 31

Answer Key
1. D 2. B 3. B 4. C 5. C 6. B 7. C 8. A 9. A 10. B
11. A 12. D 13. D 14. D 15. A 16. C 17. D 18. C 19. A 20. C
21. A 22. A 23. A 24. B 25. C 26. C 27. B 28. B 29. A 30. D
31. D 32. A 33. C 34. A 35. D

Hints and Explanations


1. Let the three-digit number be abc. 3 7 5
Its value is 1000a + 10b + c. 4. a = 4 b, b = c, c = d
2 7
On reversing it becomes cba whose values are
100c + 10b + a 3  7  5   15
Hence a =  d = d
abc – cba = 99(a – c) = 792 4  2  7   8
⇒ a – c = 8 7  5 5
(a, c) can be (9, 1) or (8, 0) b=  d = d
2  7 2
a + b + c = 18
when a = 9, c = 1, then b = 8 15 5 5
Hence a : b : c : d = d: d: d:d
when a = 8, c = 0, then 8 2 7
b = 10, which is not possible. Multiplying each term of the ratio by LCM of denomi-
Hence b = 8 nators of the coefficients of d i.e. 56,
 Choice (D) We got, a : b : c : d = 105 : 140 : 40 : 56
2. A should have got 2 out of the total 11 parts i.e., 2/11 Alternate method:
(209) = `38 Going by the choices, a : b = 3 : 4, b : c = 7 : 2 and
Ratio of actual division of `209 among A, B and C is c : d = 5 : 7 is satisfied only in Choice (C).
1/2 : 1/4 : 1/5 = 10 : 5 : 4  Choice (C)
A actually got 10 out of 19 parts i.e., 5. Let the number of bikes sand cars parked be S and C
10/19(209) = `110 respectively.
Gain of A is 110 – 38 = `72. Choice (B) S + C = 20  …… (1)
3. Let the numerator and denominator of the fraction be n Each bike has 2 tyres and each car has 4 tyres.
and d. Total number of tyres = 2S + 4C = 70
2n – 3d = 2  ….. (1) ⇒ S + 2C = 35  ……. (2)
n–d=7 ….. (2) Subtracting the second equation from the first
Solving the two equations we get n = 19 and d = 12. equation, we get C = 15,
 Choice (B) so S = 20 – C = 5 S : C = 1 : 3 Choice (C)
Quantitative Aptitude Test 6 | 1.71

6. Let the amounts with Ajay, Balu, Chetan and Dinesh be a1/a2 = b1/b2 = k1/k2 must be satisfied.
a, b, c and d respectively. Hence for the given equations, 2/6 = 1/k/2 so k = 6
1 1  Choice (A)
a + b + c + d = 240 a = (b + c + d) = (240 – a)
2 2 1 2. Given that a : b = 3 : 4
⇒ 2a = 240 – a a 3
240 so =
b 4
⇒ 3a = 240 ⇒ a = = `80
3
3a2 + 4b 2
It can be seen above that half of the total amount Dividing both numerator and denominator of
4 a2 + 3b 2
with Balu, Chetan and Dinesh becomes one-third
by b2, it becomes
of the total amount. 2
Similarly amounts with Balu and Chetan which  a  9
3  + 4 3  + 4
are one-fourth and one-fifth of the total amount  b  16  13
= =  Choice (D)
with the others become one-fifth and one-sixth of  a
2
 9 12
4  + 3 4   + 3
the total amounts.  b 16
240
Hence b = = 48 13. Let the distance travelled by the stone and the time of
5
travel of the stone be denoted by d and t respectively. d
240 = ct2 where c is a constant.
c= = 40
6 Distance travelled in the fourth second by the stone =
d = 240 – (a + b + c) = 72 Choice (B) Total distance travelled in first four seconds – the total
distance it travelled in first three seconds
Solutions for questions 7 and 8: = c(42 – 32) = 35.
On line 1, `250 was rent and `200 was call charges. On 7c = 35 ⇒ c = 5
line 2, `250 was rent and `100 was call charges. The total Total distance it falls in the first 5 seconds
chargeable calls was 300  ….. (1) = c(52).= 125 m. Choice (D)
On a single line, `250 is the rent and `450 is the call charg- 14. The shares of P, Q and R be `x, `y and `z respectively.
es, i.e. there are 450 chargeable calls  ……. (2) So x + y + z = 750
Comparing (1), (2) we conclude that the number of free calls (x + 30) : (y + 20) : (z + 10) = 10 : 8 : 9
is 150. The total number of calls (free + chargeable) on the x + y + z + 30 + 20 + 10 = 750 + 60 = 810.
first line is 150 + 200 = 350 810 × 10
7. 350 Choice (C) So x + 30 = = 300. So x = 270
(10 + 8 + 9)
8. 150 Choice (A) Share of p = `270 Choice (D)
Solutions for questions 9 to 35: 15. Let the present age of Ajay be x years.
9. Let the cost of each pen, eraser and sharpener be p, e Some time in the past, Bharat was x/4 years.
and s respectively. The age of Ajay at that time is Bharat’s present age.
2p + 4e + 5s = 36 Bharat’s present age is (x – 9) years.
3p + 7e + 9s = 63 As their difference of ages is constant,
Multiplying the first equation by 2 and subtracting the x – (x – 9) = (x – 9) – x/4. x = 24
second equation from it, p + e + s = 9 Choice (A) Sum of their present ages = 2x – 9 = 39 years
 Choice (A)
10. Let a + b – c = 3x  → (1)
b + c – a = 4x  → (2) 16. Let the incomes of A and B be 3x and 2x, respectively.
a + c – b = 5x  → (3) Let the expenditures of A and B be 5y and 4y, respec-
Adding these three equations, a + b + c = 12x  → (4) tively. Savings of A and B are 3x – 5y and 2x – 4y
2c = 9x → (4) – (1) respectively. Given that, 3x – 5y = 2(2x – 4y) ⇒ x = 3y
so c = 4.5x Ratio of income and expenditure of B = 2x : 4y
2a = 8x → (4) – (2) = 2(3y) : 4y = 3 : 2 Choice (C)
so a = 4x 17. Let the number of `1, `2 and `5 coins be x, y and z
2b = 7x → (4) – (3) respectively.
so b = 3.5x x + y + z = 40 ­ ……. (1)
a : b : c = 4x : 3.5x : 4.5x = 8 : 7 : 9 Choice (B) x + 2y + 5z = 130  …… (2)
11. If two equations should be in the form Subtracting the equation (1) from (2), y + 4z = 90
a1x + b1y = k1 and In order to satisfy the above equation, y must be divis-
a2x + b2y = k2 to have infinite solutions, ible by 2.
1.72 | Quantitative Aptitude Test 6

As z must be maximum, y must be minimum. \ x = y – 5 or x + y + 6. The least value of x is 6 – 5


Hence when y has the least value, then z is obtained as or 1. Choice (A)
22 from the above equation. Choice (D) 24. Let the number be N
18. Let the present age of Ganesh be G. \ N = 259K + 161.
Present age of Harish = 2G + 8 ⇒ N/7 = 37K + 23
Let Harish become twice the age of Ganesh k years \ N/7 leaves a remainder of 23, when divided by 37.
from now. Hence, 2G + 8 + k = 2(G + k) ⇒ k = 8.  Choice (B)
 Choice (C) 25. The given number N is 36 (36 + 1) - 37
2

1 9. Let the number of local calls, STD calls and ISD calls = 362 (37) - 37 = 37 (362 - 1) = (37) (35) (37)
made by Rohan be l, s and i respectively. \ N is not divisible by 36. Choice (C)
l + s + i = 13 and 6l + 11s + 13i = 119 26. Since the number of friends has to be the least, the
6l + 11s + 13i – 6 (l + s + i) = 119 – 6 (13) number of toffees should be as great as possible. Let
5s + 7i = 41 the toffees distributed be 29, 28, 27, 26 …..
5s ends with 0 or 5. Also, R.H.S ends with 1. \7i must Given that, 29 + 28 + 27 + 26 + …. up to n terms ≤ 290.
end with 1 or 6. Also 7i < 41. If n = 12, the number of toffees that can be distributed is
\ 7i = 21 (∴No value of 7i ends with 6) 29 + 28 + ……. + 19 + 18 = 6(29 + 18) = 282
i = 3 Choice (A) The last person gets 8 toffees. The least number of
2 0. Let Ram have f, t and w notes of denomination `5, `10 friends is 12 + 1 = 13. Choice (C)
and `20 respectively. Given that 27. Let there be m subjects in Patiala and n in Mysore. Let
5f + 10t + 20w = 540 ------ (1) (f, t, w are integers) each subject get p coins and say the number of coins
and f + w = 24  -------------- (2) left with either king is r.
(1) – 5 × (2) gives 10t + 15w = 420 \ mp + r = 33, 274 and np + r = 30, 905
420 − 15w \ (m – n) p = 2369 = (23) (103)
⇒ t =
10 As p is a 2–digit number p = 23 and m – n = 103.
420 − 15 × 2  Choice (B)
As w > 0, tmax = = 39 28. 2 × 6 + 5 × 7 can be written as 16 × 6 + 5 × 79n.
4n 7n 3n 9n n
10
As 16n can be written as 6n, the unit’s digit of the sum
 Choice (C) becomes 6 × 6 + 5 × (odd number) = 6 + 5 =1.
2 1. The vessels are filled to their capacities  Choice (B)
Let the capacities of the first, the second and the third 29. In a day, the three bells toll together = 24 times, or once
vessels be 2x, 3x and 4x respectively in every hour or 60 minutes. Let X, Y, Z ring once in
1 5 3 every x, y, z minutes respectively. The maximum value
Required ratio = (2x) + (3x) + (4x) : 3/4(2x)
1+ 3 6 8 of z is 60 and y < 60. Since y is a factor of 60, its great-
1 5 x 5 x 3x 3x x 5x est value is 30. Hence if Y rings once every 30 minutes,
+ (3x) + (4x) = + + : + + the number of times it tolls in a day will be the mini-
6 8 2 2 2 2 2 2
(24) (60)
= 1 : 1. Choice (A) mum, which is = = 48  Choice (A)
30
22. Given that a : b = 4 : 3 and b : c = 4 : 3
30. The numbers a, b, c are consecutive. Therefore there
⇒ a : b = 16 : 12 and b : c = 12 : 9
are only 2 possibilities for the types (odd / even) of a, b,
\ a : b : c = 16 : 12 : 9
c. Either a, b, c are o, e, o or they are e, o, e. We tabulate
Let a = 16k, b = 12k and c = 9k
the expressions, and the truth values (definitely true,
Also given (a + b)2 – (b2 + c2) = 2236
could be false, definitely false, i.e., dt, cf, df).
(28k)2 +(144k2 + 81k2) = 784k2 – 225k2= 2236
⇒ k2 = 4 ⇒ k = 2 dt cf df
\ b = 12k = 12 × 2 = 24. Choice (A) I 4a + 5b + 3c 4(o) + 5(e) + 4(e) + 5(o) + 
3(o) = o 3(e) = o
23. The number 23x 456 7y 4 is divisible by 4 and by 11.
II 2a + 3b + 4c 2(o) + 5(e) + 2(e) + 5(o) + 
\ y = 0, 2, 4, 6 or 8 and (2 + x + 5 + 7 + 4) – (3 + 4
4(o) = e 4(e) = o
+ 6 + y),
III a2 b3 c4 (o) (e) (o) = e (e) (o) (e) = e 
i.e., (18 + x) – (13 + y) or (5 + x – y) is also a mul-
tiple of 11. IV 3a + 2b + 4c 3(o) + 2(e) + 3(e) + 2(o) + 
4(o) = o 4(e) = e
As the least value of 5 + x – y is 5 + 0 – 9 or –4 and
the greatest value is 5 + 9 – 0 or 14, it could be 0 We see that II, IV are not definitely true
or 11.  Choice (D)
Quantitative Aptitude Test 6 | 1.73

31. When the students are arranged in 13 rows, let the 34. The given conditions represent the problem as LCM
number of students in each row be x. The total number model-2. The general form of such numbers is (LCM
of students is (13x + 3). Similarly in the second case, of 30, 36)K – [common difference i.e., 15 in this case]
the total number of students is 21y + 11. = 180K – 15.
This is LCM model II Now, when 180K – 15 is divided by 35, (175K is a
Number of students: = (LCM of 13 and 21) - 10 = 263 already divisible by 35), the remainder is given to be
Required remainder 263 ÷ 19 = 16. Choice (D) 10. Hence 180K – 25 i.e., 5K – 25 is exactly divisible
32. Divisors are 4, 5, 3. Remainders are 1, 2, 2. by 35. This will be possible when K = 5.
The least number is [(2) (5) + 2] (4) + 1 = 49. Hence the required number is 180(5) – 15 = 885.
When 49 is divided by 37, the remainder is 12.  Choice (A)
 Choice (A) 35. Remainder required
33. Let the two numbers be ha and hb respectively,where a  (220 + 3)(220 + 6)(220 + 8) 
and b are co-primes. Then LCM of the two numbers is = Rem  
 11
hab.
Given hab + h = 784 and hab – h = 756 (220 + 3) (220 + 6) (220 + 8) = (A multiple of 220 + 3.6)
⇒ hab = 770 and h = 14 Þ ab = 55. (220 + 8) = A multiple of 220 + (3) (6) (8). As 220 is
The possible values of a, b are (1, 55) and (5, 11).  (3)(6)(8) 
divisible by 11, reminder required = Rem 
When the numbers are close, the difference will  11 
bethe least, i.e., {a, b} = {5, 11}.
The least possible difference is 14 (6) = 84.  144 
= Rem  = 1. Choice (D)
 Choice (C)  1 
Quantitative Aptitude Test 7
Number of Questions: 35 Section Marks: 30

(Quadratic equations and Inequalities)


Directions for questions 1 to 35: Select the correct alterna- Ι. Roots are equal
tive from the given choices. ΙΙ. Roots are complex
1. Two students independently attempted to solve a qua- ΙΙΙ. Roots are rational
dratic equation in x. One of them copied the constant IV. Roots are real
term wrong and obtained roots as –15 and 16. The other (A) Ι and IV (B) ΙΙΙ only
student copied the coefficient of x wrong and obtained (C) Ι and ΙΙ (D) ΙΙ only
his roots as –10 and 21. Find the correct roots of the 9. All the roots of two quadratic equations are positive
quadratic equation? integers. The sum of the squares of the roots of the first
(A) (–15, 14) (B) (–14, 15) quadratic equation is equal to that of the second qua-
(C) (–25, 7) (D) (–7, 25) dratic equation. If the sum of the roots of the two equa-
tions are 10 and 8 respectively, then what is the greatest
2. 132 + 132 + 132 + ....... = ––––– possible root of these quadratic equations?
(A) 11.11 (B) 12.32 (A) 7 (B) 6
(C) 11 (D) 12 (C) 8 (D) 5
3. The range of k for which the sum as wells as the 10. If a and b are positive numbers, what is the nature of
product of the roots of 6x2 – kx + 9 – k2 = 0 are ( a + b)3
the roots of the equation (a + b) x2 + 2 abx +
negative is __________ 16
(A) (–3, 3) (B) (–∞, 3) = 0?
(C) (–∞ , –3) (D) (3, ∞) (A) Real and distinct. (B) Real and equal.
4. Find the range of k, for which −x2 + 4kx + 3k − 1, is (C) Non-real and distinct. (D) Either (B) or (C)
always negative. 11. If a positive number is increased by three and then
 1  squared, the result is 23 more than the original number.
(A)  − ,1 (B) (1, ∞) Find the original number.
 4 
(A) 1 (B) 2
 1  1 (C) 3 (D) 4
(C)  −∞, −  (D)
 −1, 
 4 4 12. Find the value of R, so that one of the roots of
5. a and b are the roots of the equation 2x2 − 15x + k = 0. x2 + 6Rx + 64 = 0 is the square of the other root.
Find the value of k if a2 − b2 = 45. −10 8
(A) (B)
1 1 3 3
(A) 5 (B) 10
16 8 5 7
(C) (D)
1 1 3 3
(C) 13 (D) 12
2 2 13. If the value of p in the equation x2 + 2(p + 1)x + 2p = 0,
6. If the sum of the roots of the quadratic equation is real, the roots of the equation are
3x2 + (2k + 1) x – k – 5 = 0 is equal to the product of (A) rational and unequal.
the roots, which of the following is true? (B) irrational and unequal.
(A) k2 – 4 = 0 (B) k2 – 9 = 0 (C) real and unequal.
(C) k2 – 16 = 0 (D) k2 – 25 = 0 (D) real and equal.
7. If the roots of the equation x – 7x – 12 = 0 are dimin-
2 14. Find the equation whose roots are twice the roots of the
ished by one and then multiplied by two, which of the equation 3x2 – 7x + 4 = 0.
following equations is formed with those values as the (A) 3x2 – 14 x + 8 = 0 (B) 3x2 + 14x + 16 = 0
roots? (C) 3x + 14 x – 16 = 0
2
(D) 3x2 – 14x + 16 = 0
(A) x2 – 10x + 24 = 0 (B) x2 – 6x – 76 = 0 15. The length of a rectangle is 1 cm more than its breadth.
(C) x – 2x – 48 = 0
2
(D) x2 – 10x – 72 = 0 If its diagonal is 29 cm, what is the measure of its
8. Which of the following statements is true about the breadth? (in cm)
roots of the equation k2 x2 – k x + (1 + 2x2) = 0, where (A) 18 (B) 20
k is a real number? (C) 17 (D) 21
Quantitative Aptitude Test 7 | 1.75

16. A is any single-digit prime number and B is any natural x x+2


number. How many equations of the form 25. If − < 0, then find the range of x.
x +1 x −1
x2 – 4 A x + 3B = 0 have both real roots?
 1
(A) 15 (B) 18 (A)  −1, −  ∪ (1, ∞) (B) (−2, −1) ∪ (0, 1)
(C) 21 (D) 24  2

17. In a class, eight students play basketball. The remain-  1   1 


(C) (−∞, −1) ∪  − ,1 (D)  − , ∞
ing students, who represent 7 times the square root of  2   2 
the strength of the class, play football. Find the strength 26. Find the range of x, for which |x + 2| − 3 |x − 1| + 4 ≥ 0.
of the class. 9
(A) 36 (B) 16 (A) −2 ≤ x ≤ 1 (B) −2 ≤ x ≤
2
(C) 64 (D) 100 3
18. If the price of a book goes down by `20 per dozen, a (C) − 4 ≤ x ≤ 4 (D) None of these
person can purchase 50 dozen books more for `30,000.
27. a, b, c and d are four positive real numbers whose sum
Find the original price of each book. abcd
(A) `10 (B) `12 is equal to 4. If p = , then find
(C) `9 (D) `8 ( abc + bcd + acd + abd )
the maximum value of p.
19. If –9 ≤ p ≤ –5 and –17 ≤ q ≤ –12 then which of the fol-
(A) 16 (B) 4
lowing can be concluded? 1
1
5 p 9 17 p 12 (C) (D)
(A) ≤ ≤ (B) ≤ ≤ 2 4
12 q 17 9 q 5
5 p 3 12 p 17 y
(C) ≤ ≤ (D) ≤ ≤ 28. If 2 < x < 5 and 10 < y < 30, then lies between
17 q 4 9 q 5 x
(A) 5 and 6 (B) 2 and 6
20. If 3 x − 4 = 5 x − 12 , then the sum of the possible val- (C) 2 and 15 (D) 6 and 15
ues of x is _____. 29. If |x| > 6 and y > −4, then which of the following is
(A) 4 (B) 6 necessarily true?
(C) –4 (D) –6 (A) |xy| > 24 (B) |xy| < 24
(C) |x| |y| > 0 (D) None of these
21. If 4 x − 9 = 7, then the values of 4 x − − x is _____.
3

30. Let f(x) = max (3x + 5, 7 − 2x), where x is any real num-
15 −15
(A) 48, −48,
(B) ber. Then the minimum possible value of f(x) is
8 8 31 27
−15 15 (A) (B)
−48, 5 5
(C) 48, (D)
8 8 21 29
(C) (D)
22. Find the range of values of x that satisfy the relation 5 5
|2x − 1| − 1 < |x − 2| + 3. 31. If 20 ≤ x ≤ 35 and 3y − 2x = 5, then the minimum value
1 x
(A) −4 < x < 4 (B) −6 < x < of is
2 x+ y
(C) −6 < x < 4 (D) −5 < x < 3
1
23. If E = |x + 4| + |x + 7| + |x − 1|, then how many integral (A) 1 (B)
3
values of x satisfy the inequality E ≤ 14?
4 4
(A) 8 (B) 10 (C) 9 (D)
(C) 11 (D) More than 11 7

24. Which of the following inequalities gives a finite range 32. If a, b and c are positive real numbers. Find the
of values for x?  1  1  1
(A) 6x3 − x2 − x < 0 minimum value of 1 + a +  1 + b +  1 + c +  .
 a  b  c
(B) x4 + x3 − 3x2 − x + 2 < 0
(C) x3 − x2 − 5x − 3 < 0 (A) 9 (B) 12
(D) x4 + 3x3 + 2x2 > 0 (C) 27 (D) 81
1.76 | Quantitative Aptitude Test 7

33. If a ≤ 25 and a + b ≥ 10, then which of the following is 10


always true? (A) 5 (B)
3
(A) a – b ≥ 40 (B) b – a ≥ −40
(C) a + b ≥ 40 (D) ab ≤ 250 10 10
(C) (D)
13 7
34. If 1 ≤ x ≤ 3, 4 ≤ y ≤ 10 and 2 ≤ z ≤ 5, what is the maxi- 18 − 2 x
y 35. Find the range of values of x for which < 3.
mum possible value of ? 4
x+ y+z
(A) 6 < x < 30 (B) –3 < x < 15
(C) –15 < x < 3 (D) 3 < x < 15

Answer Key
1. B 2. D 3. C 4. D 5. B 6. C 7. D 8. D 9. A 10. D
11. B 12. A 13. C 14. D 15. B 16. C 17. C 18. A 19. C 20. B
21. D 22. D 23. B 24. B 25. A 26. D 27. D 28. C 29. D 30. A
31. D 32. C 33. B 34. C 35. D

Hints and Explanations


1. The quadratic equation which has a and b are the roots ⇒ k < 0 and k > 3 or k < –3
is x2 – (a + b)x + ab = 0 \ k < –3 ⇒ k ∈ (–∞, –3). Choice (C)
Quadratic equation taken by the first student is 4. It is given that -x2 + 4kx + 3k - 1 < 0
x2 – (–15 + 16)x + (–15 × 16) = 0 ⇒ -(x2 - 4kx) + 3k - 1 < 0
x2 – x – 240 = 0  ––––––––(1) ⇒ -[(x - 2k)2 - 4k2] + 3k - 1 < 0
Quadratic equation taken by the second student is ⇒ -[(x - 2k)2] + 4k2 + 3k - 1 < 0
x2 – (–10 + 21)x + (–10 × 21) = 0 Now, for the above expression to be always nega-
x2 – 11x – 210 = 0  –––––––––(2) tive 4k2 + 3k - 1 < 0 ⇒ (4k -1)(k + 1) < 0
\ Required correct quadratic equation is x2 – x – 1
210 = 0 This is true when -1 < k < 4 . Choice (D)
⇒ x2 – 15x + 14x – 210 = 0
⇒ x(x – 15) + 14(x – 15) = 0 5. The given equation is 2x2 - 15x + k = 0
⇒ (x – 15) (x + 14) = 0 15
The sum of the roots, a + b = and
\ Required roots are –14 and 15. Choice (B) 2
k
2. Let X = 132 + 132 + 132 + ...... the product ab =
2
⇒ x = 132 + x It is given that a2 - b2 = 45 ⇒ a – b = 6
Squaring on both sides 15 27 3
a+b= , a - b = 6, ⇒ a = ,b= 4
⇒ x2 = 132 + x ⇒ x2 – x – 132 = 0 2 4
⇒ x2 – 12x + 11x – 132 = 0  27   3  81
⇒ (x – 12) (x + 11) = 0 \ The product of the roots ab =     =
 4   4  16
\ x = 12(\ x is always positive) Choice (D)
k 81 81 1
3. Given 6x2 – kx + 9 – k2 = 0 Now = ⇒k= = 10  Choice (B)
k 2 16 8 8
The sum of the roots =
6 − (2k + 1)
6. Sum of the roots =
9−k 2
3
The product of the roots =
6 − ( k + 5)
Product of the roots =
k 9 − k2 3
and are both negative
6 6 − (2k + 1) − ( k + 5)
=
k 9 − k2 3 3
⇒ < 0 and <0
6 6 ⇒ 2k + 1 = k + 5 ⇒ k = 4
⇒ k < 0 and 9 < k 2  Choice (C)
Quantitative Aptitude Test 7 | 1.77

7. Roots are to be diminished by one and then multiplied 11. Let the required original number be x.
by two. i.e., if A, B are roots of given equation, then (x + 3)2 = 23 + x.
2(A – 1) = A1 and 2(B – 1) = B1, where A1 and B1 are the Hence x2 + 6x + 9 = 23 + x
roots of the new equation. i.e., ⇒ x2 + 5x - 14 = 0.
A A1 + 2 B1 + 2 (x + 7) (x - 2) = 0
⇒ A = 1 + 1 = and B = ⇒ x = -7 or x = 2.
2 2 2
Since the original number is positive, x = 2.
i.e., x of the given equation is to be replaced by  Choice (B)
x+2
, to obtain the required equation. Given 12. If one of the roots is a, the other root is a .
2

2 Hence the product of the roots = a(a2).


equation is: x2 – 7x – 12 = 0.
2
a3 = 64 ⇒ a = 3 64 = 4 and a2 = 42 = 16
 x + 2  7( x + 2)
Required equation is  − − 12 = 0  6R 
 2  2 The sum of roots = -   = -6R = 4 + 16 = 20
 1 
(x + 2)2 – 7 (2) (x + 2) – 4 (12) = 0.
⇒ x2 + 4x + 4 – 14x – 28 – 48 = 0.  20  10
R=   =−  Choice (A)
⇒ x2 – 10x – 72 = 0. Choice (D)  −6  3
8. When rewritten, the equation becomes: 13. For the equation x2 + 2(p + 1) x + 2p = 0
(k2 + 2) x2 – kx + 1 = 0 b2 - 4ac = [2(p + 1)]2 - 4(2p)] = 4p2 + 8p + 4 - 8p
Discriminant, D = (k)2 – 4 (1) (k2 + 2) = 4p2 + 4 which is always positive.
= – 3k2 – 8 = – (3k2 + 8) Hence the roots of the equation are always real and
3k2 is positive for all real values of k, and hence unequal. Choice (C)
(3k2 + 8) is positive; and so – (3k2 + 8) is negative. 14. For the equation, whose roots are twice the roots of the
As the discriminant is negative, roots are complex. equation A : 3x2 – 7x + 4 = 0, the sum of the roots is
 Choice (D) twice the sum of the roots of A and the product of the
9. Let the roots of the first quadratic equation be a and b roots is 4 times the product of the roots of A.The
and those of the second equation be g and d respectively.   7   4
required equation is x2 –  2    x + 4   = 0
Given a2 + b2 = g2 + d2   3   3
Also a + b = 10 and g + d = 8. i.e., 3x2 – 14x + 16 = 0 Choice (D)
The possible values of a2 + b2 are 50, 52, 58, 68 and 82
while the possible values g2 + d2 as are 32, 34, 40 and 15. Let ℓ and b be the length and breadth in cm.
50. As only 50 is a common value, a = 5, b = 5, g = 7 Given that ℓ = b + 1
and d = 1 Also given that diagonal = 29 cm
\ The greatest possible root is 7. Choice (A) ⇒  2 + b 2 = 29
10. Dividing both sides of the given equation by a + b, By squaring on both sides, (b + 1)2 + b2 = 292
2abx ( a + b)2 ⇒ 2b2 + 2b – 840 = 0
x2 + + =0 ⇒ b2 + b – 420 = 0
a+b 16
⇒ (b + 21) (b – 20) = 0
Discriminant
2 2 2
\ b = 20 Choice (B)
 2ab  4( a + b)2  2ab   a + b 
= 
 a + b 

16
=   −
a + b  2 
 16. (4 A ) – 4 (3B) ≥ 0
2

4
Shown below is the proof that this is always non-posi- 3 A≥B
tive provided a and b are positive.
(a – b)2 ≥ 0 ⇒ a2 + b2 + 2ab ≥ 4ab As A is a single digit prime number, A can be 2, 3, 5 or
a + b 2ab if A = 2, B has 2 possibilities. If A = 3, B has 4 possi-
dividing both sides by 2 (a + b) ≥ bilities. If A = 5, B has 6 possibilities. If A = 7, B has 9
2 a+b
possibilities. A total of 21 equations are possible.
As the expressions on both sides of the inequality are  Choice (C)
a+b 2 2ab 2
positive, ( ) ≥( ) 1 7. Let the strength be x. The number of students who play
2 a+b
basketball = 8
\ D < 0 or D = 0 The number of students who play football
If D =0, the roots are real and equal. =x–8= 7 x
If D< 0, the roots are non-real and distinct.
Substituting the choices in place of x in the equation
 Choice (D)
above, only choice (C) satisfies it. Choice (C)
1.78 | Quantitative Aptitude Test 7

18. Let the initial number of books in dozens = b 22. |2x - 1| - 1 < |x - 2| + 3
Let initial price (in `) of books per dozen be p. ⇒ |2x - 1| - |x - 2| < 4
pb = 30,000. →(I) We need to consider 3 cases
(50 + b) (p - 20) = 30,000 1 1
(1) x < (2) ≤ x < 2 and (3) 2 ≤ x .
50p - 1000 + pb - 20 b = 30,000 2 2
or, 50p - 20b = 1000 1
For x < , we get
5p - 2b = 100.  →(II) 2
60, 000 -(2x - 1) + (x - 2) < 4
From (I) and (II) 5p - = 100
p ⇒ -x < 5 ⇒ x > -5
5p2 - 100p - 60,000 = 0 1
\ -5 < x <
5p2 - 600p + 500p - 60,000 = 0 2
5p(p - 120) + 500 (p - 120) = 0 ⇒ p = 120 1
120 For ≤ x < 2, 2x - 1 + x - 2 < 4
= 10  2
The price of each book = Choice (A)
12 7 1
p −p ⇒ x < \ < x < 2
3 2
19. =
q −q For x ≥ 2, (2x - 1) - (x - 2) < 4
5 ≤ –p ≤ 9 and 12 ≤ –q ≤ 17 ⇒ x < 3 \2 ≤ x < 3
−p Thus the range of x such that the given relation is
is maximum when p is maximum and q is mini- satisfied is -5 < x < 3 Choice (D)
−q
23. For x = -9, E = |-9 + 4| + |-9 + 7| + |-9 -1| = 17
mum
For x = -8, E = |-8 + 4| + |-8 + 7| + |-8 - 1| = 14
 − p 9 For x = 1, E = |1 + 4| + |1 + 7| + |1 - 1| = 13
\ Max  = = 3/4
 − q  12 For x = 2, E = |2 + 4| + |2 + 7| + |2 - 1| = 16
−p Therefore the integral values of x for which the given
is minimum when p is minimum and q is inequality is satisfied are -8, -7, -6, -5, -4, -3, -2, -1,
−q
0 and 1 i.e. a total of 10 values. Choice (B)
maximum.
2 4. We need to considering each option separately,
 − p 5
\ Min   = Option A:
 − q  17 6x3 - x2 - x < 0
5 p 3 x (2x - 1)(3x + 1) < 0
≤ ≤  Choice (C) The above inequality is satisfied for
17 q 4
1 1
20. 3 x − 4 = 5 x − 12 x < − or 0 < x <
2 2
When ever p = q it follows that p = ±q which does not give a finite range of values for x
Option B:
3x – 4 = ± (5x – 12)
x4 + x3 - 3x2 - x + 2 < 0
⇒ 3x – 4 = 5x – 12 or 3x – 4 = –5x + 12
(x - 1)2(x + 1)(x + 2) < 0 which gives the same solution
⇒ 2x = 8 or 8x = 16
set as (x + 1)(x + 2) < 0 ((x - 1)2 ≥ 0]
⇒ x = 4 or x = 2
The above inequality is satisfied for -2 < x < -1 this is
\ Required sum of the possible values of x is 6.
a finite range of values for x.
 Choice (B)
Option C:
21. a = b ⇒ a = ± b x3 - x2 - 5x - 3 < 0
|4x – 9| = 7 (x + 1)2(x - 3) < 0 which gives the same solution set as
⇒ 4x – 9 = 7 or 4x – 9 = –7 x-3<0⇒x<3
⇒ x = 4 or x = 1/2 It does not give a finite range of values for x.
3 Option D:
4 x − −x
x4 + 3x3 + 2x2 > 0
 1  1
3 x2 (x2 + 3x + 2) > 0
= 4(4) – (4) or 4   −  
3
x2 (x + 2)(x + 1) > 0
 2  2
The above inequality gives the same solution set as
= –48 or 15/8 Choice (D) (x + 2)(x + 1) > 0 [x2 ≥ 0]
Quantitative Aptitude Test 7 | 1.79

The inequality is satisfied for x < -2 or x > -1 which For x ≥ 1,


does not give a finite range of values for x. |x + 2| - 3 |x - 1| + 4 = (x + 2) - 3 (x - 1) + 4 - 2x
Note: A polynomial of odd degree can take values ≤ -9
from – ∞ to ∞ but a polynomial of even degree has \ x + 2 – 3x + 3 + 4 ≥ 0
a finite range of values for which it has values of a ⇒ 9 – 2x ≥ 0
particular sign. If the coefficient of the leading term ⇒ 2x ≤ 9
(say a) is positive, f(x) < 0 for a finite range if a < 0, 9
f(x) > 0 for a finite range. Choice (B) ⇒ x ≤
2
x x+2 9
− \ 1 ≤ x ≤
25. <0 2
x +1 x −1
Thus, the range of x satisfying the given
x( x − 1) − ( x + 2)( x + 1)
<0 3 9
( x + 1)( x − 1) inequality is - 4 ≤ x ≤  Choice (D)
2
x 2 − x − x 2 − 3x − 2 27. AM (a, b, c, d) ≥ HM (a, b, c, d)
<0
( x + 1)( x − 1) a+b+c+d 4

−4 x − 2 4 1 1 1 1
+ + +
<0 a b c d
( x + 1)( x − 1)
1 1 1 1 16
−2(2 x + 1) + + + ≥
<0 a b c d a+b+c+d
( x + 1)( x − 1)
bcd + acd + abd + abc 16
(2 x + 1) ≥
>0 abcd a+b+c+d
( x + 1)( x − 1)
abcd a+b+c+d
Multiplying both Nr & Dr by (x + 1)(x - 1) we get ≤
abc + bcd + acd + abd 16
(2 x + 1)( x + 1)( x − 1) (Q a + b + c + d = 4)
>0 1
( x + 1)2 ( x − 1)2
\ p ≤ 4 . Choice (D)
The solution set for the above inequality is the same as
that for (2x + 1)(x + 1)(x - 1) > 0 28. Given 2 < x < 5 and 10 < y < 30.
1 The value of y/x is minimum, for the minimum value of
Therefore the inequality holds true for -1 < x < − or y and the maximum value of x.
2
1 10
  \ y/x > or y/x > 2
x > 1, i.e., x ∈  −1, −  ∪ (1, ∞)  Choice (A) 5
 2
The value of y/x is maximum, for the maximum
26. |x + 2| - 3 |x - 1| + 4 ≥ 0 value of y and the minimum value of x.
For x < -2, |x + 2| - 3 |x - 1| + 4 = -(x + 2) + 3 (x - 1) 30
+4 \ y/x < or y/x < 15
2
1
\ -x - 2 + 3x - 3 + 4 ≥ 0 ⇒ 2x ≥ 1 ⇒ x ≥ ⇒ 2 < y/x < 15 Choice (C)
2
29. Given |x| > 6, y > –4.
But we have taken x < -2, thus no solution exists Consider x = 7 and y = 2; xy = 14
in this range.
⇒ |xy| = 14|xy| > 24 is not necessarily true.
For -2 ≤ x < 1, Consider x = 8 and y = 5; xy = 40; |xy| > 40 > 24
|x + 2| - 3 |x - 1| + 4 = (x + 2) + 3 (x - 1) + 4 \ The second option is not necessarily true.
⇒ x + 2 –3x + 3 + 4 ≥ 0 For y = 0; |x| |y| = 0, hence none of the given
⇒ 9 – 2x ≥ 0 options is necessarily true. Choice (D)
⇒ 9 ≥ 2x ⇒ 2x ≤ 9 30. Given f(x) = max(3x + 5, 7 - 2x)
3 f(x) has the minimum value when the two expressions
\ x + 2 + 3x - 3 + 4 ≥ 0 ⇒ 4x ≥ -3 ⇒ x ≥ - 4 are equal.
Therefore, the range of x satisfying the given con- \ 3x + 5 = 7 - 2x
3 2
dition is - 4 ≤ x < 1 5x = 2 ⇒ x =
5
1.80 | Quantitative Aptitude Test 7

\ The minimum value of f(x) is 1


 2  3(2) 2(2)  Hence, the minimum value of 1 + x + is 3 and for the
f   = max  + 5, 7 − x
 5  5 
5  given expression, it is 3(3)(3) = 27. Choice (C)
 31 31 31 33. Given a ≤ 25 and a + b ≥ 10
= max  ,  =  Choice (A) ⇒ a ≤ 25 and b ≥ 10 – a
 5 5 5
⇒ a ≤ 25 and b ≥ 10 – 25
31. 20 ≤ x ≤ 35 ⇒ a ≤ 25 and –b ≥ 15
2x + 5 ⇒ a – b ≤ 40
y=
3 ⇒ b – a ≥ -40 Choice (B)
x x y 1
\ = 3 4. = ; to maximize the given expres-
x + y x + 2x + 5 x + y + z x +1+ z
3 y y
3x 3 sion, x + z should take minimum and y should take
= =
5x + 5 5 + 5 maximum possible value.
x 1 10
\ Maximum value = 1 =
2 13  Choice (C)
This expression is positive for the given range of val- +1+
ues of x and it has its minimum value when 5/x has its 10 10
maximum value, i.e. when x = 20. 18 − 2 x
35. Given <3
3(4) 4 4
The corresponding value is = .
21 7 ⇒ | 18 – 2x| < 12
 Choice (D) ⇒ | x – 9| < 6
1 The expression |x – a| denotes the distance of the
3 2. If x is a positive number, the minimum value of x + point x from the point a on the number line.
x
|x – 9| < 6 Þ x lies within a distance 6 units from
is 2. the point 9. i.e., 3 < x < 15.
 Choice (D)
Unit II
Logical REASONING Test

Logical Ability Test 1������������������������������������������������������������������������������������������������������������������ 1.83


Logical Ability Test 2������������������������������������������������������������������������������������������������������������������ 1.90
This page is intentionally left blank
Logical Ability Test 1
Number of Questions: 35Time: 35 min

Directions for questions 1 to 5: Complete the following 16. If ‘CENTURY’ is coded as ‘AGLVSTW’, then what is
series. the code for ‘SACHIN’?
1. 4, 27, 25, 343, 121, _____ . (A) QCAFKL (B) UCEFGL
(1) 169 (B) 2197 (C) QCAJGP (D) UCAJGP
(C) 3197 (D) 2457 17. If the code for ‘AMBITION’ is ‘GSHOZOUT’, then
2. 11, 25, 77, 157, 473, _____. which of the following is coded as ‘VXOTZUAZ’?
(A) 978 (B) 1421 (A) PRINTOUT (B) PRINTING
(C) 949 (D) 1431 (C) PREDATOR (D) PROFOUND
3. 12, 30, 56, 132, 182, _____. 18. If ‘PRESIDENT’ is coded as ‘KIVHRWVMG’, then
(A) 240 (B) 300 ‘MAHENDAR’ is coded as _____.
(C) 316 (D) 306 (A) NZTVMWZI (B) NZTUMWZI
(C) NZSVMWZI (D) NZSUMWZI
4. 53, 61, 71, 79, 89, _____.
(A) 91 (B) 93 19. In a code language, if pen is called pencil, pencil is
(C) 101 (D) 95 called eraser, eraser is called paper, paper is called
book, book is called table, table is called chair and chair
5. 19, 58, 175, 526, _____. is called desk, then on which of the following do we
(A) 1578 (B) 1238 sit? (according to that language)
(C) 1458 (D) 1579 (A) Table (B) Paper
Directions for questions 6 to 10: Find the missing term. (C) Desk (D) Book
6. 24 : 576 : : 32 : _____ . 20. In a code language, if shirt means shoe, shoe means
(A) 961 (B) 1000 wallet, wallet means spectacle, spectacle means fan,
(C) 1225 (D) 1024 fan means cabin and cabin means card, then which
7. 4 : 27 : : 25 : _____. of the following do we use when we want some air?
(A) 64 (B) 216 (according to that language)
(A) Wallets (B) Spectacles
(C) 125 (D) 36
(C) Fans (D) Cabins
8. BILK : DLPP : HMTO : _____ .
Directions for questions 21 to 25: In a certain code lan-
(A) JOWQ (B) JRWS
guage, the codes for sentences given in column I are given
(C) JPVS (D) JPXT
in column II. Each word has a unique code. Answer the
9. Cricket : Game : : Kangaroo : _____. questions based on these codes.
(A) Animal (B) Team
Column I Column II
(C) Bird (D) Fish
earth gets heat from sun pep tep nep mep wep
10. Driver : Bus : : _____ : Horse
(A) Saddle (B) Jockey moon gets light from sun hep kep tep pep nep
(C) Horseman (D) Cowboy sun gave energy to plants bep pep dep zep lep
Directions for questions 11 to 15: Find the odd man out. human gets food from plants qep tep nep dep rep
11. (A) 11 (B)  21 heat and light gave life hep fep sep wep bep
(C) 31 (D) 41
life needs food, food needs fep qep gep qep gep
12. (A) 3527 (B)  2357 light hep
(C) 5723 (D) 7532
13. (A) Brown (B)  Green 21. What is the code for the word ‘food’?
(C) Yellow (D) Red (A) gep (B) fep
(C) qep (D) pep
14. (A) June (B)  May
(C) November (D) September 22. What is the code for the word ‘sun’?
(A) pep (B) nep
15. (A) Radish (B)  Carrot (C) mep (D) wep
(C) Potato (D) Cabbage
23. Which word is coded as ‘lep’?
Directions for questions 16 to 20: Select the correct alter- (A) gave (B) energy
native from the given choices. (C) to (D) Cannot be determined
1.84 | Logical Ability Test 1

24. what is the code for ‘earth sun and moon’? 33. Who is the actress?
(A) mep kep sep pep (B) mep tep nep sep (A) Anu (B) Sudha
(C) kep qep sep mep (D) sep pep rep tep (C) Mythili (D) Radhika
25. What can be the meaning of ‘fep gep zep sep hep’? 34. The order in which the artists present their programmes
(A) life needs energy and light is
(B) sun gave light and energy (A) pianist, violinist, actress, singer, dancer.
(C) human needs sun and moon (B) actress, violinist, pianist, dancer, singer.
(D) plants need sun and moon (C) actress, violinist, dancer, pianist, singer.
Directions for questions 26 to 30: These questions are (D) Cannot be determined.
based on the following data. 35. If the actress plays third, when does Sudha play?
A, B, C, D, E, F and G are the seven members in a family. (A) Immediately after Radhika.
Among them, there are two couples and each couple has (B) Immediately before the singer.
exactly two children. B, who is married, has no siblings and (C) After the dancer.
he is not married to E, a female, who is also married. D is (D) After Gowri but not immediately.
the father of G. F, the youngest in the family, has a paternal
uncle. A is unmarried while F and C are of the same gender. Directions for questions 36 to 39: These questions are
A and G are of different gender. based on the following data.
A man goes to work in his car on all days except Sun-
26. How is F related to A?
days. There are 4 different parking spaces near his of-
(A) Daughter (B) Niece
fice. Out of these, the cellar and the ground floor are the
(C) Nephew (D) Son
closest to his office while the garage and the parking lot
27. How is C related to E? are the farthest. Whenever he comes to office, he parks
(A) Mother (B) Daughter his car in one of the four parking spaces. It is known that
(C) Daughter-in-law (D) Mother-in-law (i) the parking lot is open on all days of the week but
28. How is A related to E? he can afford it for only 2 days a week.
(A) Brother-in-law (B) Husband (ii) the Garage is open on Mondays, Tuesdays and
(C) Brother (D) Father-in-law Thursdays but he can use it only once a week.
29. Which among the following is the complete group of (iii) he can use the cellar for 2 days of the week but
females in the family? he cannot use it on Tuesdays, Thursdays and
(A) E, F and G (B) A, C, E and F Saturdays.
(C) C, E, B and F (D) C, E, F and G (iv) he can use the ground floor for one day of the week,
30. How is C related to G? but not on Mondays, Wednesdays, and Fridays.
(A) Mother (B) Father (v) as he is always late on Mondays, he likes to park
(C) Uncle (D) Grandmother his car close to his office.
Directions for questions 31 to 35: These questions are 36. If he parks his car on the ground floor on Tuesday and
based on the following data. in the parking lot on Wednesday, then where should he
Five artists - a violinist, a pianist, a singer, a dancer park it on Friday?
(A) Cellar (B) Parking lot
and an actress-have to present their work one after the
(C) Ground floor (D) Garage
other, not necessarily in that order. The five artists are
Anu, Gowri, Radhika, Sudha and Mythili. Also 37. If he uses the garage on Tuesday and the parking lot on
(i) Mythili presents her work after the singer - not Thursday, where does he park his car on Wednesday?
necessarily immediately. (A) Cellar (B) Parking lot
(ii) the dancer presents her work immediately after (C) Ground floor (D) Either (A) or (B)
Radhika. 38. If he uses the garage on Tuesday and parking lot on
(iii) Gowri, the violinist, plays second. Saturday, then which is the place he uses on Thursday?
(iv) Radhika’s item is not immediately next to Gowri’s. (A) Ground floor or Parking lot
(v) Anu is not an actress. (B) Cellar
(C) Ground floor
(vi) Radhika is the singer.
(D) Parking lot
31. Who is the dancer?
39. If he uses the parking lot on Tuesday, then what must he
(A) Mythili (B) Sudha
use on Thursday?
(C) Anu (D) Radhika
(A) Parking lot
32. If the actress plays first, who plays third? (B) Garage or Ground floor
(A) Anu (B) Mythili (C) Ground floor
(C) The dancer (D) Radhika (D) Garage
Logical Ability Test 1 | 1.85

Directions for questions 40 to 42: These questions are V


X
based on the following information.
Eight persons – Anand, Brijesh, Chandak, Dweepesh,
Sayan, Jagat Rupak and Palak – are sitting around a
square table such that two persons are sitting along W
each side. The following information is known about
them. Z Y
(i) Jagat, who is sitting to the immediate right of
Rupak, is sitting opposite Chandak who is sitting Five trains – T1, T2, T3, T4 and T5 run only on two days (Sat-
to the immediate right of Brijesh. urday and Sunday), along the following routes, between
(ii) Sayan is sitting opposite Dweepesh, who sits these cities.
along the same side as Brijesh. T1 : Y – X – V
(iii) Palak is not sitting along the same side as Sayan. T2 : Z – Y – X – V
T3 : Z – Y – W – V
40. Who is sitting along the same side as Chandak? T4 : Z – X – W – V
(A) Anand (B) Palak T5 : Z – X – W
(C) Sayan (D) Rupak Route Y – W cannot be used on Sunday. On any day, no two
41. Who is sitting opposite Rupak? trains are scheduled to run on the same track connecting two
(A) Palak (B) Anand adjacent cities.
(C) Brijesh (D) Data inadequate Each train should run exactly once in these two days.
46. T4 can run
42. Who is sitting to the immediate right of Sayan? (A) only on Saturday
(A) Anand (B) Rupak (B) only on Sunday
(C) Chandak (D) Data inadequate (C) on either day
(D) only if W – Y route is used on Sunday.
Directions for questions 43 to 45: These questions are
47. Which of the following is NOT true?
based on the following information.
(A) T2 and T4 can be scheduled to run on the same day.
Three people are to be selected from a group of six
(B) T5 cannot be scheduled to run on Sunday.
people – M, N, P, Q, R and S under the following con-
(C) T3 can be scheduled to run on Saturday.
straints.
(D) T4 and T1 can be scheduled to run on the same day.
(i) If M is not selected, then N is selected.
(ii) If P is not selected, then Q is selected. Directions for questions 48 to 50: These questions are
(iii) If R is not selected, then S is selected. based on the data given below.
Six persons – A, B, C, D, E and F – stand in a row. A is
43. In how many ways can the team be selected?
to the left of B. C is to the right of D. E and F have two
(A) 2 (B) 4
persons standing between them and neither of these
(C) 8 (D) None of these
two persons is C or A.
44. Who must be there in the team?
48. What is the total number of possible arrangements?
(A) N (B) Q (A) 2 (B) 4
(C) S (D) None of these (C) 6 (D) 5
45. Which of the following is a possible team? 49. Who among the following stand at the extreme ends of
(A) P, R, S the row?
(B) P, Q, S (A) E and F (B) E and C
(C) M, Q, S (C) A and C (D) F and A
(D) More than one of the above
50. If A sits to the immediate left of E, then who sits to the
Directions for questions 46 and 47: Answer the questions immediate right of B?
on the basis of the following information. (A) D
(B) F
Shown below is the layout of the major cities of a state and (C) C
the rail tracks, connecting those cities. (D) Cannot be determined
1.86 | Logical Ability Test 1

Answer Keys
1. B 2. C 3. D 4. C 5. D 6. D 7. B 8. D 9. A 10. B
11. B 12. D 13. A 14. B 15. D 16. C 17. A 18. C 19. C 20. B
21. C 22. A 23. D 24. A 25. A 26. B 27. D 28. A 29. D 30. D
31. A 32. A 33. B 34. D 35. B 36. A 37. D 38. C 39. D 40. B
41. A 42. B 43. C 44. D 45. C 46. B 47. D 48. B 49. C 50. D

Hints and Explanations


1. The given series can be expressed as follows. 14. All the given months except May have 30 days where as
22, 33, 52, 73, 112 where 2, 3, 5, 7, 11 are prime numbers. in May there are 31 days. Choice (B)
The next in the series is 133 = 2197 Choice (B) 15. All except Cabbage, grow under the soil. Choice (D)
2. The given series can be expressed as follows. 16. Word: C E N T U R Y
(11 × 2) + 3 = 25; (25 3) + 2 = 77; (77 × 2) + 3 = 157
(157 × 3) +2, = 473; (473 2) + 3 = 949 Choice (C) Logic: –2 +2 –2 +2 –2 +2 –2
3. The given series can be expressed as follows.
32 + 3, 52 + 5, 72 + 7, 112 + 11, 132 + 13 with 3, 5, 7, 11,
Code: A G L V S T W
13 being prime numbers. The next number in the series
is 172 + 17 = 289 + 17 = 306 Choice (D) Similarly,
Word: S A C H I N
4. The given series is the series of alternate prime num-
bers. The next in the series is 101. Choice (C)
Logic: –2 +2 –2 +2 –2 +2
5. The given series can be expressed as
6 × 3 + 1 = 19; 19 × 3 + 1 = 58; 58 × 3 + 1 = 175
175 × 3 + 1 = 526; 526 × 3 + 1 = 1579 Choice (D) Code: Q C A J G P
6. 24 : (24) : : 32 : (32)
2 2
∴ QCAJGP is the code for ‘SACHIN’. Choice (C)
Square of the first number is the second number. 17. Word: A M B I T I O N
(32)2 = 1024. Choice (D)
7. 4 : 27 : : 25 : _____ Logic: +6 +6 +6 +6 +6 +6 +6 +6
(2)2 : (3)3 : : (5)2 : (6)3
This is of the form (n)2 : (n + 1)3.
(6)3 = 216 is the next number. Choice (B) Code: G S H O Z O U T

8. BILK : DLPP : : HMTO : _____ Similarly,


B I L K Code: V X O T Z U A Z
+2 +3 +4 +5
D L P P Logic: –6 –6 –6 –6 –6 –6 –6 –6
Similarly, H M T O
+2 +3 +4 +5
Word: P R I N T O U T
J P X T
Hence, JPXT is the next term. Choice (D) ‘PRINTOUT’ is coded as ‘VXOTZUAZ’ Choice (A)
9. Cricket is a type of game. 18. The code for the letter whose place value is ‘n’ is the
Similarly, kangaroo is type of animal. Choice (A) letter whose place value is (27 – n).
∴ MAHENDAR is coded as NZSVMWZI.
10. Bus is driven by a driver.
 Choice (C)
Similarly, jockey rides a horse. Choice (B)
19. We sit on a chair and chair is called desk. Choice (C)
11. All the given numbers except
21 are prime numbers where as 20. We use fans when we want air and spectacles means
21 is a composite number. Choice (B) fan.  Choice (B)
12. All the given numbers except Solutions for questions 21 to 25:
7532 are odd numbers whereas The given statements and their codes are as follows.
7532 is an even number. Choice (D) (1) earth gets heat from sun – pep tep nep mep wep
(2) moon gets light from sun – hep kep tep pep nep
13. All the given colours except brown are the colours in
(3) sun gave energy to plants – bep pep dep zep lep
(rainbow) VIBGYOR. Choice (A)
Logical Ability Test 1 | 1.87

(4) human gets food from plants – qep tep nep dep rep Hence, the code for ‘light’ is ‘hep’. The code for the
(5) heat and light gave life – hep fep sep wep bep remaining word in (6), i.e., ‘life’ is ‘fep’.
(6) life needs food, food needs light – fep qep gep qep The words ‘gets’ and ‘from’ are common for (1), (2)
gep hep and (4).
From (6), the words ‘food’ and ‘needs’ are repeated Similarly the codes ’tep’ and ‘nep’ are common. But
and the codes ‘qep’ and ‘gep’ are repeated. And now the codes for ‘gets’ and ‘from’ cannot be individually
from (4) and (6) as only the word food is repeated the obtained. Except the word ‘and’ and the code ‘sep’ in
code for ‘food’ is ‘qep’ and hence the code for ‘needs’ (5), all other words and codes are used in at least one of
is ‘gep’. the other sentences. Hence, the code for ‘and’ is ‘sep’.
From (2) and (6) the word ‘light’ and the code ‘hep’ are By using comparison and elimination procedures we
common. can find the codes for other words.

Word earth sun heat gets/ moon light gave energy/ plants human and life needs food
from to
Code mep pep wep nep/tep kep hep bep zep/lep dep rep sep fep gep qep

21. The code for ‘food’ is ‘qep’. Choice (C) 27. C is E’s husband’s mother. Hence, C is the mother-in-
22. The code for ‘sun’ is ‘pep’. Choice (A) law of E. Choice (D)
23. Either ‘energy’ or ‘to’ is coded as ‘lep’. Choice (D) 28. A is E’s husband’s brother. Hence, A is the brother-in-
law of E. Choice (A)
24. The code for ‘earth sun and moon’ is ‘mep kep sep
pep’. Choice (A) 29. C, E, F and G are the females. Choice (D)
25. The meaning of ‘fep gep zep sep hep’ can be ‘life needs 30. C is the grandmother of G. Choice (D)
energy and light’ . Choice (A) Solutions for questions 31 to 35:
Solutions for questions 26 to 30: From the given information, we have
It is given that there are seven members in the family. Gowri, the violinist, plays second. Radhika is a singer who
A, B, C, D, E, F and G. does not come immediately after Gowri, Radhika cannot
There are two couples in the family and each couple be 3rd since the dancer presents her work immediately after
has exactly two children. Radhika. Radhika cannot be 1st or 5th so, Mythili comes in
F, the youngest in the family has a paternal uncle which the 5th place. Radhika has to be 4th and dancer 5th.
implies that F’s father has a brother. B, who is a male has
So, Anu is a Pianist and Sudha is an actress and they come
no siblings and he is married. Hence, B will come in the
in the first and the third places – not necessarily in that
first generation and he has two children and a spouse.
order. Thus we have
E is married and is a female. Hence, E is the mother of
F and she is married to D. G is the child of D and A is Order Artist Profession
the brother of D. Hence, C is the wife of B. 1
F and C are of same gender. Hence, F is female. A and
2 Gowri Violinist
G are of different gender. Since A is male, G is female.
The given information can be represented in the dia- 3
gram as follows:
4 Radhika Singer
husband wife
(C) 5 Mythili Dancer
(B)
31. Mythili is the dancer. Choice (A)
son
32. If actress plays first, then Anu plays third. Choice (A)
brother son
husband (A)
33. Sudha is the actress. Choice (B)
(E)
wife (D) 34. It is not clear whether the actress or the pianist plays
first. Hence, cannot be determined. Choice (D)
35. If the actress (Sudha) plays third, then Sudha plays
(G) (F) immediately before the singer.(Radhika) Choice (B)
daughter daughter
Solutions for questions 36 to 39:
26. F is A’s brother’s daughter. Hence, F is the niece of A. Let us tabulate the days of the week when different slots are
 Choice (B) available.
1.88 | Logical Ability Test 1

Ground Parking As can be seen, on Thursday only ground floor can be


Cellar Garage used (because, if he uses the parking lot on Thursday,
Floor lot
then no day is available for the ground floor).
Monday X   Choice (C)
Tuesday X  39.
Wednesday X Mon Tue Wed Thur Fri Sat
Thursday X  Cellar  X

Friday X Ground Floor X X X X


Garage X X X X X
Saturday X
Parking Lot X 
Can use for 2 days of Only one Only one 2 days of
the week day day the week Now, garage has only Thursday and no other day left.
Hence, garage should be used on Thursday.
In addition, on Mondays he should park the car clos-
 Choice (D)
est to his office, i.e., cellar or ground floor. But since
ground floor cannot be used on Mondays, only the cel- Solutions for questions 40 to 42:
Let us represent the persons by the first letters of each
lar can be used on Mondays. Hence, garage cannot be name.
used on Mondays. Now, we can answer the questions. From (i), we get the following possibilities
3 6. If ground floor is used on Tuesday & parking lot on Case (a),
Wednesday; then we have
C
Ground Parking
Cellar Garage
Floor lot
B
Monday  X X X
Tuesday X  X
Wednesday X X X 
Thursday X X  Case (b) R J
Friday X X
Saturday X X X
Total 2 1 1 2 C B
As can be seen, he has to park his car in the cellar on
Friday. Otherwise he won’t be able to use the cellar 2
times a week. Choice (A)
3 7. On Wednesday, he can park his car in the cellar or in the R
parking lot. Choice (D)
Mon Tue Wed Thur Fri Sat J
Cellar  X X X
From (ii) as Brijesh and Dweepesh are sitting along the
Ground Floor X X X X X 
same side, case (b) is not possible.
Garage X  X X X X From (ii) we get
Parking Lot X X  C

38
Mon Tue Wed Thur Fri Sat Total B
Cellar  X X X 2
S D
Ground X X X  X X 1
Floor
Garage X  X X X X 1
R J
Parking Lot X X  2
From (iii) the possibility is as follows.
Logical Ability Test 1 | 1.89

P C run on the same track connecting two adjacent cities,


Z – Y and W – V should not run on Saturday (∵ T3 is
A B covering the route).
As T2 is covering Y – X – V on Sunday, T1 has to be
scheduled on Saturday. Similarly, T5 is to scheduled on
S D Saturday.
Finally, T1, T5, T3 are to be scheduled on Saturdays and
R J T2 and T4 are to be scheduled on Sundays.
46. T4 should on run Sunday. Choice (B)
40. Palak is sitting along the same side as Chandak. 47. From the choices,
 Choice (B) T4 and T1 can be scheduled on two different days.
41. Palak is sitting opposite Rupak. Choice (A)  Choice (D)
42. Rupak is sitting to the immediate right of Sayan. Solutions for question 48 to 50:
 Choice (B) The data is as given below:
Solutions for questions 43 to 45: (i) Six persons – A, B, C, D, E and F stand in a row.
From (i), M and N can be selected as follows. Left of
→ Only M is selected. (ii) A B
→ Only N is selected. Right
→ Both M and N are selected. (iii) D C
This implies, at least one between M and N must be E F
(iv) –– ––
selected. F E
Similarly, from (ii) and (iii) x C/A (neither C nor A)
At least one among P and Q must be selected. Let us make all the possible arrangements as per
At least one among R and S must be selected. the above data.
As only 3 persons are to be selected, both M and N, –  E/F –  –  F/E  –
both P and Q and both R and S cannot be selected. 1   2   3 4   5  6
∴ Exactly one among M and N, exactly one among Positions 2 and 5 would be occupied by E or F. A can-
P and Q and exactly one among R and S must be not be at 3 and 4 (condition (iv)) and also A cannot be
selected. at 6 (condition (ii)). Hence, A must be at position 1.
⇒ M / N, P / Q, R / S Similarly, C must be at position 6. Hence, we will get
the following arrangement:
43. The number of possible ways to select the team.
= 2 × 2 × 2 = 8 Choice (C) A  E/F  B/D  D/B  F/E  C
Therefore, the total number of arrangements are 4.
44. There is no such person who must always be there in
the team. Choice (D) 48. Total possible arrangements are four. Choice (B)
45. M, Q, S is a possible team. Choice (C) 49. A and C stand at extreme ends. Choice (C)
Solutions for questions 46 and 47: 50. The arrangement will be as shown below:
Given that the route Y – W cannot be used on Sunday. A    E    B/D    D/B    F    C
Hence T3 can be scheduled to run on Saturday. As it is Hence, the person sitting to the immediate right of B
given that, on any day, no two trains are scheduled to cannot be determined. Choice (D)
Logical Ability Test 2
Number of Questions: 35Time: 35 min

Directions for questions 1 to 4: These questions are based 6. What percentage of the residents, who own a car, also
on the following data. own at least another vehicle?
Out of a group of 315 students who went to Mumbai, 125 (A) 662/3% (B) 132/3%
visited Essel World, 140 visited Lumbini Garden and 160 (C) 28 /7%
4
(D) 331/3%
visited Film Nagar. Twenty Five of them visited all the three 7. If 15 residents do not own any of the three vehicles,
places while 200 visited exactly one of the three places. The then how many residents are there in the colony?
number of students, who visited exactly 2 out of the three (A) 100 (B) 200
places, is five times as many as those who have not visited (C) 300 (D) 400
any of the places. 8. What percentage of the colony residents own exactly
1. How many did not visit any of the three places? one type of vehicle?
(A) 75 (B) 25 (A) 15% (B) 25%
(C) 125 (D) 15 (C) 55% (D) 75%
2. How many students visited not more than one place? 9. What percentage of the residents own a scooter or a car
(A) 200 (B) 180 but not a bicycle?
(C) 250 (D) 215 (A) 65% (B) 55%
(C) 75% (D) 45%
3. If the number of students who visited at least one of the
two places, Lumbini Gardens and Film Nagar is 255, Directions for questions 10 to 12: These questions are
then how many students visited only Essel World? based on the following data.
(A) 45 (B) 25 There are three trade unions - Viram, Vishram and Be-kam
(C) 125 (D) 75 - and three thousand six hundred workers in a company.
4. If the number of students who visited at least one of the Becoming a member of a trade union is optional. A worker can
two places, Lumbini Gardens and Film Nagar is 255, be a member of more than one of the three trade unions also.
then how many students visited only one of the two There are 500 workers who are members of at least two
places, Lumbini Gardens and Film Nagar (and not any trade unions while Vishram has 1400 members. There are
of the other two places)? 100 workers who are members of only Viram and Be-kam,
(A) 215 (B) 125 whereas 200 Vishram members also are Be-kam members;
(C) 155 (D) 175 550 workers are members of only Be-kam, whereas 20% of
Viram members are members of exactly one more union.
Directions for questions 5 to 9: These questions are based An eighth of all the workers in the company are members of
on the following data. exactly two unions.
In a colony, a survey was conducted regarding the own-
10. How many workers are members of only Viram or only
ership of three different types of vehicles - car, scooter
Be-kam?
and bicycle.
(A) 3200 (B) 2700
-- The number of residents owning all three vehicles
(C) 1400 (D) 1700
is the same as those owning none.
-- The number of residents owning any two out of 11. If 10 workers give up their Be-kam membership and
the three vehicles is the same as those owning any take up Vishram membership, then how many workers
other two which in turn is the same as those own- will now have membership of all the three unions?
ing none of the three. (A) 40 (B) 50
-- The number of residents owning scooters alone is (C) 60 (D) 45
the same as those owning cars alone and each in 12. How many workers are members of Vishram but not
turn is twice those owning bicycles alone. members of Be-kam?
-- Half the number of residents who own a bicycle (A) 400 (B) 800
also own at least one of the other two vehicles. (C) 1200 (D) 1600
5. If the number of residents who own only bicycles is Directions for questions 13 to 15: These questions are
150, then what is the total number of residents in the based on the following data.
colony? In a school, 60% of the students passed in English, and 25%
(A) 500 (B) 1000 of the students who passed in English passed in the foreign
(C) 750 (D) 1250 language also, whereas 662/3% of the students who passed
Logical Ability Test 2 | 1.91

in the foreign language failed in English. Twenty students 22. A watch, which loses uniformly was observed to be 5
failed in both English and the foreign language. minutes fast at 5:00 p.m. and 4 minutes slow at 8:00
13. What percent of the students passed in exactly one of p.m. on the same day. When did the watch show the
the two subjects – English and the foreign language? correct time?
(A) 15% (B) 65% (A) 6:20 p.m. (B) 6:40 p.m.
(C) 45% (D) 75% (C) 7:00 p.m. (D) 6:30 p.m.
14. The students who failed in exactly one subject are 23. Two clocks show the correct time at 3:00 p.m. One
allowed to take a re-exam and it was found that the clock gains 4 minutes in an hour, while the other loses
number of students who passed in both the subjects 4 minutes. At 6:00 p.m. on the same day, by how much
increased by 20%. What is the least value for the per- time will the two clocks differ?
centage of students in the school who pass only in (A) 8 minutes (B) 16 minutes
English? (C) 12 minutes (D) 24 minutes
(A) 42% (B) 46% 24. The minutes hand of a clock overtakes the hours hand
(C) 34% (D) 28% after every 60 minutes of correct time. How much time
15. All the students, who failed in one or more subjects, does the clock gain or lose in eleven hours of normal
are given grace marks and it was found that the number time?
of students passing in exactly one subject went up by 4 (A) Gains 11 minutes (B) Loses 11 minutes
and the number of students who failed in both the sub- (C) Gains 60 minutes (D) Loses 60 minutes
jects dropped by 40%. What percent of the school now 25. If the time in a clock is 8 hours 20 minutes, then what
pass in both subjects? time does its mirror image show?
(A) 40% (B) 15% (A) 3 hours 20 minutes (B) 3 hours 50 minutes
(C) 12% (D) 17% (C) 3 hours 40 minutes (D) 4 hours 40 minutes
Directions for questions 16 to 35: Select the correct alter- 26. If 22nd April, 1982 was a Thursday, then what day of
native from the given choices. the week was 3rd November, 1982?
16. The angle between the two hands of a clock at 9:00 a.m. (A) Monday (B) Wednesday
is 90°. What will the angle between them be one minute (C) Friday (D) Sunday
later? 27. If 30th June, 1989 was a Friday, then what day of the
(A) 84.5° (B) 95.5° week was 17th September, 1993?
(C) 101° (D) 79° (A) Monday (B) Wednesday
17. The angle between the two hands of a clock at 5:00 (C) Friday (D) Sunday
p.m. is 150°, what will the angle between them be one 28. If 10th April, 1963 was a Wednesday, then what day of
minute later? the week was 23rd August, 1959?
(A) 144.5° (B) 155.5° (A) Sunday (B) Monday
(C) 161° (D) 139° (C) Friday (D) Tuesday
18. What is the angle between the two hands of a clock at 29. If today is Sunday, then what day of the week will be
7:28 p.m.? the 426th day from today?
(A) 56° (B) 58°
(A) Saturday (B) Friday
(C) 60° (D) 63°
(C) Tuesday (D) Wednesday
19. At what time between 6 and 7 O’ clock will the angle
between the two hands of a clock be 50°? 30. If today is Wednesday, what day will it be, 1 year and 10
(A) 6 hr.419/11 min (B) 6 hr. 237/11 min days from today?
(C) 6 hr. /11 min
2
(D) Either (A) or (B) (A) Sunday
(B) Friday
20. At which of the following times between 8 and 9 O’ (C) Monday
clock, will the angle between the hands of the clock be (D) Cannot be determined
120o?
(A) 8 hr.219/11 min (B) 8 hr. 169/11 min 31. In a year, if two successive months start with the same
(C) 8 hr. 34 /11 min
6
(D) 8 hr. 293/11 min day of the week, then the year is
(A) a century year.
21. A watch, which gains uniformly was observed to be 10
(B) a leap year.
minutes slow at 12 noon and 5 minutes fast at 6:00 p.m. (C) a non-leap year.
on the same day. When did the watch show the correct (D) only a non-leap century year.
time?
(A) 2:00 p.m. (B) 3:00 p.m. 32. If a year has 53 Sundays, how many Saturdays will be
(C) 4:00 p.m. (D) 5:00 p.m. there in that year?
1.92 | Logical Ability Test 2

(A) 52 (B) 53 write his last exam three days later and that he would
(C) 54 (D) Cannot be determined leave three days after his last exam. If his last exam is
on a Saturday, on which day of the week will my cousin
33. Three days ago I met my friend and asked him to lend
arrive?
me his maths book. He promised that he will lend it on (A) Saturday (B) Friday
the eighth day from that day. If today is Thursday, on (C) Tuesday (D) Sunday
which day will he lend me the book?
35. If the second Sunday of a month falls on the 8th, what
(A) Friday (B) Tuesday
is the date of the fourth day after the third Saturday of
(C) Monday (D) Sunday
that month?
3 4. There are three more days to go for my cousin to arrive. (A) 2 (B) 15
Eight days ago when I talked to him he said he would (C) 25 (D) 21
Answer Keys
1. D 2. D 3. A 4. C 5. B 6. D 7. C 8. D 9. A 10. D
11. B 12. C 13. D 14. A 15. D 16. B 17. A 18. A 19. D 20. A
21. C 22. B 23. D 24. C 25. C 26. B 27. C 28. A 29. A 30. D
31. C 32. D 33. B 34. D 35. C

Hints and Explanations


Solutions for questions 1 to 4: 3. Number of students who visited at least one of LG and
Total = 315 FN
= 255 ⇒ b + x + z + t + y + c = 255 … (4)
EW = 125 LG = 140
We know a + b + c + x + y + z + t
a x b
= 315 − r = 300 …… (5)
t
(5) − (4) ⇒ a = 45 visited only EW. Choice (A)
y z
4. Number of students who visited only one among LG
c and FN = b + c = 200 − a = 200 − 45 = 155.
 Choice (C)
FN = 160
r
Solutions for questions 5 to 9:
EW - Essel World 100%
LG - Lumbini Gardens C S
FN - Film Nagar 2y
x 2y
25 visited all three ⇒ t = 25 x
125 visited EW ⇒ a + x + y + t = 125 x x
⇒ a + x + y = 100 …. (1) y
140 visited LG ⇒ b + x + t + z = 140 x
⇒ b + x + z = 115 …. (2) B
160 visited FN ⇒ c + y + z + t = 160
The first three statements can be represented as shown
⇒ c + y + z = 135 …. (3)
in the diagram and hence, 5x + 5y = 100
Number of students who visited exactly 2 places
(because we have taken x & y as percentages)
= 5 times those who did not visit any.
or x + y = 20 …. (1)
⇒ x + y + z = 5r
From the fourth condition, we get (y + 3x) = B
200 students visited exactly one place = a + b + c = 200.
i.e., y = 3x …. (2)
1. Total no of students = 315 From equations (1) and (2).
⇒ a + b + c + x + y + z + t + r = 315 We get x = 5% and y = 15%.
⇒ 200 + 5r + 25 + r = 315
5. y = 15% = 150.
6r = 90 ⇒ r = 15
Hence, total number % of residents
∴ 15 did not visit any of the three places. Choice (D)
150 × 100
2. Number of students who did not visit more than one =x= = 1000. Choice (B)
15
place = Number of students who visited exactly one
place + those who did not visit any = a + b + c + r 3x 15 1
6. = = = 331/3%. Choice (D)
= 200 + 15 = 215. Choice (D) 3 x + 2 y 45 3
Logical Ability Test 2 | 1.93

15 -- 25% of 60%, i.e., 15% of the school passed in


7. x = 5% = 15. Hence total = = 300. Choice (C) both English and Foreign Language.
0.05
-- Since 662/3% of the students who passed in
8. 2y + 2y + y = 75%. Choice (D) Foreign Language failed in English, 331/3% of stu-
9. 2y + 2y + x = 65%. Choice (A) dents who passed in Foreign Language passed in
Solutions for questions 10 to 12: English also, i.e.,1/3 Foreign Language = 15%
Represent in various segments as in the following diagram, ⇒ Foreign Language = 45%
Total = 3600
So, we have only passed in (English) = 60 − 15 = 45%,
Only Foreign Language pass = 45 − 15 = 30%
Viram Vishram
Passed both in English & Foreign Language = 15%.
p u q A total of 90% passed in at least one of the subjects. So,
s
10% failed in both.
w v 20
Number of students in the school = = 200.
r 0.10
t Be-kam
13. 45% + 30% = 75%. Choice (D)
Members of at least two unions = u + v + w + s = 500 14. 20% of 15% = 3% increase in pass in both subjects.
 ….. (1) Hence, least value of pass only in English will come
Vishram members = q + s + u + v = 1400 ……… (2) when all the new people who pass in both subjects
Only Viram and Be-kam = w = 100 ……. (3) are from the group which passed only in English. i.e.,
Vishram and Be-kam = s + v = 200 ….. (4) 45 – 3 = 42%. Choice (A)
Only Be-kam = r = 550 …… (5) 15. 40% of 20 students = 8 students. Out of this, 4 students
Members of Viram who are members of only passed in exactly one subject. Hence, the remaining 4
one more union = w + u = 20% of (p + u + s + w).. (6) students (which is 2% of the school strength) pass in
u + v + w = 1/8 (Total workers) = 450. ….. (7) both subjects. So, pass in both the subjects = 15 + 2 =
From (1), (3) and (4), u = 200. 17%. Choice (D)
From equations (2),
Solutions for questions 16 to 35:
q = 1400 – u – (s + v)
= 1400 – 200 – 200 = 1000 16. As the minute hand is 5.5°/minute faster than the hour
From (7), v = 450 – 200 – 100 = 150 hand and the minute hand is ahead of the hour hand,
From (4), s = 200 – 150 = 50 the angle after one minute, i.e., at 9:01 a.m., will be
From (6), p = 1150 90° + 5.5° = 95.5°. Choice (B)
t = 3600 – (p + q + r + s + u + v + w) 17. The angle between the hands will change by 5.5°/min.
= 3600 – (1150 + 1000 + 550 + 50 + 200 + 150 + 100) In this case, since the minute hand is behind the hour
= 3600 – (3200) = 400. hand the angle will decrease by 5.5°. Hence, the angle
Now, we have all the figures and the questions can be at 5:01 = 150 – 5.5 = 144.5°. Choice (A)
answered. 11
18. The angle = (28) − 30(7) = 56°. Choice (A)
1 0. p + r = 1150 + 550 = 1700. Choice (D) 2
11. Since 10 workers have given up their Be-kam mem- 11  2
( m) − 30( h) ⇒ m = (30h ± θ)   .
bership and taken Vishram membership, it means these 19. θ =
2  11
10 workers were initially Be-kam members but not
Vishram members, i.e., they must be a part of r or w. Here h = 6 and θ = 50°
When they give up Be-kam and take up Vishram, they  2  460 or 260
⇒ m = (180 ± 50°)   = .
will move to q or u respectively. So, s does not undergo  11 11 11
any change at all. Hence, 50 is the answer. Choice (B)
9
12. q + u = 1000 + 200 = 1200. Choice (C) The two hands are 50° apart at 6 hr. 41 min and at
11
Solutions for questions 13 to 15: 7
6 hr . 23 min . Choice (D)
100% 11
E = 60% FL  2
20. m = (30h ± θ)  
 11
45% 15%
Here h = 8 and θ = 120°
 2 5 9
⇒ m = (240 ± 120)   ⇒ 65 or 21
 11 11 11
1.94 | Logical Ability Test 2

5 Number of odd days = 9 + 5 =14 = 0 odd days


If m = 65 , then the time will be more than Hence, 17th September 1993 was a Friday. Choice (C)
11
9 O’clock. 28. Number of days from 10th April, 1963 to 23rd August,
9 1963
∴ The time is 8 hr. 21 min  Choice (A) Month: April + May + June + July + August
11
Days: 20 + 31 + 30 + 31 + 23 = 135
21. In 6 hours, the watch gained 15 minutes. Number of odd days in 135 days = 135/7 = 20 days
∴ In order to gain 10 minutes, (the initial difference) 23rd August 1963 is 2 days to Wednesday i.e., Friday
it takes 4 hours. Number of odd days form 23rd August 1963 to 23rd
∴ At 4:00 p.m. it shows the correct time.Choice (C) August 1959 are five odd days.
22. In 3 hours, the watch loses 9 minutes. Hence, 23rd August 1959 is five days back to Friday is
∴ In order to lose 5 minutes, (the initial difference) it Sunday. Choice (A)
takes 5 x 3/9 hours, i.e., 1 hour 40 minutes. 29. Number of odd days in 426 days
∴ At 6 : 40 p.m., it shows the correct time. = 426/7 = 60 complete weeks + 6 odd days.
 Choice (B) 6th day after Sunday is a Saturday. Choice (A)
23. In one hour, the clocks will differ by (4 + 4) minutes 30. Whether the given year is a leap year or a non-leap
i.e., 8 minutes. year, is not given, hence the answer cannot be deter-
∴ In three hours they will differ by 24 minutes.
mined. Choice (D)
 Choice (D)
31. Two successive months can start with the same day of
5
24. As 60 < 65 , the watch is gaining time. the week, only if the previous month does not have any
11 odd days. This is possible only when February has 28
5 5 days. Hence, the year is a non-leap year. Choice (C)
The gain is 65 − 60 = 5 minutes per hour.
11 11 32. A leap year has 52 weeks and two additional days,
 60  while a non-leap year has 52 weeks and one additional
∴ In eleven hours, the watch will gain 11 
 11  day. In any year each day of the week occurs at least 52
= 60 minutes. Choice (C) times.
25. Actual time + mirror image time = 12 hours A non-leap year starts and ends with the same day of
∴ Mirror image time = 12 – 8 : 20 = 3 : 40 the week. In a leap year the first two days of the
= 3 hours 40 minutes. Choice (C) year repeat. As it is not known whether the 53rd
Sunday is the last day of the year or not, the num-
26. It is given that 22nd April, 1982 was a Thursday. ber of Saturdays could be 52 or 53. Choice (D)
Number of days from 22nd April 1982 to 3rd Novem-
ber 1982. Month: April + May + June + July + August 33. Today is a Thursday. Hence, the required day
+ September + October + November = Thursday − 3 (ago) + 8 (from 3days ago)
In each month number of odd days : = Thursday + 5 = Tuesday. Choice (B)
1 + 3 + 2 + 3 + 3 + 2 + 3 + 3 = 6 odd days 34. Eight days ago my cousin said that his last exam is three
6th day after Thursday is Wednesday. Choice (B) days later i.e. five days ago from today. He is going to
27. Number of odd days from 30th June, 1989 to 30th June, arrive three days later from today. i.e. eight days after
1993 are five. the last exam. As the last exam is on a Saturday, he is
Number of days from 30th June, 1993 to 17th arriving on a Sunday. Choice (D)
September, are 35. Since, 8th is the second Sunday, 7th is the first Saturday.
Month: July + August + September Hence, the third Saturday is on 21st and four days later
Odd days : 3 + 3 + 3 = 9 it is 25th. Choice (C)
PART II
Engineering Mathematics

Engineering Mathematics Test 1������������������������������������������������������������������������������������������������� 2.3


Engineering Mathematics Test 2������������������������������������������������������������������������������������������������� 2.9
(Ordinary Differential Equations, Calculus (Vector Calculus))��������������������������������������������� 2.9
Engineering Mathematics Test 3����������������������������������������������������������������������������������������������� 2.16
(Linear Algebra, Partial Differential Equations)������������������������������������������������������������������ 2.16
Engineering Mathematics Test 4����������������������������������������������������������������������������������������������� 2.23
(Probability And Statistics)�������������������������������������������������������������������������������������������������� 2.23
Engineering Mathematics Test 5����������������������������������������������������������������������������������������������� 2.29
Numerical Methods�������������������������������������������������������������������������������������������������������������� 2.29
This page is intentionally left blank
Engineering Mathematics Test 1
Number of Questions: 25 Time: 60 min.

Directions for questions 1 to 25: Select the correct alterna- 9. Find the maximum value of the function f(x) = 3x4 – 2x3
tive from the given choices. – 6x2 + 6x + 1 in the interval [–1, 2].
39
5x − 1 (A) (B) 2
1. Lt = 16
x→0
x2 − x + 1 − 1
(C) 21 (D) 18
(A) log e 5 (B) 2 log e 5
1 0. Let f(x) and g(x) be two continuous functions in [a, b]
(C) −2 log e 5 (D) 1 and differentiable in (a, b) and g1(x) ≠ 0 for any x ∈
(a, b) then there exists at least one value c ∈ (a, b), by
2. If f(x) = x2 + 5x – 13, if x < 1,
Cauchy mean value theorem which of the following is
x – 8 if x ≥ 1, then Lt f ( x ) = true?
x→0

(A) –8 (B) –7 f 2 ( c ) f ( b ) − g ( a)
(A) =
(C) 7 (D) does not exist g 1 ( c ) g ( b ) + f ( a)
1 g 1 (c) f ( b) + g ( b)
ex (B) =
3. f(x) = 1 . for x ≠ 0 is f 1 (c) f ( a) + f (b)
1− e x f 1 ( c ) f 1 ( b ) − f 1 ( a)
(C) =
    =0   for x = 0 g 1 ( c ) g 1 ( b ) − g 1 ( a)
(A) continuous at x = 0.
(B) not continuous at x = 0. f ' ( c ) f ( b ) − f ( a)
(D) =
(C) continuous everywhere. g ' ( c ) g ( b ) − g ( a)
(D) none of these.
4. If f(x) = a[x – 5] + b[x + 5] is continuous at 11. If the function f(x) = (sin2x) e–2x satisfies Rolle’s theo-
x = 5, then the value of a + b ([x] is the greatest integer  π
rem in the interval  0,  , then the value of c ∈
less than or equal to x) is  2
(A) 0 (B) 1
 π
(C) 5 (D) 2  0,  such that f’(c) = 0 is
2
 x −3
 ;for x ≠ 9 π π
5. If f(x) =  x − 9 is continuous everywhere, (A) (B)
 3k ; for x = 9 8 4
 π π
(C) (D)
then k is equal to –––––. 6 3
1 1
(A) (B)
2 3 12. By using Lagrange’s mean value theorem for f(x) = x (x
1 1 + 2) (x – 1), find c for c ∈ (–1, 0).
(C) (D)
9 18 −1 −3
(A) (B)
6. If y = ax + 3logx + bx has its extreme value at x = 1 and
2
3 4
x = –1, then the value of 2a – b is −2 −4
(A) 3 (B) 0 (C) (D)
3 5
(C) –1 (D) –3
7. If xy = 6, then find the minimum value of 2x + 3y, x, y  x3 − y3  ϑu ϑu
13. If u = tan–1   , then the value of x +y  is
∈ R+.  x+ y  ϑx ϑy
(A) 12 (B) 15
(A) 2 sinu (B) 2 tanu
(C) 9 (D) 6
(C) sin2u (D) 2 cos2u
8. Which of the following functions have neither maxi-
mum nor minimum? 14. If f(x, y) is a homogeneous function of degree n, then
1 the value of
(A) (B) 4x + 7
2x ϑ2 f ϑ2 f
y + x =
(C) e5x + 1 (D) All the above ϑy 2 ϑx ϑy
2.4 | Engineering Mathematics Test 1

ϑf ϑf 4
dx
(A) n.
ϑx
(B) (n – 1)
ϑy 20. Evaluate ∫ ( x − 2)( x − 3)
1
ϑf ϑf 1 1
(C) (n – 1) (D) n. (A) (B)
ϑx ϑy 2 3
2
 x3 + 2 y3 − 4 z3  (C) (D) diverges
15. If u = cos–1  7 , then the value of 3
 x − y 7 − z 7 

ϑu dx
Σ x
x, y, z ϑx
is 21. ∫ 9+ x
0
2 =

(A) 4cotu (B) – 4tanu π


(C) – 4cosu (D) – 4sinu (A) (B)
2
16. If z = ln (x2 + y3), then π π
(C) (D)
ϑz ϑz 6 3
(A) 3x = 2y
ϑx ϑy 22. The area bounded by the cure y2 = x + 2 and y = x – 4
ϑz ϑz (in sq units) is
(B) 3x + 2y =6 5 5
ϑx ϑy (A) 20 (B) 15
6 6
ϑz ϑz
(C) x −y =0 2 3
ϑx ϑy (C) 12 (D) 18 4
3
ϑz ϑz
(D) 2y + 3x =6 23. The area bounded by the curves y2 = 9ax and x2 = 9ay is
ϑx ϑy
(A) 54a2
1 (B) 48a2
∫ (x − 2 x + 3) e − xdx (C) 36a2
2
17. The value of the definite integral
0 (D) 27a2
is –––––. 24. The volume of the solid formed by the revolution of the
(A) 3 + 4e (B) 3 – 4e area A about y – axis is
4 4
(C) 3 + e (D) 3 – e (A) ∫∫ 2πdxdy (B)
A
∫∫ 2πydxdy A

∫∫ πx dxdy (D)
∫∫ πy dxdy
π 2 2
(C)
2
3sec x + 4cosec x
∫ dx = A A
18.
0
sec x + cosec x
25. The taylor’s series expansion of 2ex + 3 sin x about
7π 7 x = 0 is
(A) (B)
4 2
x3 x 4 x5
5π 7π (A) 2 + 5x + x2 + + + +…….
(C) (D) 6 12 24
4 2
x3 x 4 x5
∞ 2 (B) 2 + 5x + x2 – + + +………..
x 6 12 24
19. ∫ 5
dx =
0 (1 + x 2 ) 2 x3 x 4 x5
(C) 2 + 5x + x2 + − + +………
2 1 6 12 24
(A) (B)
3 3 x3 x 4 x5
(D) 2 + 5x + x2 + + + +…………
π π + 12 24
(C) (D)
6 3

Answer Keys
1. C 2. B 3. B 4. A 5. D 6. D 7. A 8. D 9. C 10. D
11. A 12. C 13. C 14. B 15. A 16. B 17. D 18. A 19. B 20. D
21. C 22. A 23. D 24. A 25. B
Engineering Mathematics Test 1 | 2.5

Hints and Explanations

5x − 1 x2 − x + 1 + 1  x −3
1. Lt ×  , for x ≠ 9
x→0
x2 − x + 1 − 1 x2 − x + 1 + 1 5. Given f(x) =  x − 9
 3k , for x = 9

(5x − 1) ( x 2 − x + 1 + 1)
= Lt As f(x) is continuous everywhere
x→0 x2 − x + 1 − 1
f(x) is continuous at x = 9
(5x − 1) x 2 − x + 1 + 1 \ Lt f ( x ) = f (9)
= Lt x →9
x→0 x ( x − 1)
 x − 3
⇒ xLt   = 3k
→9  x − 9 
5 −1 x
x − x +1+1 2
= Lt Lt .
x→0x x→0 x −1  1 
2 2 x
= loge5 × = 2 log e 5  Choice (C) ⇒ Lt   = 3k (By L’Hospital’s Rule)
−1 x →9  1 
1
2. we know that Lt f(x) = Lt− f(x) = Lt+ f(x) ⇒ = 3k
x →1 x →1 x →1 2 9
Lt f(x) = Lt− (x2 + 5x – 13) = –7 1
x →1− x →1
⇒ = 3k
Lt f(x) = Lt+ (x – 8) = –7 6
x →1+ x →1 1
⇒ k = . Choice (D)
\ Lt f(x) = Lt+ f(x) = –7 18
x →1− x →1

⇒ Lt f(x) = –7. Choice (B) 6. Let f(x) = ax2 + 3logx + bx


x →1
3
f1(x) = 2ax + +b
1 x
ex 0 Given at x = –1 and 1 f(x) has extreme values
3. Lt − = =0 ----------- (1)
x→0
1
1− 0 ⇒ f1(–1) = 0 and f1(1) = 0
1− e x
3
f1(–1) = 2a (–1) + +b=0
1 1
−1
ex ex
Lt = Lt +
( )
x → 0+
1
x→0 1 −1 ⇒ –2a + b = 3
1− e x
e x e x −1 ⇒ 2a – b = –3. Choice (D)
1 1 6
          = Lt + = = –1  ------------- (2) 7. Given xy = 6 ⇒ y =
x→0
−1
0 −1 x
e −1
x
3×6
2x + 3y = 2x +
From (1) and (2) Lt − f(x) ≠ Lt + f(x) x
x→0 x→0
18
\ f(x) is not continuous at x = 0. Choice (B) Let f(x) = 2x +
x
4. f(x) = a[x – 5] + b [x + 5] 18
f1(x) = 2 – 2
x
\ f(x) = a ([x] – 5) + b([x] + 5)
36
Lt f(x) =
 Lt−1 {a([x] – 5) + b ([x] + 5) ⇒ f11(x) = 3 which is always positive as x, y and R+
x → 5− x →5 x
= –a + 9b ------ (1) For maximum or minimum f1(x) = 0
18
Lt f(x) =
 Lt+ {a([x] – 5) + b ([x] + 5)} = 2– 2 =0
x → 5+ x →5 x
= 10 b  ---- (2) ⇒ x = 3
As f(x) is continuous at x = 5 6
\ y =
x
⇒ Lt f(x) = Lt f(x)
x → 5− x → 5+
⇒ y = 2
⇒ – a + 9b = 10b \ At x = 3 and y = 2, 2x + 3y has minimum value
⇒ a + b = 0. Choice (A) The minimum value is 2(3) + 3(2) = 12. Choice (A)
2.6 | Engineering Mathematics Test 1

1 ⇒ 3c2 + 2c = 0
8. Choice A: Let f(x) = ⇒ f1(x) ≠ 0 for any real value −2
2x c (3c + 2) = 0 ⇒ c = 0 or c = As c ≠ 0,
3
of x −2
Choice B: f1(x) = 4x + 7 ⇒ f11 (x) ≠ 0 for any real value c= ∈ (–1, 0). Choice (C)
3
of x
 x3 − y3  x3 − y3
Choice C: f2(x) = e5x+1 ⇒ f 21 (x) ≠ 0 for any real value 13. U = tan–1  ⇒ tan u =
 x + y  x+ y
of x
Let x = kx and y = ky then
\ None of the options have either maximum or min-
imum values. Choice (D) k 3 ( x3 − y3 )  x3 − y3 
tanu = = k2  = f(x) say
9. Let f(x) = 3x – 2x – 6x + 6x + 1
4 3 2 k ( x + y)  x + y 
f1(x) = 12x3 – 6x2 – 12x + 6 \ f is a homogeneous function of degree 2
f1(x) = 0 ⇒ 2x3 – x2 – 2x + 1 = 0
∂f ∂f
(x – 1) (x + 1) (2x – 1) = 0 \ By Euler’s theorem x + = = n(f) (n is degree
1 ∂x ∂y
⇒ x = ± 1, of f)
2
∂ ( tan u) ∂ ( tan u)
1 =
x. +y = 2 tan u
Max of f(x) in [–1, 2] = Max {f (–1), f   , f (1), f(2)} ∂x ∂y
 2

{ }
39 ∂u ∂u ∂u
= Max –6, 2, 21 = 21 . Choice (C) = x sec2u . + y sec2u = 2 tan u
16 ∂x ∂y ∂y
∂u ∂u
10. Standard result of Cauchy mean value theorem. \ x +y = sin 2u. Choice (C)
∂x ∂y
 Choice (D)
11. Given f(x) = e sin 2x
–2x 14. Given f(x, y) is a homogeneous function of degree
∂f ∂f
 π n then by Euler’s theorem we know that x  + y  
Clearly f(x) is continuous on 0,  and differentiable ∂x ∂y
 2
=nf ------- (1)
 π  π
in  0,  and f (0) = f   then there exists c ∈ Differentiating (1) w. r. to y partially we have
 2  2
 π ∂ 2 f ∂f ∂2 f ∂f
 0,  such that f1(c) = 0 x. + + y� 2 = n�
2 ∂x ∂y ∂y ∂y ∂y
f1(x) = sin 2x (–2e–2x) + 2cos2xe–2x ∂2 f ∂2 f ∂f
f1(c) = 2e–2c (cos2c – sin2c) = 0 ⇒ y + x = ( n − 1)  Choice (B)
∂y 2
∂x ∂y ∂y
⇒ sin2c = cos2c or tan2c = 1
π  π  x3 + 2 y3 − 4 z3 
⇒ c = ∈  0,  15. U = cos–1  7
8  2  x − y 7 − z 7 
π x3 + 2 y3 − 4 z3
The required value of c = . Choice (A) ⇒ cosu = = f(say)
8 x7 − y7 − z7
12. Given f(x) = x(x + 2) (x – 1) Clearly f is a homogeneous function of degree – 4
Clearly f(x) is continuous on [–1, 0] and differentiable ∂f ∂f ∂f
\ By Euler’s theorem x. + y + z = n. f
on (–1, 0) ∂x ∂y ∂z
a = –1, b = 0; ∂ ( cos u) ∂ ( cos u) ∂ ( cos u)
f(b) = 0, f(a) = (–1) (1) (–2) = 2 \ x +y +z s = −4�cos u
∂x ∂y ∂z
f1(x) = 3x2 + 2x – 2 and f1(c) = 3c2 + 2c – 2
By Lagrange’s mean value theorem  ∂u ∂u ∂u 
–  x� � (sin u) + y� (sin u) + z (sin u)
=
f ( b) − f ( b)  ∂x ∂y ∂z 
f1(c) =
b−a = –4 cosu
0−2 ∂u ∂u ∂u
\ x +y +z = 4 cot u. Choice (A)
⇒ 3c2 + 2c – 2 = ∂x ∂y ∂z
1
Engineering Mathematics Test 1 | 2.7

16. Given z = in (x2 + y3) π

∂z 2x ∂z 3 y2
2
tan 2 θ�sec2 θ
⇒ = 2
∂x x + y
and = 2
∂y x + y 3
\ ∫ 5

(1 + tan 2 θ) 2
3
0

∂z ∂z 6x2 6 y3 π
Consider 3x + 2y = 2 + 2
sin 2 θ�sec2 θ
∂x ∂y x + y 3 x 2 + y 3 = ∫ cos 2
0�sec5 θ

6 ( x2 + y3 )
0

=2 = 6. Choice (B) π


x + y3 π 2
2
sin θ 
3
1
∫ sin θ�cos θdθ = = 
2
1 =  Choice (B)
3 0 3
∫ ( x − 2 x + 3) e dx
2 −x
17. We have 0

= ( x 2 − 2 x + 3)( −e − x ) − ( 2 x − 2) (e − x ) + 2 ( −e − x )0
1
4
dx
= (–2e – 0 – 2e ) – (–3 + 2 – 2)
–1 –1
20. ∫ ( x − 2)( x − 3)
1
4
= 3 – 4e = 3 – e  –1
Choice (D) 2
dx
3
dx
4
dx
= ∫ ( x − 2)( x − 3) + ∫ ( x − 2)( x − 3) + ∫ ( x − 2)( x − 3)
1 2 3
a a
I = I1 + I2 + I3 (say)
18. We know that ∫ f ( x ) dx = ∫ f ( a − x ) dx
0 0
Clearly as I has discontinuity at x = 2, I1 diverges
π
As I1 diverges I also diverges
2
3sec x + 4cosec x 4
dx
Let I = ∫ 0
sec x + c osec x \ ∫ ( x − 2)( x − 3) diverges. Choice (D)
1

ππ  π 
3sec − x + 4 c osec −x
2
2  2  ∞
dx dx
a
1  −1 x 
a
∫0
=
π  π 
dx 21. ∫0 9 + x 2 a→∞ ∫0 32 + x 2 aLt
= Lt =
→∞ 3 
tan
3  0
sec − x + cosec −x 
2  2 
1 −1  a  π
π Lt tan   =
3cosec x + 4 sec x
2 a →∞ 3  3 6
∫0 cosec x + sec x dx
= π
The given improper integral converges to
6
I+I
π π  Choice (C)
3sec x + 4 cosec x
2 2
3cosec x + 4 sec x 22.
∫0 cosec x + cosec x ∫0 sec x + cosec x dx
= dx +
y
(7,3)
π

3 ( cosecx + sec x ) + 4 (sec x + cosecx )


2
1
∫0
=
sec x + cosecx
dx
–2 0
x

π
π
2
π
∫ 7dx = 7 ( x )02 = 7
=
0
2
(2, –2)

π y2 = x + 2 and y = x – 4
\ 2I = 7 The above two intersect at (2, –2) and (7, 3).
2
+3
π
∫ ( y + 4) − ( y − 2) dy
2
⇒ I = 7  Choice (A) The area bounded =
4 −2

∞ 3
x2  y2 y3 
19. ∫ 5
dx Let x = tanq ⇒ dx = sec2qdq L.L  2 + 4 y − 3 + 2 y
=
  −2
0 (1 + x 2 ) 2
π 125 5
When x = 0 q = 0 and U.L when x = ∞, q = = = 20 sq units. Choice (A)
2 6 6
2.8 | Engineering Mathematics Test 1

23.  3
 1
y = 2 a  (9a) 2  – (9a) 3
(9a, 9a)   27a
= 54a2 – 27a2 = 27a2. Choice (D)

x 24. Standard Result. Choice (A)


0
25. We have 2e x + 3sin x
 x2 x3 x 4   x3 x5 x7 
= 2 1 + x + + + ����� + 3  x − + − �����
 2! 3! 4!   3! 5! 7! 
Given y2 = 9ax and x2 = 9ay
The above two curves intersect at (0, 0) and (9a, 9a).  x2 x3 x4 x5 
=  2 + 2 x + 2 + 2� + 2 + 2 + �����
9a
 x2   2! 3! 4! 5! 
The required area = ∫  9ax −
9a 
dx
0  x3 x5 x7 
9a   +  3 x − 3 + 3� − 3 + �����
 3 9a
 3! 5! 7! 
 x2  1  x3 
=3 a   –   x3 x 4 x5
3 9a  3  0 + + + ����� 
  = 2 + 5x + x2 – Choice (B)
 2 0 3! 12 24
Engineering Mathematics Test 2
Number of Questions: 25 Time: 60 min.

(Ordinary Differential Equations, Calculus (Vector Calculus))


Directions for questions 1 to 25: Select the correct alterna- 7. Evaluate the surface integral ∫∫ F �n dA , where
tive from the given choices. S

1. The directional derivative of F = z 2 i + xy j + y 2 k and S is the portion of the surface


f(x, y) = x3y2 + 3xy at (1, 2) in the direction of unit vector of the cylinder x2 + y2 = 49; 0 ≤ z ≤ 5 included in first
π octant.
which makes an angle of with the x-axis
4 (A) 518 (B) 2590
is ________. 2590
(C) (D) 624
3
(A) 16 2 (B)
8 2
15 25
(C) (D) 8. If V = ( 2x − y ) − yz 2 j − y 2 z k , then evaluate ∫ V �dr
2 2 c

where C is the curve bounding the projection on the


(
2. If the vector e px - y - z i + j + k ) is solenoidal, then the semi sphere x2 + y2 + z2 = 9, z > 0; in the xy plane.
value of p is ______. (A) 18 p (B) 6 p
(A) –3 (B) 3 (C) 8 p (D) 9 p
(C) –2 (D) 2
9. If r = 2 xi + 3 y j + zk and V is the volume of the sphere
3. Evaluate ∫ ( x + 3 yz ) ds where c is the curve defined by
∫∫ r�n dA =
2
x2 + y2 + z2 = 16 then _______.
c S

 1  (A) 38 p (B) 512 p


x = 6y; z = 4 from  3, , 4 to (6, 1, 4).
 2  (C) 438 p (D) 348 p
(A) 15 37 (B)
2 37
10. The work done by the force F = 2 xyi + y 2 j + zk in
(C) 199 (D) 37 33 moving a particle over the circular path x2 + y2 = 9;
z = 0 from (3, 0, 0) to (0, 3, 0) is ________.
4. The unit normal vector to the surface xy3 + 3yz = 3 at (A) 17 (B) –12
the point (3, –1, –2) is _________ (C) –9 (D) 6
−i − 3 j − 3k −i + 3 j − 3k 11. The solution of 3(xdy + ydx) = 2xydy when x = 1,
(A) (B)
19 19 y = 1 is
(A) 3 log (xy) – 2y + 2 = 0
i + 3 j − 3k i − 3 j − 3k (B) log(xy) + y – 2 = 0
(C) (D)
19 19 (C) logx + logy + 2 = 0
(D) log(xy) – 5y + 2 = 0
12. Solve dy = (9x + y – 1)2 dx when x = 0, y = 1.
∫ ( yz + z + z ) dx + (xz – 1) dy +
2
5. Evaluate the integral
c
(A) 3 tanx = 3x + y – 1
(B) 3 tan (3x) = 9x + y – 1
(xy + x + 2xz) dz from (2, 3, 3) to (3, 4, 5).
(C) tanx = 9x + y – 1
(A) 146 (B) 107
(D) 3 tan3x = 9x – y + 1
(C) 39 (D) 185
13. The solution of the differential equation
6. Evaluate the integral ∫ ( x + 2 y ) dx + x y dy, C is the
2
x dy − ydx  y
= cos 2   dx is
c
x  x
triangle with vertices at (0, 0), (3, 0) and (3, 3) taken in
that order.  y  y
(A) tan   = log (cx) (B)
tan   = x + c
45  x  x
(A) 18 (B)
4
 y  x
37 (C) tan −1   = log (cx ) (D) tan   = log x + c
(C) (D) 15  x  y
3
2.10 | Engineering Mathematics Test 2

dy 20. The solution of the differential equation (D4 + D2 + 36D


14. The integrating factor of – y tanx – cosx = 0 is + 52)y = 0 is
dx
(A) y = (C1 + C2x)e–2x + (C3 + C4x)e2x
(A) cos x (B) sin x (B) y = (C1 + C2x + C3 cos3x + C4 sin3x)e2x
(C) sec x (D) cosec x (C) y = (C1 + C2x + (C3 cos3x + C4 sin3x)e–2x
(D) y = (C1 + C2x)e–2x + (C3 cos3x + C4 sin3x)e2x
dy x 2 + y 2
15. Solve the differential equation = with the 21. The Laplace Transform of the function f (t) = t2 sin3t,
dx 2 xy
t > 0 is _____.
boundary conditions x = 1, y = 1.
(A) x2 + y2 = 3 (B) x2 – y2 = 0 ( s − 2s + 9) (B)
2
18 ( s − 3) 2

(A)
(C) x – y = 2 (D) x2 + y2 = 2
( s + 9) ( s + 9)
2 2 2 3 2 3

16. The solution of the differential equation ydx = (x + 3y3)


dy when x = 1, y = 1 is − ( s − 2 s + 9)
2
6 ( s − 2 s + 9) 2

(C) (D)
(A) x = 3y2 – 1 (B) 3x = y(2y2 – 1) ( s + 9) ( s + 9)
2 3 2 3

(C) 2x = y(3y + 1)
2
(D) 2x = y(3y2 – 1)
1 7. The particular integral solution of the differential 22. If u (t – a) denotes the unit step function, then the
equation Laplace Transform of (t2 + 3) u (t – 2) is _______.

d3 y d2 y dy  2 4 7  2 3
(A)  3 + 2 +  e–2s (B)  3 +  e–2s
− 5 + 7 – 3y = e4x sinhx is s s s s s
dx 3
dx 2
dx
1 5x 2   1 4 7  –2s
(A) e − 2e 
3x
s   3 + 2 +  e
(C)  2 + 3 e–2s (D)
8 s s s
1 3x 23. The Laplace Transform of solution of the initial value
(B) e − xe −5 x  d2 y dy
16  problem 2 – 2 – 8y = 0, y (0) = 1 and y1 (0) = –2
dt dt
1 5x
(C) e − 8 x e 3 x  will be ____
64 
1 1
1 (A) (B)
(D)  x e − 2e 
5x 3x
( s − 4)( s + 2) s−4
4
−1 1
18. The solution of the differential equation (D3 + 5D2)y (C) (D)
( s − 4)( s + 2) s+2
= 4 is
−5 x 1 2 3  1 1 
(A) y = C1 + C2 e + x 24. The inverse Laplace Transform of  − 
5 2  s 5
s3 
−5 x 2 2 is ______.
(B) y = C1 + C2 x + C3 e + x
5
2 t t
(A) (2t – 3) (B) (4t – 3)
(C) y = (C1 + C2 x ) e + x
5x 2

5 π π
2 2 π π
(D) y = (C1 + C2 x ) e + x + e (C) (2t + 3) (D) (4t + 3)
x 5x

5 t t
19. The particular integral of the differential equation given 4(2 s + 3)
25. The inverses Laplace Tranform of is
by (D2 – 2D + 4)y = x2 ex is ( s + 4s + 20)
2

1 x 1 x
(A) e (3 x − 2) (B) (
e 2x2 − 3 )
2
(A) e2t[2cos4t – sin4t]
9 6
(B) e2t{2sin4t – cos4t]
1 x 1 x
(C) e (3 x 2 − 1) (D) e (2 x 2 − 3) (C) e–2t[2cos4t – sin4t]
8 3 (D) e–2t[2sin4t – cos4t]

Answer Keys
1. D 2. D 3. A 4. B 5. B 6. B 7. C 8. D 9. B 10. C
11. A 12. B 13. A 14. A 15. B 16. D 17. C 18. B 19. A 20. D
21. B 22. A 23. D 24. A 25. C
Engineering Mathematics Test 2 | 2.11

Hints and Explanations


1. Given f(x, y, z) = x3 y2 + 3xy h(x, y, z) = xy + x + 2xz
∂f ∂f ∂f ∂g
∇f = i+ j = (3x2 y2 + 3y) i + (2x3 y + 3x) j =z=
∂x ∂y ∂y ∂x
(∇f)(1, 2) = (12 + 6) i + (4 + 3) j = 18i + 7 j ∂f ∂h
= y + 1 + 2z =
π ∂z ∂x
Given that unit vector makes an angle with x-axis
4 ∂g ∂h
=x=
π π i+ j ∂z ∂y
\ unit vector must be b = cos i + sin j =
4 4 2 The integral is independent of the path C.
The integral is exact differential
The directional derivative of f in the direction b is ∇f. b
So there exists a function Φ
( i + j ) = 18 + 7 = 25 
(18i + 7 j ) � 2 2 2
Choice (D) Such that
∂φ
∂x
= yz + z + z 2  → (1)

(
2. Given V = epx–y–z i + j + k is solenoidal) ∂φ
= xz − 1  → (2)
∂y
We know that if V is solenoidal, div V = 0
x 1
∂V1 ∂V2 ∂V3 = ∫ 3 y 2 � dy + c = xy + x + 2xz → (3)
+ + =0 y y
∂x ∂y ∂z
∂φ
= e px − y − z � p + e px − y − z � ( −1) + epx–y–z(–1) = 0 = yz + z + z2
∂x
\ p – 1 – 1 = 0 ⇒ p = 2 Choice (D) Integrate wrt x.
f = xyz + xz + xz2 + Q(y, z) → (4)
t
3. Let x = t; y = ; z = 4 and 3 ≤ t ≤ 6. Diff wrt y
6
∂φ
= xz + f1(y, z) → (5)
 dx   dy 
2
1
2
37 ∂y
ds =   +   � dt = 1 + dt = dt
 dt   dt  36 6 Comparing (2) and (5)
6 f1(y, z) = –1 ⇒ f(y, z) = –y + R (z)
 t  37   y
∫ (x + 3 yz ) ds = ∫  t 2 + 3 �4 dt = d  tan    \ f = xyz + xz + xz2 – y + R (z)  → (6)
2
 6  6   x
C 3 ydx − xdy
= 3 y dy = xy + x + 2xz + R1(z)
 y  y y2
=
tan   = tan    Choice (A)
 x  x
Comparing (3) and (6)
4. Let f(x, y, z) = xy3 + 3yz – 3 R1(z) = 0
The normal vector to the surface f(x, y, z) is ⇒ R(z) = k
dy \ f = xyz + xz + xz2 – y + k
− y tan x − cos x
\ ∫ ( yz + z + z ) dx + ( xz − 1) dy + ( xy + x + 2 xz ) dz
2
dx
(3, 4, 5)
∇f = y 3 i + (3 xy 2 + 3 z ) j + 3 yk
= ∫ d ( xyz + xz + xz − y)
2

The normal vector at (3, –1, –2)
(2, 3, 3)
(∇f )(3, −1, − 2) = − i + (9 − 6) j − 3k = − i + 3 j − 3k
xyz + xz + xz2 – y ](2, 3, 3)
=
(3, 4, 5)
∇f
\ The unit normal vector to the surface f is =
146 – 39 = 107 Choice (B)
∇f
∫ ( x + 2 y ) dx + x
2
−i + 3 j − 3k 6. y dy
=  Choice (B) C
19
f(x, y) = x + 2y   g(x, y) = x2 y
∫ ( yz + z + z ) dx + ( xz − 1) dy + ( xy + x + 2 xz ) dz
2
5.
∂f ∂g
= 2 and = 2 xy
Let f(x, y, z) = yz + z + z2 ∂y ∂x
g(x, y, z) = xz – 1
2.12 | Engineering Mathematics Test 2

8. Given V = ( 2x − y ) i − yz 2 j − y 2 z k
y

(3, 3)
y=x ∫ V �dr = ∫∫ S
∇ × V �ndA

(By stoke’s theorem)


x
(0, 0) (3, 0)
i j k
∂ ∂ ∂
By green’s theorem ∇ ×V =
∂x ∂y ∂z
∫ ( x + 2 y ) dx + x ydy = ∫∫ (2 xy − 2)dxdy
2
2 x − y − yz 2 − y2 z
C R

3 x = i(–2yz + 2yz) – j(0) + k (0 – (–1)) = k


=∫
0
∫ (2 xy − 2) dydx
0
f(x, y, z) = x2 + y2 + z2 – 9 be the surface
3 grad f = 2 xi + 2 y j + 2 zk
= ∫ xy − 2 y ]0 dx
2 x
^ grad f 2 xi + 2 y j + 2 zk
0 n= =
3
grad f 2 x2 + y2 + z2
= ∫ ( x − 2 x )dx
3
^ xi + y j + zk
0
n=
3 3
x4  81 45
=
4
− x2  = − 9 =
0 4 4
\ ∇ × V � n = k �
∆ ( xi + y j + zk ) = z
3 3
 Choice (B)
7. Let f(x, y, z) = x2 + y2 – 49
∆ z dx dy
Surface then
∫∫ ∇ × V � n dA = ∫∫ 3
S R n�k
grad f = 2x i + 2y j z dx dy
= ∫∫
= ∫∫ dx dy
grad f 2 xi + 2 y j 3 z
1
( )

R R
n= = = xi + y j 3
grad f 2 x +y 2 2 7
Area of circular region in x-y plane = 9 p Choice (D)
Consider the projection of S on the yz plane. It is a rec-
tangle with sides 7 and 5. 9. By using divergence theorem

dA =
dy dz
=
dy dz ∫∫ r�n dA = ∫∫∫ div r dV
n�i x S V

7 r = 2 x i + 3 y j + zk
 xi + y j  xz 2 + xy 2 Div r = 2 + 3 + 1 = 6 = ∫∫∫ div r dv = ∫∫∫ 6dV = 6V
( )

F � n = z 2 i + xy j + y 2 k �   = 7
 7 
V V

V is the volume of the sphere


∆ xz 2 + xy 2 dy dz 4
\ ∫∫ F � n dA = ∫ ∫
S s
7 x = 6 × × p(4)3 = 512 p
3
Choice (B)
7
5 7 5 7 10. Given F = 2 xyi + y 2 j + zk
y  3


=
z =0
∫ (
y=0
)
z 2 + y 2 dy dz = ∫ z 2 y +  dz
z =0
3 0 Work done by the force is ∫ F �dr
C
5
343 r = xi + y j + zk ⇒ dr = dxi + dy j + dzk
∫ (7 z +
2
= )dz
z =0
3
( )(
F �dr = 2 xyi + y 2 j + zk � dxi + dy j + dzk )
5
3
7 z 343  875 1715 2590 F �dr = 2 xy dx + y 2 dy + z dz
+ z = + =
3 3 0 3 3 3
∫ F �dr = ∫ (2 xy dx + y dy + z dz
2

 Choice (C) C
Engineering Mathematics Test 2 | 2.13

Convert x, y, z in parametric form  x dy − y dx   y  dx


x = 3 cost, y = 3 sint, z = 0   sec2   =
x 2
  x x
dx = –3sint dt, dy = 3cost dt
The limit of t is 0 to π/2   y
d  tan    = d(log x)
π
2
  x
∫ F �dr = ∫ 2�3cos t�3sin t� ( −3sin t ) dt + 9sin
2
t 3cos t dt Integrating on both sides, we have
C 0
 y
π tan   = logx + logc
2  x
= − ∫ 27sin 2 t cos t dt
 y
0 tan   = log(cx). Choice (A)
π  x
Sin 3 t 
2
−27
= − 27  = = −9 Choice (C) dy
3 0 3 14. − y tan x − cos x = 0
dx
11. 3(x dy + y dx) = 2xy dy dy
⇒ − y tan x = cosx
x (3 – 2y) dy + 3y dx = 0 dx
(3 − 2 y ) dy 3dx which is in the form of
dy
+ Py = Q
+ =0 dx
y x
The integrating factor of above equation e ∫ pdx = e − ∫ tan x dx
3  dx
∫  y − 2 dy + 3∫ x = C1 = elog cosx = cosx. Choice (A)

3 logy – 2y + 3 logx = logC


dy x 2 + y 2
3 log(xy) – 2y = logC 15. =  --------- (1)
dx 2 xy
Given when x = 1, y = 1
⇒ –2 = logC (x2 + y2)dx – 2xy dy = 0
\ The required solution is 3 log(xy) – 2y + 2 = 0. which is in the form of Mdx + Ndy = 0
 Choice (A) Here M = x2 + y2, N = –2xy
dy ∂M ∂N
1 2. dy = (9x + y – 1)2dx or = (9x + y – 1)2 ……. (1) = 2y , = –2y
dx ∂y ∂x
Let 9x + y – 1 = u
1  ∂M ∂N  −1 −2
dy du ⇒ Here − = (2 y + 2 y ) =
9+ =
dx dx N  ∂y ∂x  2 xy x

du which is a function of x alone say f(x) then the


⇒ − 9 = u2 (from 1) Integrating factor (I. F) is
dx
−2
e∫
du du f ( x ) dx ∫ dx −2

= u 2 + 9 or 2 = dx =e x
= e −2 log x = e log x = x −2
dx u +9
Multiplying (1) by I. F, we have
du 1  u
∫ u2 + 9 = ∫ dx + c ⇒ 3 Tan −1  3  = x + c  y2  2y
 1 + 2
dx − dy = 0 which is in the form of M dx +
1 −1  9 x + y − 1 x  x
i.e., tan 
  = x + c or N dy = 0
3 3
3tan(3x + 3c) = 9x + y – 1, when x = 0, y = 1 \ Solution is


1 −1
tan (0 ) = C ⇒ C = 0 ∫ m dx ( taking y constant ) +
3
\ The required solution is 3 tan(3x) = 9x + y – 1.
∫ ( terms of N not containing x ) dy = C
 Choice (B)
 y2 
x dy − y dx
1 3. 2  y
= cos   dx
⇒ ∫  x 2  dx + ∫ 0 dy
1 +
x  x
 y y2
Dividing both sides by x cos2   , we have x− =C
 x x
2.14 | Engineering Mathematics Test 2

Given when x = 1, y = 1 ⇒ C = 0 e5 x − e3 x
i.e., (D – 1)2 (D – 3)y =
y2 2
\ The required solution is x − = 0 or x2 – y2 = 0.
x e5 x − e3 x 1
 Choice (B) Particular integral is �
1 6. y dx = (x + 3y3) dy
( D − 1) ( D − 3) 2
2

dx 1 e5 x 1
or y = x + 3y3 − �e 3 x
dy 2 ( D − 1)2 ( D − 3) 2 ( D − 1)2 ( D − 3)
dx x 1 e5 x 1 1 e3 x
or − = 3y2, which is a linear equation in y of the = � − �
dy y 2 (5 − 1)2 (5 − 3) 2 (3 − 1)2 ( D − 3)
dx 1
form + px = Q here P = − and Q = 3y2 e5 x 1 1 5x
dy y = − � � x � e3 x = e − 8 x e 3 x   Choice (C)
64 8 64 
1
− ∫ dy 1
\ I. F = e ∫
pdy −1
=e y
= e log y = 18. (D3 + 5D2) y = 4
y
Auxiliary equation m3 + 5m2 = 0
\ The solution is x . e I � F = ∫ Q � e� I � F �dy m2(m + 5) = 0
⇒ m = 0, 0, –5
x 1 \ C. F is (C1 + C2x)e0.x + C3e–5x = (C1 + C2x) + C3e–5x
⇒ = ∫ 3 y 2 � dy + c
y y 1
P. I = 2 4�e 0� x
x 3 y2 D ( D + 5)
= +c
y 2 1 1 1 1 2x2
     = � 2 4 =  2 �4  = .
Given when x = 1, y = 1 5 D 5  D  5
3 −1 Complementary Solution y = C.F + P.I
⇒ 1 = + c or c =
2 2
2x2
x 3y 1 2 y = C1 + C2x + C3 e–5x +  Choice (B)
\ The required solutions is = − 5
y 2 2 19. (D2 – 2D + 4) y = x2 ex
i.e., 2x = y(3y2 – 1) ex x2 1
Alternate solution: P. I = = ex � x2
Given ( D − 2D + 4) ( D + 1) − 2 ( D + 1) + 4
2 2

ydx = ( x + 3 y 3 )dy 1
ex
= x2
ydx − xdy = 3 y dy
3 D2 + 3
−1
ydx − xdy 1 x 1 D2 
= 3 y dy ⇒ e
x
x 2
= e 1 + x2
y2  D2  3  3 
3 1 +
 x  3 
d   = 3 y dy
 y
1  D2  2
e x 1 −
= x
Integrating on both sides 3 3 
 x   3 y2  1 2 1
 y  =  2  + C =
3  3  9
(
e x  x 2 −  = e x 3x 2 − 2  ) Choice (A)

2x = y(3y2 + C1)
20. (D4 + D2 + 36D + 52) y = 0
Given x = 1; y = 1
Auxiliary equation of the above is m4 + m2 + 36m + 52
2 = 3 + C1 ⇒ C1 = –1
=0
\ required solution is 2x = y (3y2 – 1). Choice (D)
By trail and error we notice m = –2, –2, are the roots
3 2
d y d y dy of the above
17. − 5 2 + 7 − 3 y = e 4 x sin h x
dx 3
dx dx \ (m + 2)2 (m2 – 4m + 13) = 0
i.e., (D – 5D + 7D – 3)y
3 2 The roots are m = –2, –2, and 2 ± 3i
\ The solution is
 e x − e − x  e5 x − e3 x
= e4 x  = y = (C1 + C2x)e–2x + e2x (C3 cos3x + C4 sin3x)
 2  2  Choice (D)
Engineering Mathematics Test 2 | 2.15

21. Let g(t) = sin 3t ⇒ f(t) = t2g(t)


3
⇒ (s 2
− 2 s − 8) y – s + 4 = 0
Now L[g(t)] = L:[sin 3t] = 2
s +9 ⇒ (s 2
− 2 s − 8) y = s – 4
d2 ( s − 4) ( s − 4)
L[f(t)] = L[t2g(t)] = 2 (L[g(t)]) ⇒ y = =
ds ( s − 2s − 8) ( s − 4)( s + 2)
2

d  d  3  1
=  2  y =
ds  ds  s + 9   s+2
The laplace transform of the solution of (1) is
d  −6s  1
=  y = L[y] =
ds  ( s 2 + 9)2  s+2
. Choice (D)

−6 ( s 2 + 9) �1 − s�2 ( s 2 + 9) �2 s 
2
3  1 1 
  24. We have to find L–1   − 
=
( s + 9)  2  s s3  
4 5
2

−6  s 2 + 9 − 4 s 2  3  −1  1  −1  1 
= 2 =  L  5  − L  3 
( s + 9) 2     
3
  s2   s 2 
18( s 2 − 3)  3 
=  Choice (B) 1

( s 2 + 9) 3  t2 t2 
3

=  − 
2   5  3
22. Let f (t) = (t2 + 3) u (t – 2)     
  2   2  
L [f (t)] = L [(t2 + 3) u (t – 2)]
L [((t – 2) + 2)2 + 3) u (t – 2)]  3 1 
∴ L [f (t)] = L [(t–2)2 + 4(t – 2) + 7) u (t – 2)] ----- (1) 3  t2 t2 
= −
Let g (t) = t2 + 4t + 7 2  3 1 
\ L [g (t)] =  L [t2 + 4t + 7] = L [t2] + 4L[t] + 7L [1]
×
2 2
π
1
2
( )π 

( )
2 4 7
\ L [g (t)] = 3 + 2 +  3 1 
s s s 3  t2 t2  t  4t 
= − = × − 2
Now from (1) 2 3 1 π  3 
 π π
L [f (t)] = L [(t – 2)2 + 4(t – 2) + 7) u (t – 2)] 4 2 
= L [g (t – 2). U (t – 2)
=
t  2t 
L [g (t)] e–2s (By second shifting theorem) =
π
×3  3 − 1
 2 4 7  –2s
=
 3 + 2 +  e . Choice (A)
s s s t
= (2t – 3). Choice (A)
23. Given initial value problem is π
d2 y dy  4(2 s + 3) 
2 5. We have L–1  2
dt 2
–2
dt
– 8y = 0  ----------- (1)  s + 4 s + 20 
Where y(0) = 1 and y1(0) = –2  2s + 3 
Applying laplace transform on both sides of (1) = 4 L–1  2 
 ( s + 4 s + 4 ) + 16 
 d2 y dy 
L  2 − 2 − 8 y  = L[0]
dt dt  2( s + 2 − 2) + 3   2( s + 2) − 1 
  = 4L–1  2  = 4L
–1
 ( s + 2)2 + 4 2 
 ( s + 2) + 4 
2
 
 d2 y   dy 
⇒ L  2  – 2L  dt  – 8L [y] = 0
 dt   2( s + 2)   1 
= 4L–1  2  – 4L
–1
 ( s + 2)2 + 4 2 
 ( s + 2) 2
+ 4   
⇒ (s 2
) ( )
y − sy(0) − y1 (0) – 2 s y − y(0) – 8 y = 0
1 1
Where y = L[y] = 8 × 4 e–2t cos4t – 4 × 4 e–2t sin 4t

⇒ s2 y – s × 1 – (–2) –2 s y + 2 × 1 – 8 y = 0 = e–2t [2cos4t – sin4t]. Choice (C)


Engineering Mathematics Test 3
(Linear Algebra, Partial Differential Equations)
Number of Questions: 25 Time: 60 min.

Directions for questions 1 to 25: Select the correct alterna- 7. The value of x3 in the solution of the system of linear
tive from the given choices. equations x1 + 2x2 + 2x3 = 4, 2x1 − 2x2 − x3 = –3, 4x1 + x2
1. If A is a square matrix of order 5 with A–1 = AT and + 2x3 = 3 is ––
non-negative determinant, then the determinant of A (A) 1 (B) −1
is ______. (C) 2 (D) −2
(A) 0 (B) 1 8. For a homogeneous system of linear equations AX = O
(C) 2 (D) 5 with four equations in four unknowns, if the number of
2. For two matrices A and B, if AB = A and BA = B, then linearly independent solutions is one, then the rank of
which of the following statements is/are correct? A is ______.
I. A is an idempotent matrix. (A) 1 (B) 2
II. B is an idempotent matrix. (C) 3 (D) 4
(A) I only
(B) II only 1 10 16 −20 
(C) Both I and II 0 −1 159 237 
(D) Neither I nor II 9. If A =   ; then the
0 0 1 −431
 
1 a bc  0 0 0 1 
3. Consider the matrix A = 1 b ca  . Which of the fol- determinant of A9 – 7A5 + 4A is _____.
 
1 c ab  (A) 4 (B) –4
(C) 16 (D) –16
lowing is NOT equal to the determinant of A?
10. If the characteristic equation of a 2 × 2 matrix A is
1 a a2 a + bc a 1 + a l2 – 4l + 1 = 0, then the trace and determinant of A
2
(A) 1 b b (B)
b + ca b 1 + b respectively are ______.
1 c c 2
c + ab c 1 + c (A) –1 and 4 (B) 1 and –4
(C) 4 and 1 (D) 4 and –1
0 a − b bc − ac 1 a + 1 a2 + a 11. If l1 and l2 are the eigenvalues of a 2 × 2 non-singular
(C) 0 b − c ca − ab (D) 1 b + 1 b2 + b matrix A, then the eigenvalues of ad joint of A are
1 c ab 1 c + 1 c2 + c ______.
(A) l1 and l2 (B) l12 and l22
4. For a non-singular square matrix A, if A3 = A, then A λ1
(C) l1 + l2 and l1 – l2 (D) l1 × l2 and
must be ______. λ2
(A) a nilpotent matrix
(B) an idempotent matrix 12. If A is a 3 × 3 matrix with the characteristic equation
(C) an involutory matrix l3 – 5l2 + 2l – 3 = 0, then 3A9 – 15A8 + 6A7 – 11A6 +
(D) None of these 10A5 – 4A4 + 10A3 – 20A2 + 8A – 9I is equal to _______.
5. If A is a matrix of order 6 × 9 with rank 5, then which 1 0 0  2 0 0 
of the following is true?  
(A) 0 1 0 (B)  
(A) All the rows of A are linearly independent.   0 2 0 
0 0 1  0 0 2 
(B) 5 columns of A are linearly independent.
(C) AAT is invertible. 3 0 0 4 0 0 
(D) ATA is invertible. 
(C) 0 3 0  (D) 
  0 4 0 
 1 2 4 −3  0 0 3   0 0 4 
 2 −3 5 −4 
6. The rank of the matrix P =   is ____. 13. Which of the following is NOT an eigenvector of the
 4 1 13 −10 
  1 0 0 
 3 −8 6 −5 
matrix A = 2 3 0  ?
(A) 1 (B) 2  
(C) 3 (D) 4  3 −1 4 
Engineering Mathematics Test 3 | 2.17

3 0 (A) x2 + y2 = f (x2 – y2)


    (B) x2 + z2 = f (x2 – z2)
(A) −3 B)
  0 (C) x2 – y2 = f (y2 – z2)
 −4   −4  (D) y2 – z2 = f (x2 – y2 – z2)
0  0  20. The solution of (p – q) (z – x p – y q) = 1 is _________
 
(C) 1 (D)  
  2 1
1  0  (A) z = ax – by + z = ax – by
(B)
a+b
1
14. If the system of linear equations (C) z = ax + by (D)
z = ax + by +
2x + 3y + 4z = 1 a−b
5x – y + z = 4
21. If u(x, y) = X(x).Y(y) be the solution of the partial differ-
3x + ay – 3z = 3
∂u ∂u
has a unique solution, then the value of a + 4 ______ ential equation 4 +5 = 0, which is obtained by
(A) must be equal to 0, ∂x ∂y
(B) should not be equal to 0 solving it by the method of separation of variables, then
(C) can be any real number, X(x) (the function of x only in u(x, y)) is ______
(D) can be any rational number [Note: Here c and k are arbitrary constants]
15. If a1, b1, c1, d1, a2, b2, c2 and d2 are any non zero real  4k 
 
numbers, then which of the following types of solution (A) X(x) = ce(kx) x2 (B) X(x) = ce  x

k
is NOT possible for the system of linear equations.  x ( )
(C) X(x) = ce  4 
(D) X(x) = ce −5 k x 2
a1x + b1y + c1z = d1
a2x + b2y + c2z = d2 22. Which of the following second order partial differential
(A) Unique solution equations is an elliptic equation?
(B) No solution
(C) Infinitely many solution ∂2 u ∂2 u ∂2 u ∂u ∂u
(A) 3 + 4 − 5 +7 − = 7x2
(D) None of these ∂x 2
∂x ∂y ∂y 2
∂x ∂y
16. The partial differential equation of ∂2 u ∂2 u ∂ 2 u ∂u ∂u
z = f (x + at) – g (x – at) is _____. (B) 3 − 4 + 5 − + = 6x2 y
∂x 2 ∂x ∂y ∂y 2 ∂x ∂y
∂2 z 2 ∂ z
2
∂z ∂z
(A) = a (B) =a ∂2 u ∂2 u ∂2 u ∂u ∂u
∂t 2
∂x 2
∂t ∂x (C) –3 +4 + 5 2 + 4x − 7 y =0
∂x 2
∂x ∂y ∂y ∂x ∂y
∂2 z ∂2 z ∂2 z ∂2 z
(C) = 2 (D) +
∂t 2
∂x ∂t 2 ∂x 2 ∂2 u ∂2 u ∂2 u ∂u ∂u
(D) +2 + −2 +5 = 6 xy 2
17. The first order partial differential equation by eliminat- ∂x 2
∂x ∂y ∂y 2 ∂x ∂y
ing the arbitary function from z = f(x3 – y3) is 1
(A) p + q = 0 (B) yp + xq = 0 23. The Fourier cosine series of the function f(x) = ,
2
(C) y2p + x2q = 0 (D) 2y2p + 3x2q = 0
0 ≤ x ≤ 1 is
18. The general solution of the partial differential equation (A) 1 (B)
x3 (y – z) p + y3(z – x)q = z3(x – y) is (C) 0 (D)
 1 1 1 1 a0
(A) φ  xyz , + +  = 0 24. The Fourier series of f(x) = e2x in the interval
 x y z 4 2
 1 (0, 2p) is
1 1 1 1 1
(B) φ  2 + 2 + 2 , + +  = 0 ∞ ∞
x y z x y z
f(x) = + ∑ an cos nx + ∑ bn sin nx , then the value of
n =1 n =1
1 1 1 1 1 1
(C) φ  − − , 2 + 2 − 2  = 0 a0
x y z x y z  is
2
 1 1 1 1 1 1 e4π − 1 e2π − 1
(D) φ  2 − 2 − 2 , − +  = 0 (A) (B)
x y z x y z 4π 2π
19. The solution of the partial differential equation e4π − 1 e2π − 1
(C) (D)
xy2z2p + x2yz2q = x2y2z is 2π 4π
2.18 | Engineering Mathematics Test 3

25. The Fourier series of the function 1  nπ 


 −π
(A)
nπ sin 2 − cos nπ 
−2 for − π < x < 2
 4  nπ 

f(x) = 0 for
−π
<x<
π (B)
nπ cos 2 − cos nπ 
 2 2
 π 1  nπ 
2 for 2 < x < π (C)
4nπ cos nπ − cos 2 

a0
∞ ∞ 4  nπ 
is f(x) = + ∑a n cos nx + ∑ bn sin nx , then bn = (D)
nπ sin 2 − sin nπ 
2 n =1 n =1

Answer Keys
1. B 2. C 3. B 4. C 5. B 6. B 7. A 8. C 9. D 10. C
11. A 12. C 13. D 14. B 15. A 16. A 17. C 18. B 19. C 20. D
21. C 22. B 23. B 24. A 25. B

Hints and Explanations


1. Given A–1 = AT and det of A is non-negative bc a 1
\ A.AT = AT.A = I5
=
ca b 1 + 0
⇒ Det of (A.AT) = Det of I5
⇒ |A.AT| = |I5| ab c 1
⇒ |A| |AT| = 1
⇒ |A| . |A| = 1 (∵ |A| = |AT|) 1 a bc
⇒ |A|2 = 1 ⇒ |A| = ±1 = –1 1 b ca
\ The determinant of A = 1. 1 c ab
(∵ |A| is non-negative) Choice (B)
= –Det of A ≠ Det of A
2. Given AB = A and BA = B \ The determinant given in option B is NOT equal to
Consider BA = B det of A. Choice (B)
⇒ A (BA) = AB
4. Given A is non-singular and A3 = A
⇒ (AB)A = A (Q AB = A)
⇒ AA2 = A
AA = A ⇒ A2 = A
⇒ A–1 (AA2) = A–1 A
⇒ A is an idempotent matrix  → (1)
⇒ (A–1A)A2 = A–1 A
Consider AB = A
⇒ A2 = I
⇒ B (AB) = BA
⇒ A must be an involutory matrix. Choice (C)
⇒ (BA)B = B (Q BA = B)
⇒ BB = B ⇒ B2 = B 5. Given A is a matrix of order 6 × 9.
⇒ B is an idempotent matrix  → (2) Rank of A = 5
\ From (1) and (2), both I and II are correct. \ Maximum number rows/columns of A that are lin-
 Choice (C) early independent = 5.
\ Option (B) is TRUE. Choice (B)
1 a bc 
3. Given A = 1 b ca   1 2 4 −3 
 
1 c ab   2 −3 5 −4 
6. Given matrix is P =  
Consider the determinant given in option (B)  4 1 13 −10 
 
a + bc a 1 + a a a 1+ a bc a 1 + a  3 −8 6 −5 
b + ca b 1 + b = b b 1 + b + ca b 1 + b R2 → R2 – 2R1, R3 – 4R1 and R4 → R4 – 3R1
c + ab c 1 + c c c 1+ c ab c 1 + c 1 2 4 −3
bc a 1 bc a a 0 −7 −3 2 
~  
=
0 + ca b 1 + ca b b 0 −7 −3 2 
 
ab c 1 ab c c 0 −14 −6 4 
Engineering Mathematics Test 3 | 2.19

R3 → R3 – R2 and R4 → R4 – 2R2 8. Given the system of equations AX = O has


Number of unknowns = n = 4
1 2 4 −3
0 −7 −3 2  Number of equations = 4
~   \ A is a 4 × 4 matrix
0 0 0 0  Also given the number of linearly independent solu-
  tions = 1. We know that the number of linearly inde-
0 0 0 0 
pendent solutions of a system of homogeneous linear
−1 equations.
R2 → R2
7 AX = O is = n – r
−3  Where n = the number of unknowns and r = the rank
1 2 4
0 of A.
1 3 / 7 −2 / 7  \ n – r = 1
\ P ~  
0 0 0 0  ⇒ 4 – r = 1
  ⇒ r = 4 – 1 = 3
0 0 0 0 
\ The rank of A = 3 Choice (C)
Which is in Row Echelon form.
\ The rank of P = the number of non-zero rows in its 1 10 16 −20 
Row Echelon form = 2. Choice (B) 0 −1 159 237 
9. Given A =  
7. Given system of equations is 0 0 1 −431
x1 + 2x2 + 2x3 = 4  
0 0 0 1 
2x1 − 2x2 − x3 = –3 → (1)
4x1 + x2 + 2x3 = 3 The eigenvalues of A are 1, –1, 1 and 1.
It can be written in matrix form as If λ = –1 is an eigenvalue of A, then (–1)9 – 7(–1)5 +
AX = B → (2) 4(–1) = 2 is an eigenvalue of A9 – 7A5 + 4A.
Also if l = 1 is an eigenvalue of A then
1 2 2   x1  4 (1)9 – 7(1)5 + 4(1) = –2 is an eigenvalue of A9 – 7A + 4A
Where A = 2 −2 −1 ; X = x2 and B = −3
    
\ The eigenvalues of A9 – 7A5 + 4A are –2, 2, –2 and
     
 4 1 2   x3   3  –2.
\ The determinant of A9 – 7A5 + 4A = Product of the
Consider the augmented matrix
eigenvalues of A9 – 7A5 + 4A = (–2) (2) (–2) (–2)
1 2 2 4 = –16. Choice (D)
[A/B] = 2 −2 −1 −3 10. Given the characteristic equation of a 2 × 2 matrix A is
l2 – 4l + 1 = 0
4 1 2 3
Let l1 and l2 be the eigenvalues of A.
R2 → R2 – 2R1 and R3 → R3 – 4R1 \ Trace of A = sum of the eigenvalues of A
= l1 + l2 = (–(–4) = 4
1 2 2 4 
Determinant of A = Product of the eigenvalues of
~ 0 −6 −5 −11 A = l1 . l2 = 1. Choice (C)
 
0 −7 −6 −13
a b 
R3 → 6R3 – 7R2 11. Let A =   be a 2 × 2 matrix with l1 and l2 as its
c d 
1 2 2 4  eigenvalues.
\ [A/B] ~ 0 −6 −5 −11 The characteristic equation of A is
 
0 0 −1 −1  a−λ b
|A – lI| = 0 ⇒ =0
Hence the system of equations that has same solution c d −λ
as that of AX = B is ⇒ (a – l) (d – l) – bc = 0 ---- (1)

1 2 2   x1   4   d −b 
The adjoint of A is adj(A) =  
      −c a 
0 −6 −5  x2  =  −11
0 0 −1  x3   −1  \ The characteristic equation of adj(A) is

⇒ x1 + 2x2 + 2x3 = 4 d − λ −b
=0
–6x2 – 5x3 = –11 −c a − λ
–x3 = –1 ⇒ x3 = 1. Choice (A)
2.20 | Engineering Mathematics Test 3

⇒ (d – l) (a – l) – bc = 0  ---- (2) x 1 


As (1) and (2) are one and the same and l1 and l2 being
X = y and B =  4  consider the augmented matrix
 
the roots of (1), l1 and l2 will be the roots of (2).    
\ The eigenvalues of adj(A) are l1 and l2.  z   3 
 Choice (A) 2 3 4 1 
12. The characteristic equation of A is l3 – 5l2 + 2l – 3 = 0 5 −1 1 4 
[A|B] =  
\ By Cayley Hamilton theorem, we have  3 a −3 3 
A3 – 5A2 + 2A – 3I = 0  ----- (1)  
 
Consider
R2 → 2R2 –5R1, R3 → 2R3 – 3R1
3A9 – 15A8 + 6A7 – 11A6 + 10A5 – 4A4 + 10A3 – 20A2
+ 8A – 9I 2 3 4 1
= 3A6(A3 – 5A2 + 2A – 3I) – 2A6 + 10A5 – 4A4 + 10A3 ~ 0 −17 −18 3
– 20A2 + 8A – 9I  
0 2a − 9 −18 3
= 3A6 × 0 – 2A3(A3 – 5A2 + 2A – 3I) + 4A3 – 20A2
+ 8A – 9I R3 → R3 – R2
(From (1)) = 0 – 2A3 × 0 + 4(A3 – 5A2 + 2A – 3I) + 3I 2 3 4 1
(From (1)) = 4 × 0 + 3I (From (1)) [A|B] ~ 0 −17 −18 3 

 
1 0 0   3 0 0  0 2a + 8 0 0 
3I = 3 0 1 0  = 0 3 0  
= Choice (C) The given system of equations has a unique solution, if
0 0 1  0 0 3  P(A) = p([A/B]) = 3 (= The no. of unknowns)
This is possible only if 2a + 8 ≠ 0
⇒ a + 4 ≠ 0. Choice (B)
1 0 0 
1 5. Given system of equations is
13. Given A = 2 3 0 
  a1x + b1y + c1z = d1  ______(1)
 3 −1 4  a2x + b2y + c2z = d2
The eigenvalues of A are 1, 3 and 4 It can be written in matrix form as
AX = B
 x1 
x
If x =  x2  is an eigenvector of A, then x should satisfy  a1 b1 c1     d1 
  Where A =   ; x =  y  and B =  d 
 x3  a b
 2 2 2 c  2
 z 
any one of the three conditions. AX = x, AX = 3X and
Here two possibilities arise
AX = 4X
(i) P (A) ≠ P(A/B])
From the options given, it can be easily observed that
In this case, (1) has no solution
the vectors given in options (A), (B) and (C), will sat-
(ii) P (A) = P ([A/B]) < 3(= The no. of unknowns)
isfy one of these three conditions.
In this case, (1) has infinitely many solutions.
Consider the vector given in option (D),
So, the given system (1) do not have a unique solu-
1 0 0   0   0  0  tion. Choice (A)
AX = 2 3 0   2  =  6  ≠ λ  2  1 6. z = f (x + at) – g (x – at)
      
 3 −1 4  0   −2   0  ∂z
= f 1 ( x + at ) − g 1 ( x − at )
For, l = 1, 2 or 3 ∂x
\ Its not an eigenvector of A Choice (D)
∂2 z
= f 11 ( x + at ) − g 11 ( x − at )
14. Given system of linear equations is ∂x 2
2x + 3y + 4z = 1
∂z
5x – y + z = 4 = af 1 ( x + at ) + ag 1 ( x − at )
∂t
3x + ay – 3z = 3  --------- (1)
It can be written in matrix form as AX = B ∂2 z
= a2f11(x + at) – a2g11(x – at)
2 3 4  ∂t 2
Where A = 5 −1 1  ; ∂2 z 2 ∂ z
2
  = a  Choice (A)
 3 a −3 ∂t 2 ∂x 2
Engineering Mathematics Test 3 | 2.21

17. Given z = f(x3 – y3) Considering the last two fractions of (2), we have
Let x3 – y3 = u y dy = z dz. On integrating, we get
z = f(u) y2 – z2 = C2  …………… (4)
∂z ∂z ∂u From (3) and (4) the general solution of (1) is
= � = f �(u).3x2 x2 – y2 = f (y2 – z2). Choice (C)
∂x ∂u ∂x
20. Given (p – q) (z – x p – yq) = 1
∂z ∂z ∂u 1
= � = − f �(u) 3 y 2 z = x p + yq +
∂y ∂u ∂y p−q
∂z ∂z This is a clairaut equation and its solution is
y2 + x2 = f �(u) 3 x 2 y 2 − f 1 (u) 3 x 2 y 2 = 0
∂x ∂y 1
z = ax + by +  Choice (D)
∴ The first order partial dE is y2p + x2q = 0 a−b
 Choice (C) 2 1. Given u = x(x), y(y) ------ (1) is the solution of the PDE
1 8. x3(y – z) p + y3 (z – x)q = z3(x – y) ∂u ∂u
4 +5 = 0 ----------- (2)
The subsidiary equation of the given differential equa- ∂x ∂y
tion is
Obtained by solving (2) by the method of separation
dx dy dz of variables
= 3 = 3  …………. (1)
x ( y − z) y (z − x) z (x − y)
3
∂u ∂u
\  = x1 y and = xy1
1 1 1 ∂x ∂y
using the multipliers , 2 and 2 each fraction of dY
2
x y z dx
where x1 = and y1 =
1 1 1 dx dy
dx + 2 dy + 2 dz
x 2
y z \ (2) becomes
(1) is equal to
0 4x1y + 5xy1 = 0
1 1 1 4 x1 y1
⇒ dx + 2 dy + 2 dz = 0 ⇒ +5 = 0
x 2
y z x y
On integrating the above, we get 4 x1 y1
⇒ = −5 = k (say) where k is a constant
1 1 1 1 1 1 x y
− − − = C or + + = C1  ………… (2)
x y z x y z 4 x1 −5 y1
⇒ = k and =k
1 1 1 x y
using the multipliers , 3 and 3 each of the frac-
3
x y z kx −ky
⇒ x1 =   y1 =
4 5
1 1 1
dx + 3 dy + 3 dz
x 3
y z kx ky
tion (1) equal to ⇒ x1 – = 0 ⇒ y1 + =0
0 4 5
On integrating both the sides, we get dX kx dx k
⇒ = ⇒ = dx
1 1 1 dx 4 x 4
2
+ 2 + 2 = C2  …………. (3)
x y z dx k k
From (2) and (3) the general solutions is ⇒ ∫ = ∫ dx ⇒= x + c1
x 4 4
1 1 1 1 1 1 kx kx
ϕ  + + , 2 + 2 + 2  = 0. Choice (B) ⇒ x = e 4 + c1 = e 4 c1
x y z x y z 
 kx 
  1
4
⇒ x = ce ; where c = e c
19. xy2z2p + x2yz2q = x2 y2 z ……………… (1)  kx 
 
The subsidiary equations of (1) are \ x (x) = ce 4
.  Choice (C)
dx dy dz
= =  …………. (2) 22. A PDE is of the form
xy 2 z 2 x 2 yz 2 x 2 y 2 z
Auxx + Buxy + Cuyy + F(x, y, u, ux, uy) = 0 ----- (1)
Considering first two fractions of (2) we have x is elliptic, if B2 – 4AC < 0
dx = y dy. From the PDE in the options, consider the PDE in
On integrating we get x2 = y2 or x2 – y2 = C1 ……… (3) option (B)
2.22 | Engineering Mathematics Test 3

Comparing it with (1), we have 1


A = 3, B = –4 and C = 5 24. In the Fourier series of e2x; a0 is given by


π ∫ f ( x )dx
\ B2 – 4AC = (–4)2 – 4 × 3 × 5 = – 44 < 0 0

⇒ B2 – 4AC < 0 1

1
∫e
2x 2π
Hence the PDE given in option (B) is elliptic a0 = dx = e 2 x 
π 0
2π   0
Also, it can be easily observed that the PDE given in
options (A), (C) and (D) do not satisfy the property, 1 a e4π − 1 e4π − 1
  = e 4 π − 1 0 = = . Choice (A)
B2 – 4AC < 0. Choice (B) 2π 2 2 × 2π 4π
23. The coefficients of Fourier cosine series are given by 1
2 5. In the Fourier series of f(x), bn is given by
1 π π
2
a0 =
1 ∫ f ( x)dx and −π
∫ f ( x ) sin nxdx
0

2
1
 nπx   −π π
π
1 2

2
an = f ( x ) cos  dx
1 0
 1  bn =
π  −∫π
( −2 ) sin nxdx + ∫ 0�sin nxdx + ∫π 2 sin nxdx
 −π
2 2
1
2 1
∫ 2 dx = x ]
1
\ 
a0 = =1 −π
1 0
0
2  cos nx  2 2  − cos nx  π
= +
1 1
π  n  − π π  n  π
2 1 2
an =
1 ∫0 2 cos( nπx)dx = ∫ cos( nπx)dx  π π
2  cos n 2 cos nπ cos nπ cos n 2 
0

= − − +
sin( nπx ) 
1
π  n n n n 

  = =0 
nπ  0  
1 4  nπ 
\ The required series is f(x) =
2
. Choice (B) =
nπ cos 2 − cos nπ  . Choice (B)
Engineering Mathematics Test 4
Number of Questions: 25 Time: 60 min.

(Probability and Statistics)

Directions for questions 1 to 25: Select the correct alterna- (A) 1 : 70 (B) 1 : 69
tive from the given choices. (C) 69 : 70 (D) 70 : 1
1. If A, B and C are mutually exclusive such that 5P(B) 7. Kids and Toys factory is transporting balls of 5 differ-
= 8 P(A), 4P(C) = 3P(B) and 19P(A ∪ B) = 13 then ent colours – yellow, blue, red, green and white. Mr.
P(A ∪ B ∪ C) is ________. Bholeram, a worker in the factory has to separate these
(A) 0.75 balls as per their colours into different boxes and label
(B) 1 them with the corresponding coloured labels.
(C) 0.625
(D) Cannot be determined Mr. Bholeram, after separating the balls, sealed the
boxes and then labelled the boxes at random. What
2. 240 passengers travelling in a plane from Hyderabad is the probability that all the boxes are incorrectly
to Sharjah like one or more of the three meals among labelled?
sandwich, burger and pizza as given below. 140 passen-
(A) 1 (B) 0
gers like sandwich,110 passengers like burger and 90
passengers like pizza. 40 of them like both sandwich 11 11
(C) (D)
and burger, 50 of them like both Sandwich and Pizza, 120 30
40 of them like both burger and pizza where as 30 of
them like all the three meals. What is the probability 8. While shuffling a pack of cards, 4 cards are acciden-
that a randomly selected passenger likes pizza only? tally dropped. The probability that all of them are num-
(A) 0.125 (B) 0.250 bered cards (2 to 10) of the same suit is
(C) 0.375 (D) 0.500 ( 9 C4 )
4
4 × 9 C4
3. Let S be the set of all 4 digit numbers that can be formed (A) 52
(B) 52
C4 C4
using the digits 2, 3, 5, 7, 8 and 9. Probability that a
randomly selected number of S has all digits distinct
( 9 C1 )
4
4 × 9 C1
is ______. (C) 52
(D) 52
5 5 C4 C4
(A) (B)
12 18
5 5 9. Arpit and Bipin pick up a ball at random from a bag
(C) (D) containing 5 violet, 2 red and 3 orange balls one after
24 36
the other, replacing it every time till one of them gets
4. An unbiased coin is tossed until it shows up the same an orange ball and the one who first gets an orange ball
face in two consecutive throws. What is the probability is declared a winner. If Arpit begins the game, then the
that the number of tosses is not more than 4? probability of Bipin winning the game is
3 1
(A) 4 (B) 10 7
8 (A) (B)
1
17 17
7
(C) (D) 4 7 3
8 (C) (D)
10 10
5. What is the probability that a quadratic equation ax2 +
bx + c = 0 has equal roots if a, b and c are distinct and
are taken from {1, 2, 3, 4, 6, 8, 9}? 10. An urn A contains 6 white balls and 7 black balls. And
1 2 urn B contains 8 white balls and 6 black balls. A person
(A) (B) draws a ball at random from one of the two urns. It
35 35
turns out to be black. What is the probability that the
1 2 ball was drawn from urn A?
(C) (D)
105 105 7 49
(A) (B)
6. A bag contains 4 five rupee coins, 3 two rupee coins 14 88
and 3 one rupee coins. If 6 coins are drawn from the 39
bag at random, what are the odds in favour of the draw (C) (D) None of the above
88
yielding maximum amount?
2.24 | Engineering Mathematics Test 4

11. The bivariate probability distribution of X and Y is as 3 2


follows. (A) (B)
7 3
   Y 0 1 2 1 4
X (C) (D) 7
3
1 2 3
0 17. A random variable X has the following probability
40 40 40
distribution.
2 3 1
1
40 40 40 X = xi 0 1 2 3 4
P(x = xi) K 2K 3K 5K 4K
3 1 7
2
40 40 40 Then find P(X ≥ 2).
4 5 8 4 1
3
(A) 5 (B)
40 40 40 5
2 1
Find P(X ≤ 1, Y = 2). (C) (D)
5 15
2 1
(A) (B) 18. The standard error is _____.
5 5
3 1 (A) accepting the null hypothesis when it is false.
(C) (D) (B) rejecting the null hypothesis when it is true.
10 10
(C) the standard deviation of the sampling distribution
12. In a book of 500 pages, there are 50 typing errors. of a statistic.
Assuming that the number of errors per page follows (D) the probability that the test statistic does not lie in
poission distribution, find the probability that randomly the critical region.
chosen 5 pages will contain no error. 19. In large sampling, the sampling distribution of means
(A) 0.6065 (B) 0.6078 follows _____
(C) 0.6538 (D) 0.3935 (A) Normal distribution
(B) t – distribution
13. The continuous random variable X is uniformly distrib-
(C) F – distribution
uted with mean 2 and variance 12. Find P(X > 0).
(D) x2 – distribution
1 4
(A) (B) 5 20. Which of the following distributions is used to test the
3
equality of variances of two populations from which
2 1
(C) (D) two small random samples are drawn?
3 5 (A) Normal distribution
14. X and Y are two independent normal variates with (B) t – distribution
means 3, 6 and variances, 1, 9 respectively. Find the (C) F – distribution
value of k such that P(X + Y ≤ k) = P (9X – Y ≥ 2k). (D) x2 – distribution
(A) 9.3 (B) 9.6 21. If a statistic s follows t – distribution with ν = 10
(C) 8.6 (D) 10.3 degrees of freedom, then s2 follows F – distribution
with degrees of freedom ( ν1 , ν2 ) = _____.
15. Bag A contains 9 white balls and 5 green balls. Bag B
contains 6 white balls and 7 green balls. One ball is (A) (1,9) (B) (1,10)
drawn from bag A and is placed in bag B. Now one ball (C) (1,11) (D) (9,1)
is drawn at random from bag B. It is found that the ball 2 2. In testing of hypothesis, if the test statistic is outside the
is green. Find the probability that white ball is trans- critical region, then we will
ferred from bag A. P : Accept the null hypothesis
20 63 Q : Reject the null hypothesis
(A) (B) R : Accept the alternative hypothesis
103 103
S : Reject the alternative hypothesis
80 75
(C) (D) Which of the following is true?
103 103 (A) P only
16. A dice is rolled twice the sum of the numbers appear- (B) R only
ing is 7, what is the probability that at least one dice (C) P and S only
shows 3? (D) Q and R only
Engineering Mathematics Test 4 | 2.25

23. Three letters are placed into three addressed envelopes 24. The variance of the data x, x + 3, x + 5, x + 7, x + 10 is
randomly. A random variable X denotes the number of (A) 11.2 (B) 11.6
letters placed into corresponding envelopes. Find the (C) 11.6 + x (D) 11.2 + x
variance of X. 25. The median of the following data can be 3, 8, 12, 28,
5
(A) (B) 2 16, 15, x
6 (A) 13 (B) 14
(C) 1 (D) 3 (C) 15 (D) Any of the above

Answer Keys
1. B 2. A 3. B 4. C 5. C 6. B 7. D 8. A 9. B 10. B
11. D 12. A 13. C 14. B 15. B 16. C 17. A 18. C 19. A 20. C
21. B 22. C 23. C 24. B 25. D

Hints and Explanations


1. Given 5P(B) = 8P(A) and 4P(C) = 3P(B) n ( P ) − n ( P ∩ S ) − n ( B ∩ P ) + n (S ∩ B ∩ P )
5 3 =
⇒ P(A) =  P ( B) and P(C) = 4 P(B)  ----- (1) 240
8 90 − 40 − 50 + 30 1
13 = = = 0.125. Choice (A)
Now 19 P(A ∪ B) = 13 ⇒ P (A ∪ B) = 240 8
19
13 3. The number of 4 digit numbers that can be formed
P(A) + P(B) = (∵ A and B mutually exclusive) using the digits 2, 3, 5, 7, 8 and 9
19
= The number of elements of S = 64
5 13
⇒ P (B) + P (B) = The number of 4 digit numbers of S that have all digits
8 19 distinct = The number of 4 digit numbers that can be
13 13 formed using the digits 2, 3, 5, 7, 8 and 9 = 6P4
⇒ P (B) =
8 19 \ Probability that a randomly selected number of
8 6
⇒ P (B) = P 5
19 S has all digits distinct = 44 =  Choice (B)
6 18
Now P (A ∪ B ∪ C) = P (A) + P (B) + P(C)
5 3 4. The number of tosses may be 2 or 3 or 4.
= P ( B) + P ( B) + P ( B) The possible cases and their corresponding probabilities:
8 4
2
1
→ 2  
5 3 19 19 8
=  + 1 +  P ( B) = 8 P ( B) = 8 × 19 = 1� Case 1 : HH OR TT
 2
8 4
3
 Choice (B) 1
Case 2 : HTT OR THH → 2  
2. The total number of passengers = 240  2
Let S, B and P denote the sets of passengers who like 4
sandwich, burger and pizza respectively.  1
Case 3 : HTHH OR THTT → 2  
 2
\ n (S) = 140, n (B) = 110, n (P) = 90, n(S ∩ B) = 40,
n (B ∩ P) =
 40, n (P ∩ S) Hence, the required probability is
= 50 and n (S ∩ B ∩ P) = 30 1 1 1  7
2 + +  =  Choice (C)
S B  4 8 16  8
5. Considering different values of a, b and c from the set
{1, 2, 3, 4, 6, 8, 9}, we get different quadratic equations.
P As a, b and c are distinct, 7P3 = 210 different quadratic
\  Probability that a randomly selected passenger equations can be formed.
likes only \ Total ways are 210
pizza For the quadratic equation ax2 + bx + c = 0 to have equal
roots, b2 = 4ac.
 =
(The number of passengers who like only pizza ) The possible combinations of a, b and c respectively
(The total number of passengers) are 1, 6, 9 and 9, 6, 1.
2.26 | Engineering Mathematics Test 4

Hence favourable cases are 2 Probability of Bipin winning is the same as probability
2 1 of Arpit losing i.e.,
\ Required probability = = . Choice (C) 10 7
210 105 \ P(B) = P ( A) = 1 − =  Choice (B)
6. We have 4 five rupee coins, 3 two rupee coins and 3 one 17 17
rupee coins. Note: If ‘p’ is the probability of success (in this case
For the draw to yield a maximum amount, of the 6 coins picking up an orange ball), the probability that the
drawn 4 should be five rupee coins and 2 should be two 1
beginner wins the game =
rupee coins. The required probability is 2− p
4
C 4 × 3 C2 3 1
= = 1
10
C6 210 70 10. Probability of selecting urn A is P ( A) = .
2
Hence, odds in favour are favourable ways : unfavoura- 1
ble ways = 1 : 69. Choice (B) and that of selecting urn B is P ( B ) =
2
7. There are 5 boxes and 5 labels. Hence the boxes can be Probability of drawing a black ball (event E) when urn
labelled in 5! i.e. 120 different ways 7
 E C
1 1 1 1 A is selected P   = 13 1 and probability of E when
P (all labelled incorrectly) = − + −  A C1
2! 3! 4! 5!
6
44 11  E C
= =  Choice (D) urn B is selected P   = 14 1
120 30  B C1
8. There are 9 numbered cards in each suit. Probability of selecting black ball
P(all the 4 cards are numbered cards of same suit)  E  E
9
C4 + 9 C4 + 9 C4 + 9 C4 4 × 9 C4 = P ( A) � P   + P ( B ) � P  
= 52  A  B
= 52  Choice (A)
C4 C4
1 7 C1 1 6 C1
3   � + �
9. The probability of picking up an orange ball is 2 13 C1 2 14 C1
10
7 1 7 C1
while not picking up an orange ball is . �
10 2 13 C1
Required Probability =
We compute the probability of Arpit (the beginner) 1 7 C1 1 6 C1
� + �
winning the game. 2 13 C1 2 14 C1
Let A and B be the events of Arpit and Bipin picking up 7
7
an orange ball respectively 13 7 × 14 49
The winning sequence of Arpit can be = = 13 = =  Choice (B)
7 6 98 + 75 176 88
+
A, A B A, A B A B A, � � � � � � 13 14 13 × 14
As the above sequence indicates, Arpit may pick an 11. The marginal distributions are given below.
orange ball right in the 1st trial with a probability of
3    Y 0 1 2 Px(x)
(or) in the third trial (as the 2nd trial is made by X
10
1 2 3 6
Bipin, and for Arpit to win, Bipin should not be getting 0
2 40 40 40 40
 7 3
an orange ball). The probability here being   ×
 10  10 2 3 1 6
4 1
 7 3 40 40 40 40
(or) in the fifth trial with a probability of   ×
 10  10
3 1 7 11
and so on. 2
40 40 40 40
2 4
3  7 3  7 3
\ P(A) = +   × +   × + ����� 4 5 8 17
10  10  10  10  10 3
40 40 40 40
3
10 30 10 10 11 19
     = 2
= = 1
 7 51 17 Py(y)
1−   40 40 40
 10 
Engineering Mathematics Test 4 | 2.27

P(X ≤ 1, Y = 2) = P(X = 0, Y = 2) + P(X = 1, Y = 2) Given P(x + y ≤ k) = p(9x – y ≥ 2k)


3 1 4 1 P(z ≤ z1) = p(z ≥ z2)
= + = =  Choice (D) P(z ≤ z1) = p(z ≤ –2z)
40 40 40 10
k − 9 −(2k − 21)
50 1 = = =k–9
12. Average number of errors per page l = = 10 90
500 10
1 1 −2k + 21
Average number of errors per 5 pages = 5 x = =
10 2 3
λk 3k – 27 = –2k + 21
Probability of k errors per page is P(x = k) = e −λ �
k! 48
5k = 48 ⇒ k = = 9�6  Choice (B)
5
\ here k = 0
15. Let B1: transfer of white ball to bag B.
λ k −λ − 12 B2: transfer of green ball to bag B.
\ P(k = 0) = e =e
k!
9 5
P(B1) = ; P ( B2 ) =
\ Probability that a random sample of 5 pages has no 14 14
error = e–0.5 = 0.6065 Choice (A)
Let E be the event of drawing a green ball from bag B
13. We know that X is uniform random variable in the after transfer.
1
interval [a, b] then p(x) = , a < x < b and mean =  E
b−a P   = probability of drawing green ball if white
 B1 
a+b (b − a)2
, variance = 7
2 12 ball is transferred to bag B =
14
a+b
Given mean = 2 ⇒ =2  E
2 P   = probability of drawing a green ball if green
 B2 
⇒ a + b = 4  ---- (1) 8
ball is transferred to bag B = .
(b − a) 2
14
Variance = 12 ⇒ = 12
12  E  E
\ 
P(E) = P(B1) . P   + P(B2) . P  
⇒ (b – a)2 = 144 ⇒ b – a = 12  ------ (2)  B1   B2 
Solving (1) and (2) a = –4, b = 8
9 7 5 8 63 40 103
1 1 1 = � + � = + =
\ P(x) = = = 14 14 14 14 196 196 196
b − a 8 − ( −4) 12
 B1 
b 8
1 1
8
8 2 \ The required probability P  
∫ p( x )dx = ∫ dx = x  = E
P(x > 0) =
12 12  12 = 3
0 0 0
 E 9 7
 Choice (C) P ( B1 )�P  
 B1  14 � 14 63
1 4. Given mean of X = 3 = = =  Choice (B)
P( E ) 103 103
Variance of X = 1 196
Mean of Y = 6
Variance of Y = 9 16. Let A be the event that the number 3 appears atleast
X = N(3, 1) Y = N(6, 3) and X and Y are independent. once.
Let u = x + y; and v = 9x – y B be the event that sum of the numbers appearing is 7.
Then u, v are also normal variates A ∩ B be the event that the sum is 7 and 3 appear atleast
U = x + y = N(3 + 6, 1 + 9) = N(9, 10) 6 2
once P(B) = ⇒ P(A ∩ B) =
V = 9x – y = N(9(3) – 6, 81(1) + 9) = N(21, 90) 36 36
By definition A/B denotes atleast one number show 3 while the sum
u−9 k −9 of the numbers is 7.
Z= and for u = k ⇒ Z = =z
10 10 2
 A  P ( B ∩ A) 36 1
v − 21 2k − 21 P  = = =  Choice (C)
Again z = and for v = 2k ⇒ Z = = z1  B P ( B) 6 3
90 90 36
2.28 | Engineering Mathematics Test 4

17. We know ∑P(X = x1) = 1 \ mean (M) = ∑Xi P (x = xi)


\ k + 2k + 3k + 4k + 5k = 1 2 3 0 1
1 0 × +1× + 2 × + 3 ×
=
15k = 1 ⇒ k = 6 6 6 6
15
0 + 3+ 0 + 3 6
P(X ≥ 2) = P(X = 2) + P(X = 3) + P(X = 4) = = =1
6 6
12 4
= 3k + 5k + 4k = 12k = =  Choice (A) Variance = ∑xi2 P(x = xi) – µ2
15 5
18. By definition. Choice (C) 2 3 1
0 × +1× + 4 × 0 + 9 × −1
=
6 6 6
19. Standard Result. Choice (A)
3+ 9
20. Standard Result. Choice (C) = − 1 = 2 – 1 = 1. Choice (C)
6
21. We know that, if a statistic ‘s’ follows t-distribution 24. We know that variance (x, x + 3, x + 5, x + 7, x + 10)
with degrees of freedom = ν, then ‘s2’ follows = variance (0, 3, 5, 7, 10)
F–distribution with degrees of freedom (1, ν)
0 + 3 + 5 + 7 + 10 25
AM(0, 3, 5, 7, 10) = = =5
Here ν = 10 5 5
∴ ‘s2’ follows F-distribution with degrees of freedom
= (1, ν) = (1,10). Choice (B) ∑( x1 − A)2
Variance (0, 3, 5, 7, 10) =
n
22. When the test statistic is outside the critical region, it (0 − 5) + (3 − 5) + (5 − 5) + (7 − 5)2 + (10 − 5)2
2 2 2

lies in the acceptance region. So, we will accept the null =


5
hypothesis and reject the alternative hypothesis.
25 + 4 + 0 + 4 + 25 58
∴ Both P and S are true. Choice (C) = =
5 5
23. Three letters are placed into 3 addressed envelopes ran- 58
domly in 3! = 6 ways. Variance = = 11�6 . Choice (B)
5
X denotes the number of letters placed into correspond-
ing addressed envelopes. The probability distribution 25. The ascending order of the given data except x is 3, 8,
table is as follows. 12, 15, 16, 28
If x < 12, the fourth observation is 12 hence median is
X = xi 0 1 2 3
12 if x > 15, the fourth observation is 15, hence median
2 3 0 1 is 15. If 12 < x < 15, the fourth observation is x, hence
P(x = xi) median is x.
6 6 6
Median is always lies between [12, 15]. Choice (D)
Engineering Mathematics Test 5
Number of Questions: 25 Time: 60 min.

Numerical Methods

Directions for questions 1 to 25: Select the correct alterna- 7. The following table gives the velocity n of a particle at
tive from the given choices. time t.
1. In the process of finding an approximate root of T(in seconds) 0 2 4 6 8 10 12
f(x) = 0 in [a, b] (where f(a) and f(b) are of opposite n(in m/sec) 6 10 16 20 22 30 40
signs) by Regula – Falsi method, we assume that the
curve f(x) = 0 in between x = a and x = b can be approx- The distance moved by the particle in 12 seconds,
imated to ________. when calculated by the Trapeziodal rule with h = 2
(A) a parabola is ________.
(B) a straight line (A) 200 meters (B) 210 meters
(C) a hyperbola (C) 242 meters (D) 262 meters
(D) a rectangular hyperbola 8. A curve is drawn to pass through the points given by
2. The iterative formula to find a root of the equation the following table
f(x) = x3 – 5x + 7 = 0 by Newton Raphson method X 1.0 1.5 2.0 2.5 3.0 3.5 4.0
is _______. Y 1.0 1.7 2.5 3.4 4.1 3.7 2.9
x3 + 5x − 7 2 xk3 + 5 x The area bounded by the curve, the x – axis and the
(A) xk + 1 = k 2 xk + 1 =
(B)
3 xk + 5 3 xk2 + 7 lines x = 1 and x = 4, when calculated by the Simpson’s
3
2 xk3 − 7 x3 − 5x th Rule is _____ square units.
(C) xk + 1 = xk + 1 = k 2
(D) 8
3 xk − 5
2
3 xk + 7
(A) 8.7562 (B) 5.7435
3. With x0 = 0.5 as the initial approximation, the value of (C) 6.7134 (D) 8.4296
the root of f(x) = x + sin x – 1 = 0, after first iteration by 9. The absolute error (correct up to 4 decimal places) in
Newton Raphson method is _______. calculating the value loge2 by trapezoidal rule, with 4
2
(A) 0.7456 (B) 0.5110 dx
(C) 0.4998 (D) 0.2644 intervals using the formulae loge2 = ∫ is _______.
1
x
4. Applying the secant method, the first approximation to (A) 0.1314 (B) 0.0039
the root of f(x) = xex – 2 = 0, starting with function (C) 0.0000 (D) 0.0004
value at x = 0.5 and x = 1 is _______.
(A) 1.1756 (B) 0.4035 10. With reference to finding solution of a differential
(C) 0.8104 (D) 0.5473 equation by numerical methods, which of the following
methods is NOT a predictor correct method?
5. The extreme (minimum or maximum) point of a func- (A) Picard’s method
df ( x )
tion f(x) is to be determined by solving = 0 using (B) Modified Euler’s method
dx (C) Adams – Bash forth method
the Newton Raphson method. Let f(x) = x3 – 4x2 + 5 and (D) Milne’s method
x0 = 3 be the initial guess of x. The value of x after first dy
iteration (x1) is ______. 11. The differential equation – x2 = y; y(0) = 1 is to be
dx
(A) 2.70 (B) 4.33
(C) 3.30 (D) 1.77 solved by the modified Euler’s method. With h = 0.1,
the value of y1 correct to four decimal places is ______.
π (A) 1.2046 (B) 1.1058
2

∫ (x + sin 2 x + 5) dx using
6. In the process of evaluating 3 (C) 0.9954 (D) 0.8764
0 12. Using Taylor’s series method, the solution of the differ-
π dy
Simpson’s Rule with h = , the absolute value of the ential equation – xy = 1 with y(0) = 3 at x = 0.1 with
8 dx
error does not exceed ______. h = 0.1 is correct upto three decimal places is _______.
(A) 0.12351 × 10–4 (B) 1.03503 × 10–4 (A) 3.1153 (B) 2.9847
(C) 3.01243 × 10–4 (D) 6.2475 × 10–4 (C) 4.1572 (D) 3.7893
2.30 | Engineering Mathematics Test 5

dy linear form Y = a + bX, X and Y respectively stand


13. The solution of the differential equation = x + y; for _____.
dx
1 x2 1 x
y (0) = 0 at x = 0.2 by Runge Kutta method of fourth (A) 2 and (B) and
order with h = 0.2 is ______. x y x y
(A) 1.0034 (B) 0.0456 y y
(C) 0.9984 (D) 0.0214 (C) x2 and 2 (D) x and
x x
14. Consider an equation f(x) = 0 for which x = 4.50 is an
20. In the process of fitting a curve exp(y) = abx to a given
exact root. In the process of finding a root of f(x) = 0
set of n pairs of values of x and y by converting it into
by a numerical method, the approximations obtained in
a linear form y = A + Bx, A and B respectively stand
four successive iterations are 4.45, 4.54, 4.47 and 4.52
for _____
respectively. Then these approximate values of the root
(A) ln a and ln b (B) ln a and log10b
of f(x) = 0 are _____.
(C) log10a and ln b (D)log10a and log10b
(A) precise but not accurate
(B) not precise but accurate 21. If ∆ denotes the forward difference operator then
(C) both precise and accurate the value of ∆18 (1 + 2 x 3 )(1 − 3 x 4 )(1 + 4 x 5 )(1 − 5 x 6 )
(D) neither precise nor accurate
is _____.
1 5. For an equation f(x) = 0, if xe­ is the exact root and (A) 5 ! × 18 !
xa is the approximate root, then the percentage error (B) 6 ! × 18 !
is _____. (C) 5 ! × 17 !
(A) ( xe − xa ) × 100 (B)
xe − xa × 100 (D) 6 ! × 17 !

(C)
( xe − xa ) × 100 (D)
xe − xa
× 100
22. The central difference operator δ is defined as
xe xe yr – yr–1 = δy 1
r−
2
16. Consider the following two statements Then which of the following is an identity? (Note that ∆
P: Truncation error in numerical analysis arise when and ∇ denote the forward and the backward difference
approximations are used to estimate some quantity. operators respectively)
Q:  Round off error in numerical analysis occurs
(A) ∆y5 = ∇y4 = δy 7
because of the computing devices inability to deal
2
with certain number.
Then (B) ∆y5 = ∇y4 = δy4
(A) Both P and Q are true (C) ∆y4 = ∇y5 = δy 9
(B) P is true but Q is false 2

(C) P is false but Q is true (D) ∆y4 = ∇y5 = δy4


(D) Both P and Q are false
23. Match the following
17. In the process of fitting a quadratic equation of the
form y = a + bx + cx2 to a set of n points (x1, y1), (x2,y2), Group – I Group – II
….., (xn,yn) by the method of least squares, which of the P. To extrapolate the 1. Newton’s divided
following is not a normal equation? values of y to the left difference formula
(A) ∑ yi = na + b ∑ xi + c ∑ xi 2 of y0 when x values are
equally spaced
(B) ∑ xi yi = a ∑ xi + b ∑ xi 2 + c ∑ xi 3 Q. To interpolate the values 2. Lagrange’s
(C) ∑ xi yi 2 = a ∑ xi 2 + b ∑ xi 3 + c ∑ xi 4 of y near the end value interpolation formula
yn when x values are
(D) ∑ xi 2 yi = a ∑ xi 2 + b ∑ xi 3 + c ∑ xi 4 equally spaced
R. To split the given func- 3. Newton’s forward
18. If y = 3x + 7 is the best fit for 6 pairs of values of x and tion into partial fractions interpolation formula
y by the method of least squares and ∑ y = 150, then ∑
S. To interpolate the values 4. Newton’s backward
x is _____. of y when x values are interpolation formula
(A) 144 (B) 102 unequally spaced.
(C) 46 (D) 36
(A) P – (1), Q – (2), R – (3), S – (4)
x2 (B) P – (3), Q – (2), R – (4), S – (1)
1 9. In the process of fitting a curve y = to a given
ax + b (C) P – (3), Q – (4), R – (2), S – (1)
set of n pairs of values of x and y by converting it into a (D) P – (2), Q – (1), R – (4), S – (3)
Engineering Mathematics Test 5 | 2.31

24. The 9th divided difference of a polynomial of degree 8 25. If f(0) = − 12, f(3) = 6 and f(4) = 12, then the value of
is _____. f(6) obtained by the Lagrange’s interpolation formula
(A) zero is _____.
(B) a non – zero constant (A) 18 (B) 24
(C) a linear polynomial (C) 20 (D) 26
(D) a quadratic polynomial

Answer Keys
1. B 2. C 3. B 4. C 5. A 6. B 7. C 8. A 9. B 10. A
11. B 12. A 13. D 14. B 15. D 16. A 17. C 18. D 19. B 20. A
21. A 22. C 23. C 24. A 25. B

Hints and Explanations


1. Standard Result. Choice (B) ∴ We have to find the approximate root of g(x) =
2. Given, f(x) = x – 5x + 7 = 0
3 0 after first interaction by the Newton Raphson
⇒ f1(x) = 3x2 – 5. method with x0 = 3.
By Newton Raphson’s method, the interative formulae \  g1(x) = 6x – 8
to find a root is By Newton Raphson method

f ( xk ) (x 3
− 5 xk + 7 ) g ( x0 )
xk + 1 = xk – = xk –
k x1 = x0 –
f 1 ( xk ) (3 x 2
− 5) g 1 ( x0 )
k

2 xk3 − 7 (3(3)2 − 8(3))


\ xk + 1 = . Choice (C)    = 3– = 2.7. Choice (A)
3 xk2 − 5 6(3) − 8
3. Given, f(x) = x + sin x – 1 = 0 π
⇒ f1(x) = 1 + cos x and x0 = 0.5 2

By Newton Raphson’s method 6. We have ∫ ( x 3 + sin 2 x + 5)dx .


0
f ( x0 )
x1 = x0 – Let y = f(x) = x3 + sin2x + 5.
f 1 ( x0 )
The absolute value of the maximum error in Simpson’s
(0�5 + sin(0�5) − 1) Rule is
 = (0.5) –
(1 + cos(0�5))
(b − a)h4
  = 0.5110. Choice (B) |E|max = m ----------- (1)
180
4. Here, f(x) = xe – 2 = 0
x

By the secant method, the approximate root of f(x) = 0 Where m = max – { y0( iv ) , y2( iv ) , y4( iv ) }
after first iteration is given by π π
Here, h = and y(iv) = 16 sin2x, a = 0, b =
x0 f ( x1 ) − x1 f ( x0 ) 8 2
x2 =  ---------- (1)
f ( x1 ) − f ( x0 )  π
\ m = max [16sin0, 16sin  2 ×  ,
 4
Here, x0 = 0.5 and x1 = 1
\ f(x0) = f(0.5) = –1.1756 and f(x1) = f(1) = 0.7183  π
16 sin  2 ×  ] = 16
Substituting these in (1) we have  2
(0�5)(0�7183) − (1)( −1�1756) From (1)
x2 =
(0�7183) − ( −1�1756)
π   π
\ x2 = 0.8104. Choice (C)    
2 8
5. Given, f(x) = x3 – 4x2 + 5 |Emax| = × 16 = 1.0350 × 10–4
180
df ( x )
= 0 ⇒ 3x2 – 8x = 0 The absolute value of the maximum error cannot
dx exceed 1.03503 × 10–4.
Let, g(x) = 3x2 – 8x = 0.  Choice (B)
2.32 | Engineering Mathematics Test 5

7. Given velocity of the particle at various times is The absolute error in calculating loge2 by the trapezoi-
0 2 4 6 8 10 12
dal rule = 0.6970 – 0.6931 = 0.0039. Choice (B)
T
n 6 10 16 20 22 30 40
10. A predictor corrector method is one in which we predict
the solution first and then we improve it for accuracy
12
Picard’s method is not a predictor corrector method
Distance traveled in 12 seconds = ∫ νdt .
0
and all other methods are predictor corrector methods.
 Choice (A)
By trapezoidal rule dy
1 1. Given different equation is – x2 = y and y(0) = 1
12
h dx
∫ νdt = 2
[(n0 + n6) + 2 (n1 + n2 + n3 + n4 + n5)] dy
0 ⇒ = x2 + y
2 dx
= [(6 + 40) + 2(10 + 16 + 20 + 22 + 30)] \ f(x, y) = x2 + y, x0 = 0, y0 = y(x0) = 1 and h = 0.1
2
By Euler’s method
= 242 meters. Choice (C)
y1(0) = y0 + h (f(x0, y0) = y0 + h ( x02 + y0 )
8. Let y = f(x) be the curve, that pass through the points
=
1 + (0.1) (0 + 1)
X 1.0 1.5 2.0 2.5 3.0 3.5 4.0
y1(0) = 1.1
Y 1.0 1.7 2.5 3.4 4.1 3.7 2.9
By modified Euler’s method
∴ The area bounded by the curve y = f(x), x – axis h
4
y1(1) = y0 +  f ( x0 , y0 ) f ( x1 y1(0) )
 
andthe lines x = 1 and x = 4 is ∫ f ( x)dx . 2
1
h
3 =
y0 + ( x02 + y0 ) + ( x12 + y1(0) )
2  
By Simpson’s th Rule, we have
8 (0�1)
4
3h =
1+ [(0 + 1) + (10.1)2 + 1.1)] = 1.1055
∫1 f ( x)dx = 8 [(y0 + y6) + 3(y1 + y2 + y4 + y5) + 2y3] 2
h  f ( x0 , y0 ) + f ( x1 , y1(1) )
3 × (0�5) y1(2) = y0 +  
= [(1.0 +  2.9) + 3(1.7 + 2.5 + 4.1 2
8 h
+ 3.7) + 2 × 3.4] = 8.7562. =
y0 + ( x02 + y0 ) + ( x12 + y1(1) )
2  
 Choice (A)
(0�1)
1 =
1+ [(0 + 1) + ((0.1)2 + 1.1055)]
9. Let y = f(x) = 2
x
=
1.1058
Here, a = 1, b = 2 and n = 4
h  f ( x0 , y0 ) + f ( x1 , y1(2) )
b−a y1(3) = y0 +  
\ h = = 0.25 2
n
h
=
y0 + ( x02 + y0 ) + ( x12 + y1(2) )
X 1 1.25 1.5 1.75 2 2  
F(x) 1 0.8 0.667 0.5714 0.5 (0�1)
=
1+ [(0 + 1) + ((0.1)2 + 1.1058)]
2
2
dx = 1.1058
We have loge2 = ∫
1
x The solution of a given different equation at x1 = 0.1 is
y1 = 1.1058. Choice (B)
By the trapezoidal rule, we have
du
2
dx
2 1 2. Given differential equation is – xy =1 and y(0) = 3
h dx
∫1 x = ∫ f ( x)dx =
1 2
[(y0 + y4) + 2(y1 + y2 + y3)]
dy
⇒ = 1 + xy
dx
0�25
[(1+ 0.5) + 2(0.8 + 0.6667 + 0.5714)] Here f(x, y) = 1 + xy, x0 = 0, y0 = y(x0) = 3 and h = 0.1
2
2 By Taylar’s series we have
dx
\ loge2 = ∫ = 0.6970  ---------- (1) y1 = y(x1) = y0 + h0 y10 +
h2 11
y0 +
h3 111
y0 + ….. ∞ 
1
x 2! 3!
The exact value of loge2 = 0.6931  -------- (2) ----(1)
Engineering Mathematics Test 5 | 2.33

 dy  14. The four approximations given are not relatively close


y10 =   = f(x0, y0) = 1 + x0 y0 = 1 + (0)(3) = 1 to each other. So, they are not precise. All the four
 dx  x = 0
approximations are close to the exact root x = 4.50. So,
d2 y d  dy  d they are accurate. Choice (B)
=   = dx (1 + xy) = xy + y
1
dx 2
dx dx 15. By definition. Choice (D)
 d2 y 16. By definitions of the round off and the truncation errors.
\ y11
0 =  dx 2  = x0 y10 + y0 = 0 × 1 + 3 = 3  Choice (A)
atx = x 0
17. Given set of n points are (x1, y1,),
3
d y d  d y 2
d (x2, y2),…..,(xn, yn)
=  dx 2  = dx (xy + y) = xy + 2y
1 11 1

dx 3 dx We have to fit the quadratic equation


d3 y y = a + bx +cx2  → (1)
\ y111
0 = = x0 y11
0 + 2 y0 = 0 × 3 + 2 × 1 =2
1
to the given set of n points
dx 3 atx = x0
Here a, b and c are constants to be determined such that
d4 y d  d3 y d S = ∑  yi − ( a + bxi + cxi2 ) 
2
= = (xy11 + 2y1) = xy111 + 3y11 → (2)
dx 4  3 
dx  dx  dx
is minimum
d4 y S is minimum for those values of a, b and c at which
y0( iv ) = = x y111
0 + 3 y0 = 0 × 2 + 3 × 3 = 9
11

dx 4 atx = x0 0 ∂S ∂S ∂S
= 0, = 0 and = 0�
Substituting these in (1), we have ∂a ∂b ∂c
y1 = 3 + (0.1) × 1 +
(0�1)2
2!
×3+
(0�1)3
3!
×2+
(0�1) 4
4!
× (∑  y − (a + bx + cx )) = 0
i.e., −2 i i
2
i

9 + … = 3.1153. Choice (A) −2 x ( ∑  y − ( a + bx + cx )) = 0


i i i
2
i
dy
and −2 x ( ∑  y − ( a + bx + cx )) = 0
13. Given differential equation is = x + y, y(0) = 0 2 2
dx i i i i

Here f(x, y) = x + y, x0 = 0; y0 = 0 and h = 0.2


∴ the normal equations are
\ x1 = x0 + h = 0.2
By R – K method of fourth order we have ⇒ ∑ yi = na + b∑ xi + c ∑ xi2
Yat x = 0.1 = y1 = y0 + Dy  ---------- (1)
1
∑ x y = a∑ x + b ∑ x + c ∑ x
i i i
2
i
3
i

Where Dy = [k1 + 2k2 + 2k3 + k4]  ------- (2)


6 And ∑ x y = a∑ x + b∑ x + c ∑ x
2
i i
2
i
3
i
4
i

Here k1 = h f(x0, y0) = h(x0 + y0) ∴ the equation given in option (C) is not a normal
    = (0.2) (0 + 0) equation.  Choice (C)
\ k1 = 0 18. Given that y = 3x + 7 is the best fit for 6 pairs of values
 h k1  of x and y also given ∑ y = 150
k2 = h f  x0 + , y0 + 
 2 2 ∴ We know that
 h  k  ∑ y = 3∑ x + n7
  =h   x0 +  +  y0 + 1   (Here n = number of points = 6)
  2   2 
⇒ 150 = 3 ∑ x + 6 x 7
 (0�2)   0  ⇒ 3 ∑ x = 108
  =(0.2)  0 +  +  0 +  
  2 2  ⇒ ∑ x = 36. Choice (D)
2
\ k2 = 0.02 x
1 9. Given curve is y =
K3 = 0.022 and k4 = h f(x0 + h, y0 + k3) ax + b
= h [(x0 + h) + (y0 + k3)] 1 ax + b
⇒ =
= (0.2) [(0 + 0.2) + (0 + 0.022)] y x2
\ k4 = 0.0444 a b
\ From (2), ⇒ = +
y x x
1
Dy = [0 + 2 × 0.02 + 2 × 0.022 + 0.0444] x b
6 ⇒ = a+
y x
Dy = 0.0214
x b
\ From (1), y1 = y0 + Dy = 0 + 0.0214 = 0.0214. ⇒ = a+
 Choice (D) y x
2.34 | Engineering Mathematics Test 5

Which is of the linear from Y = a + bX Among the options given, we can get option (C) by tak-
1 x ing r = 5 in (4),
Where X = and Y =  Choice (B)
x y ∴ ∆y4 = ∇y5 = δy 9  Choice (C)
2
20. Given curve is exp(y) = abx
23. Standard result Choice (C)
i.e., ey = abx
Applying logarithm (ln) on both sides, 24. We know that the n divided difference of any polyno-
th

mial of degree less than n is always zero. Choice (A)


We have
ln (ey) = ln (abx) 25. Given pairs of values of x and f(x) are
⇒ y = lna + lnbx X 0 3 4
⇒ y = lna + xlnb f(x) –12 6 12
Which is of the form
By Lagrange’s interpolation formula, we have
y = A + Bx
( x − x1 ) ( x − x2 )
Where A = ln a and B = ln b. Choice (A) f(x) = f(x0)
( x0 − x1 ) ( x0 − x2 )
21. We have ∆18 (1 + 2 x 3 )(1 − 3 x 4 )(1 + 4 x 5 )(1 − 5 x 6 ) ( x − x0 ) ( x − x2 ) ( x − x0 ) ( x − x1 )
+ f(x1) + f(x )
( x1 − x0 ) ( x1 − x2 ) ( x2 − x0 ) ( x2 − x1 ) 2
= ∆18 2 x( −3) x 4 x( −5) x18 + k1 x17 + k 2 x16 + ���� + k15 x 3 + 1
( x − 3) ( x − 4) ( x − 0) ( x − 4)
= 5! ∆18 [x18] = × (–12) + ×6
( 0 − 3) (0 − 4) ( 3 − 0) (3 − 4)
(∵ ∆18[xn] = 0 for n <18)
= 5 ! × 18 !. Choice (A) ( x − 0) ( x − 3)
+ × 12
( 4 − 0) (4 − 3)
22. We know that ∆ yr-1 = yr – yr – 1  → (1)
By taking x = 6 on both sides,
∇ yr = yr – yr – 1  → (2) We have
and given that δy = yr − yr −1  → (3) (6 − 3)(6 − 4) (6 − 0)(6 − 4)
r−
1 f (6) = × ( −12) + ×6
2 3× 4 3 × ( −1)
From (1), (2) and (3), we have (6 − 0)(6 − 3)
+ × 12
∆ yr–1 = ∇ yr = δy  → (4) 4 ×1
1
r−
2 ∴ f(6) = 24 Choice (B)
 | 3.1

PART III 
CIVIL ENGINEERING

Unit I  Engineering Mechanics


Engineering Mechanics Test 1����������������������������������������������������������������������������������������������������� 3.5
Engineering Mechanics Test 2����������������������������������������������������������������������������������������������������� 3.9

Unit II  Solid Mechanics


Solid Mechanics Test 1���������������������������������������������������������������������������������������������������������������� 3.21
Solid Mechanics Test 2���������������������������������������������������������������������������������������������������������������� 3.27

Unit III  Structural Analysis


Structural Analysis Test 1���������������������������������������������������������������������������������������������������������� 3.35
Structural Analysis Test 2���������������������������������������������������������������������������������������������������������� 3.44

Unit IV  Construction Materials and Management


Construction Materials and Management Test 1��������������������������������������������������������������������� 3.53
Construction Materials and Management Test 2��������������������������������������������������������������������� 3.56

Unit V  Concrete Structures


Concrete Structures Test 1��������������������������������������������������������������������������������������������������������� 3.63
Concrete Structures Test 2��������������������������������������������������������������������������������������������������������� 3.68
Unit VI  Steel Structures
Steel Structures Test 1���������������������������������������������������������������������������������������������������������������� 3.77
Steel Structures Test 2���������������������������������������������������������������������������������������������������������������� 3.83

Unit VII  Geotechnical Engineering


Soil Mechanics Test 1������������������������������������������������������������������������������������������������������������������ 3.91
Soil Mechanics Test 2������������������������������������������������������������������������������������������������������������������ 3.92
Geotechnical Engineering Test 3����������������������������������������������������������������������������������������������� 3.97
Foundation Engineering Test 4������������������������������������������������������������������������������������������������ 3.103

Unit VIII  Fluid Mechanics and Hydraulics


Fluid Mechanics Test 1������������������������������������������������������������������������������������������������������������� 3.109
Fluid Mechanics Test 2������������������������������������������������������������������������������������������������������������� 3.114
Hydraulics Test 3����������������������������������������������������������������������������������������������������������������������� 3.121

Unit IX  Water Resources Engineering


Hydrology Test 1������������������������������������������������������������������������������������������������������������������������ 3.129
Irrigation Test 2������������������������������������������������������������������������������������������������������������������������� 3.133
Water Resource Engineering Test 3���������������������������������������������������������������������������������������� 3.138

Unit X  Environmental Engineering


Water Supply Engineering Test 1�������������������������������������������������������������������������������������������� 3.145
Waste Water Engineering Test 2���������������������������������������������������������������������������������������������� 3.149
Solid Waste Management and Pollution Test 3���������������������������������������������������������������������� 3.154
Environmental Engineering Test 4������������������������������������������������������������������������������������������ 3.157

Unit XI  Transportation Engineering


Transportation Infrastructure Test 1�������������������������������������������������������������������������������������� 3.163
Traffic Engineering Test 2��������������������������������������������������������������������������������������������������������� 3.166
Transportation Engineering Test 3������������������������������������������������������������������������������������������ 3.174
Highway Engineering Test 4����������������������������������������������������������������������������������������������������� 3.179

Unit XII  Geomatics Engineering


Geomatics Engineering Test 1�������������������������������������������������������������������������������������������������� 3.185
Geomatics Engineering Test 2�������������������������������������������������������������������������������������������������� 3.188
Unit I
Engineering Mechanics
This page is intentionally left blank
Engineering Mechanics Test 1
Number of Questions: 25 Time: 60 min.

Directions for questions 1 to 25: Select the correct alterna- 8. Two metallic balls having potential energy in the ratio
tive from the given choices. 3 : 5 are made to slide down a frictionless inclined
1. The value of coefficient of restitution is one for plane with zero position. What will be the ratio of their
(A) perfectly elastic collision kinetic energy when they reach at bottom of inclined
(B) perfectly inelastic collision plane?
(C) neither plastic nor elastic collision (A) 5 : 3 (B) 3 : 5
(D) None of these (C) 1 : 1 (D) 2 : 3
2. The radius of gyrations for a sphere and cylinder of 9. Two forces form a couple only when
radius ‘R’ are respectively. (A) magnitude is same have parallel lines of action
(A) 0.6324 R and 0.707 R and same sense
(B) 0.6234 R and 0.77 R (B) magnitude is different, have parallel lines of action
(C) 0.6432 R and 1.414 R but same sense
(D) 0.6324 R and 1.414 R (C) magnitude is same have non parallel lines of
action but same sense
3. Which of the following relation represents motion
(D) magnitude is same and have parallel lines of action
under variable acceleration?
and opposite sense
dv dv
(A) v = a (B) a=v 10. A wheel is rolling on a straight road as shown below.
ds ds
For this wheel the acceleration of the center ‘O’ and its
1 dv
(C) v = (D) None of these instantaneous center are
a ds
B
4. If a projectile motion with usual notations is expressed ω
is A C V
O
gx 2 D
y = xP – (a = Angle of projection), then ‘P’
2u 2 Q 2
and ‘Q’ are (A) ω2r and O (B) ω2r and D
(A) tana and cos2a (C) V /r and D
2
(D) zero and O
(B) tana and cosa 11. A particle moving from rest moves in a straight line. Its
(C) tana and sec a acceleration is given by the equation
(D) tana and sec2a a = 10 - 0.006 S2
Velocity of the particle when it has travelled 40 m is
5. A mechanism has 5 numbers of joints and 6 members.
(A) 19.16 m/s (B) 23.32 m/s
The number of additional members needed to make it a
(C) 26.84 m/s (D) 30.14 m/s
perfect frame will be
(A) 4 (B) 3 12.
(C) 2 (D) 1 B

6. The rate of change of velocity and the rate of change of


momentum of a moving body respectively are
(A) acceleration and impulse O
(B) acceleration and force
(C) displacement and force
A
(D) force and displacement
7. In the equation of virtual work, which of the following
force is neglected? A wheel of radius 1 m rolls on a flat horizontal ground
(A) reaction at any smooth surface with which the without slipping as shown in figure. Resultant velocity
body is in contact at point B is 1 m/s. Angular velocity of the wheel about
(B) reaction of rough surface of a body which rolls on its centre in rad/s is
1
it without shipping (A) (B) 2
(C) reaction at a point on an axis fixed in space, around 2
which a body is constrained to turn. 1
(C) 1 (D)
(D) All of these 2
3.6 | Engineering Mechanics Test 1

13. Acceleration of a particle is given by 19. Time taken by the projectile to reach the ground after
a = t3 -3t2 + 5 firing is
Where t = time in seconds and (A) 9.8 s (B) 8.6 s
a = acceleration in m/s2. Velocity of particle when (C) 7.2 s (D) 5.1 s
t = 2 sec is 8 m/s. Velocity of the particle when t = 4 20. Horizontal range of the projectile is
sec is (A) 220.7 m (B) 208.5 m
(A) 22 m/s (B) 25 m/s (C) 192.6 m (D) 186.1 m
(C) 28 m/s (D) 32 m/s
Common Data for Questions 21 to 23:
Statement for Linked Data Questions 14 and 15: 10 kg

14. A body of mass 5 kg falls from a height of 50 m and A


penetrates into the ground by 90 cm. Average resistance
to penetration is
(A) 2668 N (B) 2774 N
(C) 2814 N (D) 2892 N
15. Time taken for penetration is B 5 kg
(A) 19.7 s (B) 12.7 s
(C) 17.4 s (D) 15.4 s Block A of mass 10 kg placed on a rough horizontal plane is
connected to another block B of mass 5 kg by a string pass-
16. Angular displacement of a body is given by ing over a pulley as shown in figure. Coefficient of friction
q = 6t2 + 3t + 10 between block A and horizontal plane is 0.25. If the system
Where t is in seconds. Angular velocity and angular is released from rest and block B is falling,
acceleration of the body when
21. Tension on the string is
t = 10 seconds are
(A) 28.43 N (B) 33.41 N
(A) 123 rad/s, 12 rad /s2
(C) 37.62 N (D) 40.88 N
(B) 135 rad /s, 14 rad /s2
(C) 142 rad/s, 16 rad/s2 22. Acceleration of block B is
(D) 153 rad/s, 18 rad/s2 (A) 1.268 m/s2 (B) 1.635 m/s2
(C) 1.824 m/s 2
(D) 2.116 m/s2
Statement for Linked Answer Questions 17 and 18: 23. Velocity acquired by block B when it falls through a
A ball can be projected with a maximum velocity of 50 m/s. vertical distance of 1 m, is
On an inclined plane, the maximum range obtained on pro- (A) 1.24 m/s (B) 1.56 m/s
jecting the ball is 190 m. (C) 1.81 m/s (D) 2.35 m/s
17. Inclination of the plane to the horizontal is Common Data for Questions 24 and 25:
(A) 20° (B) 18° Angular displacement of a particle, moving in a circular
(C) 16° (D) 14° path of 150 m radius is given by
q = 18t + 3t2 - 2t3
18. The projection angle from horizontal is
(A) 68° (B) 65° 24. Angular acceleration at 2 seconds from start is
(C) 60° (D) 55° (A) 15 rad/s2 (B) 18 ard/s2
(C) -15 ard/s (D)
2
-18 rad/s2
Statement for Linked Answer Questions 19 and 20: 25. Maximum angular velocity is
A projectile is fixed at an angle of 30° in a horizontal level (A) 16.4 rad/s (B) 19.5 rad/s
ground with a velocity of 50 m/s. (C) 22.3 rad/s (D) 25.4 rad/s

Answer Keys
1. A 2. A 3. B 4. B 5. D 6. B 7. D 8. B 9. D 10. D
11. B 12. D 13. A 14. B 15. C 16. A 17. A 18. D 19. D 20. A
21. D 22. B 23. C 24. D 25. B
Engineering Mechanics Test 1 | 3.7

Hints and Explanations

I min Integrating,
2. Radius of gyration = v2 0.006S 3
m = 10 S − +C
I = mass moment of inertia about the central axis
2 3
R = radius of sphere or cylinder. When S = 0, v = 0
2 2 \ C = 0
For sphere: I = mR or mK = mR
2 2 2
2
5 5 v 3
\ = 10 S 0.002 S
k = radius of gyration 2
When S = 40 m
2
\ k = R = 0.6324 R v2
5 = 10 × 40 - 0.002 × (40)3
2
For cylinder:
v = 23.32 m/s. Choice (B)
mR2 mR2 1 2. Resultant velocity at B
I= or mk 2 =
2 2 R = 2 n = 2 w r
\ k = R = 0.707R. Choice (A) \ 1 = 2 w × 1
5. Kutzbach equation 1
w =  radian. Choice (D)
F = 3( – 1) – 2j – h 2
F = degree of freedom
 = number of links or member dv 3
13. a = = t − 3t 2 + 5
J = number of lower pair dt
H = number of higher pair dv = (t3 - 3t2 + 5) dt
When 5 joints and 6 members are there Integrating,
4 v = ∫(t3 - 3t2 + 5) dt
5
t4 3
3  = − t + 5t + C
2 6 4
when t = 2, v = 8
1
24
=6 \ 8 = − 23 + 5 × 2 + C , C = 2
4
J=7
\ F = 1(kinematic chain) t4 3
\ v = − t + 5t + 2
For perfect frame, F = 0 4
\ When 5 joints and 7 members then When t = 4
=7 44
J = 9 v= - 43 + 5 × 4 + 2 = 22 m/s. Choice (A)
\ F = 0 (perfect frame) 4
Number of additional member = 1. Choice (D) 14. m = 5 kg, h = 50 m
x = 90 cm = 0.9 m
8. Since plane is frictionless, so KE at ground will be
equal to P. E at top. v1 = 2gh = 4.43 h = 4.43 50 = 31.32 m/s
1 1 v2 = 0
m1gh= m1v12 and m2 gh = m2 v2 2 Let R be the average resistance of penetration
2 2
1
KE1 m1 gh PE1 3 m(v22 - v12) = (mg - R) x
= = =  Choice (B) 2
KE2 m2 gh PE2 5 1
× 5(0 - 31.322) = (5 × 9.81 - R) 0.9 - 2724.84
dv dv dS dv 2
11. a = = . = ν. = 49.05 - R
dt dS dt dS
R = 2773.89 N. Choice (B)
dv
\ v = 10 - 0.006S2 15. Applying impulse momentum equation,
dS F × t = m(v2 - v1)
vdv = (10 - 0.006S2) dS F = mg - R = 49.05 - 2773.89 = - 2724.84 N
3.8 | Engineering Mechanics Test 1

\ -2724.84 × t = 5(0 - 31.32) 21. Let mass of block B be m1 and block A be m2


t = 17.4 sec. Choice (C) T
m2
1 6. q = 6t2 + 3t + 10
Angular velocity

w= = 12t + 3
dt
when t = 10 T
w = 12 × 10 + 3 = 123 rad/s m1
Angular acceleration
dω m1g - T = m1a
a= = 12 rad/s2. Choice (A) T - mm2g = m2a
dt
17.
\ a = g
( m1 − µm2 )
( m1 + m2 )
m1m2 g (1+ µ )
T =
m1 + m2
5 × 10 × 9.81(1 + 0.25)
T = = 40.88 N. Choice (D)
α 5 + 10
β
g ( m1 − µm2 )
2 22. a =
u m1 + m2
Rmax =
g (1 + sin β ) 9.81(5 − 0.25 × 10 )
a= = 1.635 m/s2. Choice (B)
50 2 5 + 10
190 =
9.81(1 + sin β ) 23. v2 = u2 + 2as = 0 + 2 × 1.635 × 1
sin b = 0.3413 v = 1.81 m/s. Choice (C)
b = 19.95° or 20°. Choice (A) 2 4. q = 18 t + 3t2 - 2t3
1 8. b = 19.95° = 0.3482 radian Angular velocity

1π  w= = 18 + 6t - 6t2
 − β = α − β dt
2 2
Angular acceleration
1π 
 − 0.3482 = α − 0.3482 dω d 2θ
2 2 α= = 2 = 6 − 12 t
dt dt
a = 0.9595 radian = 54.98 or 55°. Choice (D)
When t = 2
1 9. Time taken is given by a = 6 - 12 × 2 = - 18 rad/s2. Choice (D)
2u sin α 2 × 50 × sin 30 2 5. Angular velocity is maximum when
t= =
g 9.81 dω
=0
  = 5.097 sec or 5.1 sec . Choice (D) dt
2 0. Horizontal range i.e., 6 - 12 t = 0 or t = 0.5 sec
= Horizontal velocity × time of flight = (u cos a) × t Maximum angular velocity
= 50 cos 30 × 5.097 = 220. 7 m. Choice (A) = 18 + 6 × 0.5 - 6 × (0.5)2 = 19.5 rad/s. Choice (B)
Engineering Mechanics Test 2
Number of Questions: 30 Time: 75 min.

Directions for questions 1 to 30: Select the correct alterna- 7. A truck weighing 150 kN and traveling at 2 m/s impacts
tive from the given choices. with a buffer spring, which compresses 1.25 cm per
1. The velocity-time graph of a body is passing through 10 kN. The maximum compression of the spring is
the velocity axis with intercept of 4. If the slope of the
graph is 3, the distance travelled by the body in 6 sec-
onds would be
(A) 40 m (B) 60 m
(C) 78 m (D) 80 m
2. A circular disc of radius ‘R’ rolls without slipping at (A) 20 cm
a velocity ‘V’. The magnitude of the velocity at point (B) 22.85 cm
P(see figure) is (C) 27.65 cm
P
(D) 30 cm
R
30° 8. A particle moving in space with velocity J = 3t2i + 4tj –
V
7t3k. The acceleration of the particle at t = 1 will be
(A) 3i + 8j – 7k
(B) 6i + 4j + 21k
(C) 6i + 4j – 21k
3 (D) zero
(A) 3V (B) V
2 9. Match the following
V 2
(C) (D) V List – I List – II
2 3
a. Two parallel forces acting 1. Collision
3. A particle starts with velocity 2 m/s and accelerates on a body moving with
uniform velocity
at a rate of 3 m/s2 for 15 seconds and then retards at
6 m/s2 until it stops. The total distance covered is b. A moving particle 2. Forces in equilibrium
(A) 184.08 m (B) 551.58 m c. Two coplanar forces 3. Kinetic energy
(C) 367.5 m (D) None of these equal in magnitude but
opposite in direction
4. A stone is projected horizontally from a cliff at 10 m/s d. Co-efficient of 4. Couple
and lands on the ground below at 20 m from the base restitution
of the cliff. Find the height ‘h’ of the cliff. Use g = 10
m/s2. a b c d
(A) 18 m (B) 20 m (A) 4 3 2 1
(C) 22 m (D) 24 m (B) 1 2 3 4
(C) 2 3 4 1
5. Two cars are going with constant speeds, round con-
(D) None of these
centric circles of radii r1 and r2 and take the same time
to complete their circular paths. Their speeds will cor- 10. For the truss shown in the figure, the force (N) in the
respond to the ratio member BC is
r1
(A) 1 : 1 (B) W W
r2
B C
2
 r1  r2
(C)   (D) 30°
 r2  r1
60°
A 30° D
6. A point ‘P’ moves along a straight line as per the law
x = 4t2 + 12t + 1, the velocity of the point after 3rd and
4th seconds are respectively.
(A) 36 m/s and 48 m/s (A) 0 N(compressive)
(B) 36 m/s and 44 m/s (B) 0.577 W(tensile)
(C) 34 m/s and 44 m/s (C) 0.577 W(compressive)
(D) 34 m/s and 46 m/s (D) 0.866 W(compressive)
3.10 | Engineering Mechanics Test 2

11. of kinetic friction for blocks A and B are 0.1 and 0.35
respectively the frictional forces on A and B are
(A) 46.1 N, 369 N (B) 49.44 N, 398 N
(C) 52.14 N, 404 N (D) 56.48 N, 410 N
16.
B
150 N 3N
A 1 kg
300 N
P

30°
A body of mass 1 kg is resting on a plane surface as
Two blocks A and B weighing 300 N and 150 N respec- shown in figure. A force of 3 N is gradually applied on
tively are placed on a rough inclined plane of angle 30° one side as shown. Coefficient of static friction is 0.35
and connected through a string over a pulley as shown and coefficient of kinetic friction is 0.3. The friction
in the figure. Coefficient of friction of the contact sur- force acting is
faces are 0.25. Force P required on block A for impend- (A) 3.4335 N (B) 2.943 N
ing motion of the blocks is (C) 3 N (D) 0 N
(A) 22.43 N (B) 25.24 N 17.
(C) 28.62 N (D) 30.14 N P
12. A ball of mass 5 kg moving with a velocity of 6 m/s
makes impact with another ball of mass 3 kg moving
in the same direction with a velocity of 4 m/s. If coef-
200 kg
ficient of restitution is 0.5, velocities of the balls after 100 N
impact are
(A) 4.875 m/s, 5.875 m/s
(B) 4.962 m/s, 6.125 m/s
(C) 5.125 m/s, 6.536 m/s A body of mass 200 kg rests on a horizontal surface as
(D) 5.565 m/s, 6.926 m/s shown in the figure. Coefficient of friction between the
body and surface is 0.2. If a horizontal pull of 100 N
Common Data for Questions 13 to 15: can be exerted on the body, the vertical force P required
100 kg to move the body is
(A) 1462 N (B) 1418 N
B
(C) 1360 N (D) 1322 N
50 kg
Common Data for Questions 18 and 19:
A
300 mm

Two blocks A and B of mass 50 kg and 100 kg are placed on


an inclined plane, by connecting them by a string as shown
in them figure. Coefficient of friction between block A and
inclined plane is 0.15 and between block B and inclined
50 N
plane is 0.4
13. Inclination of the plane when the two blocks just start
to move is
1000 mm
(A) 10.15° (B) 17.57° 200
(C) 14.24° (D) 16.28° mm

14. Tension on the string is A band brake is used to control the speed of a flywheel as
(A) 77.9 N (B) 31.96 N shown in figure. The coefficient of friction between the band
(C) 74.3 N (D) 67.4 N and flywheel is 0.3. Radius of the flywheel is 300 mm. A
15. Angle of the inclined plane is increased to 20° and force of 50 N is applied at the end of the lever as shown in
the connecting string is removed. If the coefficients the figure
Engineering Mechanics Test 2 | 3.11

18. Torque applied on the flywheel when it is rotating 23.


clockwise is 5 kN 5 kN
(A) 262 Nm (B) 280 Nm
(C) 315 Nm (D) 326 Nm E F
19. Torque applied on the flywheel when it is rotating
counter clockwise is
(A) 94 Nm
(B) 82 Nm 45° 45° 45° B
A
(C) 76 Nm 5m C 5m D 5m
(D) 68 Nm
Common Data for Questions 20 and 21:

For the truss loaded as shown in figure, force in the


member CD is
(A) 5 kN (B) 2.5 kN
5
C (C) kN (D) 5 2 kN
150 N 2
B
24.
A 200 N P A B

20° 70°
Block A weighing 200 N is placed on plane floor and block
B weighing 150 N is placed over block A. Block B is con-
strained by a string C and a force P is applied on block A as
shown in figure. For the contact surfaces, coefficient of stat- C
ic friction is 0.3 and coefficient of kinetic friction is 0.25.
20. The smallest force P required to start block A moving is
(A) 143 N 200 N
(B) 150 N
(C) 156 N A weight of 200 N is hung using a cable as shown in
(D) 160 N the figure. Tensions in portions of cable AC and BC are
respectively
21. If a force P of 160 N is applied, the resultant friction
(A) 59.6, 171.7 N
forces exerted on block A is
(B) 62.4, 176.8 N
(A) 110 N (B) 120 N
(C) 62.5, 182.7 N
(C) 125 N (D) 150 N
(D) 68.4, 187.9 N
22.
6 kN 8 kN 25.
A

E D
30° C
P
45°

30 ° 60 ° B 30 ° 60 °
A C

5m 5m
6 kN B

A simply supported structure is loaded as shown in the Two steel truss members AC and BC with cross section
figure. Force in the member AB is area 100 mm2 is subjected to a horizontal force P kN as
(A) 10.26 kN shown in figure. Maximum value of P such that axial
(B) 13.42 kN stress in any of the members does not exceed 50 MPa is
(C) 15.75 kN (A) 10.15 kN (B) 9.22 kN
(D) 17.83 kN (C) 7.92 kN (D) 6.83 kN
3.12 | Engineering Mechanics Test 2

26. (A) 6.624 rad/s (B) 8.368 rad/s


(C) 10.472 rad/s (D) 14.376 rad/s

Common Data for Questions 28 and 29:


A pile of mass 500 kg is driven by a mass 350 kg falling
A
on it vertically through a distance of 1 m. After impact, the
B 90 N falling mass and pile remain in contact and move together.
The pile is moved 150 mm at each blow.
28. Energy lost in each blow is
(A) 1676 Nm (B) 1762 Nm
30° (C) 1915 Nm (D) 2020 Nm
Refering to the figure given above, coefficient of fric- 29. Average resistance against the pile is
tion for all surfaces of contact is 0.3. The minimum (A) 17.765 kN (B) 18.625 kN
weight of block A required to keep block B in position (C) 20.516 kN (D) 22.835 kN
is 30. A body of mass 5 kg falls from a height of 50 m and
(A) 35.6 N (B) 38.4 N penetrates into the ground by 90 cm. Average resistance
(C) 41.6 N (D) 44.5 N to penetration is
2 7. A wheel at rest is accelerated uniformly from rest to (A) 2668 N (B) 2774 N
3000 rpm in 30 seconds. Its angular acceleration is (C) 2814 N (D) 2892 N

Answer Keys
1. C 2. A 3. B 4. B 5. B 6. B 7. C 8. C 9. C 10. C
11. A 12. A 13. B 14. A 15. A 16. C 17. A 18. B 19. D 20. B
21. C 22. B 23. A 24. D 25. D 26. C 27. C 28. D 29. A 30. B

Hints and Explanations


1. the disk is rotating with respect to this center with an
V angular velocity.
V
w=
R
22
4 Velocity of P = w . OP
OP = R2 + R2 − 2 R2 cos120°
t
   = 3R 2 = 3R
a = 3.
v = u + at = 4 + 3 × 6 V
VP = . 3R = 3V  Choice (A)
  = 22 m/s R
Distance travelled = Area under the curve
1 1 2
= 6 × 4 + ×18 × 6 = 24 + 54 = 78 m Choice (C) 3. S = ut + at
2 2
2. u = 2 m/s
a = 3 m/s2
1
S1 = 2 (15) + × 3 (15) = 367.5 m
2
A P
2
30°
V v = u + at = 2 + 3(15) = 47m/s
v2 – u2 = 2as
0 – 472 = 2(–6)s2
O
472
s2 = = 184.08 m/s
12
the instantaneous center of rotation of the disk is O, i.e, Total distance covered is = S1 + S2 = 367.5 + 184.08
its point of contact with ground every other point on = 551.58 m/s Choice (B)
Engineering Mechanics Test 2 | 3.13

4. At t = 1
dJ
Vo = 10 m/s Acceleration, = 6i + 4j – 21k. Choice (C)
dt
10. Considering pin A
h
RAB (assume)

20m 60°
A RAD (assume)
20
time of flight = = 2 sec
10 RA = W
Since horizontal component of velocity remains con-
SFy= 0
stant therefore
\ RAB sin 60° + RA = 0
1 2 1 \ RAB = – 1.1547W
h = gt = × 10 × 2 = 20 m
2
Choice (B)
2 2 Considering tress member AB
2πr1 RAB
5. We know t1 = B
v1
2πr2
And t2 = Compressive
v2
A
But t1 = t2
RAB
v1 v2
Therefore =
r1 r2 RAB = + 1.1547W
Considering pin B
Since v = ωr, therefore
W
ω1r1 ω 2 r2
=
r1 r2 B
RBC
w1 = w2
30°
RBD
v1 v2 RAB
=
r1 r2
Sfx = 0
v1 r1 \ RAB sin 30° + RBC = 0
\ =  Choice (B) \ RBc = – 1.1547W sin30°
v2 r2
RBc = – 0.577W
6. X = 4t2 + 12t + 1 Considering tress member BC
B C
dX RBC
Velocity, v = = 8t + 12 RBC
Compressive
dt
(v) t = 3sec = 8(3) + 12 = 36 m/s \ RBc = 0.577W(compressive) Choice (C)
(v) t = 4sec = 8(4) + 12 = 44 m/s Choice (B)
11. Consider free body diagram of block B. Let T be the
tension on the string
1 1
7. mV 2 = KX 2 T
2 2
w 150 × 103 B 0.25 NB
m= = = 15290 kg
g 9.81
10000
15290 (2)2 = x2 30°
1.25 × 10 −2
⇒ x = 27.65 cm. Choice (C) NB
8. Velocity, J = 3t2i + 4tj – 7t3k 150 N

dJ NB = 150 cos 30 = 129.9 N


Acceleration, = 6ti + 4j – 21t2k
dt T = 150 sin 30 + 0.25 NB = 75 + 0.25 × 129.9 = 107.48 N
3.14 | Engineering Mechanics Test 2

Consider free body diagram of block A T + 981 sin q = 392.4 cos q  -----------(2)
T From (1) and (2)
(490. 5 sin q - 73.575 cos q) + 981 sin q = 392.4 cos q
A 1471.5 sin q - 465.975 cos q = 0
465.975
tan q = = 0.3167
1471.5
P 30° q = 17.57°. Choice (B)
NA
0.25 NA
14. T = 490.5 sin 17.57 - 73.575 cos 17.57 = 77.925 N.
 Choice (A)
300 N
15. Consider block A for static condition
NA = 300 cos 30 = 259.8 N
T = 490.5 sin q - 73.575 cos q
P + 300 sin 30 = T + 0.25 NA
As there is no connecting string
P = 107.48 + 0.25 × 259.8 - 150 = 22.43 N.
490.5 sin q - 73.525 cos q = 0
 Choice (A)
tan q = 0.15
1 2. m1 = 5 kg, u1 = 6 m/s q = 8.525°
m2 = 3 kg, u2= 4 m/s, e = 0.5 As q is given as 20° the block moves down
Applying momentum equation Frictional force = mk × N1 = mk × W1 cos q
m1u1 + m2u2 = m1v1 + m2v2 = 0.1 × 50 × 9.81 cos 20 = 46.1 N
i.e., 5 × 6 + 3 × 4 = 5 × v1 + 3 × v2 For the static condition of block B
5v1 + 3v2 = 42  -----------------(1) 981 sin q = 392.4 cos q
From Newton’s law of collision of elastic bodies, tan q = 0.4
v −v q = 21.8°
e= 2 1
u1 − u2 \ The block is stationary at an angle 20° and static
v2 - v1 = e(u1 - u2) = 0.5 (6 - 4) friction is active
v2 - v1 = 1  ---------------------(2) Frictional force = ms × N2 = ms × W2 cos q
From (1) and (2) = 0.4 × 100 × 9.81 cos 20
v1 = 4.875 m/s = 368.735 N. Choice (A)
v2 = 5.875 m/s. Choice (A) 16. Weight of the body W = mg = 1 × 9.81 = 9.81 N
1 3. Let T be the tension in the spring Static friction force = W × ms = 9.81 × 0.35 = 3.4335 N
Considering block A alone Kinetic friction force = W × mk = 9.81 × 0.3 = 2.943 N
W1 The applied force is 3 N. The body will start moving
θ T only when applied force exceeds the static friction
force. After this the kinetic friction force will come
A into action.
So the friction force acting is 3 N. Choice (C)
µ N1 N1
17. Weight of the body = mg = 200 × 9.81 = 1962 N
θ
Frictional force = (mg - P)m = (1962 - P)0.2
N1 = W1 cos q For moving the body
W1 sin q = T + m N1 = T + 0.15 W1 cos q 100 ≥ (1962 - P) × 0.2
T = 50 × 9.81 sin q - 0.15 × 50 × 9.81 cos q 100
1962 - P =
  = 490.5 sin q - 73.575 cos q  --------(1) 0.2
Considering block B P = 1962 - 500 = 1462 N. Choice (A)
W2
18.
θ

B θ

T
µ2N2 N2 T2

N2 = W2 cos q T1
T + W2 sin q = m2 N2 = 0.4 × 100 × 9.81 cos q
Engineering Mechanics Test 2 | 3.15

For clock wise rotation of the flywheel, band connected 21. Static friction forces to overcome for the movement of
to the lever is slack side and corresponding tension is T2 A = 150 N
T1 For force P above 150 N
= e µθ Kinetic friction is active
T2
Considering block A
3π N1 = 150 N
where q = wrapping angle = 270° = radian
2 µkN1

T1 0.3 ×
= e 2 = 4.11
T2 200 N

T2 × 200 = 50 × 1200 F 160 N


T2 = 300 N A
\ T1 = T2 × 4.11 = 300 × 4.11 = 1233 N
Torque on the fly wheel = (T1 - T2) r µkN2
=(1233 - 300) 0.3 = 280 Nm. Choice (B)
N2
1 9. For anti clockwise rotation tight side of the band is con-
nected to the lever i.e., T1 acts on the lever Resultant friction forces = mkN1 + mkN2
\ T1 × 200 = 50 × 1200 = 0.25 × 150 + 0.25 (150 + 200)
T1 = 300 N = 37.5 + 87.5 = 125 N. Choice (C)
T1 22.
= 4.11 6 kN 8 kN
T2
T1 300
T2 = = = 73 N E D
4.11 4.11
Torque = (T1 - T2)r
= (300 - 73)0.3 = 68 Nm. Choice (D)
2 0. Consider block B, A
30 ° 60 ° B 30 ° 60 °
C
Let T be the tension in the string B
5m 5m
W1 = 150 N

6 kN

T VA + VC= 6 + 8 + 6 = 20 kN
Taking moment about A
B 5×3  5 × 3
VC × 10 = 6 × + 6 × 5 + 8 5 + 
4  4 
µsN1
VC = 12.25 kN
N1 \ VA = 20 - 12.25 = 7.75 kN
N1= W1 = 150 N considering joint A TAE sin 30 = VA
T = ms × N1 = 0.3 × 150 = 45 N 7.75
Consider Block A TAE = = 15.5 kN
0.5
N1
µsN1 TAB = TAE cos 30 = 15.5 × cos 30 = 13.42 kN.
 Choice (B)
200 N 23.
5 kN 5 kN
P E F
A
5
µsN2 5

N2 45° 45° 45°


A 5 C 5 D 5 B
N2 = N1 + 200 = 150 + 200 = 350 N VB
P = msN1 + msN2 = 45 + 0.3 × 350 = 150 N. Choice (B) VA
3.16 | Engineering Mechanics Test 2

By symmetry 26.
WA
VA = VB = 5 kN
Considering joint B, θ
T
TFB sin 45 = VB = 5
TDB = TFB cos 45 = 5 kN as sin 45 = cos 45
considering joint D TCD = TDB = 5 kN. Choice (A) A µ N1

24.
A B θ = 30° N1
20° 70°
Consider the free body diagram of block A
N1 = WA cos 30  -------------(1)
90°
m N1 + WA sin 30 = T
C
N1 WB
110° 160°
θ

200 N
µ N1 B
Applying Lami’s theorem
TAC T 200
= BC = = 200
sin160 sin110 sin 90 µ N2 N2
TAC = 200 × sin 160 = 68.4 N
TBC = 200 × sin 110 = 187.94 N Choice (D) Consider free body diagram of B
N1 + WB cos q = N2
25. i.e., N1 - N2 = -90 × cos 30
= -77.942  ---------------(2)
A T1
m(N1 + N2) = WB sin 30
30° P
0.3(N1 + N2) = 90 × 0.5
45° C
N1+ N2 = 150  ------------(3)
From (2) and (3)
T2 2 N1 = 72.06
N1 = 36 N
Substituting in (1)
B 36 = WA cos 30
\ WA = 41.6 N. Choice (C)
2 7. w0 = 0
Considering the joint P,
2πN 2π × 3000
T1 cos 30 + T2 cos 45 = P w= = = 100 p rad/s
60 60
T1 sin 30 = T2 sin 45
t = 30 s
T1 sin 45
= = 1.414 w = w0 + a t
T2 sin 30 100 p = 0 + a × 30
T1 = 1.414 T2 a = 10.472 rad/s. Choice (C)
1.414 T2 cos 30 + T2 cos 45 = P 28.
T2 = 0.5177 P m1 350 kg
T1 = 0.732 P
Maximum force is in the member AC 1m
TAC = T1 = 0.732 P kN
0.732 × 103 P
Axial stress = = 7.32 P N/mm2 m2
100 500 kg
= 7.32 P MPa
\ 7.32 P ≤ 50 MPa
P ≤ 6.83 kN. Choice (D) R
Engineering Mechanics Test 2 | 3.17

m1 = 350 kg, m2 = 500 kg 29. Let R be the average resistance against the pile.
S = 1 m, x = 150 mm = 0.15 m Net work done = kinetic energy after impact
Let V1 be the velocity of mass m1 when hitting m2 1
(R - m1g - m2g)x = (m1 + m2)V2
V1 = 2 gS = 2 × 9.81 × 1 = 4.43 m/s 2
V2 = Velocity of pile before impact (R - 350 × 9.81 - 500 × 9.81) 0.15 = 1413.96
V = Common velocity after impact R = 17765 N = 17.765 kN. Choice (A)
m1V1 + m2V2 = (m1 + m2)V 30. m = 5 kg, h = 50 m
350 × 4.43 + 0 = (350 + 500)V
x = 90 cm = 0.9 m
\ V = 1.824 m/s
1 v1 = 2gh = 4.43 h = 4.43 50 = 31.32 m/s
Kinetic energy before impact = m1V1
2
1 v2 = 0
= × 350 × (4.43)2 = 3434.36 Nm Let R be the average resistance of penetration
2
1 1
Kinetic energy after impact = (m1 + m2) V2 m(v22 - v12) = (mg - R) x
2 2
1 1
= (350 + 500) × (1.824)2 = 1413.96 Nm × 5(0 - 31.322) = (5 × 9.81 - R) 0.9
2 2
Energy lost in blow = 3434.36 - 1413.96 ⇒ 49.05 – R = 2724.84
= 2020.4 N/m. Choice (D) R = 2773.89 N Choice (B)
This page is intentionally left blank
Unit II
Solid Mechanics
This page is intentionally left blank
Solid Mechanics Test 1
Number of Questions: 25 Time: 60 min.

Directions for questions 1 to 25: Select the correct alterna- 4.


tive from the given choices. w/unit length

1. A
B

5 N/m
3m
5 N/m
6m

A cantilever beam of 6 m span is subjected to a uni- The free end of the cantilever AB is supported by a
formly varying load as shown. The bending moment at prop. The cantilever is loaded by a uniformly distrib-
the middle of the beam is uted load as shown in the figure. Assuming that there is
no deflection at the free end, force on the prop is
(A) 27.5 N–m (B) 15.0 N–m
4 3
(C) 22.0 N–m (D) 18.7 N–m (A) wL (B) wL
5 8
2. 3 3
(C) wL (D) wL
260 MPa 4 5
5. A solid conical bar of uniformly varying cross section
60 MPa is hung vertically as shown
20 cm φ
100 MPa 100 MPa

60 MPa 260 MPa


1m

Figure shows state of stress at a point in a stressed


body. Radius of Mohr’s circle representing the state of
stress is
(A) 60 (B) 80
(C) 120 (D) 100 If specific weight is 80000 N/m3 and modulus of elas-
ticity is E = 2 × 105 N/mm2, extension of its length due
3. to self weight is
500 mm 800 mm
(A) 6.67 × 10–5 m (B) 1.33 × 10–4 m
(C) 1 × 10 m –4
(D) 4.45 × 10–5m
6. The bulk modulus is K, modulus of elasticity E, and
20 kN 20 kN 1
B C passion ratio is then which of the following is true?
A m
200 mm2 160 mm2  2  1
(A) E = 3K 1 +  (B)
E = 3K 1 − 
 m  m
A bar ABC with cross sectional area 200 mm2 at por-
 2  1
tion AB and 150 mm2 at portion BC is subjected to an (C) E = 3K 1 −  (D)
E = 3K 1 + 
axial pull of 20 kN. If E = 2 × 105 N/mm2, strain energy  m  m
stored in the bar is 7. A solid circular shaft is subjected to bending & twist.
(A) 15.5 Nm (B) 18.6 Nm The ratio of maximum shear to maximum bending
(C) 8.7 Nm (D) 10.5 Nm stress at any point would be (M = T)
3.22 | Solid Mechanics Test 1

(A) 1 : 1 (B) 1 : 2 (A) 37.5, 12.64 (B) 41.2, -15.74


(C) 2 : 1 (D) 2 : 3 (C) 52.8, -17.92 (D) 49.3, -16.78
8. The shear stress distribution diagram of a beam of rec- 14. The direct stress in N/mm that produces same strain on
2

tangular cross section, subjected, to transverse loading that produced by the principal stresses is
will be (Poisson’s ratio is 0.3)
(A) 36.78 (B) 52.76
d d (C) 45.92 (D) 39.62
Statement for Linked Answer Questions 15 and 16:
(A) (B)
1 kN 1 kN 1 kN
A B
d 1m 1m 1m 1m
d ♦

(C) (D)
Where ‘d’ is the depth of the beam A simply supported beam AB is loaded as shown in the fig-
9. Proof resilience is the maximum energy stored at ure. The beam has a rectangular cross section of 100 mm
(A) limit of proportionality width and 240 mm depth.
(B) elastic limit 15. At a section 1.5 m from A maximum shearing stress is
(C) plastic limit (A) 0.0625 N/mm2
(D) None of these (B) 0.0848 N/mm2
10. Which of the following will give the value of deflection (C) 0.0313 N/mm2
at any point? (D) 0.0565 N/mm2
dy 16. Principal stresses at a point in neutral axis of the above
(A) EI = M (B) EIY=M section in N/mm2 is
dx
(A) +0.0313, 0
(C) E I Y = ∫M (D)
E I Y = ∫ ∫M (B) +0.0313, -0.0313
11. (C) +0.0625, 0.0313
10 kN/m (D) +0.0625, -0.0625
17. A 2 m long wooden column bottom fixed and top end
2m free, has a square cross section and has to take a load
of 100 kN. Modulus of elasticity is 12 GPa. Size of
5m the column, using Euler’s formula and a factor of safety
3, is
The displacement of the free end of the cantilever beam
(A) 148.5 mm (B) 135.8 mm
shown in figure is
(C) 162.3 mm (D) 156.7 mm
[Take E = 2 × 105 N/mm2,
 I = 180 × 106 mm4] Common Data for Questions 18 and 19:
(A) 16.39 mm (B) 14.93 mm
(C) 12.72 mm (D) 10.68 mm Brass
A B
1 2. During an experiment on a steel column using Rankine’s Steel T
C
formula, the following results were available 80 mm 50 mm
Slenderness ratio 65 160
Average stress at failure 200 N/mm2 70 N/mm2 200 mm 1m

Rankine’s constant for the material of the column is For the stepped shaft ABC, fixed at A, portion AB is made
(A) 1.865 × 10-4 (B) 2.194 × 10-4 of brass and portion BC is made of steel, Allowable shear
(C) 1.623 × 10
-4
(D) 1.373 × 10-4 stress for brass is 80 N/mm2 and for steel is 100 N/mm2.
Common Data for Questions 13 and 14: Modulus of rigidity for brass is 40 kN/mm2 and for steel is
At a cross section in a shaft of diameter 100 mm it is sub- 80 kN/mm2
jected to a bending moment of 2.5 kNm, and a twisting mo- 18. Maximum value of torque that can be applied at the end
ment of 5 kNm. of the shaft is
13. Maximum principal stresses induced in the section in (A) 8042 Nm (B) 6053 Nm
N/mm2 are (C) 2454 Nm (D) 3064 Nm
Solid Mechanics Test 1 | 3.23

19. Total rotation at the free end in degrees is (A) 1.8 mm (B) 2.2 mm
(A) 4.18° (B) 3.04° (C) 1.5 mm (D) 2.8 mm
(C) 3.62° (D) 2.51° 23. A cylindrical tank of 750 mm internal diameter and 4 m
Common Data for Questions 20 and 21: length is made of 18 mm thick sheet. If it is subjected to
an internal fluid pressure of 2 N/mm2, maximum inten-
10 kN/m
C B sity of shear stress induced is
A
(Take E = 2 × 105 N/mm2 and m = 0.3)
2m 2m (A) 11.78 N/mm2 (B) 9.57 N/mm2
(C) 8.62 N/mm 2
(D) 10.42 N/mm2
2 4. Change in volume of the tank in cm3 is
The simply supported beam loaded as shown above has a (A) 699 (B) 1119
flexural rigidity 833.33 kN - m2 (C) 1520 (D) 386
20. Slope at end A is 25.
(A) 12 rad (B) 15 rad Y
(C) 10 rad (D) 17 rad A B

21. Maximum deflection occurs between A and C. Distance P (40, 25)


from A is
250 mm X
(A) 2.00 m (B) 1.00 m
(C) 1.84 m (D) 1.95 m
22. D C
B 400 mm

10 kNm On a short masonry column of cross section as shown


above, a concentrated load of 500 kN is applied at point
2m P, + 40 mm from y axis and + 25 mm from x - axis.
Moment of inertia about x- axis = 520.833 × 106 and
A cantilever AB of length 2 m and 100 mm breadth moment of inertia about y - axis = 1333 × 106
and 200 mm depth is fixed at end A. It is subjected to a Stress developed at point D is
moment of 10 kNm at the free end B. Flexural rigidity (A) 2.10 N/mm2 (B) 1.15 N/mm2
is 13,340 kN-m2. Magnitude of maximum deflection is (C) 1.00 N/mm 2
(D) 2.67 N/mm2

Answer Keys
1. B 2. D 3. D 4. B 5. A 6. C 7. B 8. D 9. B 10. D
11. A 12. D 13. B 14. C 15. C 16. B 17. A 18. C 19. B 20. B
21. C 22. C 23. D 24. A 25. C

Hints and Explanations


1. The load acting can be split in to two as shown in figure Bending moment due to uniformly varying load at
(i) A uniformly varying load of 10 N/m at fixed end x = 3 m.
and zero at free end. Bending moment due to the uniformly distributed load
(ii) A uniformly distributed negative load of 5 N/m wx 2 5 × (3)2
acting through out the beam. =+ = = 22.5 Nm
2 2
Total bending moment = 22.5 – 7.5 = 15 Nm
 Choice (B)
10 N/m
2
 px − p y 
 2  + q
2
2. Radius of Mohr’s circle is =
wL
x
2 2
 260 − 100 
 + 60
2
5 N/m = 
2
3.24 | Solid Mechanics Test 1

= 80 2 + 60 2 10 × 54 10 × 34 10 × 33 × 2 
= − +
8 EI  8 EI 6 EI 
= 6400 + 3600
1 590
= 781.25 − (101.25 + 90 ) =
= 1000 = 100.  Choice (D) EI  EI
20000
3. Stress in portion AB s1 =  = 100 N/mm2 E = 2 × 105 N/mm2 = 2 × 108 kN/m2
200 I = 180 × 106 mm4 = 180 × 10–6 m4
20000
Stress in portion BC s2 = = 200 N/mm2 590
100 \ Deflection =
2 × 108 × 180 × 10 −6
 σ2 
Strain energy stored = ∑  × volume = 0.01639 m = 16.39 mm. Choice (A)
 2E 
12. Rankine’s formula is
(100)2 200 2
= × (200 × 500 ) + × (100 × 800 ) σc A Pcr σc
2 × 2 × 105 2 × 2 × 105 Pcr = Or =
( ) ( )
2 2
1+ a  k A 1+ a 
= 10,500 N - mm = 10.5 Nm. Choice (D) k
4. Deflection of beam at the free end if no prop is there
wL4 When  k = slenderness ratio = 65
=
8 EI
Pcr
FL3  = 200 N/mm2
Upward deflection of the end due to the force F = A
3 EI
When there is no resultant deflection σc
\ 200 =  -----------(1)
1 + a (65)
2
FL3 wL4
=
3 EI 8 EI
When  k = 160
3
\ F = wL . Choice (B)
8 Pcr
= 70
5. Extension of vertical conical bar due to self weight A
wl 2 80, 000 × 12 σc
= = m \ 70 =  ---------(2)
6E 6 × 2 × 105 × 106 1 + a (160 )
2

= 6.67 × 10–5 m Choice (A)


(1) ÷ (2) gives,
16T
7. τ = 3 200 1 + a(160)2
πd =
70 1 + a(65)2
32 M
s=
πd 3 200 + 845,000 a = 70 + 1792,000 a
947000 a = 130
τ 16T πd 3
= × = 1 : 2 Choice (B) a = 1.373 × 10–4. Choice (D)
σ πd 3 32 M
13. M = 2.5 kNm = 2.5 × 106 Nmm
11.
T = 5 kNm = 5 × 106 Nmm
Maximum principal stress
a b
16 
p1 = M + M2 +T2 
πd 3  
16 
   = 2.5 + 2.52 + 52  × 106 = 41.2 N/mm2
π1003  
The given uniformly distributed loading of the canti-
lever can be treated as a combination as shown above. 16 
p2 = M − M 2 + T 2  × 106
i.e., a full loading + a negative loading through length a πd 3  
\ Deflection at free end
16 
w 4  wa4 wa3 .b       = 2.5 − 2.52 + 52  × 106
= − + π1003  
8 EI  8 EI 6 EI      = - 15.74 N/mm2. Choice (B)
Solid Mechanics Test 1 | 3.25

14. Let p be the direct stress 18.


p p1 µp2 T
= − T
E E E
P = 41.2 - 0.3 (-15.74) = 45.92 N/mm2. Choice (C)
15. T T

1 kN 1 kN 1 kN Both portions are subjected to same torque T


x For brass portion maximum torque that can be applied
π 3 π
1m 1m 1m 1m
Tb = d τb = (80)3 × 80
16 16
1.5 m x
RA= 1.5 kN RB= 1.5 kN   = 8042,477 N-mm = 8042.477 Nm
π
Shear force at the section xx is 1.5 - 1.0 = 0.5 kN For steel portion, Ts = (50)3 × 100
16
0.5 × 1000 500
= 2454,369 N-mm
Average shearing stress = =
bd 100 × 240 = 2454.369 Nm
So maximum torque that can be applied at the end of
Compression
the shaft is 2454 Nm = 2454 kN mm Choice (C)

N A
TLb TLs
19. Rotation of the free end = qB + qs = +
Gb J b Gs J s
32  200 1000 
= 2454 × +
π  40 × 80 4 80 × 50 4 
Tension Shearing
Bending Stress
Stress
= 0.053 rad = 3.04°. Choice (B)
Shearing stress is maximum at neutral axis 20.
Maximum shearing stress 10 kN/m
= 1.5 × average shearing stress A C B
2m 2m
500
= 1.5 × = 0.03125 N/mm2. Choice (C)
24000
VA + VB = 2 × 10 = 20 kN
16. At Neutral axis bending stress (Normal stress) is zero VA × 4 = 2 × 10 × 3
\ px = 0, py = 0 and q = 0.03125 N/m2 VA = 15 kN
Principal stress VB = 5 kN
2 Using Macaulay’s method
px + p y  px − p y 
0 + (0.03125) 10 ( x − 2)
2 2
= +  + q2 = 0 ± d2 y
2  2  Mx = 5 x - = ΕΙ 2
2 dx
= ± 0.03125 N/mm2. Choice (B) 2
5 ( x − 2)
3
dy 5x
ΕΙ = C1 + −
17. Critical load = working load × factor of safety dx 2 3
= 100 × 3 = 300 kN
5 x 3 5 ( x − 2)
4

Applying Euler’s formula EIy = C2 + C1x + −


2 3 3 4
π 2 EI
Critical load = At x = 0, y = 0
L2 0 = C2 + 0 + 0
When L = Effective length = 2 × actual length At x = 4, y = 0
= 2 × 2 = 4 m = 4000 mm 5 43 5 24
0 = 0 + 4C1 + × − ×
π × 12 × 10 × I
2 3
2 3 3 4
\ 300 × 103 =
(4000)2 - 4C1 = 53.33 - 6.67 = 46.66
C1 = - 11.67
a4
I = 40,528,473 = 5 x 2 5 ( x − 2)
3
dy
12 ΕΙ = −11.67 + −
dx 2 3
\ a = 148.5 mm. Choice (A)
3.26 | Solid Mechanics Test 1

At A, x = 4 ML2 10 × 103 × 22
ymax = ymax = =
 dy  5× 4 5×2 2 3
2ΕΙ 2 × 13340
\ EI   = −11.67 + −
 dx  A 2 3 = 1.5 mm Choice (C)
40 2 3. d = 750 mm
= -11.67 + 40 - = 14.996 rad. t = 18 mm
3
d 750
 Choice (B) = = 41.67 > 15
t 18
21. Maximum deflection occurs where slope is zero
\ Thin cylinder formula can be applied
5x2 5
− ( x − 2) = 0 2 × 750
3
\ -11.67 + pd
2 3 Hoop stress s1 = = = 41.67 N/mm2
2t 2 × 18
5
2.5x2 - ( x − 2) = 11.67
3
pd 41.67
3 Longitudinal stress s2 = = = 20.83 N/mm2
4t 2
\ x = 2.16 m (distance from C)
σ1 − σ 2 20.83
\ Distance from A = 4 - 2.16 = 1.84 m Choice (C) qmax = = = 10.42 N/mm2. Choice (D)
2 2
22.
x δd σ µσ
24. Diametral strain = e1 = 1 − 2
d E E
41.67 20.83
= − 0.3 × = 17.71 × 10–5
At any section bending moment is 10 kNm 2 × 10 5
2 × 105
i.e., Mx = 10 kNm δL σ σ
Longitudinal strain = e2 = 2 − µ 1
d2 y L E E
ΕΙ 2 = 10 kNm
dx 20.83 − 0.3 × 41.67
dy = = 4.165 × 10–5
ΕΙ = 10 x + C1 2 × 105
dx
δV
dy Volumetric strain = 2e1 + e2
At x = 0, =0 V
dx
= (2 × 17.71 + 4.165) × 10–5 = 39.58 × 10–5
\ C1 = 0
d V = 39.58 × 10–5 × V
10 x 2
ΕΙy = + C2 39.58 π × (0.75)
2
2 = × × 4 m3
At x = 0, y = 0 105 4
= 69.94 × 10–5 m3 = 699.43 cm3. Choice (A)
10 x 2
\ ΕΙy =
2 p Mx My
25. s = + y+ x
10 × 22 A Ιx Ιy
 EI ymax =
2 500 × 103 500 × 103 × 25 500 × 103 × 40
Flexural rigidity EI = 13, 340 kN - m2  = + y + x
250 × 400 520.833 × 106 1333 × 10
20
\ ymax = m
 1 25 × 125 40 × 200 
13340 fD = 500 × 103  − −
 250 × 400 520.833 × 10 1333 × 10 
6 6
= 1.5 × 10–3 m
= 1.5 mm or   = -1 N/mm2. Choice (C)
Solid Mechanics Test 2
Number of Questions: 30 Time: 75 min.

Directions for questions 1 to 30: Select the correct alterna- 8. A bar of 3m in length 30 mm breadth and 20 mm thick-
tive from the given choices. ness is subjected to a compressive stress of 50 kN/m2.
1. If Poisson’s ratio of a material is 0.5, modulus of elas- What will be the final volume of the bar if the poisson’s
ticity of the material is ratio is 0.30 and modulus of rigidity is 90 GN/m2.
1 (A) will increase by 0.4615 mm3
(A) times the shear modulus (B) will decrease by 0.5625 mm3
3
(C) will decrease by 0.4615 mm3
(B) 3 times the shear modulus
(D) will increase by 0.5625 mm3
(C) 4 times the shear modulus
(D) equal to the shear modulus 9. When ‘C’ is the modulus of rigidity and ‘q’ is the inten-
sity of shear stress, the strain energy due to shear is
2. A circular shaft subjected to torsion undergoes a twist given by
of 1° in a length of 1.6 m. If the maximum shear stress
q
induced is 10,000 N/cm2 and if modulus of rigidity is (A) × volume of block
8 × 106 N/cm2 then radius of the shaft is 2C
27 36 q2
(A) cm (B) cm (B) × volume of block
π π 2C
π π q2
(C) cm (D) cm (C) × volume of block
36 27 2C
q
3. A 2 m long mild steel bar of 2000 mm2 cross sectional (D) × Volume of block
2
area is subjected to an axial load of 40 kN. If Young’s
10. According to maximum shear stress failure criterion,
modulus for the shaft is 2 × 105 N/mm2, extension of
yielding in material occurs when maximum shear
the shaft in mm is
stress is
(A) 0.5 mm (B) 1 mm
(C) 0.2 mm (D) 2 mm 1
(A) yield stress (B) 2 yield stress
2
4. A steel bar of 1 m length is heated from 30°C to 60°C.
Coefficient of linear expansion is 12 × 10-6/°C and 2
(C) yield stress (D) 2 yield stress
Young’s modulus is 2 × 105 MN/m2. Stress developed 3
in the bar is 1 1. Stress-strain behaviour of a material is shown in the
(A) 18 N/mm2 (B) zero figure. Proof Resilience in Nm/m3 is
(C) 36 N/mm2 (D) 72 N/mm2
5. Slope of a beam under load is 140
(A) rate of change of deflection
120
(B) rate of change of bending moment
(C) rate of change of bending moment x flexural 90
Stress
rigidity (MPa) 60
(D) rate of change deflection x flexural rigidity
30
6. Relationship between modulus of elasticity E, modulus
of rigidity G and bulk modulus K is
6 KG 9 KG 0.004 0.008 0.012
(A) E = (B) E=
3K + G 3K + G Strain (mm/mm)

3K + G 3K + G
(C) E = E=
(D) (A) 10 × 104 (B) 15 × 104
6 KG 9 KG (C) 76 × 104 (D) 130 × 104
7. A column with 80mm diameter is fixed at both the 12.
ends. If the crippling load calculated by Rankine for- P
mula is 750 kN then what will be the crushing load of A B C
the column. The length of column is 8m. E = 180 GPa
(A) 949.4 kN (B) 994.4 kN
(C) 317.77 kN (D) 984.6 kN L L L
3.28 | Solid Mechanics Test 2

A beam is made of 2 bars AB and BC hinged at B, fixed w/m 3


at A and simply supported at C. If it is loaded at mid c. 3. w 2
point of BC as shown in figure, bending moment at A is l 8
PL w/m
(A) PL (B) w 2
2 d. 4.
2 2l 4
(C) 2PL (D) PL
3
Codes:
13. A cantilever beam is loaded as shown in the figure a b c d a b c d
P (A) 3 2 4 1 (B) 3 4 2 1
(C) 1 2 3 4 (D) 4 3 2 1
x
B
A C Directions for questions 17 and 18: A circular bar made
L of C.I is to resist on occasional torque of 2.2 kNm acting
in transverse plane. The allowable stresses in compression,
If L is the length of the beam and EI, the flexural rigid- tension and shear are 100, 50, 35 MN/m2 respectively.
ity, slope at point C at a distance x from fixed end is Take G = 40 GN/m2.
Px Px 17. The diameter of the bar will be
(A) (2 L − x ) (B) - (2 L − x )
EI 2EI (A) 64.8 mm (B) 68.4 mm
Px Px (C) 66.8 mm (D) 67.4 mm
(C) ( L − x ) (D)
− ( L − x) 18. The angle of twist under the applied torque per meter
2EI EI
length of bar will be
14. The extension of a circular bar tapering uniformly from (A) 1.86° (B) 1.26°
diameter d1 to d2 is same as that of a uniform circular (C) 1.46° (D) 1.16°
bar of same length, under same load. Diameter of the 19. If diameter of long column is reduced by 20% then per-
uniform bar is centage of reduction in Euler buckling is
(A) d1d2 (B) d1 − d2 2 2
(A) 4 (B) 36
d + d2 d1 − d2 (C) 49 (D) 59
(C) 1 (D) 20. A shaft subjected to a maximum bending stress of
2 2
80 N/mm2 and maximum shearing stress equal to
15. A brass bar having a cross sectional area of 1000 mm2 30 N/mm2 at a particular section. If the yield point in
is subjected to axial forces as shown in the figure. The the tension of the material is 280 N/mm2, and maxi-
total change in length of the bar is. mum shear stress theory of failure is used, then the fac-
Take E = 1.05 × 105 N/mm2. tor of safety obtained will be
A B C D (A) 2.5 (B) 2.8
(C) 3.0 (D) 3.5
50 kN 8 kN 10 kN 21. A Cantilever beam AB is connected to another beam
20 kN BC with a pin joint at B as shown in the figure. For the
loading as shown in the figure, the magnitude of bend-
0.6 m 1m 1.2 m ing moment at A (in kN-m) is
10 kN
(A) – 0.15 mm (B) + 0.15 mm A B C
(C) – 0.1143 mm (D) + 0.1143 mm
16. Match the following List – I (Loaded beam) and 1m
List – II (Maximum bending moment).
4m 2m

List – I List – II
(A) 5 (B) 10
a. l /2 l /2 1. w 2 (C) 20 (D) 50
2
2 2. A solid metal tube with modulus of elasticity E and
w/m Poisson’s ratio m is constrained on all faces. It is heated
w 2
b. 2. so that temperature rises uniformly. If coefficient of
6
l
thermal expansion is a, the compressive stress devel-
oped in the cube due to the heating is
Solid Mechanics Test 2 | 3.29

Eα∆T Eα∆T 2P  a b 
(A) (B) (C)  + 
2 (1 − 2µ ) (1 − 2µ ) πDE  D d 
2 Eα∆T Eα∆T 2P  a b 
(C) (D) (D)  + 
(1 − 2µ ) 3 (1 − 2µ ) πdE  D d 
23. A bar of length L, breadth b and thickness t is subjected 28.
to an axial pull of P. If ex is the strain in the direction of py
pull, volumetric strain produced is (m = Poisson’s ratio) 3 MPa
(A) ex (1 + 2m) A B
(B) ex (1 - 2m) 3 MPa
(C) ex (1 + m) pt
(D) ex (1 - m)
24. A simply supported beam of length L has a cross sec-
6 MPa
d
tion of depth d and width . If it is loaded with a uni- 2 MPa
2 45°
formly distributed load of w/unit length, maximum
deflection is (Young’s modulus = E) C
5 wL4 5 wL4
(A) (B) At a point in a stressed body stresses acting are as
8 Ed 4 16 Ed 4
3
shown in the figure. Value of Py is
5 wL 5 wL3
(C) 4
(D) (A) -8 MPa (B) 8 MPa
8 Ed 16 ed 4
(C) -4 MPa (D) 4 MPa
2 5. When a material is subjected to uniaxial tension,
to avoid failure due to shear in 45° planes, the shear 29.
P
strength of the material should be atleast
(A) half the tensile strength d
1 L
(B) times tensile strength
2
(C) tensile strength
3
(D) 4 times tensile strength θ
θ
26. At a point in a strained material, direct stresses
120 N/mm2 (tensile) and 100 N/mm2 (compressive) are
acting. If major principal stress is 150 N/mm2, maxi-
mum shearing stress at the point is A cantilever beam of varying width and constant depth
(A) 87 N/mm2 is loaded as shown in the figure. Maximum bending
(B) 140 N/mm2 stress at the fixed end of the beam is
(C) 130 N/mm2 PL 2P
(D) 280 N/mm2 (A) 2 (B)
d tan θ d 2 tan θ
27. 3P 3PL
P (C) (D)
D D d P d tan θ
2
d tan θ
2

a b
30.
10 N

A bar is having uniform diameter D for a length a and


tapering diameter from D to d for a length b as shown
2m
in figure. If the bar is subjected to an axial pull P, the
4m
extension produced is
4P  a b  A beam with cross section 10 cm width and 20 cm
(A)  + 
πDE  D d  depth is loaded as shown in the figure. Maximum shear
stress at a section 1 m away from end is
4P  a b  (A) 0 (B) 0.375 MPa
(B)  + 
πdE  D d  (C) 3.75 MPa (D) 37.5 MPa
3.30 | Solid Mechanics Test 2

Answer Keys
1. B 2. B 3. C 4. B 5. A 6. B 7. C 8. C 9. B 10. A
11. B 12. B 13. B 14. A 15. C 16. A 17. B 18. C 19. D 20. B
21. C 22. B 23. B 24. B 25. A 26. B 27. A 28. A 29. C 30. B

Hints and Explanations


1. E = 2G (1 + m) t = 20 mm = 20 × 10–3m
= 2G (1 + 0.5) ⇒ E = 3G Choice (B) s = 50 kN/m2
2. L = 1.6 m m = 0.3
π G = 90 × 109 N/m2
q = 1° = radian V = b t = 3000 × 30 × 20
180
  = 18 × 105 mm3
G = 8 × 106 N/cm2 = 8 × 1010 N/m2
E = 2G (1 + m)
τ = 10, 000 N/cm2 = 1 × 108 N/m2
E = 2 × 90 × 109 (1 + 0.3) = 234 × 109 N/m2
τ Gθ E = 3K (1 –2m)
=
R L 234 × 109 = 3K (1 – 0.6)
τL 108 × 1.6 × 180 0.36 36 234 × 109
⇒ R = = m = m = cm. K= = 195 × 109
Gθ 8 × 10 × π
10
π π 3 × 0.4
 Choice (B) Compressive stress
K=
3. L = 2 m = 200 mm; A = 2000 mm ; 2
volumetric strain
P = 40 kN = 40 × 103 N
E = 2 × 105 N/mm2 50 × 103
195 × 109 =
PL 40 × 103 × 2000  dv 
Extension dL = = = 0.2 mm.  
AE 2000 × 2 × 105 v
 Choice (C) dv 50 × 103
= = 0.256 × 10 −6
4. As the expansion of the bar is not blocked, no stress is v 195 × 109
developed. Choice (B) dv = 0.4615 mm3
7. If both the ends fixed Will decrease by 0.4615 mm3. Choice (C)
L 8 1 1. Proof resilience is the strain energy at elastic limit
Le = = = 4m 1
2 2 = (stress × strain × volume)
E = 180 × 109N/m2 2
π 2 EI π 2 × 180 × 109 × I Proof resilience/ unit volume
Euler’s load PE = = 1
Le2
42 =× stress × strain at elastic limit
2
d = 80mm = 80 × 10–3m 1
=× 75 × 10 × 0.004 = 15 × 104 Nm/m3
6
π 4 π
(80 × 10 −3 ) = 2.01 × 10–6 m4
4
I= d = 2
64 64  Choice (B)
π 2 × 180 × 109 × 2.01 × 10 −6 12.
PE = = 223.2 kN P
16 × 1000
1 1 1 B C
= + L L
PR PE PC MA RB
A RC
1 1 1 L
= + RB
750 223.2 PC RA

1 1 1 223.2 − 750 Free body diagram is as shown above


= − = P
PC 750 223.2 750 × 223.2 RB = RC =
2
PC = 317.77 kN Choice (C)
Bending moment at A
8.  = 3m = 3000 mm PL
MA = RB­ × L = . Choice (B)
b = 30 mm = 30 × 10–3 m 2
Solid Mechanics Test 2 | 3.31

13. 17. T = 2.2 kNm


sC = 100 MN/m2
P(L – x)
PL st = 50 MN/m2
(–)
t = 35 MN/m2
A B G = 40 GN/m2
x C
L T τ max 2.2 × 103 35 × 106
  = = =
BM diagram J R π 4 d
d
BM diagram 32 2
Slope at C d = 68.4 mm Choice (B)
QC = QC - QA as QA = 0
τ max Gθ 35 × 106 40 × 109 × θ
Area of bending moment diagram 18. = = =
R L  68.4  1
between A and C  
   = 2000
EI
q = 0.026 radians × 180/p
−  PL + P ( L − x ) x − (2 PL − Px ) q = 1.46° Choice (C)
   = = x
2 EΙ 2 EΙ
π EI
2

− Px 19. Feuler =
   = (2 L − x )  Choice (B) L2
2 EΙ
F∝I
PL π 4
14. Extension = I= d
AE 64
PL PL F ∝ d4
\ =
πd 2 πd1d2 F1 − F2 d14 − d2 4
E E =
4 4 F1 d14
⇒ d = d1d2  d14 − ( 0.8d1 )
4
Choice (A)  1 − 0.84 
D2 = 0.8d1 = =  × 100 = 59%
15. d14  1 
A B
 Choice (D)
50 kN 50 kN
20. sb = 80 N/mm 2

t = 30 N/mm2
0.6 m
sy = 280 N/mm2
1 280
τ max = σ 2 + 4τ 2 =
p 50 × 103 × 600 2 FS × 2
δl1 = =
AE 1000 × 1.05 × 105 1 280 280
= 80 + 4 (30 ) =
2 2
= 50 =
dl1 = 0.2857 mm 2 FS × 2 FS
B C 280
F . S = factor of safety =  = 2.8 Choice (B)
30 kN 30 kN 50 × 2
21. Bending moment at hinge B
MB = 0
1000 10 kN
A B C
p 30 × 103 × 1000
δl2 = =  = – 0.2857mm (–)
AE 1000 × 1.05 × 105
4m 1m 1m
C D
10 kN 10 kN
Considering the portion BC, reaction at C,
Rc = 5 kN
1.2 Let MA be the bending moment at A
Taking moments about A
pl 10 × 103 × 1200 10 × 5 - 5 × 6 = 20 k Nm. Choice (C)
δl3 = =  = – 0.1143 mm
AE 1000 × 1.05 × 105
2 2. Let side length be L
dl = dl1 + dl2 + dl3 = –0.1143 mm Choice (C) Elongation due to heating = aDTL
3.32 | Solid Mechanics Test 2

As elongation is blocked compressive strain in the 28. Normal stress on plane AC


x-direction px + p y px − p y
σ µσ y σ pn = + cos 2θ + q sin 2θ
ex = a ∆T = x − −µ z 2 2
E E E
pn = 2MPa; px = 6 MPa
As σ x = σ y = σ z = σ q = 3 MPa; q = 45°
σ 6 + py py
ex = [1 − 2µ ] \ 2 = + 0 + 3 ×1 = 3 + +3
E 2 2
σ Eα∆T py
i.e., [1 − 2µ ] = aDT ⇒ s = (compressive) ⇒ = −4 ⇒ py = -8 MPa
E (1 − 2µ ) 2
 Choice (B) = 8 MPa (compressive) Choice (A)
23. Volumetric strain M
29. Maximum bending stress f =
ev = ex + ey + ez = ex + 2(-ex . m) = ex(1 - 2m) Choice (B) Z
24. where M = bending moment
d
2
and Z = section modulus
At fixed end,
M=P×L
d
bd 2 2 L tan θd 2
Z= =
6 6
P × L×6 3P
 d 3 \ f =  =  2  Choice (C)
3  ⋅ d 2 Ld tan θ d tan θ
2
bd 2 d4
Moment of inertia = = =
12 12 24 30. Shear force diagram is given below
4
5wL 5 wL4 24
Maximum deflection = = × 4 5kN
384 EI 384 E d 5kN
1m
5 wL4 4m
=  Choice (B)
16 Ed 4 Shear force at a distance 1m from end F = 5kN
2
 px + p y  10 cm
 2  + q
2
25. Maximum shear stress =
10 cm
y
2 NA
 px  px
=   + 0 =
2 2
1
Maximum shear stress = the uni axial tensile stress y = 50 mm
2
Area above neutral axis
 Choice (A)
a = (100)2 mm2
26. px = 120 N/mm , py = -100 N/mm
2 2
breadth b = 100 mm
p1 = 150 N/mm2 shear stress is maximum at neutral axis and is given by
p1 + p2 = px + py = 120 - 100 = 20 N/m2
F
⇒ 150 + p2 = 20 ⇒ p2 = -130 q= ay
p1 − p2 bΙ
Maximum shearing stress = 5 × 103 × (100 ) × 50
2
2 qmax = = 0.375 MPa
100 × (200 )
3
(150 + 130) 100 ×
= = 140 N/mm2 Choice (B) 12
2
πDd Alternately,
2 7. Mean area for tapering portion = F
4 Average shear stress qav =
Pa Pb bd
Total extension produced =  +
πD E πDd E
2
Maximum shear stress
4 4 F 5 × 103 × 1.5
qmax = qav × 1.5 = × 1.5 = = 0.375 MPa
4P  a b  bd 100 × 200
=
   +  Choice (A)
πDE  D d   Choice (B)
Unit III
Structural Analysis
This page is intentionally left blank
Structural Analysis Test 1
Number of Questions: 25 Time: 60 min.

Directions for questions 1 to 25: Select the correct alterna- (A) 12.5 kN-m (B) 7.5 kN-m
tive from the given choices. (C) 15 kN-m (D) None
1. Which of the following statements regarding statically 7. The Influence line diagram (I.L.D) shown is for the
determinate structures are correct? member –––––
(a) Can be analyzed by equilibrium equations
a b
(b) Stresses are caused due to temperature changes
(c) BM at a section does not depend on material or
sectional properties of structure. c d
(A) a, b, c are correct (B) a, b are correct
(C) a, c are correct (D) b, c are correct
2. The pin jointed frame shown in the figure is Straight line

60° c
100kN 100kN 100kN d
L/2 L/2 L/2

(A) Perfect frame (B) Redundant frame Straight line


(C) deficit frame (D) None
3. Which of the following method comes under the cate- (A) ab (B) ac
gory of displacement method to analyze statically inde- (C) cd (D) ad
terminate structure?
8. The ILD for shear force at a section ‘c’ of simply sup-
(A) Elastic center method
ported beam of length ‘L’, when unit load moves from
(B) Minimum strain energy theorem
one end to other is
(C) Moment distribution method
a b
(D) Column analogy method A B
c
4. The cantilever beam AB of length ‘L’ fixed at A and
free at B is subjected to a concentrated load ‘W’ at its L
free end. The strain energy (U) stored in a beam is a/L
[EI: Constant)
(+)
W 2 L2 WL3 (A) A B
(A) (B) (–) C
4EI 6EI
W 2 L3 WL b/L
(C) (D) ab/L
6EI EI
(B) (+)
5. The Bending moment induced at fixed end of cantilever
A B
beam of span ‘L’ if the free end undergoes a unit dis-
placement without rotation is b/L

3EI 5EI (+)


(A) 2 (B) (C)
L L2 (–)

6EI 4 EI a/L
(C) (D)
L2 L2 ab/L
6. The value of support moment M for the beam shown (+)
below (in kN-m) (D)
(–)
30 kN
M 3m 1m 1m 3m b/l

9. Which of the following statements below are correct?


(a) The stiffness coefficient kji indicates force at j due
to a unit deformation at i.
3.36 | Structural Analysis Test 1

(b) Stiffness matrix is a square symmetric matrix.


(c) Stiffness matrix is possible for both stable and
unstable structures also. +
(B)
(A) a, b, c are correct (B) a, b are correct B C D
(C) a, c are correct (D) b, c are correct (–)

10. Which of the following statements are correct?


(a) The internal forces at any c/s of an arch are SF,
BM only (C)
(b) The effect of arching a beam is to reduce BM in
the span A B C D
(c) A two hinged arch is indeterminate by one degree (D) A B C D
(d) The internal forces at any c/s of an arch are SF,
14. A beam ABCD is 15m long and is simply supported at
BM and normal thrust also.
B and C 8 m apart. Overhangs BA and CD are 3m and
(A) a, b, c, d are correct
4m respectively. A train of two point loads of 150 kN
(B) a, b, c are correct
and 100 kN, 3 m apart, crosses the beam from left to
(C) b, c, d are correct
right with 100 kN load leading. The maximum sagging
(D) a, c, d are correct
B.M. under 150 kN load anywhere is
1 1. Determine the stiffness matrix for a beam for the given (A) 150 kN-m (B) 250 kN-m
coordinates shown in the figure (C) 360 kN-m (D) 400 kN-m
1 2 1 5. In the truss shown below, indicate how many number of
Ι 2Ι Ι
members with zero forces
5m P Q
3m 3m

 4 EI   4 EI  X
R
 4 EI 3   4 EI 3 
(A)  
 (B) 
 4 EI 52 EI   4 EI 52 EI  S
 3 T
 5  15  V U
W
 52 EI −4 EI   +4 EI 
(A) 4 (B) 5
 15 3   4 EI 3 
(C)   
(D)  (C) 6 (D) None
 −4 EI 4 EI   4 EI 4 EI  1 6. A simply supported beam of length L = 10 m and
 3   3  depth = 0.5 m is subjected to a temperature differen-
tial of 80oC at top and 160°C at bottom. Determine the
12. A cable carrying a load of 40 kN/m run of horizontal vertical deflection of beam at its mid point (c) due to
span, is stretched between supports 150m apart. The temperature gradient take a = 10 × 10–6 /°C.
supports are at same level and the central dip is 15 m. 80°c
The greatest tension and least tension in cable are 0.5m
(A) 8100 kN, 7500 kN A
C
(B) 10,000 kN, 7500 kN 160°c B
(C) 9500 kN, 6000 kN 10m
(D) None (A) 25 mm (B) 30 mm
13. For the continuous beam shown below, the I.L.D for (C) 40 mm (D) None
Reaction at A is 1 7. What is the rotation of the member at ‘C” for a frame as
Hinge shown in figure below?
B C
A C
D B

L L/2 L/2 30 kN–m

4m
Straight line
(A)
A
A B C D 3m
Structural Analysis Test 1 | 3.37

30 60 23. In the frame shown below; what are the distribution


(A) (B) factors for members BA, BC and BD respectively
3EI 7EI
90 75 A
(C) (D)
EI EI
L
18. In the portal frame shown in the given figure, the
E
ratio of sway moments in column AB and CD will be C
B
equal to
2 kN/m L

C
B D

3m 2I L L
2I 5m

D
A (A) 0, 0.4, 0.6 (B) 0, 0.3, 0.7
(C) 0, 0.4, 0.3 (D) None
(A) 25/9 (B) 3/5
(C) 2/3 (D) 15/8 24. Examine the given truss below
19. U1 and U2 are the strain energies stored in a prismatic
bar due to axial tensile force w1 and w2 respectively.
The strain energy ‘U’ stored in the same bar due to
combined action of w1 and w2 be
(A) U = U1 U2 (B) U > U1 + U2
(C) U < U1 + U2 (D) U = U1 + U2
2 0. The right triangular truss is made of members having
equal c/s area of 1000 mm2 and youngs modulus of (A) statically determinate
2 × 105 MPa. The horizontal deflection at B is
(B) statically indeterminate but kinematically deter-
B 100 kN minate
(C) statically indeterminate and kinematically indeter-
4m minate
(D) statically determinate and kinematically indeter-
A C minate
3m
25. The horizontal deflection at C for the following frame
(A) 15 mm (B) 20 mm shown below
(C) 12 mm (D) None
2P
2 1. A two spans continuous beam having equal spans each
of length L/2 is subjected to a uniformly distributed B
C
L/2
load 2w per unit m length. The beam has constant flex-
ural rigidity. The reaction at middle support is
3wl 3wl L
(A) (B)
4 8
5wl 5wl
(C) (D) A
4 8
22. Using the data in Q No 21; find the Bending moment at PL2 PL3
the middle support (A) (B)
2 EI 3 EI
wl 2 wl 2
(A) (B)
16 4 PL3 2 PL2
(C) (D)
wl 2 3wl 2 2 EI EI
(C) (D)
8 16
3.38 | Structural Analysis Test 1

Answer Keys
1. C 2. A 3. C 4. C 5. C 6. B 7. D 8. C 9. B 10. C
11. B 12. A 13. B 14. C 15. B 16. C 17. B 18. A 19. B 20. C
21. C 22. A 23. C 24. C 25. C

Hints and Explanations


1. Statements (a) and (c) are correct Choice (C) [Due to sinking of support ‘B’; Rotations develop at A
2. m = 2j – 3 and B; But fixed end does not allow rotations and hence
m = number of members = 11 6 EI δ
fixed end moments of magnitude develop]
j = number of joints = 7 L2
m = 2(7) – 3 = 14 – 3 = 11  Choice (C)
Hence, a perfect frame Choice (A) 6. Because of symmetry,
3. Moment distribution method comes under the category 15kN
of displacement method. Choice (C) 3m
4. 1m
W 15kN–m

A B From concept of carry over moment half the moment is


x
L transferred to fixed end in some direction.
15
i.e., = 7.5 kN-m Clockwise
2
Strain energy (U) is given by
 Choice (B)
M x2 dx
L

Ux = ∫ 11.
0
2 EI I 1 2 I
2I
A D
MX = –wx
B C 5m
L
(WX )2 3m 3m
Ux = ∫ dx
0
2 EI
2
1
L
w X 
2 3
W L 2 3
A D
  =   =  Choice (C)
2 EI  3  o 6 EI B C

5. Given, free end undergoes unit displacement and  K1 K12 


without rotation, so the beam can be shown as K= 
 K 21 K 22 
l
K11: moment at (1) due to unit rotation at (1)
K21: moment at (2) due to unit rotation at (1)
6EIδ/L2
So at first coordinate given unit rotation in C.W
A
B direction
δ=1
1 2
A =1
12EIδ 6EIδ
L3 =1
L2 D
12EIδ B C
4EI 4EI 2EI
L3 L L L

6 EI δ  4 EI   4 EI 
Moment at A = K11 =  +
L2  L  BA  L  BC

d=1
6 EI  4 EI   4 E (2 I )  12 EI
  =  +    = 3 = 4EI
MA =
L2  3   3 
Structural Analysis Test 1 | 3.39

 2 EI  2 E (2 I ) 4 EI CD deforms due to downward movement of C. At ‘B’,


K21 =    = = there won’t be any displacement because of support of
 L  BC 3 3
B. Since the beam is statically determinate the ILD is a
K22: moment developed at (2) due to unit rotation at (2) straight line, not a curve. Choice (B)
K12: moment developed at (1) due to unit rotation at (2) 1 4. To get maximum Bending moment under a load, the
So apply unit rotation at (2) Resultant and load should be kept equidistant from
center of span.
1 2
150kN 100kN
=1
A θ=1
x 3m
B C D
2EI 4EI 4EI R
3 3 5
100 × 3 300
x= = = 1.2 m
 4 EI   4 EI  =  4 E (2 I )  +  4 EI  150 + 100 250
K22 = K 22 =  +    
 L CB  L CD  3   5 

1.8

8 EI 4 EI 40 EI + 12 EI 52 EI 150kN 100kN
  = + = =
3 5 15 15 3.4 0.6 0.6m

 2 EI   2 E (2 I )  4 EI A D
K12 =   = = 3 P E
 L CB  3  B 4m
R
4m C

 4 EI  3m 8m 4m
 K11 K12   4 EI 3 
[K] =  =  Choice (B)
 K 21 K 22   4 EI 52 EI  Vb + VC = 150 + 100 = 250 kN
 3 15  (VC)(8) – 100(6.4) – 150(3.4) = 0
12. VC = 143.75 kN
VA VB VB = 106.25 kN
R 150m R Maximum B.M at P = (VB) (3.4)
HA HB = 361.25 kN-m Choice (C)
15m 15.
4OkN/m
P Q
R

wl 40 × 150 3 S
VA = VB = = = 3000 kN T
2 2 1
4 V U 2
W
Wl 2 40 × 150 2
HA = HB = =
8h 8 × 15 Go by Joint no’s.
At joint (1); Since no load is acting at ‘s’ force in these
5 × 150 × 150 members to be zero
  = = 7500 kN
15 At Joint (2) i.e., at T
Maximum tension = R = V + H
2 2
R
FTR
= (3000)2 + (7500)2 = 8077 kN
U
FTU T
Minimum tension occurs at deepest point and is equal
to Horizontal thrust
w
Tmin = 7500 kN Choice (A)
1 3. By Muller Breslau’s principle, apply a unit vertical dis- Three forces are acting at a joint; [i.e., W; FTR; F ]
placement at A. The resulting deflection profile is the and two of the forces are in same line i.e., W and FTR
I.L.D at A. Since A, B and C are one part and CD are the force in FTU to be zero since for equilibrium of a
the other part; ABC deforms as a single member and joint
3.40 | Structural Analysis Test 1

At Joint (3) i.e., at X; M 12 EI + 9 EI


=
FXP θ 12
FXQ
M 21EI
X FXU
=
θB 12
12 M 12 × 30
The same statement as above; Hence the force in θB = ⇒ θB =
21EI 21EI
FXQ = 0
120
At Joint (4) i.e., at V θB =
7 EI
From slope deflection method
Fvp
2 EI  3δ 
M CB =  2θC + θ B −  + M FCB
Rv L  L
V Fvu
d = 0, MFCB = 0
FVP = 0 MCB = 0 [Since hinge support]
∴ no of members with zero forces = 5. Choice (B) 2 EI
1 6. Since the vertical deflection at center of beam is to be
0=
3
[2θC + θ B ]
determined 2 EI 120 

Apply a unit load at center of beam
3 2θC + 7 EI  = 0
1 kN
120
A C B 2θc + =0
7 EI
5m 5m −120
RA = 0.5 kN RB = 0.5 kN θC =
14 EI
From symmetry; +60
θC = [Neglect sign] Choice (B)
RA = RB = 0.5 kN 7 EI
L
m α ∆ Tm dX
dC = ∫ 6 EI δ 6 E (2 I ) δ 12 EI δ
C 18. MAB a  = =
0
L2 (3)2 9
m: Internal virtual moment in beam
a = 10 × 10–6/°c 6 EI δ 6 E (2 I ) δ 12 EI δ
MCD a  = =
∆TM : temperature difference between mean temp and L2 52 25
the temp at the top or bottom of beam
M AB 12 EI δ (25)
 80 + 160  = = 25/9 Choice (A)
=  − 80 = 120 – 80 = 40°C M CD 9 (12 E I δ )
 2 
0.5 19. U1 = strain energy stored in bar due to W1
C: mid depth of beam = = 0.25m
2 P 2 L W12 L
U1 = =
10 m
(0.5 x ) (10 × 106 °C) ( 40°C) 2 AE 2 AE
δv = ∫
0
0.25m
dx U2 = strain energy stored in bar due to W2
W22 L
 0.5 × 10 × 10 × 40 x −6 2
10
U2 =
=
   2 AE

 0.25 2 0 U = strain energy stored in bar due to combined
0.5 × 10 × 10 −6 × 40 × 10 2 (W1 + W2)
=
  
(W + W )
2
2 × 0.25 1 2 L
U=
dV = 0.04m 2 AE
dV = 40 mm Choice (C) ∴ U > U1 + U2 Choice (B)
17. Rational stiffness at a joint B; PkL
20. d = Σ
M 3 EI 3 EI 3 EI 3 EI AE
S= = + = +
θ L L 3 4 P: Force in member due to applied loads
Structural Analysis Test 1 | 3.41

K: Force in member for a unit load in the direction in Joint C:


which deflection is desired
L: Length of members FBC
B 1 kN
100 kN
FAC
R = 1.33 kN
4m
FAC = 1 kN (Comp)
 HC = 100kN FBC = 1.33 kN (Comp)
A C
3m At Joint A:
B
RA RC FAB

(R)(3) = 100 × 4 A C
100 × 4 1.33 kN
RA = RC = R = = 133.33 kN
3
At Joint A: FAB sinθ = 1.33 kN
FAB 1.33 1.33
FAb = =
sinθ 415

FAB = 1.66 kN (tension)
FAC
R Take tension: + ve
4 compression: – ve
sinθ = 5
Member Length (L) P K PkL
cosθ = 3/5
AB 5 166.67  1.66 1383.36
FAB sinθ = R
BC 4 –133.33 –1.33   709.31
R 133.33
FAB = = = 166.67 kN(Tension) AC 3 –100 –1 300
sinθ 4 /5
FAB cosθ = FAC ΣPKL = 2392.67
⇒ FAC = 166.67 × 3/5 = 100 kN(compression) ΣPkL
δ=
At joint C: AE
B
1
  =  (2392.67)
FBC
100 kN (1000 mm ) (2 ×105 N mm2 )
2

FAC
C
133.33 kN d = 11.96 mm Choice (C)
ΣFy = 0 21.
FBC = 133.33 kN 2w/m
FBC = 133.33 kN (Compressive)
FAC = 100 kN (Compressive) C
A
Due to unit load B
B 1 kN L/2 L/2
EI EI

5
Wl 2 − (2w ) ( L 2) − (2w ) ( L 4 ) −WL2
4 22

MFAB = – = = =
A  12 12 12 24
1 kN
+ wl 2 (2w ) ( L 2) +WL2
2
3 C
MFBA = = =
RC = RA = R = 1.33 RC 12 12 24

1×4=R×3 −WL2 +Wl 2


MFBC = , M FCB =
R = 4/3 = 1.33 kN 24 24
3.42 | Structural Analysis Test 1

Relative Distribution 23.


Joint Member Σk
stiffness k – I/L factor = k/ Σk A
3 I 6I 12I 6I 4L
BA = = 1 L
4 L 2 4L 4L 4L(12I) 2
B E
3 I 6I C
BC = 1/2 B
4 L 2 4L
L

A B C D
1 1
2 2
L L
–WL2 WL2 –WL2 +WL2
F EM
24 24 24 24
Joint Members K = I/L ΣK DF = K/ ΣK
Release +WL 2 –WL2 BA 0 0
moment 24 24
WL2 –WL2 BC I/L I2L
= 0 .4
C.O.M 48 48 L.5I
5
I/L
B 3 2 3I 2L 6
Final O O BD I/L = = 0 .3
WL2 –WL2 4 4L 5I 20
16 16
3
BE I/L 0.3
4

2w/m 2w/m  Choice (C)


B
C
2 4. Internal static indeterminacy, Dsi = m – (2j – 3)
A
L/2 L/2 m = 16, j = 8
RB RB
Dsi = 16 – (2 × 8 – 3)
wl2 Dsi = 3
16
External indeterminacy Dse = r – 3 = 3 – 3= 0
Ds = 3 + 0 = 3
For SPAN AB: Kinematic indeterminacy, Dk = Nj –C
2 Dk = 2 × 8 –3
L  L   L  wL
( RB ) = 2w     =
 2   4  16
=0 Dk =13
2 statically and kinematically indeterminate Choice (C)
L 2 × 2 × wL WL2
2
25. Since no horizontal load acting at c, apply a fiction load
RB = + ‘Q’ at ‘c’.
2 2×8 16
2P
L 5wL2
RB = w B
2 16 Q
L/2 C
5WL
RB = RB = L
8

For Span BC:


L WL2 L  L
( RB ) = = (2w )   = 0 A
2 16 2  4
Span BC:
5WL 2P
RB =
8
Q Q
5WL 5WL 5WL x
∴ Prop Reaction at B = + = PL 2P
8 8 4
L/2
 Choice (C)
Structural Analysis Test 1 | 3.43

MX = – 2PX My = PL + Qy
∂ Mx
∂ My
 = 0 = y y = 0 to L
∂Q ∂Q
L
 ∂ M y  dy
L2
∂M dX
(Mx ) x
( )
(dH)AB = ∫ M y  
(dH)BC = ∫
0
∂ Q EI
=0 0  ∂ Q  EI
L
dy
Span AB:    = ∫ ( PL + Qy ) ( y )  
0
EI
2P
L
B PL  PLy 2  PL3
Q    =   =
y
 2 EI  0 2 EI
(dH)c = (δH)Bc + (δH)AB
PL3
A
(dH)c =   Choice (C)
2 EI
Structural Analysis Test 2
Number of Questions: 30 Time: 75 min.

Directions for questions 1 to 30: Select the correct alterna- c. Castiglianos theorem of mini- 3.
mum strain energy
tive from the given choices.
d. Moment distribution method 4.
1. The frame shown below is
Q R Codes:
a b c d a b c d
(A) 2 1 2 1 (B) 1 2 1 2
(C) 1 1 2 2 (D) 1 2 2 1
P S 6. The cantilever beam AB of length ‘’ fixed at A and free
at other end is subjected to a concentrated load W at
(A) statically indeterminate but unstable its free end. The strain energy (U) stored in a beam is
(B) unstable (EI const)
(C) determinate and stable W 2 2 W 3
(A) (B)
(D) none of the above 4 EI 6 EI
2. The static indeterminacy of a continuous beam with an W 3 3 W
internal hinge shown below is (C) (D)
6 EI EI
7. The bending moment induced at fixed end of cantilever
beam of span ‘’ if the free end undergoes a unit dis-
placement without rotation is
(A) zero (B) 1 3EI 5EI
(C) 2 (D) None (A) 2 (B)
 2
3. The degrees of freedom of the rigid frame with clamped 6EI 4EI
ends at P and Q as shown in figure is (C) 2 (D)
 2
S R
8. The slope deflection equation at the end Q of member
QR for the frame shown in the given figure below is
P Q
50 kN Q R
(A) 2 (B) 3 EI
(C) 4 (D) zero 4m EI
EI 6m
4. A truss, as shown in figure is carrying 180 kN at B. the P
force in member BD is S
180 kN
B 5m

2 EI
(A) MQR = (2θQ + θR)
5
A C 2 EI
D (B) MQR = (2θQ − θR)
5
(A) 180 kN (Tensile) 2 EI
(C) MQR = (2θR − θR)
(B) 180 kN (compressive) 5
(C) 180 2 kN (Tensile) EI
(D) MQR = (2θQ + θR)
(D) zero 5
5. Match List – I and List – II and select the correct answer 9. Which of the following cross sections has the highest
using the code given below. shape factor?
(A) Rectangle (B) Diamond
List – I List – II (C) Triangle (D) Circle
a. Column analogy method 1. Stiffness
10. The stiffness coefficient Kji indicates
b. Kanis rotation contribution 2. Force method (A) force at j due to a unit deformation at i
method
(B) force at I due to a unit deformation at j
Structural Analysis Test 2 | 3.45

(C) deformation at j due to a unit force at i 15. For the truss shown in the figure, the force in member
(D) deformation at I due to a unit force at j AB is
1 1. The force in the member QS of the truss shown in the L
figure is; A B
150kN

S L
T

C
D
W
P R
Q
W
(A) W (B)
150 2
(A) 150 kN (B) kN (C) zero (D) None
2
(C) zero (D) 35.5 kN 16. Which of the following statements is true with regard
12. The pin jointed 2-D truss is loaded with a horizontal to the flexibility method of analysis?
force of 20 kN at joint ‘S’ and another 20 kN vertical (A) the method is used to analyse determinate structures.
force at joint ‘U’ as shown. Find the force in the mem- (B) the method is used only for manual analysis of inde-
ber RS (in kN). (taking tension as positive and com- terminate structures
pression as negative) (C) the method is used for analysis of flexible structures.
4m 4m 4m (D) the method is used for analysis of indeterminate
structures with lesser degree of static indeterminacy.
R S 20kN
17. The strain energy stored in the member AB of the pin
jointed truss shown below. A and E is same for all
Q V U T members.
A C
4m 20 k N
W
L
(A) zero (B) 20 kN
(C) 40 kN (D) −20 kN D
B
1 3. For the plane truss shown in the figure, the number of L 2P
zero force members for the given loading is
W2
4P2 L
(A) (B) zero
2 AE
P2 L 2P 2 L
(C) (D)
AE 3 AE
W1
18. The unit load method used in structural analysis is
(A) 8 (B) 4
(A) applicable only to indeterminate structures.
(C) 11 (D) 13
(B) derived from castigliano’s, theorem
1 4. The force in members p, q, r in the truss shown (C) another name for stiffness method
2W D (D) none

q
19. For linear elastic systems, the type of displacement
p function for strain energy is
(A) quadratic (B) linear
A C
B r (C) cubic (D) quartic

(A) 2W, W, 0 (B) 2W, W, W 20. The right triangular truss is made of members having
W W W equal cross sectional area of 1500 mm2 and young’s of
(C) , W, 0 (D) , ,0
2 2 2 105 × W5Mpa. The horizontal deflection at join Q is
3.46 | Structural Analysis Test 2

Q 100 kN (A) 1/5, 2/5, 2/5 (B) 2/5, 1/5, 1/5


(C) 1/3, 1/3, 1/3 (D) none
24. A propped cantilever beam PQ with fixed edge ‘P’ is
4m
propped at ‘Q’ and carries a UDL of w/m over the entire
span. If the prop displaces upward by 2 mm, which one
P R of the following is true? (if prop reaction = RQ moment
at P = MP)
3m (A) both RQ and MP increase
(B) RQ increases, and MP decreases
(A) 7.79 mm (B) 4.60 mm (C) RQ decreases and MP increases
(C) 8.20 mm (D) none (D) both RQ and MP decreases
2 1. In a redundant joint model, three bar members are pin 2 5. Identify the correct deflection diagram corresponding
connected at Q as shown in figure. Under some load to the loading in the plane frame
placed at Q, the elongation of the members MQ and P
C
OQ are found to be 40 mm and 25 mm. then the hori- B
zontal displacement ‘U’ and vertical displacement ‘V’
of the node Q1 in mm will be respectively,
N

M O A
MN: 400 mm
NO: 500 mm (A) (B)
NQ: 500 mm
Q X, U

Y, V
(C) (D) None
(A) 8.84 mm, 44.20 mm (B) 5 mm, 20 mm
(C) 0 mm, 44.20 mm (D) 7 mm, 35 mm
2 2. The strain energy started in the member AB of the pin
jointed truss shown aside when E and A are same for all
members is 26. Rotational stiffness coefficient, K11 at joint ‘C’ for the
A EI
C frame having two members of equal is given by

L a Fixed EI, l

D c
B
1.5P
b EI, l
1.5P 2 L P2 L Hinge
(A) (B)
AE AE
7EI 5EI
2P 2 L (A) (B)
(C) zero (D)  
AE
4EI 6EI
(C) (D)
23. In the portal frame shown below, what are the distribu-  
tion factors for member BA, BC BD respectively?
3P 4P
27. A propped cantilever beam of span ‘L’ is loaded with
Q C
UDL of intensity w/unit length, all through the span.
Bending moment at fixed end is _____
w/m 2I 2L
A B WL2 WL2
I (A) (B)
2I 2L 8 2
D
2L WL2 WL2
(C) (D)
12 24
Structural Analysis Test 2 | 3.47

28. Influence lines for redundant structures can be obtained 30. Consider the beam PQRS and the influence line as
by shown below. The influence line pertains to
(A) Castigliano;s theorem
(B) Muller Berslau principle P Q R S
(C) Unit load theorem
(D) Maxwell-Betti reciprocal theorem L L 2L
29. A homogeneous, simply supported prismatic beam of
Width B, depth D and span ‘L’ is subjected to a con- 1
centrated load of magnitude P. The load can be placed P Q R S
anywhere along the span of beam. The maximum flex-
ural stress developed in the beam is (A) Reaction of P, RP
3 PL 4 PL (B) Shear force Q, VQ
(A) 4 2 (B) 3 BD 2
BD (C) Shear force on left of Q, VQ−
3 PL 2 PL (D) Shear force on right of Q, VQ+
(C) 2 (D)
2 BD 3 BD 2

Answer Keys
1. B 2. A 3. B 4. D 5. A 6. C 7. C 8. A 9. C 10. A
11. C 12. A 13. A 14. A 15. A 16. D 17. B 18. B 19. B 20. A
21. A 22. C 23. A 24. B 25. A 26. A 27. A 28. B 29. C 30. B

Hints and Explanations


1. Given rigid jointed plane frame with internal hinge J : number of joints = 4
at Q C : number of restraints = 3 + 3 = 6
Ds = (3m + r) − 3J – C \ DK = (3) (4) − 6 = 12 – 6
Where m number of members = 3 DK = 6 [with axial deformations]
r : no of reactions = 2 + 1 = 3 DK = 6 – 3 = 3 [neglecting axial deformation]
J : number of joints = 4  Choice (B)
C : number of releases = (2 – 1) = 1 4.
\ Ds = [3(3) + 3] − 3(4) – 1 180kN
B
  = 12 – 12 – 1 = –1
⇒ Ds < 1
The given frame is unstable Choice (B)
2. Degree of static indeterminacy = Ds = Dsi + Dse − C
Dse : external indeterminacy A C
r : − 2 [since only vertical loading so ‘2’ equilibrium D
equations]
r : number of reactions = 3 At joint D
Dse = 3 − 2 = 1
Dsi = Internal indeterminacy = 0 FDB
C : number of releases = 1
[Due to the presence of internal hinge, we get an addi- FDA D FDC
tional equation since it does not allow moment from
one part to other] If two forces are in same line the force in third member
\ Ds = Dse + Dsi − C is equal to zero when three forces are acting at a joint
  = 1 + 0 – 1 \ FBD = 0 Choice (D)
Ds = 0 Choice (A) 5. These are basic method of structural analysis to analyse
3. Kinematic indeterminacy/Degrees of freedom DK = NJ statically indeterminate structures.
−C Column analogy method and castiglianos theorem of
Where N : number of degrees of freedom of each joint minimum strain energy comes under force method/flex-
= 3 [since rigid plane frame] ibility method/equilibrium method and kanis rotation
3.48 | Structural Analysis Test 2

contribution method and moment distribution method 10. Choice (A)


comes under compatibility method/stiffness coefficient 11. Consider joint Q
method/displacement method. Choice (A) S

6.
W P R
Q
A
B Three forces are acting at joint Q. two of them are in
x
same line; the force in third member (QS) to be zero
form equilibrium of joint. Choice (C)
Strain energy (U) is given by 12. SFy = 0
L
M X2 d X RW + RJ = 20kN
U= ∫
O
2 EI SMT = 0
(RW)8 + (20)(4) − (20(4) = 0
Where Mx : moment at a section (x)
RW = 0
  =− wx where x varies from o to L
RT = 20kN
L
( −WX )2 d X X
U= ∫
O
2 EI
R S
20kN

L L
W 2 X 2 W 2 X 3  W 2  2
   = ∫ =   =  Choice (C)
O
2 EI  6 EI O 6 EI T
X 20kN
7. Given free end undergoes unit displacement and with- RT = 20
out rotation so the beam can be shown as
RW = 0
6EIδ A B
l2  δ=1
l Draw a section (X) – (X) passing through (RS)
6EIδ
 Taking equilibrium of right hand side
l2
12EIδ
L3 FRS
12EIδ 20kN
L3

\  Moment at [Due to sinking of support B; U


6EIδ
A= rotations develop at A & B; but
2
fixed end does not allow rotations 20kN 20kN
Where d = 1
and hence fixed end moments of
6EI Taking moments about U
MA = 6EIδ
2 magnitude 2 develops] (FRS)(4) + (20)4 − 20(4) = 0

(FRS) = 0 Choice (A)
 Choice (C)
13.
2EI 3δ W2
8. MQR = (2θQ + qR − ) + MFQR
L L
5
d = 0; MFRQ = 0 6 8
4 2
3 7 1
2EI
\ MQR = (2θQ + qR) Here L = 5m Choice (A)
L
W1
9. C/S sections Shape factors values
(a) Rectangle 1.5 Using the principle : If three forces act at a joint and
(b) Diamond 2.0 two of them are in same line; the force in third member
(c) triangle 2.34 will be zero.
(d) circle 1.7 Choice (C) \ Total number of zero forces = 8 Choice (A)
Structural Analysis Test 2 | 3.49

14. 20.
2W D Q 100kN
E

q 4m
p

P R 100
A C
B r
3m
RA
RB RQ = 133.33kN
RP = 133.33

RA + RB = 2W RP × 3 = 100 × 4
2W
Due to symmetry RA = RB = =W RP = 100 × 4/3 = 133.33kN
2
Q 100kN
RB and q are in same line at joint C
\ RB = q = W 5m 4m
so the value of r = 0
at joint E; P R 100kN
2W and p are in same line
So, p = 2W Choice (A) 3m

133.33 133.33kN
15. At joint C; the force W and FBC are in same line so the
force in member DC will be zero
At joint (P):
At joint B:
Q
A B
5m
45°
W
P R
D 3m
133.33kN
SFX = 0
FAB = FBD cos45° 3 4
cosθ = ; sinθ = 5
SFY = 0 5
FDB sin45° = W FPQ sinθ =133.33 KN
FDB = W 2 133.33 133.33
FAB =W 2 cos45° FPQ = =
sin θ (4)15
FAB = W Choice (A)
FPQ = 166.66kN
17. FPQ cosθ = FPR
HA = 2p FPR = (166.66)(315)
A C FPR = 99.99kN
2p
FRQ = 133.33kN
L R×3=1×4
R = 4/3 = 1.33 kN
2p
B
D At joint P:
L 2P Q

At joint B; P R
Reaction 2p and force in BD are in same line
Therefore force in member AB = 0 R:133.33kN
P2 L
Strain energy = FPQ sinθ = 1.33
2 AE
1.33 1.33
p = 0; FPQ = =
= 0 Choice (B) sin θ 415
UAB
3.50 | Structural Analysis Test 2

FPQ = 1.662 kN FAB = 0


FPQ cosθ = FPR P2 L
3 U= = 0 Choice (C)
FPR = 1.662 × = 0.9975 kN AE
5 23.
FRQ = 1.33 kN Distribution

Member P K  Joint Members Relative Sk factor  K 


stiffness(k)  
∑K 
PQ 166.66 1.662 5
BA I/2L 1/5
QR 133.33 1.33 4
B BC 2I/2L 5I/2L 2/5
PR 99.99 0.9975 3
BD 2I/2L 2/5
ΣPKL
dH =
AE  Choice (A)
166.66 × 1.662 × 5 + (133.33) 2 4. Choice (B)
(1.33) (4) + (99.99) (0.9975) (3) 25. Choice (A)
=
  
1500 × 200 26. Rotational stiffness at joint = Sum of rotational stiffness
dH = 7.978mm Choice (A) of each member
21. 4EI 3EI 7EI
N
= + =  Choice (A)
  
M O
27.
A W/unit length
1 2
B
Q l
X, U

Y, V
MN: 400mm −WL2 WL2
Initial FEM
NO: 500mm 12 12
NQ: 500mm
400mm
sinθ1 = = 0.624 −WL2 −WL2
400 2 + 500 2 Balancing
12 12
500
cosθ1 = = 0.780 −WL2
400 2 + 500 2 Carry over
24
500
sinθ2 = = 0.707
500 2 + 500 2 Final FEM −WL2 0
500 8
cosθ2 = = 0.707
500 2 + 500 2  Choice (A)
By resolving the displacements along OQ; 2 9. To get maximum BM place load at enter of beam,
−U sinθ2 + V cosθ2 = 25 - (1) M
Along member MQ; Maximum bending stress f = max
Z
U sinθ1 + V cosθ1 = 40 - (2) PL
U = 8.84mm 4 3 PL
V = 44.20mm Choice (A) = =  Choice (C)
BD 2 2 BD 2
22. At joint B; 6
Reaction at B and force in member CD are in same line.
Therefore the force in member AB = 0 30. ILD shown is for shear force at Q, VQ Choice (B)
Unit IV
Construction Materials
and Management
This page is intentionally left blank
Construction Materials and Management Test 1
Number of Questions: 25 Time: 60 min.

Directions for questions 1 to 25: Select the correct alterna- (iii) Strength increases with increase in slenderness
tive from the given choices. ratio
1. The bearing strength of M25 grade concrete in limit (iv) Strength decreases with decreases in cube size
state method of design as per IS 456:2000 is (A) (i) and (ii) are correct
(A) 25 MPa (B) 11.25 MPa (B) (i), (ii), (iii) are correct
(C) 15 MPa (D) None (C) (i) and (iii) are correct
(D) All the above
2. Modulus of Elasticity of M30 grade concrete is
(A) 25000 N/mm2 (B) 27386 N/mm2 12. Which of the following statements regarding properties
(C) 30000 N/mm2 (D) None of concrete are correct?
(p) Modulus of elasticity of M25 grade of concrete is
3. Flexural tensile strength of M25 grade concrete as per
25000 MPa.
IS 456-2000 is
(q) Approximate value of shrinkage strain of concrete
(A) 12.5 N/mm2 (B) 25 N/mm2
is 0.0003
(C) 22.5 N/mm 2
(D) 3.5 N/mm2
(r) pH value of water used in concrete construction
4. Minimum grade of concrete used for pretensioned and should not be less than 6.
post tensioned prestressed concrete are (A) p and q are correct
(A) M40 and M30 (B) M40 and M20 (B) p and r are correct
(C) M30 and M40 (D) None (C) q and r are correct
5. The 7-days strength of M30 grade concrete should be at (D) all the above are correct
least 13. The long term modulus of elasticity of M25 grade con-
(A) 30 MPa (B) 20 MPa crete with q value at 7 days to be 2.2 is
(C) 25 MPa (D) none (A) 25000 MPa (B) 7812.5 MPa
6. The target mean strength (fm) for concrete mix design (C) 3500 MPa (D) None
obtained from the characteristic strength (fck) and stand- 14. The probability of failure of a structure as per
ard deviation (s) as defined in IS456-2000 is: IS456-2000 (according to the concept of limit state design)
(A) fck + 1.35s is ____
(B) fck + 1.65s (A) 0.0975 (B) 0.95
(C) fck + 1.45s (C) 0.975 (D) 0.20
(D) fck + 1.55s
15. Group – I contains some properties of concrete / cement
7. Minimum cement content to be used in Reinforced and Group II contains list of some tests on concrete /
cement concrete for mild exposure is cement.
(A) 300 kg/m3 (B) 320 kg/m3 Match the property with corresponding test.
(C) 340 kg/m 3
(D) 450 kg/m3
Group – I Group – II
8. Nominal cover to main reinforcement in case of slabs
P. Direct tensile strength of 1. Cylinder splitting test
with mild exposure should be
conc rete
(A) 30 mm (B) 25 mm
Q. Workability of concrete 2. Surface area test
(C) 20 mm (D) 40 mm
R. Bond between steel and 3. Vee – bee tests
9. The individual variation in compressive strength of concrete
three cubes in the sample should not exceed
S. Fineness of cement 4. Fineness modulus test
(A) ±10% (B) ±15%
5. Pullout test
(C) ±20% (D) ±25%
10. The pozzolanas added to improve the properties of con- Codes:
crete are P Q R S P Q R S
(A) Fly ash (B) Silica fume (A) 1 3 5 4 (B) 5 2 1 3
(C) Slag (D) All the above (C) 2 3 1 4 (D) 2 1 5 3
11. Which of the following statements regarding the cube 1 6. Consider the following statements regarding the air
strength of concrete are correct? entrained concrete?
(i) Strength increases with decrease in cube size (1) Increased resistance to freezing and thawing
(ii) Strength decreases with increase in slenderness (2) Improvement in workability.
ratio (3) Increase in strength.
3.54 | Construction Materials and Management Test 1

(4) Permits reduction in water content of these, (q) Factors affecting the durability of concrete are w/c
(A) 1, 2, 4 are correct and maximum cement content
(B) 2, 3, 4 are correct (r) Minimum cement content is not based on expo-
(C) 1, 3, 4 are correct sure conditions.
(D) All the above are correct (A) p, q, r are correct (B) p and q are correct
(C) p and r are correct (D) only p is correct
17. Which of the following statements regarding admix-
tures are correct? 22. Consider the following statements regarding the addi-
(A) Retards the setting of cement tion of pozzolanas to cement causes
(B) Accelerates the setting of cement (p) Increase in strength
(C) Improves the workability of concrete (q) Less heat of hydration
(D) All the above (r) Decrease in workability
The true statements are
18. Consider the following statements: (A) p, q, r are correct (B) p and q are correct
I. The compressive strength of concrete decreases (C) p and r are correct (D) q only is correct
with increase in water cement ratio of the concrete 23. The composition of air entrained concrete is given
mix. below:
II. Water is added to the concrete mix for hydration Water : 180 kg/m3
of cement and workability. Ordinary Portland cement: 360 kg/m3
III. Creep and shrinkage of concrete are independent Sand : 601 kg/m3
of the water cement ratio in the concrete mix. Coarse aggregate: 1160 kg/m3
The true statements are Assume the specific gravity of OPC, sand and coarse
(A) I and III (B) I, II, III aggregate to be 3.10, 2.65 and 2.74 respectively, the air
(C) II and III (D) I and II content in liters/m3 is _______
19. Consider the following statements: (A) 53 liters/m3 (B) 50 liters/m3
I. Modulus of elasticity of concrete increases with (C) 45 liters/m 3
(D) None
increase in compressive strength of concrete 24. Consider the following statements
II. Brittleness of concrete increases with decrease in (p) Nominal maximum size of coarse aggregate to be
compressive strength of concrete. used in R.C.C is 20 mm
III. Shear strength of concrete increases with increase (q) As per IS456-2000; fine sand to be used in R.C.C
in compressive strength of concrete. should confirm to zone II and medium sand.
The true statements are (r) Minimum grade of concrete to be used in R.C.C is
(A) I and III (B) I, II, III M30
(C) II and III (D) I and II The true statements are
(A) p and q are true (B) p and r are true
20. Consider the following statements: (C) p, q and r are true (D) q and r are true
(p) Nominal mix proportions for M20 grade concrete
is 1 : 1.5 : 3 25. Which of the following statements given below are
(q) Weight batching is preferred compared to nominal correct?
(volume) batching (p) In mild environment, surface crack width should
(r) Maximum cement content as per IS456-2000 is not exceed 0.3 mm as per IS456-2000.
450 kg/m3 (q) Crack width increases with increase in stress in
(A) p, q are correct (B) p, r are correct reinforcement bar.
(C) q, r are correct (D) p, q and r are correct (r) Concrete and steel exhibit high strength after
being subjected to high temperature.
21. Which of the following statements given below are (A) p and r are correct
correct. (B) p, q and r are correct
(p) Nominal cover to reinforcement is based on ser- (C) p and q are correct
viceability or durability requirements (D) None

Answer Keys
1. B 2. B 3. D 4. A 5. B 6. B 7. A 8. C 9. B 10. D
11. A 12. D 13. B 14. A 15. A 16. A 17. D 18. D 19. B 20. D
21. B 22. D 23. A 24. A 25. C
Construction Materials and Management Test 1 | 3.55

Hints and Explanations


1. Bearing strength of concrete = 0.45 fck Ec 5000 × 25
13. Ece = =
= 0.45 (25) 1+ θ 1 + 2.2
= 11.25 MPa Choice (B)     =7812.5 N/mm2 Choice (B)
2. Ec = 5000 fck 14. The structure may fail when
(i) The load exceeds the design load.
  = 5000 30
(ii) The strength is less than the characteristic strength
  = 27386N/mm2 Choice (B) (iii) Both load exceeds design load and strength less
3. Flexural tensile strength of concrete = 0.7 f ck than characteristic strength.
The probability of load exceeding design load p1 = 5%
= 0.7 25 = 3.5 MPa  Choice (D) The probability of strength less than characteristic
2 strength p2 = 5%
5. 7 days strength = cube strength The probability of failure = 1 –q1 q2 = 1 – (0.95)2
3 = 0.0975 Choice (A)
2
= × 30  = 20 N/mm2 Choice (B) 15. Group I correctly matches with Group II for Choice (A).
3
 Choice (A)
6. fck = fm – 1.65 s
16. There is an increased resistance to freezing and thawing
⇒ fm = fck + 1.65 s Choice (B) & improvement in workability and also permits reduc-
7. Minimum cement content to be used in reinforced con- tion in water content in case of air entrained concrete.
crete for mild exposure is 300 kg/m3 Choice (A)  Choice (A)
8. Minimum cover to Reinforcement for slabs = 20 mm 17. All the above statements regarding admixtures are cor-
 Choice (C) rect. Choice (D)
18. I and II statements are correct. Choice (D)
9. Individual variation is compressive strength of 3 cubes
should not exceed ±15%. Choice (B) 19. I, II, III statements are correct. Choice (B)
20. Statements p, q and r are correct. Choice (D)
10. Fly ash, silica fume and slag are the three types of poz-
zolanas used in concrete to improve the properties of 21. p and q statements are correct. Choice (B)
concrete. Choice (D) 22. Statement ‘q’ only is correct. Choice (D)
11. Statements (i) and (ii) are correct M c M s M cA
Strength increases with decrease in cube size because 23. + + + Vw + Va = 1
ρc ρs ρcA
of better homogeneity and strength decreases with
increase in slenderness ratio. Choice (A) 360 601 1160 180
+ + + + Va = 1
3.10 × 1000 2.65 × 1000 2.74 × 1000 1000
12. All statements are correct.
Modulus of elasticity of concrete of M25 grade is Va = 0.053 m3/ m3
Ec = 5000 25 = 5000 f ck 25000 MPa and approxi-   =0.053 × 1000 litres/m3 [∵ 1 m3 = 1000 litres]
  =53 litres/m3 Choice (A)
mate value of shrinkage strain of concrete is 0.0003.
and pH value of water used in construction should not 24. Statements (p) and (q) are true. Choice (A)
be less than 6. Choice (D) 25. Statements (p) and (q) are correct. Choice (C)
Construction Materials and Management Test 2
Number of Questions: 30 Time: 75 min.

Directions for questions 1 to 30: Select the correct alterna- 7. Which of the following pairs are incorrect with refer-
tive from the given choices. ence to ordinary Portland cement?
1. The probability distribution taken to represent the com- (i) Initial setting time - 30 minutes
pletion time in PERT analysis is _____ (ii) Final setting time – 10 hours
(A) Gamma distribution (iii) Normal consistency – 10%
(B) Normal distribution (iv) All are correct
(C) Beta distribution (A) iv (B) ii and iii
(D) Log normal distribution (C) iii only (D) i and iii
2. The probability of completion of any activity within its 8. The most commonly used admixture to accelerate the
expected time is _____ initial setting time of concrete is _____.
(A) 50% (B) 84.1% (A) Gypsum
(C) 99.9% (D) 100% (B) Calcium Carbonate
(C) Calcium Chloride
3. The proportion of cement : F.A : C.A in a given con-
(D) Calcium Ferrate
crete is 1 : 2 : 4; then the mix refers to
(A) M20 (B) M15 9. A sand is said to be unsuitable for construction if it has
(C) M10 (D) M5 F.M. more than _____
(A) 2.9 (B) 3.2
4. Negative slack occurs when
(C) 3.4 (D) 3.9
(A) Dummy activities are large in number
(B) Events stick to their schedule 10. The standard size of specimen for conducting the mod-
(C) Dummy activities do not exist ules of rupture of concrete is
(D) Deficiency of resources occurs (A) 15 × 15 × 60 cm
(B) 15 × 15 × 65 cm
5.
(C) 15 × 15 × 70 cm
3
(D) 15 × 15 × 75 cm
6 8
B
C 11. To make one cubic meter of 1 : 2 : 4 by volume
1 4 E 2 centrete, the volume of coarse aggregate required
1 2 5 6
A G is ____
D F (A) 0.95 m3 (B) 0.85 m3
9 5
4 (C) 0.75 m 3
(D) 0.65 m3
12. In drawing AOA network and making time computa-
Duration of the project shown in the network is ____
tions, following processes are involved.
(Note : Durations given above are in days)
1. Activity listing
(A) 18 (B) 17
2. Work breakdown structure
(C) 16 (D) 15
3. Activity time allotment
6. In the network shown in the figure, the activity ‘F’ can 4. Consideration of available resources for each
be started only when activity
E 5. Activity dependencies
3 6
6. Float computations
B H 7. Backward path computation
8. Project duration
A D 9. Forward path computation.
1 2 7
What is the correct sequence of the process given
C G above?
(A) 1,2,4,3,9,7,5,6,8
4
F 5 (B) 4,1,3,2,9,5,7,8,6
(C) 2,1,4,3,5,9,8,7,6
(A) Activity B is completed (D) 1,3,2,4,5,9,7,8,6
(B) Activity C is completed 13. A building project consist of 10 activities, represented
(C) Activity D is completed by the network shown below. Find the critical path of
(D) Activity C and D both are completed the network?
Construction Materials and Management Test 2 | 3.57

D c. Item rate 3. Adopted for buildings, roads,


3 6
4 H contract bridges and electrical works.
B
E 4 8 d. Labour contract 4. Petty works and regular
2
I maintenance work.
A
1 2 5 8
5 7 Codes:
6 2 a b c d
F J
C 2
(A) 1 2 3 4
3 7 (B) 2 3 4 5
4
G (C) 5 4 3 2
(D) 4 3 2 1
(A) 1 - 2 - 4 - 7 - 8 (B) 1 - 2 - 3 - 5 - 8
17.
(C) 1 - 2 - 4 - 5 - 8 (D) 1 - 2 - 3 - 6 - 8
List – I List – II
1 4. For a given probability distribution curve,
1. Setting time of cement a. Le- chatelier’s apparatus
2. Consistency of cement b. Air Permeability test
3. Soundness of cement c. Vicat apparatus
4. Fineness d. Pycnometer
Probability

Choose the correct one from the following


(A) 1 - c, 2 - c, 3 - b, 4 - a
(B) 1 - c, 2 - a, 3 - a, 4 - d
(C) 1 - c, 2 - c, 3 - a, 4 - b
4 8 12
(D) 1 - c, 2 - a, 3 - a, 4 - d
Duration (in days) 18.
Type of work Slump recommended
Match the following:
(i) Optimistic time (a)  12 1. Concrete for road works 20 - 28mm
(ii) Most likely time (b)  4 2. Ordinary RCC work 50 - 100mm
(iii) Expected time (c) 8 3. Mass concrete 75 - 175mm
(iv) Pessimistic time 4. Columns – retaining walls 12 - 25mm
(A) i - b, ii - b, iii - c, iv - a
(B) i - a, ii - b, iii - b, iv - c Which of the following pairs are correctly matched
(C) i - a, ii - c, iii - c, iv - b (A) 1,3 and 4 (B) 1 and 3
(D) i - b, ii - c, iii - c, iv - a (C) 3 and 4 (D) 2 and 4
1 5. For the path of a certain network shown below, the 19. The fineness modulus of C.A and F.A are given as 7.6
expected time and standard deviation will be _____ and 2.78 respectively. The economical value of fineness
3 – 5 – 10
modulues of combined aggregate is 6.4, then the pro-
10 20 portion of fine aggregate is _______
4 – 9 – 11 (A) 25% (B) 33.33%
40 50
(C) 50% (D) 66.67%
0–0–0 20. Considering following strengths of concrete, choose
3–6–9 the correct sequence in increasing order.
(1) Cube strength
30
(2) Cylinder strength
(3) Split tensile strength
(4) Modulus of rupture
(A) 15 and 1.75 (B) 20 and 1.93
(A) 3,4,2,1 (B) 3,4,1,2
(C) 15 and 1.5 (D) 20 and 1.75
(C) 4,3,2,1 (D) 4,3,1,2
1 6. Match List – I with List – II
21. Number of bricks required for one cubic meter of brick
List – I List – II masonry is ________.
a. Piece work 1. Not practiced in government (A) 450 (B) 500
contract (C) 550 (D) 600
b. Lump sum 2. Payment made by detailed 22. UPV method of non – destructive testing for concrete is
contract measurement of different items used to determine ______
3.58 | Construction Materials and Management Test 2

(1) Compressive strength Activity duration is 6. Then, Match the List - I, with
(2) Existence of voids List - II
(3) Tensile Strength List – I List – II
(4) Static Modulus of concrete
a. Free float 1.  5
(5) Dynamic modulus of concrete
b. Total float 2.  9
(A) 1, 2, 3 and 4 (B) 1 and 3 only
(C) 2 and 5 only (D) 3 and 5 only c. Interfering float 3. 13

23. In shape test of aggregate which one of the following d. Independent float 4. 18
gives the correct slot for flakiness index for a material Codes:
passing 50 mm sieve and retained on 40 mm sieve? a b c d
(A) 25 mm (B) 27 mm (A) 4 3 1 2
(C) 81 mm (D) 30 mm (B) 3 4 1 2
2 4. Identify the sequence of determination of components (C) 3 4 2 1
of a concrete mix as per IS method of mix design Select (D) 4 3 2 1
the correct answer using codes given below. 2 8. Match the List - I with List - II
List – I List – II List – I List – II
a. Cement content 1. First step a. PERT 1. Activity based
b. Aggregate content 2. Second step b. Node 2. Event oriented
c. Water content 3. Third step c. Dummy 3. End of job
d. Water cement ratio 4. Fourth step d. CPM 4. Imaginary activity

a b c d (A) a - 1, b - 4, c - 3, d - 2
(A) 1 2 3 4 (B) a - 2, b - 4, c - 3, d - 1
(B) 3 2 4 1 (C) a - 2, b - 3, c - 4, d - 1
(C) 1 4 2 3 (D) a - 1, b - 3, c - 4, d - 2
(D) 3 4 2 1 29.
4 2
2 5. The relation between the strength of brick masonry fw, 3 5 6
the strength of bricks fb, and the strength of mortar fm is 3
given by (where kw is coefficient based on layout of the
2
bricks and the joints) 1 2 3
f f
(A) fw = fW = KW b
k w b (B) 3
fm fm
4
(C) fW = k w , f b f m (D)
fW = KW fb . fm In the network shown, total float for the activities 2 - 4
and 3 - 5 are respectively
26. Match List - I (Wood elements) With List - II (A) 0 and 0 (B) 2 and 2
(Description) (C) 2 and 1 (D) 1 and 1
List – I List – II 3 0. Match List - I with List - II
a. Pith 1. Inner most portion of the tree List – I List – II
b. Sap wood 2. Inner annual rings (Cement mortar for (Cement : Sand in
Surrounding the pith different works) mortar)
c. Heart wood 3. Outer most annual rings a. For normal brick work 1. 1:4
d. Cambium layer 4. Thin layer of Sap between b. For plastering work 2. 1:3
Sapwood and inner bark
c. For grouting the 3. 1:6
cavernous rocks
Codes:
a b c d d. For guniting 4. 1 : 1.5
(A) 1 3 2 4 Codes:
(B) 2 4 1 3 a b c d
(C) 1 4 2 3 (A) 3 2 4 1
(D) 2 3 1 4 (B) 4 1 3 2
2 7. For an activity i – j, the EST,EFT, LST and LFT are (C) 4 2 3 1
given as are 5,24,9, and 29, respectively. (D) 3 1 4 2
Construction Materials and Management Test 2 | 3.59

Answer Keys
1. C 2. A 3. B 4. D 5. A 6. D 7. C 8. C 9. B 10. C
11. B 12. C 13. C 14. D 15. B 16. D 17. C 18. B 19. B 20. A
21. B 22. C 23. B 24. D 25. D 26. A 27. B 28. C 29. C 30. D

Hints and Explanations


1. Beta distribution Choice (C) 13.
2. 7/8 11/12
D
3 6
4
B H
4 8
2 E
0/0
A 5/5 I
Probability

1 2 5 8 20/20
5 7
6 J
C 2
2 J

4 3
7
t0 tL–tE tP G
11/11 14/18
Duration (in days)
Critical path : 1 – 2 - 4 – 5 – 8 Choice (C)
tE = Expected time
14.
So, 50% Choice (A)
3. M10 – 1:3:6
M15 – 1:2:4
M20 – 1:1.5:3 Choice (B)
Probability

4. Negative slacks are occur when there is a deficiency in


available resources. Choice (D)
5.
7/
3 t0 tm tP
8 Duration (days)
6 C
0/ B 16/ 18/18
1/
4 E
1
1
2 5
2
6 to + 4tm + t p to + 4tm + t p 4 + 4 (8) + 12
A G te = = = 8 days
D F 6 6 6
9 5
\ Expected time = 8days Choice (D)
4
11/
15.
3 – 5 – 10
10 20
To find the duration of project, forward computa- 4 – 9 – 11
tion is enough. No need of backward computation of 40 50
durations.
So, total duration of the project is 18 days. Choice (A) 0–0–0
3–6–9
6. To start one particular activity, the preceding activities
must get completed. So to start an activity F, both C
30
and D activities are need to get completed. Choice (D)
11. To make 1m3 of wet concrete, 1.5 m3 of dry concrete is
required. t 0 + 4t m + t p
te =
Volume of C.A = proportion of C. A × 1.5 m3 6
4  3 + 20 + 10   3 + 24 + 9 
= 7 × 1.5= 0. 857m3 Choice (B) Expected time te =   +  +
 6  6
12. Correct sequence is:  4 + 36 + 11 120
  = = 20days
2 -1 -4 -3 – 5 – 9 – 8 – 7 – 6 Choice(C) 6 6
3.60 | Construction Materials and Management Test 2

 t p − to 
2 50 + 40
variance σ = 
2 23. Mean dimension = = 45mm
 6  2
2 2 2 3
 7  6  7 Slot for flakiness index = × 45 = 27mm Choice (B)
S.D =   +   +   = 1.93 Choice (B) 5
6 6 6
16. A – 4, B – 3, C – 2, D – 1 Choice (D) 27. Given, TEi = 5 ; TEj = 24 , TLi = 9, TLi = 29 and t ij = 6
17. 1 – c, 2 – c, 3 – a, 4 – b Choice (C) Total float = (LFT – EST) – Duration
18. Columns retaining walls - 75 -150mm = TLj − TEi − t ij = (29 – 5) – 6 = 18
Mass concrete – 25 -50mm Free float = TEj − TEi − T ij
Vibrated concrete – 12 -25mm Choice (B)
Free float = (24 – 5) – 6 = 13
19. Proportion of F.A in Interfering float = Total float – Free float = 18 – 13 = 5
F.M of C.A-Desired F.M Independent float = (TEi − TLi ) − t ij
= × 100
Desired F.M-F.M of C.A =
24 – 9 – 6 = 9 Choice (B)
7.6 − 6.4 29.
Proportion of F.A = × 100 = 33.33% 5/5 10/10 12/12
6.4 − 2.78 4 2
3 5 6
 Choice (B)
3
20. Cube strength > Cylinder strength > Modulus of rup- 0/0 2/2
ture > Split tensile Strength Choice (A) 2
1 2 3
2 1. Nominal size of brick is 19 × 9 × 9cm
With mortar in brick masonry, size = 20 × 10 × 10cm 3
\ Volume of one brick in Masonry 4
= 0.2 × 0.1 × 0.1 m3 7/7
= 2 × 10–3 m3
For an activity 2 – 4,
For 1m3; Number of bricks required is
Total float = (7 – 2) – 3 = 2
1 For an activity 3 – 5,
= = 500 Choice (B)
2 × 10 −3 Total float = (10 – 5) – 4 = 1 Choice (C)
Unit V
Concrete Structures
This page is intentionally left blank
Concrete Structures Test 1
Number of Questions: 25 Time: 60 min.

Directions for questions 1 to 25: Select the correct alterna- 8. What is the value of minimum percentage of reinforce-
tive from the given choices. ment in case of Fe 250 steel in slabs?
1. The following two statements are made with reference (A) 0.1% (B) 0.12%
to a simply supported Under Reinforced RCC beam. (C) 0.2% (D) 0.15%
I.  Steel reaches ultimate stress prior to concrete 9. Minimum grades of concrete to be used for pre ten-
reaching ultimate stress. sioned and post tensioned structural elements are
II. There is a shift in neutral axis upwards as the load respectively [As per IS1343-1980]
is increased. (A) M30 and M40
(A) Both the statements are false (B) M40 and M30
(B) I is true but II is false (C) M20 and M30
(C) I is false but II is true (D) M30 and M30
(D) Both statements are true.
10. Which of the following are subjected to primary
2. As per the provisions of IS 456-2000, the (short torsion?
term) modulus of elasticity of M40 grade concrete (in (A) Isolated L-beam
N/mm2) can be assumed to be (B) Ringbeam of circular water tank
(A) 31600 (B) 28500 (C) A and B
(C) 30000 (D) 36000 (D) Grid system
3. As per IS 456-2000, the effective length of column in a 11. A singly reinforced rectangular section of 300mm wide
Reinforced concrete building frame is independent of and 550 mm effective depth, reinforced with 3 bars of
(A) height of column 16 mm diameter Fe415 steel bars. The concrete used is
(B) loads acting on frame M20 grade. The ultimate moment of resistance of beam
(C) frame type is _____ (in kN-m)
(D) span of beam (A) 110 (B) 150
4. Maximum strains in an extreme fiber in concrete and in (C) 120 (D) 100
the tension Reinforcement (Fe-250 grade and Es = 200 12. A reinforced beam of size 300 mm width and 700
kN/mm2) in a balanced section at limit state of flexure mm overall depth is subjected to a service moment of
are respectively 80 kN-m. If M25 and Fe 415 is used, it is be designed
(A) 0.0035 and 0.0041 as (use effective cover = 50 mm)
(B) 0.002 and 0.0038 (A) doubley reinforced section
(C) 0.0035 and 0.0030 (B) singly reinforced section
(D) 0.002 and 0.0018 (C) over reinforced section
5. An isolated foot bridge has a slab of 4 m width. The (D) None
central supporting beam is of 8 m length, width of web- 13. The effective flange width of T-beams spaced at 4 m
350 mm. The effective width of flange is, with web depth of 1.2 m, web width of 0.5 m spanning
(A) 1.25 m (B) 1.68 m 10 m with a flange slab of 150 mm thickness is
(C) 1.75 m (D) 2.0 m (A) 4 m (B) 2 m
6. The percentage of minimum shear Reinforcement as (C) 3 m (D) 5 m
per IS456 using HYSD bars of Fe 415 grade is
14. An RC beam of 350 mm width and effective depth 550
(A) 0.11% (B) 0.15%
mm is subjected to a factored shear force of 120 kN.
(C) 0.3% (D) 0.2%
M20 grade concrete is used for the beam. For shear
7. Which of the following is the correct expression to esti- reinforcement 6 mm diameter two legged mild steel
mate the development length of deformed reinforced stirrups are used. The spacing of shear reinforcement
bars used in compression as per IS456 in limit state of in beam is [take τc,max = 2.8 MPa and τc = 0.60 MPa]
design? (A) 1500 mm (B) 410 mm
φσ s φσ s (C) 90 mm (D) 300 mm
(A) (B)
8τ bd 4 τ bd 15. A bar of 12 mm diameter is embedded in concrete for
a distance of 15 cm. Calculate the maximum load
φσ s φσ s
(C) (D) which the bar can take if bond stress is not to exceed
64 τ bd 5τ bd 0.5 N/mm2?
3.64 | Concrete Structures Test 1

(A) 1.4 KN (B) 3.5 KN Assume the specific gravity of OPC, sand and coarse
(C) 2.0 KN (D) 2.8 KN aggregate to be 3.0, 2.68 and 2.70 respectively. The air
16. The minimum extension of steel bars of 12 mm diame- content in liters/m3 is ______
ter bar of Fe250 grade steel in M25 grade concrete with (A) 30 (B) 40
a design bond strength of 1.6 MPa and 135° standard (C) 50 (D) None
bend at the end is 22. A column of size 300 × 550 mm has unsupported
(A) 410 mm (B) 265 mm length of 4.0 m and is braced against side sway in both
(C) 150 mm (D) None directions. According to IS456-2000, the minimum
17. A reinforced concrete beam of 12 m effective span and eccentricities (in mm) with respect to major and minor
1.5 m effective depth is simply supported. If the total principle axes are
udl on the beam is 5 kN/m, the design shear force for (A) 18 mm, 26.33 mm
the beam is (B) 26.33 mm, 18 mm
(A) 22.5 KN (B) 50 KN (C) 20 mm, 18 mm
(C) 30 KN (D) 40 KN (D) 26.33 mm, 20 mm
18. At the limit state of collapse, an RCC beam is subjected 2 3. A simply supported reinforced concrete beam of length
to total flexural moment of 300 kN-m, shear force of 12 m sags while undergoing shrinkage. Assuming
30 kN, torque of 18 kN-m, the beam is of 350 mm wide a uniform curvature of 0.006 m–1 along the span, the
and 450 mm gross depth with an effective cover of maximum deflection at mid span is
30 mm. The equivalent nominal shear stress (τve) as cal- (A) 0.20 m
culated using the code turns out to be lesser than the (B) 0.11 m
design shear strength (τc) of concrete. The equivalent (C) 0.30 m
shear force is (D) 0.25 m
(A) 30 KN 2 4. In a pre stressed concrete beam section shown in fig-
(B) 115 KN ure. The net loss is 15% and the final prestressing force
(C) 80 KN applied at y is 700 KN. The initial fiber stresses (in
(D) 200 KN N/mm2) at the top and bottom of beam were:
19. Based on the above data, the equivalent bending 300 mm
moment (Me1) for designing the longitudinal tension
steel is 300 mm
(A) 400 KN-m
(B) 300 KN-m
(C) 325 KN-m 300 mm Y
(D) 18 KN-m 150 mm
20. A rectangular column of 350 mm × 650 mm is rein-
forced with 0.8% reinforcement based on gross area. (A) + 49 and 58
Fe500 steel and M25 grade concrete is used. The ulti- (B) – 58 and – 49
mate load carrying capacity of column is (C) – 49 and 58
(A) 4.8 MN (D) – 58 and 49
(B) 2.8 MN 2 5. A concrete column carries an axial load of 350 KN and
(C) 3.2 MN a bending moment of 45 KN-m at its base. An isolated
(D) 5.6 MN footing of 3 m × 5m with 5m side along the plane of
21. The composition of an air entrained concrete is given bending moment, is provided under column. The C.G.
below: of column and footing coincides, the net maximum
Water : 180 kg/m3 and minimum pressures in KN/m2 on the soil under the
Ordinary Portland cement: 360 kg/m3 footing are respectively?
Sand : 600 kg/m3 (A) 150, 98 (B) 100, 75
Coarse aggregate : 1200 kg/m3 (C) 150, 120 (D) 134, 98

Answer Keys
1. D 2. A 3. B 4. C 5. B 6. A 7. A 8. D 9. B
10. C 11. A 12. B 13. C 14. C 15. D 16. B 17. A 18. B 19. B
20. B 21. A 22. D 23. B 24. C 25. D
Concrete Structures Test 1 | 3.65

Hints and Explanations

2. Ec = 5000 fck  = 5000 40 Calculation of xu:


0.36 fckbxu = 0.87 fyAst
   = 31622 N/mm2 Choice (A)
π
4. Maximum strain in steel at extreme tension fiber 0.87 × 415 × 3 × × 162
0.87 fy Xu = 4
≤ 0.002 + 0.36 × 20 × 300
Es
0.87 × 250 Xu = 100.82 mm
= 0.002 + Xu < xumax ⇒ under reinforced section
2 × 105
∴ M = T× lever arm
= 0.0030.
Miximum strain in concrete = 0.0035 Choice (C) Mu = 0.87 fyAst (d – 0.42 xu)
π
o Mu = 0.87 × 415 × 3 × × 162 (550 – 0.42 × 100.82)
5. bf = + bw ≤ b 4
o
+4 Mu = 110.55 KN-m Choice (A)
b
12. b = 300 mm;
D = 700 mm; d = 650 mm
lo : 8m
Design moment, Mu = 80 × 1.5 = 120 KN-m
fck = 25 MPa
fy = 415 MPa
b : 4m d1 = 50 mm
Mu, limit = 0.138 fck bd2 = 0.138 × 25 × 300 × 6502
Isolated T-beam : so use the above formula = 437.28 KN-m
8 8 Mu ≤ Mu, lim ⇒ singly Reinforced section. Choice (B)
bf = + 0.35  = + 0.35
8 6
+4 13. C = 4 m
4
bw = 0.5 m
bf = 1.68 m Choice (B) Df = 150 mm
o = 10 m
Asv 0.4
6. =  10
b × sv 0.87 fy bf = o + bw + 6Df ≤ c = 4 m = + 0.5 + 6 (0.15)
6 6
0.4
% minimum shear Reinforcement = × 100 bf = 3.06 m Choice (C)
0.87 × 415
= 0.110% 14. b = 350 mm
 Choice (A) d = 550 mm
φσ s Vu = 120 KN
7. Ld = fck = 20 MPa
4 × τ bd τcmax = 2.8 MPa
For HYSD bars increase τbd by 60% and for compres- τc = 0.60 MPa
sion increase τbd by 25% τv = ?
Nominal (average) shear stress due to external load is
φσ s φσ s
Ld = =  Choice (A) V 120 × 103
4 × 1.6 × 1.25τ bd 8τ bd τv = u =
bd 350 × 550
11. Given τv = 0.623 N/mm2
Singly Reinforced section τv< τcmax ⇒ No. diagonal compression failure.
b = 300 mm τv > τc ∴ not safe in shear
d = 550 mm
π Shear reinforcement is required for
Ast = 3 × × (16)2 Vus = vu – τc (bd)
4
   = 120 × 103 – 0.60 (350 × 550) = 4500 N
fy = 415 N/mm2
Vus = 4.5 KN
fck = 20 N/mm2
Mu= ? 0.87 fy Asv d
Vus =
Xumax = (0.48) (d) = 0.48 × 550 = 264 mm sv
3.66 | Concrete Structures Test 1

π 2 Critical section for shear is at a distance ‘d’ from


0.87 × 250 × 2 × × 6 × 550 support
4500 N = 4
wl 5 × 12
sv ∴ Design shear force = – wd = – 5(1.5)
2 2
Sv = 1503 mm
F = 22.5 kN Choice (A)
Sv based on minimum shear RFM
Asv 0.4 18. Mu = 300 kN-m
= Vu = 30 kN
bsv 0.87 fy
Tu = 18 kN-m
( Asv )(0.87 fy ) b = 350 mm
Sv = D = 450 mm
(0.4)(b)
d1 = 30 mm
 π 2 d = 420 mm
 2 × × 6  (0.87 × 250) τve < τc
4
Sv = Equivalent shear force;
0.4 × 350
1.6Tu 1.6 × 18
Sv = 87.85 mm Ve = Vu + VT = Vu + = 30 +
Sv = 0.75 × 550 = 412.5 mm b 0.35
Sv should be minimum of Vu = 112.28 KN Choice (B)
(1) sv calculated based on vertical stirrups 19. Longitudinal tension steel is based on Mu only.
(2) sv based on minimum shear reinforcement [ignoring the effect of torsion] If τve<τc
(3) ≤ 0.75 d ∴ Me1 = Mu = 300 KN-m Choice (B)
(4) 300 mm
∴ sv = 87.85 mm Choice (C) 20. b = 350 mm
D = 650 mm
15. τbd = 0.5 N/mm2 Asc = 0.8% (Ag)
Ld = 15 cm 0.8
φ = 12 mm     = (350× 650)
P=? 100
P = (τbd) (∏φLd) Asc = 1820 mm2
  = (0.5) (π × 12 × 150) Ac = Ag – Asc
P = 2827.43 N    =(350 × 650) – (1820)
P = 2.82 kN Choice (D) Ac = 225680 mm2
16. φ = 12 mm Pu = (0.4 fckAc) + (0.67 fyAsc)
fy = 250 MPa    =(0.4 × 25 × 225680) + (0.67 × 500 × 1820)
fck = 25 MPa Pu = 2866 KN
τbd = 1.6 MPa Pu = 2.8 MN Choice (B)
135° bend
(Ld) required = (Ld)straight bar – anchorage value Mc M s M a
21. + + + vw + va = 1
ρc ρs ρa
φσ s 12 × 0.87 × 250
(Ld)straight bar = =
4 τ bd 4 × 1.6 360 600 1200 180
+ + + + va = 1
=407.81 mm 3 × 1000 2.68 × 1000 2.7 × 1000 1000
Anchorage value for a 135° bend Va = 0.0316
= 12φ = 12 × 12 = 144 mm    =0.0316 × 1000
(Ld)required = 407.81 – 144    =31.67 Choice (A)
= 263.81 mm Choice (B) L b
22. emin = + or 20 mm which ever is more
17. L = 12 m 500 30
d = 1.5 m 4000 550
exx = + = 26.33 mm
5 kN/m 500 30
4000 300
eyy = + = 18.0 mm
500 30
12 m
d Subject to a minimum of 20 mm,so, take 20 mm.
 Choice (D)
Concrete Structures Test 1 | 3.67

d2 y 1.15 × 700 × 103 1.15 × 700 × 103 × 150


23. EI = Mx   = ± × 300
dx 2 300 × 600 3003
300 ×
d2 y Mx 12
=
dx 2 EI   =4.47 ± 53.66
  =– 49.18 and 58.13 (in N/mm2) Choice (C)
d2 y
=ρ 2 5. P = 350 KN
dx 2
M = 45 KN-m
dy M 45
= ρx e= = = 0.128 m
dx P 350
ρx 2 σ max  P P.e
y=
2   = ±
σ min  A z
2
 L P P.e(6)
(0.006)   ±
 2   =
y = A bd 2
L
at x =
2
2 P  6.e 
  = 1±
(0.006) (6)2 bd  d 
    = = 0.108 m Choice (B)
2 350  6 × 0.128 
  = 1±
24. Pi = 1.15 × 700 = 805 KN 3 × 5  5 
P M σmax = 134.58 kN/m2
σi = ± ×y
A I σmin = 98.74 kN/m2 Choice (D)
Concrete Structures Test 2
Number of Questions: 30 Time: 75 min.

Directions for questions 1 to 30: Select the correct alterna- 10. Calcium lignosulphate is an example of
tive from the given choices. (A) retarder (B) accelerator
1. Minimum Grade of concrete for structural purpose (C) dispersal agent (D) hardness agent
(A) M15 (B) M20 11. If the following figure represents the idealized σ – ε
(C) M25 (D) M30 curve of concrete in compression.
2. Balanced Neutral axis depth for a singly reinforced
cross section depends on
Compression
(A) Grade of concrete fck
(B) Grade of steel
(C) Amount of steel reinforcement σ
(D) All the above
ε 0.0035
3. A rectangular concrete beam of width 230 mm and
effective depth 300 mm is reinforced with 4 – 12 mm Then the σ – ε curve of concrete in Tension is
bars in tension zone. M20 Grade concrete and Fe 415
steel are used. Find the Moment of Resistance of the
1
beam. fck
10
(A) 42.23 KNm (B) 35.26 KNm
(C) 64.13 KNm (D) 72.54 KNm
σ
(A)
4. Minimum percentage of steel in both directions in a ε
slab when HYSD bars are used is _____% of (bD)
(A) 0.1% (B) 0.15%
(C) 0.12% (D) 0.2% 1
fck
10
5. In design for shear in Reinforced concrete structures,
which of the following is considered explicitly. σ
(A) Dowel action (B)
ε
(B) Aggregate interlocking
(C) Concrete in compression zone
ε
(D) All the above (C)
σ
6. As per IS: 456 – 2000, span/depth ratio of two way sim-
ply supported slabs is
(A) 40 (B) 30 1 fck
10
(C) 20 (D) 35
7. A hook of Fe 415 Grade is provided in Compression ε
(D)
in M20 Grade Concrete with tbd = 1.2 MPa, Then the σ
development length Ld = ______ φ
(A) 44.17 (B) 37.6
1 fck
(C) 21.61 (D) Not allowed (hook) 10
8. In a plain concrete pedestal of M35 Grade, the max-
imum bearing pressure at the base is found to be Common Data for Questions 12 and 13:
40 N/mm2. Find the depth of footing if the projection A reinforced concrete beam, size 250 mm wide and
beyond the column is 300 mm 400 mm deep effective is simply supported over a span of 6 m.
(A) 3.1 m (B) 2.6 m It is subjected to a point load of P at centre of the beam. The
(C) 2.4 m (D) 1.9 m. point load is increased gradually. Beam is reinforced with
9. In the limit state design of serviceability the deflection 5 HYSD bars of Fe 415 grade of 12 mm diameter placed
after erection of partitions and erection of finishes is at an effective cover of 40 mm on bottom face and nom-
limited to inal shear reinforcement. The characteristic compressive
(A) span/250 (B) san/325 strength and bending tensile strength of concrete are 20MPa
(C) span/350 (D) span/150 and 2.2MPa respectively.
Concrete Structures Test 2 | 3.69

12. Ignoring the pressure on tension reinforcement, find (A) a1 = 1.82 m and a2 = 2.14 m
the value of load P in KN when the first flexural crack (B) a1 = 2.14 m and a2 = 1.82 m
will develop on the beam (C) a1 = 1.24 m and a2 = 2.72 m
(A) 26.4 KN (B) 11.83 KN (D) a1 = 2.72 m and a2 = 1.24 m
(C) 16.4 KN (D) 35.5 KN 18. A reinforced concrete beam of 12m effective span and
13. The theoretical failure load of the beam for attainment 1 m effective depth is simply supported. If the total udl
of limit sate of collapse in flexure is of the beam is 10MN/m the design shear force for the
(A) 33 KN (B) 48 KN beam is
(C) 52 KN (D) 64 KN (A) 20MN (B) 30MN
14. A reinforced beam of size 230 mm width and 350 mm (C) 40MN (D) 50MN
overall depth is subjected to a working moment of 19. A column is of 500 × 300 mm and unsupported length
65 KNm. If M20 grade concrete and Fe415 grade of 3 m. The design criteria of the column as per IS 456-
steel are used, it is to be designed as (effective cover = 2000 will be.
50 mm) (A) short along long and short dimensions.
(A) Balanced section (B) long along short and short along long dimension
(B) Singly reinforced section (C) long along long and short dimensions
(C) Doubly reinforced section (D) long along long and short along short dimensions
(D) Over reinforced section 20. The factored load carrying capacity of a column of 300
15. A hall of 10 m × 24 m consists of a number of beams mm × 500 mm size with minimum percentage of steel
4 m centre to centre parallel to the shorter span of is (M20 and Fe415)
the hall. Width of web = 300 mm. Thickness of slab (A) 1234KN
120 mm, the beams are cast monolithic with the col- (B) 1468KN
umns at their ends. The effective width of flange of an (C) 1524KN
intermediate beam is (D) 1632KN
(A) 1.52m (B) 0.94m 21. Match the following with reference to RCC.
(C) 2.01m (D) 2.19m
List – I List – II
16. Match List – I with List – II and select the correct
answer from the codes given below. a. Torsional Analysis 1. Truss Analogy
b. Shear stress Analysis 2. Skew Bending Theory
List – I (Admixtures) List – II (Example)
c. Limit state method 3. Semi Probabilistic
P. Retarder 1. Volcanic Tuff Approach
Q. Accelerator 2. Natural wood resins d. Working stress method 4. Deterministic
R. Pozzolona 3. Calcium sulphate Codes:
S. Air entraining agent. 4. Calcium chloride a b c d
Codes: (A) 1 2 3 4
P Q R S (B) 2 1 3 4
(A) 1 2 3 4 (C) 2 1 4 3
(B) 1 2 4 3 (D) 1 2 4 3
(C) 3 4 1 2 Common Data for Questions 22 and 23:
(D) 2 1 3 4 A post tensioned concrete beam 120mm wide and 300mm
1 7. Two columns A and B carrying loads are shown in figure deep is prestressed by three cables each with a cross sec-
below with different notations. Soil bearing capacity of tional area of 60 mm2 and with an initial stress of 1100 MPa.
soil qo = 200 KN/m2. width of footing for both columns All the three cables are straight and located 100mm from
is 2.4 m. Find a1 and a2 shown in figure (assume footing the sofit of the beam. If the modular ratio is 6, the loss of
weight as 10% of column load) stress due to elastic shortening in the beam
1800 KN 1500 KN 22. When simultaneous tensioning and anchoring of all the
three cables is done will be.
(A) 24.51 MPa (B) 43.92 MPa
a1 A B a2
a = 3.6 m (C) 78.26 MPa (D) zero
23. When successive tensioning of the three cables are
done (one at a time)
qo (A) 24.51MPa (B) 43.92MPa
(C) 78.26MPa (D) zero
3.70 | Concrete Structures Test 2

24. Consider the following statements regarding the Effective cover = 50mm. If 3 - 20φ bars reinforcement
PORTLAND PUZZOLANA CEMENT is provided and M20 Grade concrete and Fe415 steel
(1) It produces less heat of hydration are used. Find the moment of Resistance of the beam.
(2) Addition of Pozzolano does not contribute to the (A) 92KNm (B) 106KNm
strength at early stages (C) 114KNm (D) 138KNm
(3) Strength of this cement at any time is always less 28. A concrete column carries an axial load of 500 KN and
than the strength of the Portland cement a bending moment of 50 KN–m at its base. An isolated
(4) It is particularly useful in marine and hydraulic footing of 2 m × 3m, with 3m side along the plane of
construction bending moment is provided under column. The CG of
(A) 1, 2, 3 are correct (B) 2, 3, 4 are correct column and footing coincides. The net maximum and
(C) 1, 3, 4 are correct (D) 1, 2, 4 are correct minimum pressures in KN/m2 on the soil under the
25. A reinforced concrete beam of rectangular cross section footing are respectively
of breadth 300 mm and effective depth 500 mm is sub- (A) 100 and 66.67 (B) 95 and 55.32
jected to maximum factored shear force of 400KN. The (C) 72 and 46.18 (D) 120 and 75
grades of concrete and steel are M25 and Fe415 respec- 2 9. Unfactored maximum bending moments at a section of
tively. Based on the area of main steel provided grade of a reinforced concrete beam resulting from a frame anal-
concrete the design shear stress tc as per IS 456 : 2000 ysis are 330, 420 and 150KN-m under dead, live and
is 0.64N/mm2. If 2 - 16φ bars are used as bent up bars wind loads respectively. The design moment(KN – m)
α = 50°) Design shear reinforcement spacing of as per IS 456 : 2000 for the limit state of collapse is
(8 mm φ) 2 legged vertical stirrups (tcmax = 3.1 N/mm2) (A) 720KN – m (B) 840KN – m
(A) 90 mm (B) 110 mm (C) 1125KN – m (D) 1530KN – m
(C) 130 mm (D) 280 mm
3 0. In under reinforced concrete beam, which of the fol-
26. A rectangular beam of 500 mm × 700 mm with effec- lowing statements are correct?
tive cover of 40mm is subjected to a factored values 1. Actual depth of neutral axis is less than the critical
of shear force 12KN, Bending Moment 150KNm and depth of neutral axis
a torsional moment 15KNm. Find the design bending 2. Concrete reaches ultimate stress prior to steel
moment for the design in KN- m. Use share resistance reaching the ultimate stress.
of the cross section tc = 1.3MPa. 3. Moment of resistance is less than that of balanced
(A) 100 (B) 114 section
(C) 150 (D) 171 4. Lever arm of resisting couple is less than the bal-
27. A T – beam roof section has the following particulars. anced section.
Width of flange = 600 mm, Thickness of slab = 120 mm. (A) 1 and 2 only (B) 1 and 3 only
Width of web = 250 mm depth of web = 300mm (C) 2, 3 and 4 (D) 1, 2 and 4

Answer Keys
1. B 2. B 3. A 4. C 5. A 6. D 7. D 8. A 9. C 10. A
11. C 12. B 13. B 14. C 15. D 16. C 17. B 18. D 19. A 20. C
21. B 22. D 23. B 24. D 25. A 26. C 27. C 28. A 29. C 30. B

Hints and Explanations


1. 2. Neutral axis can be found from either s diagram or
M5 e diagram.
M7
M10 Non – structural purpose
By considering s diagram:
M15 Compressive force = tensile force
M20
:M25 0.36 fck bx = 0.87 fy Ast
:
M55  0.87 f y Ast 
: X =  .d   --------- (1)
: Structural purpose  0.36 f ck (bd ) 
M80
By considering e diagram
M 20 can be used for both structural and non structural
purpose. Minimum Grade for structural purpose starts εc εs
=
from M 20. Choice (B) x (d − x)
Concrete Structures Test 2 | 3.71

0.87 fy
+ 0.002
0.0035 Es 300
=
x d−x
d α
On simplifying
  q0
 0.0035 
X =  d  -------- (2) 100 × 40
 0.005 + 0.87 fy  tana ≥ 0.9 +1
 Es  30
d
From (1) and (2) equations tana =
We can say that 300
N.A depends on only d 100 × 40
≥ 0.9 +1
(i) Grade of steel 300 30
(ii) Depth of cross section. Choice (B) d
≥ 10.43
π 300
0.87 f y Ast 0.87 × 415 × 4 × × 122
3. X =   =   4  = 98.63 mm d ≥ 3129 mm
0.36 f ck b 0.36 × 20 × 230   ≥ 3.13 m
Balanced Neutral axis for Fe415 Grade \ d ≃ 3.1 m. Choice (A)
Xb = 0.48 × d = 0.48 × (300) = 144 mm 9. The final deflection of horizontal members should not
X < xb ⇒ under reinforced. exceed span/250.
\ M = T. a = (0.87 fy Ast) (d – 0.42x) The deflection after construction of partitions or appli-
π cation of finishes should not exceed span/350 or 20 mm
   = 0.87 × 415 × × 4 ×122 whichever is less. Choice (C)
4
× (300 – 0.42 × 98.63) 10. Calcium lignosulphate is an example of retarder.
   = 42.23 × 106 N mm  Choice (A)
   = 42.23 KNm. Choice (A) 11. If compressive stress is considered positive, Tensile
4. Minimum % of steel in slab stress is considered negative
= 0.15% b D (for Mild steel) If strain in compression is positive, strain in tension is
= 0.12% b D (for HYSD bars). Choice (C) negative
1
5. Aggregate interlocking and concrete in compression Tensile stress in concrete = (compressive stress in
10
Zone effect are considered implicitly 1
But dowel action is taken explicitly like in inclined bars concrete) =  fck
10
d
Vus = 0.87 fy Asv (sin b + cos b). Choice (A) In compression dl is reduction, In tension dl is
Sv
increasing.
20(beams) \ if s – e of compression is in I quadrant
span same as beam for one ways labs s – e of tension is in III quadrant.

6. =  .
depth
35(two way simply supported slabs) ε
40(2way continuous slabs) σ

 Choice (D) 1
fck
10
7. In actual sense for every 45° bend, anchorage value  Choice (C)
= 4f 12.
250
And for hook it is 180° = 4(45°)
\ (Ld)req = Ld – 4(4f)
But maximum angle of bend in HYSD is only 135°
440
\ Hook is not allowed with HYSD bars. Choice (D)
100 qo
8. tana ≥ 0.9 +1 d = 400 mm
f ck
D = 400 + 40 = 440 mm
q0 = 40 Mpa b = 250 mm
fck = 35 Mpa l=6m
3.72 | Concrete Structures Test 2

P Mu = 65 k N m
 /2  /2 Mu > Mu limit
\ Design as doubly reinforced beam. Choice (C)
15.

10 m
M = P /2 4 4 4 4 4 4
x x x x x

From bending equation Beam monolithic with columns means beam are fixed
M f to columns
=
I y L0 = 0.7l = 0.7 × 10 = 7m
M 
fcr = bf = 0 + bw + 6Df.
Z 6
M 7
   = + 0.3 + (6 × 0.12)
2.2 = 6
250 × 440 2
bf = 2.19 m 
6 use minimum
bf ≯ C = 4m 
M = 17.75 KN – m
\ bf = 2.19 m. Choice (D)
Bending Moment is maximum at centre.
Therefore flexural crack develops at centre 16. Retarder – calcium sulphate
P Accelerator – calcium chloride
= 17.75 Pozzolone – volcanic Tuff
4
Air entraining agent – Natural word resins.Choice (C)
P ×6
= 17.75 ⇒ P = 11.83 KN. Choice (B) 1 7. a = 3.6 m
4
B = 2.4 m
SBC = q0 = 200 KN/m2
0.87 f y Ast
13. X = P1 × 1.1 P2 × 1.1
0.36 f ck b = 9.9 = A1; = 8.25 = A2
q0 q0
π
0.87 × 415 × 5 ×× 122 A1 A
   = 4 L1 = = 4.125 L2 = 2 = 3.44
0.36 × 20 × 250 B B
X = 113.43 mm L1 + L2
= 3.78 > 3.6 = a
Balanced Neutral axis 2
Xb = 0.48 d = 0.48 × (400) = 192 mm \ combined footting is provided
X < xb ⇒ under reinforced beam ( P1 + P2 ) × 1.1
\ Moment of Resistance A = = 18.15 m2
q0
M = T.a
   = 0.87 fy Ast(d – 0.42x) A
π L = = 7.56 m
B
   = 0.87 × 415 × 5 × × 122 (400 – 0.42 × 113.43)
4 Distance of centroid from column A
   = 72 KNm P2 a
P x = = 1.64 m
P1 + P2
For simply supported with point load at centre M =
4 L
P ×6 a1 + x =
72 = ⇒ P = 48 KN. Choice (B) 2
4
a1 = 2.14 m
14. b = 230 mm a2 = 1.82 m Choice (B)
D = 350 mm 18.
Effective cover d1 = 50 mm
C
Effective depth d= 300 mm
Xu max = 0.48d = 144 mm 10MN/m
A B
Mu limit = 0.36 fck bxumax (d – 0.42 xumax) d
= 0.36 × 20 × 230 × 144 (300 – 0.42 × 144) S
12 m
= 57.12 KNm
Concrete Structures Test 2 | 3.73

Design shear force at critical section. i.e., at d = 1 m vus = Vu – Vc = 400 – 96 = 304 KN


from one end 304KN = Vbentup + Vstirrups
Vu = RA – 10d Vbentup = 0.87 fy Asb × sin a
10 × 12 π
   = – 10 × 1 = 50 MN. Choice (D) = 0.87 × 415 × 2 × × 162 × sin (50°)
2 4
 3000 Vbentup = 111KN
19. = = 10 < 12
b 300 Vstirrrups = 304 – 111 = 193 KN
 3000
= = 6 < 12 304
D 500 Vbentup ≯ 50% (Vus) = = 152KN
2
\ short columns along both directions. Choice (A)
Satisfied
2 0. Minimum % of steel = 0.8% (Ag)
0.8 0.87 fyAsv d
ASC = × (300 × 500) = 1200 mm2 Vstirrups =
100 Sv
Ac = Ag – ASC = 148.8 × 103 mm2 π 2
0.87 × 415 × 2 × × 8 × 500
Pu = 0.4 fck Ac + 0.67 fy ASC 193 × 103 = 4
   = 0.4 × 20 × (148.8 × 103) + (0.67 × 415 × 1200) Sv
   = 1524 × 103N = 1524 KN. Choice (C) Sv ≯ 94 mm
22. ≯ 0.75 d = 375 mm
b = 120 mm ≯ 300 mm
\ 2 legged 8f @ Sv = 90 mm. Choice (A)
1.6Tu
26. Ve = Vu + VT = Vu +
D = 300 mm b
100 mm
1.6 × 15 × 1000
Sofit =
12 + = 60KN
500
In simultaneous tensioning and anchoring, there will
Ve 60 × 103
be no losses tvc = =
\ losses = 0. Choice (D) bd 500 × (700 − 40)
23. prestressing force in each cable =0.182 MPa
P = ss. AS = 1100 × 60 = 66KN tve < tc
P Pe ⇒ Ignore Torsion effect
fc = + (e) \ Mu = 150 KN – m. Choice (C)
AC I
2 7. xb = 0.48d = 0.48 × (420 – 50) = 177.6 mm
66 × 103 66 × 103 × 50 Let NA lies in flange
  = + × 50
120 × 300 120 × 3003 0.36 fck bf x = 0.87 fy Ast
12 π
0.36 × 20 × 600 × x = 0.87 × 415 × 3 × × 202
fc = 2.44 MPa 4
n( n − 1) X = 78.77 mm
Loss = [mfc] X < Df = 120 mm \  NA is with in flange
2
X < xb ⇒ under reinforced
3(3 − 1) M = T.a
Given m = 6 = [ 6 × 2.44]
2    = 0.87 × fy × Ast (d – 0.42x)
= 43.92 MPa. Choice (B) π
   = 0.87 × 415 × 3 × × 202 × (370 – 0.42 × 78.77)
24. Portland puzzolana cement produces less heat of 4
hydration and so less cracks and used in hydraulic    = 114.65 KNm. Choice (C)
construction.
Less strength at early ages, but more strength at later τ max 
 P M 500 50
ages. Choice (D) 28. = s ± f = ±  = ±
τ min  A Z 2 × 3 2 × 32
2 5. τ cmin = 0.64 MPa < τ cmax = 3.1 MPa  6
Vc = ( τ cmin) bd smax = 100 KN/m2
   = (0.64) × (300 × 500) = 96 KN smin = 66.67 KN/m2. Choice (A)
3.74 | Concrete Structures Test 2

29. Design load for collapse 30. For under reinforced beam
(i) 1.5 DL + 1.5 LL (i) x < xb(balanced)
= 1.5 (330 + 420) X = depth of neutral axis
= 1125 KN – m (ii) Steel yields prior to concrete
(ii) 1.5 DL + 1.5 WL (iii) MUR = 0.87 fy Ast (d – 0.42x)
= 1.5 (330 + 150) Which is also = 0.36 fck bx(d – 0.42x)  -------- (1)
= 720 KN – m As x was found by
(iii) 1.2 DL + 1.2 LL + 1.2 WL 0.36 fck bx = 0.87 fy Ast
= 1.2 (330 + 420 + 150) MB = 0.36 fck bxb (d – 0.42 xb)  -------- (2)
= 1080 KN – m From (1) and (2)
Design moment = maximum of above combinations ⇒  MUR < MB (as x < xb)
= 1125 KN – m. (iv) Lever arm = (d – 0.42x)
 Choice (C) (d – 0.42x) > (d – 0.42 xb) [∵ x < xb] Choice (B)
Unit VI
Steel Structures
This page is intentionally left blank
Steel Structures Test 1
Number of Questions: 25 Time: 60 min.

Directions for questions 1 to 25: Select the correct alterna- 7. ______ beams are used for large spans and light loads
tive from the given choices. (A) ISLB
1. How are the most commonly produced and used (B) Tubular beams
structural elements in frames, floor beams, with high (C) Castellated beams
moment of inertia about X-axis, are designated? (D) ISWB
(A) ISWB – section 8. A steel c/s has less capacity to resist torsion, when
(B) ISLB – section (A) Shear center is above center of gravity
(C) ISMB – section (B) Shear center is below center of gravity
(D) ISHB – section (C) Shear center coincides with Center of gravity
2. Match Group – A with Group – B and select the correct (D) Not related with their locations
answers 9. The design normal strength of a fillet weld is
Group – A Group – B
fy
(A) fu (B)
P. Used when two plates are 1. Fillet weld 3
placed one below the other. fu
(C) fy (D)
Q. Pressure applied continuously 2. Plug weld 3 γ mw
R. Member subjected to direct 3. Slot weld
axial loads 10. Maximum pitch for a tension member whose thickness
S. Joining two surfaces in two 4. Seam weld is “t” is
different planes. (A) 12t or 200 mm
5. Butt weld (B) 16t or 200 mm
(C) Least of a and b
Codes:
(D) Only 200 mm
P Q R S
(A) 4 5 3 2 11. Determine the safe load P that can be carried by the joint
(B) 4 3 2 1 shown in figure below. The bolts used are 20 mm diame-
(C) 5 4 1 2 ter of grade 4.6. The thickness of flange of I-section is 9
(D) 3 4 5 3 mm and that of bracket plate is 10 mm
3. For economical spacing of roof truss, if t, p, r are the P
cost of truss, purlin and roof coverings respectively,
250mm
then
(A) t = p + r
30 mm
(B) t = 2p + r
(C) t = p + 3r 80 mm
(D) t = p + 2r 80 mm
4. Gantry girders are usually designed
80 mm
(A) formultistorey buildings
(B) using channel sections only 30 mm
(C) as laterally supported beams
(D) as laterally unsupported beams
5. Bearing stiffners in a plate girder is used wherever there 140mm
is concentrated load to
(A) increase shear resistance
(B) prevent excessive deflection (A) 93.68 kN
(C) prevent buckling of web (B) 89.49 kN
(D) to transfer the load from compression flange to the (C) 65.68 kN
tension flange (D) 72.92 kN
6. Which of the bolted connections have maximum 12. Boiler plates of t = 15 mm thickness are lap jointed
efficiency? with bolts 18 Φ of 4.6 Grade. If the diameter of boiler
(A) Zig – Zag (B) Diamond is 1m determine maximum pressure that can be allowed
(C) Chain (D) Both A and B in boiler.
3.78 | Steel Structures Test 1

150
10 mm thick
12mm 1m

250

10 mm thick

(A) 1543 kN (B) 1728 kN


80mm = g (C) 1648 kN (D) 1483 kN
16. Determine the flenural design strength of the follow-
ing welded members. The girders are simply supported
and have continuous lateral support. Consider that only
flanges resist BM.
70mm Flanges: 250 × 12mm
Web: 1200 × 8mm
Span: 12m
(A) 0.12 N/mm2
(A) 1652.72 kN (B) 826.36 kN
(B) 0.26 N/mm2
(C) 727.19 kN (D) 1454.38 kN
(C) 0.38 N/mm2
(D) 0.45 N/mm2 17. A built up section is composed of an I section ISMB
400 and C section ISMC 300 connected on top of I
13. Match List – I of types of sections of beams with
section as shown in figure. The minimum radius of
List – II the cases for which beams are designed.
gyration of built up section in cm is ____
List – I List – II
ISMB 400 ISMC 300
P. Plastic section 1. Elastic design
Izz = 20458.4 cm4 Izz = 6362.6 cm4
Q. Compact section 2. Indeterminate frames
R. Semi-compact section 3. Plate Girdes Iyy = 422.1 cm4 Iyy = 310.8 cm4

S. Slender section 4. Simply supported beams A = 78.46 cm2 Cyy = 2.36 cm


tw = 7.6 cm
Codes:
P Q R S A = 45.64 cm2
(A) 2 4 1 3
(B) 4 2 3 1 ISMC300
(C) 3 1 2 4
(D) 1 3 4 2
14. Statement I: In long columns the axial buckling stress
remains below the proportional limit and they buckle ISMB400
elastically.
Statement II: The failure loads for such columns are
proportional to the flexural rigidity (EI) of the column (A) 10.6 cm (B) 13.5 cm
and independent of the strength of steel. (C) 12.1 cm (D) 14.7 cm
(A) Both I and II are true and II is correct explanation 18. Match List – I with List – II and select the correct
of I answer using the codes given below the lists:
(B) Both I and II are true and II is not correct explana-
tion of I List – I (Methods of List – II (Conditions
Analysis) satisfied)
(C) I is true and II is false
(D) Both I and II are false a. Exact Plastic 1. Equilibrium, sufficient
Analysis plastic hindges and non-
15. Determine the block shear strength of the welded ten- violation of plastic moment
sion member shown in figure. Plates are of Fe 410 capacity.
grade b. Mechanism method 2. Equilibrium and non-
P.S.F for yielding = 1.1 of plastic analysis violation of plastic moment
capacity
P.S.F for ultimate stress = 1.25
Steel Structures Test 1 | 3.79

c. Equilibrium method 3. Equilibrium, continuity and 22. A circular penstock of mild steel of grade Fe 410, 1.0 m
of plastic analysis non-violation of plastic diameter is fabricated in works shop with 12 mm thick
moment capacity plates. The plates are secured by 8 mm size fillet weld
4. Equilibrium and sufficient provided on inside and outside of lapped ends as shown
plastic hindges. in figure. Determine safe internal pressure than can be
allowed in the penstock. (in N/mm2)
Codes:
Fillet weld
a b c
(A) 3 1 2
(B) 1 2 4 P
(C) 3 4 2 12mm thick
(D) 1 4 2 0.5m
1 9. Find out the collapse load for the following cantilever
beam in figure below
(A) 3.55 (B) 7.01
A B Wu (C) 4.24 (D) 2.36
2MP C Common Data for Questions 23 and 24:
2MP
23. Find the design strength of Lap joint between 2 plates
L/2 L/2 shown in figure Bolts 20 Φ, 4.6 Grade plates E 250
(A) 0.75 Mp/L (B) Mp/L Fe(410) are used.
(C) 2 Mp/L (D) 1.5 Mp/L P 16mm
P
2 0. Two framing angles ISA 150 × 150 × 10 mm are used to 12mm
make beam to column connection. One angle is placed
1
on either side of the web of the beam as in figure. 3 40
bolts of 16 mm diameter of 4.6 Grades are used to
connect the angle legs to the beam web. Determine the 80
reaction that can be transferred through the joint. Given 80
pitch P = 65 mm and end distance e = 40 mm
80
Column ISHB 300 @ 618.03 N/m
tf = 10.6 mm 80
Beam section ISMB 350 @ 514.04 N/m
tw = 8.1 mm 40
Grade of steel is Fe 410 2
ISHB300
35 70 80
ISMB350 (A) 271 kN (B) 362 kN
(C) 308 kN (D) 420 kN
24. Find the efficiency for the bolted connection in problem
no 23.
(A) 82% (B) 50%
(C) 32% (D) 41%
2 5. Match List – I (of different types of structural beams)
List – II (functions of the beams)
ISA 150 × 150 × 10 mm Side view
Front view List–I List–II

(A) 198.36 kN (B) 174.14 kN a. Girder 1. Provided in buildings to support roofs.


(C) 200.68 kN (D) 183.42 kN b. Purlins 2. These carry roof loads in trusses
2 1. A circular plate, 200 mm in diameter is welded to c. Joists 3. Supports a number of joists
another plate by means of 6 mm fillet weld. Calculate d. Spandrels 4. Carry part of floor that of the exterior
the ultimate twisting moment capacity that can be wall.
resisted by the weld use steel grade Fe 410 and shop Codes:
welding a b c d a b c d
(A) 49.97 kNm (B) 36.31 kNm (A) 1 2 3 4 (B) 2 1 4 3
(C) 57.68 kNm (D) 61.31 kNm (C)
3 2 1 4 (D) 2 3 4 1
3.80 | Steel Structures Test 1

Answer Keys
1. C 2. D 3. B 4. D 5. C 6. B 7. C 8. C 9. D 10. B
11. D 12. B 13. A 14. A 15. C 16. B 17. D 18. D 19. C 20. A
21. A 22. C 23. B 24. D 25. C

Hints and Explanations


7. Castellated Beams P1 × 250 × 138.92
F2 = = 0.3365 P1
103200
70
cos θ = = 0.5038
120 2 + 70 2
F1
F2

Welded

 Choice (C)
rn
1 1. For Fe 410 Grade of steel fu = 410 MPa 120mm
For 4.6 Grade bolts fub = 400 MPa
Partial safety factor for the material of bolt γmb = 1.25 70mm
π
Net Area Anb = 0.78 × × 202 = 245 mm2 Resulted force
4
Diameter d = 20 mm ⇒ do = 22 mm R = F12 + F22 + 2 F1 F2 cos θ
Pitch P = 80 mm 2
P  P 
=  1  + ( 0.3365 P1 ) +  2 × 1 × 0.3365 P1 × 0.5038
2
Edge distance (e) = 30 mm
 8  8 
Strength of bolt in single shear
f  A  400 245 R = 0.4138 P1
Vdsb = ub ×  nb    = × × 10 −3
3  γ mb  3 1.25 Resulted force ≤ Strength of bolt
0.4138P1 ≤ 45.26
=
45.26 kN P1 ≤ 109.38 kN
fu Service load
Strength of bolt in bearing Vdbb = 2.5 Kbdt
γ mb P1 109.38
P= =
e 30 load factor 1.5
Kb = = = 0.454
3do 3 × 22    = 72.92 kN Choice (D)
P 80 1 2. d = 18mm
  = − 0.25 = − 0.25 = 0.96.
3do 3 × 22 do = 20mm
Asb = 254mm2
f ub 400 Anb = 198mm2
  = = = 0.975 = 1
f u 410 Considering width of plate g = 80mm
∴ Kb = 0.454 Strength of Bolt:
410 1  400 
Vdpb = 2.5 × 0.454 × 20 × 9 × × 10–3 = 67 kN Shear Vdsb = (198) = 36.58 kN
1.25 1.25  3 
∴ Strength of bolt Vsd = 45.26 kN 2 × Bolt value = 73.16 kN
No. of bolts n = 8 73.16 × 103
P1 P1 Stress in the plate/area = = 13.06 N/mm2
Direct shear force in each bolt F1 = = 70 × 80
h 8
Pd P × 1.5 × 10
3

P.eo rn Hoop stress = =


Force due to torque in bolt F2 = 2t 2 × 15
Σr 2
P × 1.5 × 103
13.06 =
( 40 + 80)
2
rn = + 70 2 = 138.92mm 2 × 15
Σr2 = 4 [(40 + 80)2 + 702 + 402 + 702] = 103200 mm P = 0.26 N/mm2 Choice (B)
Steel Structures Test 1 | 3.81

15. 19.
Wu
150 mm
10 mm thick
2MP
X 2MP
250
L/2 L/2
mm
There is a change of forming one plastic hindge
X
Case (i): Plastic hindge at the center where c/s changes
10 mm thick θ ∆

Avg = Avn = 2(150 × 10) = 3000 mm2 Internal work done


L
Atg = Atn = 2(250 × 10) = 5000 mm2 =
Wu × ∆ = Wu × θ
2
Avg f y 0.9 Atn f u
Tdb1 = + External work done = Mp θ
3γ mo γ ml
L
Mp θ = Wu × θ
 3000 × 250 0.9 × 5000 × 410  −3
2
= +  × 10
 3 × 1.1 1.25  2M p
Collapse Load Wu =
L
= 1869.65 kN
Case (ii): Plastic hindge at the fixed end
0.9 Avn f u Atg f y
Tdb2 = + θ

3 γ ml γ mo
 0.9 × 3000 × 410 5000 × 250  Internal work done = Wu∆ = Wu × L θ
= +  × 10–3 External work done = 2 MP θ
 3 × 1.25 1.1  WuL θ = 2 MP θ
= 1647.66 kN 2M P
∴ Block shear strength = 1648 kN Choice (C) Wu =  Choice (C)
L
fy 20. For Fe 410 grade of steel fu = 410 MPa
16. Md = Af × ×d
γ mo For 4.6 Grade bolts fub = 400 MPa
T1 d = 16mm do = 18mm
γmb = γml = 1.25
d
Bolts connecting both angle legs to web of beam are in
T2 double shear,
f n A +n A 
Vdsb = ub  n nb s sb 
250 γ mb  3 
Md = (250 × 12) × × (1212)
1.1 Π
Asb = × 162 = 201.06 mm 2
   = 826.36 × 106 Nmm 4
   = 826.36 kNm Choice (B)
Anb = 0.78 × Asb = 156.83 mm2
17. Izz = Izz(I) + Izz(C) 400
Vdsb = (201.06 + 156.83) × 10–3 = 66.12 kN
  =26821cm4 1.25 × 3
A1 C yy + A2 ( 7.6 + 20) Strength of bolt in bearing,
Centroid =
A1 + A2 dt f u
Vdpb = 2.5 Kb
( 45.64 × 2.36) + (78.46 × (7.6 + 20)) γ mb
=
45.64 + 78.46 e 40
Kb = = = 0.74
= 18.32 cm 3do 3 × 18
Iyy = [422.1 + 78.46 × (18.32 – 20)2 + 310.8 + 45.64 × P 65
(18.32 – 2.36)2] = 12579 cm4   =  – 0.25 =  – 0.25 = 0.95
3do 3 × 18
I yy 12579 f ub 400
rmin = = = 14.7cm Choice (D)   = = = 0.975 = 1
A ( 45.64 + 78.46) f u 410
3.82 | Steel Structures Test 1

∴ Kb = 0.74 Single shear nn = 1 ns = 0


410 Shear strength
Vdpb = 2.5 × 0.74 × 16 × 8.1 × × 10–3
1.25 f [nn Anb + ns Asb]
Vdsb = ub
    = 78.64 kN 3 γ mb
∴ Strength of one bolt = 66.12 kN
As 3 bolts are provided, maximum end reaction that 400
= [245 × 1] = 45.27 KN
can be transferred 3 × 1.25
= 3 × 66.12 = 198.36 kN Choice (A)
Bearing strength
2 1. Diameter D = 200 mm 1
Fe 410 Grade ∴ fu = 410 MPa Vdpb = [2.5 Kbdtfu]
γ mb
Shop weld: γmw = 1.25
Ultimate twisting moment capacity e 35
Tu = Pdw × r Kb = = = 0.53
3do 3 × 22
fu 200
  = Lw × tt × × p 70
3 × γ mw 2 = – 0.25 = – 0.25 = 0.81
3do 3 × 22
410 200
  =(π × 200) × (0.7 × 6) × × f ub 400
3 × 1.25 2 = = = 0.975 = 1
f u 410
  = 49.97 × 106 Nmm = 49.97 kNm Choice (A)
Kb = 0.53
2 2. Fe 410 fu = 410 MPa
Size of weld S = 8 mm 1
Vdpb = [2.5 × 0.53 × 20 × 12 × 410]
Throat thickness = KS = 0.7 × 8 = 5.6 mm 1.25
γmw = 1.25    =104.3 kN
Let internal pressure = p Plate yield of gross c/s:
pd
Hoop force/mm length Fh =
2 Tdg =
Ag f y
=
(320 ×12) × 250
γ mo 1.1
Design strength of fillet weld/mm length
fu 410    = 872.7 kN
Pw = tt × × 2 = 5.6 × ×2 Tearing/Rupture:
3 × rmw 3 × 1.25
0.9 An f u
Pw = 2120.95 N Tdh =
γ ml
For safe conditions
Hoop force/mm length = Strength of weld/mm length An = (320 – 4 × 22) × 12 = 2784 mm2
pd p × 1000 0.9 × 2784 × 410
2120.95 = = Tdh = = 821.8 kN
2 2 1.25
p = 4.24 N/mm2 Choice (C) Joint Strength:
23. Strength of bolt in shear = 45.2 × 8 = 362 kN
Bolt Plate Strength of bolt in bearing = 104 × 8 = 832.kN
d = 20 mm t1 = 12 mm Yielding = 872 kN
do = 22 mm t2 = 16 mm
Rupture = 821 kN
Strength of Joint = 362 kN Choice (B)
π
Asb = × 202 = 314 mm2 fu = 410 MPa
4 strength of Joint
24. Efficiency η = × 100
Anb = 0.78Asb = 245 mm 2
fy = 250 MPa yield strength
fub = 400 MPa γmb = 1.25
362
fyb = 0.6 × 400 = 240MPa = × 100 = 41% Choice (D)
872
Steel Structures Test 2
Number of Questions: 30 Time: 75 min.

Directions for questions 1 to 30: Select the correct alterna- 8. While designing, for a steel column of Fe250 grade, a
tive from the given choices. base plate resting on a concrete pedestal of M30 grade,
1. Minimum edge distance of bolted joint for hand flame the bearing strength of concrete (in N/mm2) in limit
cut edges is not less than state method of design as per IS 456 : 2000 is
(A) 1.5 × diameter of bolt hole (A) 3.84 (B) 11
(B) 1.5 × diameter of bolt (C) 13.5 (D) 15
(C) 1.7 × diameter of bolt hole 3
9. In a roof truss if pitch is and slope is 3 , the angle
(D) 1.7 × diameter of bolt 2
2. The type of weld used for joining two surfaces in two of inclination with horizontal would be
different planes is (A) 60° (B) 30°
(A) Fillet weld (B) single V butt weld (C) 45° (D) 90°
(C) double groove weld (D) None 10. In an industrial building gantry girder of effective span
3. If Bolts in any Bolt group is subjected to shear and 25.0 m carries a manually operated crane of 350 kN.
tension; the interaction equation need to be satisfy as The maximum deflection as per IS 800 – 2007 is
per IS 800 : 2007 is [Vb, Vdb : Factored shear force and (A) 50 mm (B) 30 mm
design shear strength and Tb, Tdb : Factored tensile force (C) 25 mm (D) 75 mm
and design tensile strength] 11. Two plates 12 mm and 20 mm thick are to be joined by
V   T  a double cover butt joint with 8 mm thick packing plate.
(A)  B  +  b  ≤ 2.0 What will be the effect of packing on the design shear
Vdb   Tdb  strength of bolt?
2 2 (A) decreases by 10% (B) increases by 10%
V  T 
(B)  b  +  b  ≤ 2.0 (C) decreases by 15% (D) increases by 15%
Vdb   Tdc  12. Calculate the design tensile capacity of M20 bolt of
2 2 grade 4.6 is
V  T  (A) 50 kN (B) 68 kN
(C)  b  +  b  ≤ 1.0
Vdb   Tdb  (C) 75 kN (D) 35 kN
(D) None 13. Calculate the number of bolts required for a lap joint
4. The maximum length of a tension member with mini- between two plates of 12 mm and 24 mm thick to trans-
mum radius of gyration of 20 mm carrying load rever- mit a factored load of 100 kN using M16 bolts of grade
sals other than wind or earthquake forces as per IS 800 4.6 and Fe410 plates. (assume minimum end distance =
is _____? 27 mm; minimum pitch = 40 mm and thread intercept
(A) 5.0 m (B) 1.5 m shear plane)
(C) 3.6 m (D) 6.0 m (A) 2 (B) 6
(C) 4 (D) 8
5. Which of the following is the most efficient section for
column for a given equal cross section area? 14. Determined the service load which can be applied to
(A) solid circular section the fillet weld for the figure shown with a weld size
(B) Angle section of 5 mm. Use plates of grade 410 steel and workshop
(C) I-section welding.
(D) tubular section 200ISF12

6. When the column is effectively held in position and


restrained against rotation at both ends, the effective
length of column is ‘K’ times the unsupported length 200 PS
(L) of column where K is –––––––
(A) 1.2 (B) 0.8
(C) 0.65 (D) 1.0
200
7. A beam section is selected and provided on the basis of
(A) shear (B) deflection (A) 450 kN (B) 500 kN
(C) section modulus (D) All the above (C) 350 kN (D) 265 kN
3.84 | Steel Structures Test 2

15. When length of side fillet weld is 400 times the effec- 19. The moment rotation curve shown in figure is that of a
tive throat thickness. Then the design shear capacity of (i)
fillet weld is
(ii)
(A) decreased by 33%

Moment
(B) increased by 33% (iii)
(C) decreased by 20%
(D) decreased by 66%
16. Two 12 mm thick plates are joined in the field by a single
‘V’ bolt weld. The effective length of weld is 250 mm.
Rotation
Determine the design strength of welded joint. The
yield and ultimate tensile strength of weld and steel are (A)   (i) Semi Rigid Joint
250 Mpa and 410 Mpa respectively. (ii) Rigid.
(A) 330.50 kN (iii) Flexible
(B) 437.50 kN (B)   (i) Rigid Joint
(C) 530.50 kN (ii) Semi Rigid.
(D) 357. 50 kN (iii) Flexible
17. Four the bolt systems shown in figure, the bolts are (C)  (i) Flexible Joint
at a distance of 50 mm from Center of gravity of bolt (ii) Semi Rigid.
group. The resultant force in the critical bolt is (iii) Rigid
(D) None
P
100 mm 20. What is the net effective sectional area of plate of
thickness 12 mm as shown in the figure for carrying
tension?
50 mm 30 mm [take dh : 18 mm]
30 mm 40 mm

50 mm

40 mm P 100 mm

(A) P/4 (B) 0.50P 300 mm P


100 mm
(C) 0.70P (D) 1.0P
1 8. Each bolt shown in figure is capable of resisting design 50 mm
shear capacity of 25 kN and design tension capacity of
20 kN. The interaction equation between of forces as 40 mm
per limit state method of IS 800 : 2007?
(A) 2500 mm2 (B) 3048 mm2
200 mm
P
(C) 2750 mm 2
(D) 3500 mm2
2 1. Compute the tensile strength of an angle section ISA
150 × 150 × 10 mm of Fe 410 grade of steel connected
120 mm with guest plate as shown in figure based on gross sec-
tional yielding.
120 mm

150 mm
 P  P
(A)  + ≤ 10
100   24  10 mm
2 2
P P
(B)   +   ≤ 1.0
 50   24 
2 3
P P
(C)   +   ≤ 1.0
 50   30  50mm
2 2
 P   p (A) 450 kN (B) 350 kN
(D)  + ≤ 1.0
100   24  (C) 660 kN (D) none
Steel Structures Test 2 | 3.85

22. The best tension member section will be a is increased by 15% for wind and earth quake loads,
(A) bolted single angle section then the load factor is
(B) welded single angle section (A) 1.95 (B) 1.40
(C) channel section (C) 1.65 (D) 1.80
(D) double angle section on opposite side of gusset 2 8. A propped cantilever beam AB of length ‘L’ fixed at ‘A’
plate and propped at B is subjected to a concentrated load
23. Determine service axial load on column section ISMB ‘w’ at its center. By kinematic approach calculate the
350. Given that the height of column is 3.2 m and that ultimate collapse load (w) in terms of Mp.
is fixed on both ends. Assume fy : 250 Mpa, fu 410 Mpa 2Mp 6Mp
and E = 2 × 105 Mpa (properties of ISMB 350 are (A) w = w=
(B)
L L
A = 6670 mm2, tf = 14.2 mm, tw = 8.1 mm, b = 140 mm, 4Mp 8Mp
h = 350 mm, rzz = 143 mm and ryy = 28.4 mm (C) w = w=
(D)
L L
(A) 720 KN (B) 850 KN
(C) 350 KN (D) None 29. The distance of plastic neutral axis from top of T-section
shown below:
24. A built up column consists of ISMC 300 channels 100 mm
placed back to back at a spacing of 250 mm and carries
working load of 2000 kN, the double lacing provided 20 mm
with an angle of 50° with longitudinal axis. As per IS
800 – 2007 lacing member should be designed to resist
design axial load of 100 mm
(A) 19.5 kN (B) 24.5 kN
(C) 30.8 kN (D) 54.2 kN
25. In laced columns, end tie plates are provided to 20 mm
(A) check the buckling of column as a whole
(B) check the buckling of component column (A) 15 mm (B) 60 mm
(C) check the distortion of the column sections at (C) 20 mm (D) 40 mm
ends because of unbalanced horizontal force from 3 0. The number of possible independent mechanisms for a
lacings portal frame shown in fig. is
(D) keep the column components in position 2p
26. A steel beam of circular c/s is clamped at both ends. 2P
Deformation is just observed when the U.D.L on the
beam is 20 kN/m. At the instant of collapse, the load on
the beam will be
(A) 15 kN/m (B) 30 kN/m
(C) 20 kN/m (D) 45 kN/m
27. For an rectangular beam the shape factor is 1.5. The (A) 1 (B) 3
factor of safety in bending is 1.5. If the allowable stress (C) 4 (D) 2

Answer Keys
1. C 2. A 3. C 4. C 5. D 6. C 7. D 8. C 9. A 10. A
11. A 12. B 13. C 14. D 15. A 16. B 17. C 18. D 19. B 20. B
21. C 22. D 23. A 24. B 25. C 26. D 27. A 28. B 29. C 30. D

Hints and Explanations


1. For hand flame cut edges miniumum edge distance = 3. Interaction equation need to be satisfy as per IS : 8OO
1.7 × dia. of bolt hole. Choice (C) 2 2
V   T 
is  b  +  b  ≤ 1.0  Choice (C)
2. For joining two surfaces in different planes, Fillet weld  vdb   Tdb 
is used. Choice (A)
3.86 | Steel Structures Test 2

 400 π
4. ≤ 180    = [1 × 0.78 × × 162] = 28.974 kN
r 3 × 1.25 4
 = (180) (20)
2.5dt K b f u
 = 3600mm Vdpb =
γ mb
 = 3.6m Choice (C)
5. For a given cross section area, tubular section is more e P f
Kb is least of ; − 0.25; ub ; 1.0
efficient due to more (r) min. Choice (D) 3dh 3dh fu
6. Given end conditions are translations and rotations
e 27
restrained at both ends. (i.e., fixed at both ends)  =  = 0.50
Effective length constant K = 0.65 Choice (C) 3dh 3 × 18
7. A beam section is selected based on all of the above. P 40
 Choice (D) − 0.25 − − 0.25 = 0.490   Take least
3dh 3 × 18
8. Maximum allowable bearing strength = 0.45 × 30 among these
f ub 400
= 13.5 N/mm2 Choice (C) = = 0.97
f u 410
9. Slope of roof truss;
tan θ = 3 where q : Angle of inclination with Kb = 0.490
horizontal 2.5 × 16 × 12 × 0.490 × 400
θ = tan−1 ( 3 ) = 60° Choice (A) Vdpb = = 75.264 kN
1.25
Span \ Vdb = 28.974 kN
10. Limiting deflection =
500 P 100
n= = = 3.45 ~ 4 Choice (C)
25000 Vdb 28.97
= = 50mm Choice (A)
500 14. Given S = 5mm
11. Design requirement:
Packing plate Design action ≤ design strength
t = Bmm \ P = Pdw
t : 20mm Design shear strength of fillet weld
t = 12mm
fu
(Pdw) = (Lw × tt) ×
3γ mw
Reduction factor (β pkg) : [tpkg > 6mm]
410
Βpkg = 1 − 0.0125 tpkg  = 600 × 0.70 × 5 × = 397.678 kN
3 × 1.25
= 1 − 0.0125 (8) = 0.90
\ Design shear strength of bolt decrease by 10% \ P = Pdw = 397.678 kN
 Choice (A) P 397.678
1 2. Design tensile capacity of bolt = Tdb Ps = = = 265.11 kN Choice (D)
1.5 1.5
0.9 f ub f 15. LJ = 400tt
Tdb = × Anb ≤ yb Asb
γ mb γ mo If LJ > 150tt ⇒ a reduction factor need to be applied for
design shear capacity of fillet weld.
0.9 × 400 π 240 π Reduction factor (βLW):
   = × 0.78 × (20)2 ≤ × (20)2
1.25 4 1.10 4 0.20 × 400t
0.20 LJ
   =70.572 kN ≤ 68.543 βLW = 1.20 − = 1.20 −
150tt 150tt
Tdb : 68.543 kN Choice (B)
= 1.20 – 0.533 = 0.66
P So strength reduced by 33% Choice (A)
1 3. Number of bolts n =
Vdb 5
1 6. Design strength of single ‘v’ butt weld = Lw ×  × T
8
Vdb is least of Vdsb, Vdpb
fy
×
f ub γ mw
Vdsb = (nn Anb + ns Asb) [ns = 0 since thread inter-
3γ mb 5 250
= 350 × × 12 × = 437.500 kN Choice (B)
cept shearplane] 8 1.50
Steel Structures Test 2 | 3.87

17. Load/moment is lying in plate of bolt group. fy


So bolt group is subjected to Direct concentric load (P) 21. Tdg = × Ag
rmo
and Inplane moment/twisting moment (M = Pe)
Vertical shear force in any bolt due to direct concentric Ag = 150 × 10 + (150 − 10) × 10 = 2900 mm2
load is Fa 250
P P Tdg = × 2900 = 659.090 kN Choice (C)
Fa = = 1.10
n 4
Shear force in any bolt due to twisting moment (M = h 350
23. For = = 2.5 > 12 and tf < 40mm
Pe) is bf 140
p .e .r p. (100 )(50 ) Buckling class about z-z axis – a
Fm = =
Σr 4 (50 ) Buckling class about y-y axis is – b
2 2

a = 0.21 (Buckling class a)


40 4
Fm = 0.5 p  cos θ = =    = 0.34 (Buckling class b)
50 5
KL
(FR)max: maximum Resultant force Effective slenderness ratio
rmin
2 2
 P  P  P  P 4 K = 0.65 for ends fixed
=  +   + 2    
 4  2  4 2 5 Effective slenderness ratio

= 0.71 P Choice (C) 0.65L 0.65 × 3.2


= = = 73.23mm
rmin 28.4
1 8. Load/moment is not lying in plane of bolt group.
So bolt group is subjected to direct Concentric load (P) Non dimensional effective slenderness ratio = λ
and bending moment (M) 2
Vertical shear force in any bolt due to P is  KL 
fy 
 r  250 (73.25)
2
P P
q1 = = = Vb λ= =
n 4 π2 E π 2 × 2 × 10 2
Stress in any bolt due to M is q2 λ = 0.82
1 f = 0.5 (1 + a (λ − 0.2) + λ2)
M yn
q2 =   = 0.5 (1 + 0.34 (0.82 − 0.2) + 0.822)
Σy 2   = 0.9416
assume CG of bolt coincides with bending axis Design strength of axial compression
M p.e fy
M1 = =
2 2 rmo fy
f cd = ≤
p (200 )(60 ) 5P φ + (φ 2 − λ )
2 0.5 rmo
Tb = q2 =   Tb =
(
2 2 (60 )
2
) 6 250
1.10
=
\ Interaction equation is 0.9416 + (0.94162 − 0.822 )
0.5

2 2
 Vb   Tb 
  +   ≤ 1.0 =161.82 Mpa ≤ 22727Mpa
Vdb   Tdb  Design axial load on column P = f cd × Ae
2
 P 1   5P 1  =
161.82 × 6670
 4 × 25  +  6 × 20  ≤ 1.0 =
1079.3 kN
2 2 1079.3
 P  P Service load = = 719.53 kN Choice (A)
100  +  24  ≤ 1.0 Choice (D) 1.5
24. Design load on column P = 1.5 × 2000 = 3000 kN
P12 P2 25
20. Anet = (B − n.dh)t + t1 + 2 t2 Transverse shear (V) = × 2000 × 1.5 = 75 kN
4 g1 4 g2 100
V
Force in lacing member (F) =
40 2 × 12 40 2 × 12 4 sin θ
=
[300 − 3(18)]12 + +
4 × 100 4 × 100 75
= = 24.47 kN Choice (B)
=
3048mm2 Choice (B) 4 × sin 50
3.88 | Steel Structures Test 2

26. Given deformation just taken place L


wl 2 d= θ
i.e., My = 2
12 From principle of virtual work,
w l2 External work done by loads = Internal work done
At collapse Mp = c
16 L
Shape factor S = 1.70 w θ = Mp θ + Mp (θ + θ)
2
wc l 2 × 12 wc × 12
1.70 = = 6M p
16 × wl 2 16 × 20 w=  Choice (B)
L
Wc = 45.33 kN Choice (D)
2 7. Load factor = factor of safety × shape factor 29. Area of flange = 100 × 20 = 2 × 103 mm2
fy  fy  Area of web = 100 × 20 = 2 × 103 mm2
=  ×S=   × 1.50 Area of flange and area of web are equal so plastic
f 1.15 f 
NA lies at junction of flange and web
 1  \ distance of N.A. from top = 20mm Choice (C)
=
  × 1.5 × 1.5 = 1.95 Choice (A)
1.15 
30.
28. Number of plastic hinges = Ds + 1 = n
Ds = (2 + 1) − 2 = 1 P 2P
n=1+1=2 B D
C
one plastic hinge is formed at fixed end and other at
point load.
W A E

A B Number of Independent mechanisms = N – Ds


Where N – number of possible location of plastic hinges
L
Ds – Degree of static Indeterminacy
Mp
Ds = r – 3 = 5 – 3 =2
Mp N = A, B, C, D = 4
δ
Number of independent mechanisms = 4 – 2 =2
 Choice (D)
Unit VII
Geotechnical Engineering
This page is intentionally left blank
Soil Mechanics Test 1
Number of Questions: 25 Time: 60 min.

Directions for questions 1 to 25: Select the correct alterna- 11. Match the following
tive from the given choices.
Source of Transportation Type of Soil
1. Which of the following is a type of chemical weathering 1. River i. colluvial soil
(A) oxidation (B) wedging
2. Gravitation ii. Aeolian soil
(C) abrasion (D) temperature effect
3. Wind iii. Alluvial soil
2. Which of the following is the characteristic of a floccu-
4. Lakes iv. Lacustrine soil
lated clay structure
(A) low shear strength (A) (1 – iii), (2 – i), (3 – ii), (4 – iv)
(B) low permeability (B) (1 – ii), (2 – iii), (3 – iv), (4 – i)
(C) low compressibility (C) (1 – iv), (2 – iii), (3 – i), (4 – ii)
(D) None of these (D) (1 – i), (2 – iv), (3 – ii), (4 – iii)
3. In oven drying method a temperature of 60°C to 80°C 12. A sample of soil deposit has a void ratio of 1. If the
is preferred when void is reduced to 0.3 by compaction, the % volume
(A) high organic soils are present loss is
(B) Gypsum is present (A) 58% (B) 56%
(C) inorganic particles are present (C) 54% (D) 34%
(D) high clay content is present
13. The following data is obtained from the liquid limit test
4. When Cu > 4 and Cc is lies between 1 and 3 the soil can
conducted on soil sample
be classified as
(A) uniformly graded soil No. of blows 20 25 30 35 40
(B) well graded soil Water content 64.2 63.9 62.5 61.9 61.8
(C) gap graded soil
(D) Coarse grained soil (A) 61.9% (B) 61.8%
(C) 63.9% (D) 64.2%
5. The notations GP and SM represent
(A) silty gravel and silty sand 14. In falling head permeability test on a sample 13.4 cm
(B) clayey gravel and clayey sand high and 48.4 cm2 in cross sectional area, the water level
(C) well graded gravel and well graded sand in a stand pipe of 5.25 mm internal diameter dropped
(D) poor graded gravel and silty sand from a height diameter dropped from a height of
65 cm to 25 cm in 20 min. The coefficient of permeabil-
6. A soil is said to be highly permeable when
ity (× 10–4 cm/sec) is
(A) K > 10–1 cm/sec (B) K > 10–3 cm/sec
(A) 0.58 cm/sec (B) 0.47 cm/sec
(C) K < 10 cm/sec
–1
(D) K < 10–3 cm/sec
(C) 0.53 cm/sec (D) 0.54 cm/sec
7. The process of softening of soil due to increase in
water content caused by melting of ice formed in 15. A glass container with pervious bottom has a sand
soil is with void ratio = 0.6. If the specific gravity of sand par-
(A) frost heave (B) frost boil ticles = 2.65, area of c/s = 20 m2, head of water required
(C) Thawing (D) Capillary fringe to cause quick sand condition is (take L = 10 m)
(A) 10.1 m (B) 11.3 m
8. Effective stress in soil increased if the flow is (C) 10.8 m (D) 10.3 m
(A) downwards (B) zig-zag
(C) upwards (D) uniform 16. In a flow net there are 10 flow channel and 20 equipo-
tential drops, the quantity of seepage if head loss is 4 m
9. Space between any two adjacent flow lines and adjacent and k = 3 × 10–5 m/s is
equi potential lines is called (A) 24 × 10–5 m3/s (B) 6 × 10–5 m3/s
(A) flow net (B) flow line (C) 8 × 10 m /s
–5 3
(D) 22 × 10–5 m3/s
(C) flow field (D) flow path
10. The chart used to find the vertical stress on wester- Common Data for Questions 17 and 18:
gaard’s equation is known as A soil profile consists of a surface layer of sand 4 m thick
(A) influence chart (g = 1.8 t/m3), an intermediate layer of clay 3.8 m thick
(B) isobar chart (g = 2.3 t/m3) and the bottom layer of gravel 5 m thick
(C) Fenske’s chart (g = 1.98 t/m3). The water table is at upper surface of clay
(D) None of the above layer (takew = 0.98 t/m3)
3.92 | Soil Mechanics Test 1

17. Effective stress at 7.8 m from the surface is 22. The Plastic limit and liquid limit of the soil are 33%
(A) 8.58 t/m3 (B) 8.64 t/m3 and 45% respectively. The percentage of clay fraction
(C) 12.21 t/m3 (D) 8.58 t/m3 30%. The activity of clay is
(A) 0.3 (B) 0.4
18. Effective stress at 12.8 m from the surface is (C) 2.5 (D) 2.8
(A) 14.9 t/m3 (B) 17.21 t/m3
23. The unit weight of sand back fill was found to be 1746
(C) 14.8 t/m
3
(D) 15.3 t/m3
kg/m3. The water content is 6.6% and unit weight of
A saturated clay has water content 39.3% and bulk
soil constituents is 2.6 g/cc. In laboratory the void ratio
sp-gravity 1.84
of loosest and densest states were found to be 0.842 and
19. Specific gravity of soil is 0.622 respectively. The relatively density of soil is
(A) 2.73 (B) 2.78 (A) 1.23 (B) 1.86
(C) 2.74 (D) 2.79 (C) 1.18 (D) 1.15
24. A soil has the liquid limit of 50% and plastic limit of
20. Void ratio of soil is 30%. Then the classification of soil will be
(A) 1.05 (B) 1.2 (A) CL (B) CI
(C) 1.07 (D) 1.8 (C) CH (D) MH
21. (i) Soil with largest void ratio has less permeability 25. Sedimentation method generally used in the field of
(ii) Permeability of partially saturated soils is consid- soil mechanics is
erably smaller than that of fully saturated soils (A) successive sedimentation
(B) observation of the amount of sediment per unit
(A) (i) is true and (ii) is false
volume at a given point
(B) (i) is false and (ii) is true
(C) observation of total amount of soil in suspension
(C) (i) and (ii) are false above a given elevation
(D) (i) and (ii) are true (D) observation of total sedimentation soil

Answer Keys
1. A 2. C 3. B 4. B 5. D 6. A 7. B 8. A 9. C 10. C
11. A 12. D 13. C 14. B 15. D 16. B 17. C 18. B 19. C 20. C
21. D 22. B 23. D 24. B 25. B

Hints and Explanations


V1 1 + e1 14. In falling head permeability test
12. =   given: e1 = 1
V2 1 + e2 a.L h 
e2 = 0.3 K= loge  1 
A.t  h2 
V1 1+1 2
= = Given
V2 1 + 0.3 1.3
πd 2 π
= (52.5 × 10 −1 ) = 0.216 cm 2
2
V1 = 1.53 V2 a=
4 4
V1 − V2 L = 13.4 cm
% loss in volume = × 100
V1 A = 48.4 cm2
h1 = 55 cm
1.53V2 − V2
= × 100 h2 = 25 cm
V1 t = 20 min
0.53V2 0.216 × 13.4  65 
= × 100 = 34% Choice (D) k= × log e  
V1 48.4 × 20  25 
13. The liquid limit of soil is water content at which part    = 28.61 cm/min = 0.47 × 10–4 cm/sec Choice (B)
of soil cut by a groove will flow together for a distance
15. Head required to cause quick sand condition is
of 12 mm under an impact of 25 blows in casagrande’s
apparatus. r1 L + q
h=
wL = 63.9% Choice (C) rw
Soil Mechanics Test 1 | 3.93

r1 = rsat – rw 19. (Gm)sat = 1.84


rw (G + e ) γ sat
= 1.84
rsat = γw
1+ e
9.8 ( 2.65 + 0.6) e=
wG
[ s = 1]
  = = 19.9 kN/m3 s
1 + 0.6
r = 19.9 – 9.8 = 10.1 kN/m3
1 (G + e )
= 1.84
In the given problem q = 0 1+ e
10.1 × 10 G + 0.393 G
\ h = = 10.3m  Choice (D) = 1.84
9.8 1 + 0.393 G
16. Seepage quantity G = 2.74 Choice (C)
 Nf  wG
q = K .H .  20. e =
 N d  s
39.3
Nf = 10 e = 2.74 × = 1.07 Choice (C)
Nd = 20 100
K = 3 × 10–5 m/s lp
22. Activity of clay =
H = 4m c
 10  WL + WR 45 33
q = 3 × 10 −5 × 4 ×          = =  = 0.4 Choice (B)
 20  C 30
  =6 × 10–5 m3/s Choice (B)
emax − eo
23. Relative density =
17. Effective stress = total stress – Pore water pressure total emax − emin
stress at 7.8 m from surface
γ r G
= (1.8 × 4) + (3.8 × 2.3) γd = = W
= 15.94 t/m3 1 + w 1 + eO
Pore water pressure 1.746 1 × 2.6
=
= 3.8 × 0.98 = 3.724 t/m3 6.6 1 + eO
∴ Effective stress = 15.94 – 3.72
1+
100
= 12.216 t/m3 Choice (C) 2.6
1.637 =
18. Total stress at 12.8m from the surface is 1 + eO
= (1.8 × 4) + (3.8 × 2.3) + (5 × 1.98)
= 25.84 t/m3 eo = 0.587
∴ Effective stress = 25.84 – (0.98 × 8.8) 0.842 0.587
Relative density = = 1.15 Choice (D)
= 17.216 t/m3 Choice (B) 0.842 0.622
Soil Mechanics Test 2
Number of Questions: 25 Time: 60 min.

Directions for questions 1 to 25: Select the correct alterna- 10. The unsupported vertical cut of the embankment if
tive from the given choices. C = 40 kN/m2, g = 30 kN/m3 and ka = 1 is
1. The intensity of radial shear stress at a point 8 m below (A) 5.23 m (B) 5.33 m
vertically and 5 m horizontally below a point load of (C) 5.43 m (D) 5.53 m
3 tonnes is 11. A retaining wall of height 10 m retains dry sand. The
(A) 1.438 t/m2 (B) 1.583 t/m2 soil is loose and has a void ratio of 0.8, gd = 18.8 kN/m3
(C) 1.875 t/m
2
(D) 2.013 t/m2 and Φ = 50°. The backfill is compacted to a state of
0.5, gd = 20.8 kN/m3 and Φ = 65°. The ratio of initial
2. The curves indicating the distribution of excess hydro-
passive thrust to the final passive thrust according to
static pressure in the soil are known as
Rankine’s earth pressure theory is
(A) isobars (B) isochrones
(A) 0.35 (B) 2.9
(C) isotopes (D) isohyts
(C) 0.33 (D) 2.7
3. The soil which is compacted to the dry of optimum has 12. A cohesive soil yield a maximum dry density of
(A) low swelling 1.4 g m/cc at OMC of 16% during a standard proctar
(B) low shear strength test. If the value of G = 2.65 then degree of saturation
(C) high swelling of the soil is
(D) None of the above (A) 1.23 (B) 1.86
4. The critical shear plane will have an angle of _______ (C) 1.43 (D) 1.69
with reference to major principle plane 13. A saturated clay of 6m thick takes 1.6 years for 50%
Φ Φ primary consolidation when drained on both sides. Its
(A) 45° + (B) 35° +
2 2 coefficient of volume charge is 1.5 × 10–3 m2/kN. The
Φ coefficient of permeability of soil will be
(C) 90° + Φ (D) 135° + (A) 0.013 m/yr (B) 0.016 m/yr
2
(C) 1.58 m/yr (D) 2.54 m/yr
5. Under active pressure condition the failure wedge
moves 14. A footing of 3 m × 2 m exerts uniform pressure of
(A) towards right 150 kN/m2 on soil.
(B) towards left Assuming a load dispersion of 2 vertical to 1 horizon-
(C) towards upward tal, the average vertical stress (kN/m2) at 1 m below the
(D) towards downward footing is
(A) 50 (B) 80
6. Westergaard’s theory is applicable for which type of (C) 45 (D) 100
soils
15. Match the following:
(A) Sandy soils (B) Startified soils
(C) humus soils (D) gravel Compaction equipments Usage
7. During consolidation process 1. Tampers i. for cohesive and non
cohesive soils
(A) effective pressure on soil decreases
(B) void ratio increases 2. pneumatic tyred rollers ii. for clays

(C) degree of saturation remains same 3. sheep foot roller iii. For confined trenches
(D) excess hydrostatic pressure increases 4. Vibratory compactors iv. For granular soils

8. The compactive effort in the modified proctor test is (A) 1 – iii, 2 – i, 3 – ii, 4 – iv
about ________ times than that of the standard proctor (B) 1 – iv, 2 – iii, 3 – ii, 4 – i
test (C) 1 – iii, 2 – ii, 3 – iv, 4 – i
(A) 4.85 (B) 4.65 (D) 1 – ii, 2 – i, 3 – iii, 4 – iv
(C) 4.25 (D) 4.55
16. (i) In rankine’s theory the retaining wall is assumed
9. The sensitivity of soil indicates the to be smooth and vertical.
(A) moisture holding capacity of soil (ii) In coulomb’s wedge theory the retaining wall is
(B) shear strength of the soil assume to be rough.
(C) consolidation of soil (A) (i) is true, (ii) is false
(D) weakening due to remolding of soil (B) (i) is false, (ii) is true
Soil Mechanics Test 2 | 3.95

(C) (i) and (ii) are true (A) 0.048 kN/m2 (B) 0.096 kN/m2
(D) (i) and (ii) are false (C) 0.049 kN/m2 (D) 0.035 kN/m2
22. With the increase of water content incompaction the
Common Data for Questions 17 and 18:
maximum dry density will
When an unconfined compression test is conducted on a
(A) increase
cylinder of soil, it fails under axial stress of 4.3 kg/cm2. The
(B) decrease
failure plane makes an angle of 60° with horizontal
(C) first increase and then decrease
17. The cohesion of soil is (D) first decrease and then increase
(A) 0.58 kg/cm2 (B) 0.63 kg/cm2
2 3. Which of the following property increases with increase
(C) 0.68 kg/cm2 (D) 1.24 kg/cm2
in compaction
18. The angle of internal friction of soil is (A) permeability
(A) 6.5° (B) 20° (B) shear strength
(C) 30° (D) 10° (C) void ratio
(D) compressibility
19. A 5 m thick clay has coefficient of consolidation 0.025
24.
cm2/min and final settlement 10 cm. The time required
for 80% of settlement to occur is Well graded soil

Low Plastic silt


U Tv
Q
80% 0.567
25% 0.078

The type of soil present at Q is


(A) 5.6 × 106 minutes (B) 6.5 × 106 minutes
(A) high plastic clay
(C) 7.5 × 106 minutes (D) 5.8 × 106 minutes
(B) high plastic silt
20. Time required for 2.5 cm settlement is (C) low plastic clay
(A) 7.5 × 105 min (B) 7.8 × 105 min (D) poorly graded soil
(C) 7.6 × 105 min (D) 7.2 × 105 min 2 5. When the OCR = 1 then the type of clay is known as
(A) over consolidated clay
21. A 50 kN load acts on the surface of an infinite elas-
(B) Normal consolidated clay
tic medium the vertical pressure intensity in kN/m2 at
(C) under consolidation clay
10 m below and 4 m away from the load will be
(D) None of these

Answer Keys
1. C 2. B 3. C 4. A 5. D 6. B 7. C 8. D 9. D 10. B
11. C 12. D 13. B 14. C 15. A 16. C 17. D 18. C 19. A 20. B
21. D 22. B 23. C 24. B 25. B

Hints and Explanations


T  4C
1. Radial shear stress Trz = σ z   10. Unsupported vertical cut H C =
 z γ ka
Given σ2 = 3 tonnes = 3 × 103 kg 4 × 40
= 3 × 104 N HC = = 5.33 m  Choice (B)
30 × 1
r = 5m
11. Initial passive thrust
z = 8m
PP = KP × σv   σv = g × z
 5 σV = g × z = 18.8 × 10 = 18.8 kN/m
Trz = 3 × 103 × 10 ×  
 8 1 + sin (50 )
KP = = 7.54
= 18750 N/m 2
1 − sin (50 )
= 1.875 t/m2. Choice (C) (PP)1 = 7.54 × 188 = 1419.1 kN/m
3.96 | Soil Mechanics Test 2

Final passive thrust Q = A × q = (3 × 2) (150) = 900 kN/m


σV = r × z = 20.8 × 10 = 208 kN/m 900
∴ σ = = 45 KN/m 2  Choice (C)
1 + sin (65) (3 + 2 ) ( 2 + 2 )
kp = = 20.34
1 − sin (65) 16. Choice (C)
(PP)2 = 208 × 20.34 = 4232.0 kN/m 17. Unconfined compression test σ = 2 × Cu × tan(45 +
Φ/2)
( PP )1 1419.1 4.3 = 2 × Cu × tan60°
= = 0.33  Choice (C)
( PP )2 4232.0 ∴ Cu = 1.24 kg/cm2. Choice (D)
12. eG = Gw φ
18. 45 + = a= 60° Angle of internal friction
γ G 2
γd = w
1+ e = 7.5°.f = (60 – 45) × 2 =30° Choice (C)
16 1 9. CV = TV d /t
2
∴ 0.25 × S = 2.65 ×
100 0.025 = TV × (500)2/t
1 × 2.65 t = 10 × 106 TV
1.4 =    S = 1.69 when U = 80%, TV = 0.567
1+ e
∴ t = 10 × 106 × 0.567
e = 0.25 Choice (D)   = 5.67 × 106 minutes. Choice (A)
CV × t 2.5
13. Tv = 20. U = × 100 = 25%
d2 10
H When U = 25%, TV = 0.078
d= → double drainage
2 ∴ t = 10 × 106 TV
  = 10 × 106 × 0.078
CV × t Cv × t × 4 CV − 3 × 16 × 4
∴ TV = =  =   = 7.8 × 105 minutes. Choice (B)
( 2)
2
H H2 62 5
2
2  
π  50   
TV for 50% consolidation is =  ×   Q 3  1
4  100  2 1. σz = 2 × 
Z 2π   r   2

π  50 
2
CV × 3 × 1.6 × 4 1 +   
∴ ×  =  z 

4 100 62
∴ CV = 1.103 r= 10 2 + 4 2 = 100 + 16 = 10.7m
k
CV = \ K = CV mv rw 5
mv rw 2
 
= 1.103 × 1.5 × 10–3 × 9.8 50 
3  1 
= 0.016 m/year. Choice (B)   = × 
100 2π   10.7   2

1 4. The average vertical stress 1 +   


 10  
Q
  =   = 0.035 kN/m2 Choice (D)
( ) ( B + z)
L + z
Geotechnical Engineering Test 3
Number of Questions: 30 Time: 75 min.

Directions for questions 1 to 30: Select the correct alterna- 7.


tive from the given choices. n

1. If w represents natural water content & wL, wp, ws repre-


sents Liquid limit, Plastic limit, Shrinkage limit respec-
Pressure
tively; choose the incorrect pair from the following.
(A) Plasticity Index (Ip) = wL – wP (3)
(B) Shrinkage Index (Is) = wp – ws
w − wp
(C) Liquidity Index (IL) = (2)
wL − w p (1)

w − wL
(D) Consistency Index (IC) =
w L − wP Wall movement

Identify the correct one from the following. (po, pa, pp


2. The symbol ‘SM’ indicates
indicates at rest, active and passive earth pressures
(A) Sandy silt
respectively)
(B) Medium silt
(A) (1) – po,  (2) – pa,  (3) – pp
(C) Silty sand
(B) (1) – pa,  (2) – po,  (3) – pp
(D) Medium sand
(C) (1) – pp,  (2) – po,  (3) – pa
3. For a given flow net, if number of flow channels and (D) (1) – po,  (2) – pp,  (3) – pa
number of potential drops are found as 12 and 8; then 8. In a falling head permeability test the initial head of
what would be the shape factor of the flow net? 1.2 m dropped to 0.4 m in 4 hours, the diameter of the
(A) 4 stand pipe being 5 mm. The soil specimen was 300 mm
(B) 1.67 long and of 150 mm diameter. The coefficient of per-
(C) 1.5 meability of the soil is _____
(D) 1.71 (A) 2.54 × 10–5 cm/sec
(B) 2.54 × 10–6 cm/sec
4. The figure given below represents the contact pressure
(C) 2.54 × 10–4 cm/sec
distribution underneath a
(D) 2.54 × 10–7 cm/sec
9. Sand Bath method is used to determine ___
Settlement (A) Specific gravity
(B) Unit weight
(C) Moisture content
(D) Particle Size distribution
Contact 10. The number of blows observed in a Standard Penetration
pressure test for different penetration depths are given as follows.
Penetration of Sampler Number of blows
(A) Rigid footing on Sand
   0 – 100 mm  2
(B) Flexible footing on Clay 100 – 200 mm  4
(C) Flexible footing on Sand 200 – 350 mm  7
(D) Rigid footing on Clay 350 – 400 mm 10

5. If uniform surcharge of 120 kN/m2 is placed on the The observed ‘N’ value is ______
backfill with Φ = 30°, the increase in pressure is 11. For a Sand deposit having Specific gravity 2.65 and
(in kN /m2) moisture content 25%; what will be the relative den-
6. The cohesion and density of a soil are 1.8 t/m2 and sity in saturated condition if loose and Compacted void
2 t/m2. If Stability number is taken as maximum, for a ratios are given as 0.92 and 0.41 respectively
factor of safety against 2.5 what will be the safe height (A) 49% (B) 51%
of the slope in meters? _____ (C) 39% (D) 31%
3.98 | Geotechnical Engineering Test 3

12. For a particular soil sample, if D10, D30 and D60 is given 17. Match List – I (Roller type) with List – II (Soil type)
as 425 μ, 2.36 mm and 4.75 mm respectively. Then List – I List – II
match the following
1. Sheep foot roller a. Gravel in WBM Road
Group – I Group – II 2. Pneumatic roller b. Dry sand
1. Coefficient of Curvature a. 1.81 3. Smooth heavy roller c. Hearting of earthen dam
2. Coefficient of Uniformity b. 2.76
4. Vibratory roller d. Casing of earthen dam
3. Permeability c. 11.2
(A) 1 – b, 2 – c, 3 – a, 4 – d
(A) 1 – a, 2 – b, 3 – c (B) 1 – c, 2 – d, 3 – a, 4 – b
(B) 1 – c, 2 – b, 3 – a (C) 1 – d, 2 – c, 3 – a, 4 – b
(C) 1 – a, 2 – c, 3 – b (D) 1 – d, 2 – c, 3 – b, 4 – a
(D) 1 – b, 2 – c, 3 – a
18. The water content of saturated soil and the specific
13. In a 8m thick stratum of fine sand having submerged gravity were found to be 30% and 2.65 respectively.
density of 11 kN/m3, quick sand condition occurred at Assuming the unit weight of water to be 10 kN/m3, sub-
a depth of 5.2 m of excavation. What is the depth of merged unit weight (in kN/m2) and porosity of the soil
lowering of ground water table required for making an are _____
excavation 6 m deep? (A) 19,0.6 (B) 9,0.45
(Take γw as 10 kN/m3) (C) 9,0.6 (D) 19,0.78
(A) 1.76 m (B) 1.68 m
19. When an unconfined compression test is conducted on
(C) 0.88 m (D) 0.74 m
a cylinder of soil, it fails under axial stress of 1.5 kg/
14. The vertical stress at some depth below the centre of cm2. If an angle of internal friction of the soil is 30°,
3 m × 4 m rectangular footing due to certain load inten- what will be the cohesion of the soil? _____
sity is 100 kN/m2. What will be the vertical stress in (A) 0.43 kg/cm2 (B) 0.67 kg/cm2
kN /m2 below the corner of 1.5 m × 2 m rectangular (C) 0.75 kg/cm 2
(D) 0.35 kg/cm2
footing at the same depth and same load intensity?
20. A square footing of size 5 m × 5 m is resting on the sur-
15. An unsupported excavation is made to the maximum face of a deposit of saturated clay having an unconfined
depth in a clay soil having g = 21 kN/m3,C = 80 kN/m2 compressive strength of 54 kPa, What will be the net
and Φ = 30°. What will be the active earth at pressure safe bearing capacity of the footing (in kPa) if factor of
the base level of excavation, according to Rankine’s safety is given as 2.5? _____
theory?
21. A square group of 16 piles was driven into soft clay
(A) 184.8 kN/m2 (B) 92.4 kN/m2
extending to a large depth. The diameter and length
(C) 1462.4 kN/m 2
(D) 277.18 kN/m2
of the piles were 30 cm and 12 m respectively. If the
16. For the sampler shown in the figure Area ratio, Inside cohesion of clay is given as 5 t/m2, for the pile spacing
clearance and outside clearance are respectively 100 cm c/c what is the capacity of the pile group? (Take
adhesion factor as 0.75 and FOS = 1.75)
25 mm
(A) 733 t (B) 417 t
(C) 550 t (D) 623 t
22 mm 22. In a constant head permeameter with cross section
Sampling tube area of 10 cm2, when the flow was taking place under a
hydraulic gradient of 0.6 the amount of water collected
in 60 seconds is 720 CC. The permeability of the soil
is ____.
Cutting edge
(A) 0.2 cm/sec (B) 0.02 cm/sec
(C) 0.002 cm/sec (D) 2 cm/ sec
20 mm
23. What is the shear strength in terms of effective stress on
a plane with in the saturated soil mass at a point where
28 mm
total normal stress is 245 kPa and pore water pressure
is 80 kPa. The effective shear stress parameters are c1 =
12 kPa, and φ’ = 30°.
(A) 96%, 12%, 10% (A) 105.3 kPa (B) 106.3 kPa
(B) 49%, 11%, 9% (C) 107.3 kPa (D) 108.3 kPa
(C) 49%, 9%, 11% 24. Sieve analysis on a dry soil sample of mass 1000 g
(D) 96%, 10%, 12% showed that 980 g and 270 g of soil pass through
Geotechnical Engineering Test 3 | 3.99

4.75mm and 75 μ sieve, respectively. The liquid limit consolidation under the condition that ‘clay is drained
and plastic limits of the soil fraction passing through on the top surface only’.
425 μ sieves are 55% and 25% respectively. The soil (A) 15.4 years (B) 61.7 years
may be classified as ______ (C) 85.6 years (D) None
(A) GC (B) SM 29.
(C) GM (D) SC
25. Identify the incorrect pair from the following
(A) Alluvial soils - Transported by running water H
(B) Lacustrine soils - Deposited at the bottom of lakes
(C) Talus - soil transported by gravitational force
(D) Loess - soil transported by glaciers
26. In a plate load test with size of plate 30 × 30 cm; bear-
ing capacity and settlement were noted as 15 kPa & GL
6 mm respectively in a sandy soil. Then find the bear-
ing capacity and settlement under a footing of size
2.1 m × 2.1 m under the same pressure intensity? D Soft soil
(A) 15 kPa and 42 mm
(B) 15 kPa and 18.4 mm
(C) 105 kPa and 18.4 mm
Hard stratum
(D) 105 kPa and 42 mm
27. A footing carries a load of 1200 tons and is of 3.2 m For the given slope, the failure expected to be ___
square. It rests in dense sand of 8 m thickness overlay- (A) Toe failure
ing a clay layer of 2.8 m. The clay layer overlies hard (B) Base failure
rock. The depth of foundation is 1.5 m Liquid limit of (C) Face failure
clay is 48% and void is 0.95. The saturated unit weight (D) Can’t say/ Data inadequate
of sand and clay are given as 1.86 t/m3 and 1.76 t/m3 3 0. Identify the false statements from the list given.
respectively Take the load distribution as 2V to 1H. (i) Skempton’s theory is suitable for clays only.
Assume that the site is flooded and determine the ulti- (ii) The discharge between any two adjacent flow lines
mate settlement due to consolidation of clay layer? is constant
(A) 150 mm (B) 170 mm (iii) 
If water table rises s1 and u increases but s
(C) 190 mm (D) 210 mm decreases.
28. If a clay test specimen of 25 mm thick, under double (iv) The westergaard analysis is suitable for stratified
drainage condition attained 50% of primary consolida- soils
tion in 50 minutes. How long will it take for the same (A) (iii) and (iv) (B) (i) and (iii)
clay layer of 10 m thick to reach the same degree of (C) (iii) only (D) (iv) only

Answer Keys
1. D 2. C 3. C 4. C 5. 40 kN/m2 6. 1.37 to 1.40 7. B 8. B
9. C 10. 21 11. B 12. D 13. B 14. 25 15. B 16. D 17. B 18. B
19. A 20. 80 21. B 22. D 23. C 24. D 25. D 26. C 27. B 28. B
29. B 30. C

Hints and Explanations


wL − w 5. Increase in pressure = Ka. q
1. consistency Index, Ic =  Choice (D)
wL − w p 1
1 − sin φ 1−
Nf Ka = = 2 =1 = 1
3. Shape factor = 1 + sin φ 1 3 3
Nd 1+
2
12 1
= = 1.5 Choice (C) Ka. q = × 120 = 40 kN/m2  Ans: 40 kN/m2
8 3
3.100 | Geotechnical Engineering Test 3

C 21 (2.8) – 10 (8 – x1) = 0
6. Sn = x1 = 2.12 m
Fc.γH
1.8 x1
0.261 =
2.5 × 2 × H
H = 1.38m 5.2 m
 Ans: 1.37 to 1.40
(8 – x1)
7. pa < po < pp Choice (B)
aL h 
8. K = 2.303 log10  1 
At  h2  2.8 m
Given; h1= 1.2m and h2 = 0.4m
π To make an excavation of 6 m deep,
Area of stand pipe (a) = × 52 = 19.635mm2
4 21 (2) – 10 (8 – x2) = 0
π x2 = 3.8 m
Area of soil specimen (A) = × 1502 = 17671.46mm2
4 \ Lowering of WT = 3.8 – 2.12 = 1.68 m.
19.635 × 300  Choice (B)
\ k = 2.303 ×  log10 14.
17671.46 × 4 × 60 × 60
4
K = 2.54 × 10–5 mm/sec
   = 2.54 × 10–6 cm/sec Choice (B)
9. Oven drying method, pycnometer method, calcium car- 1.5
bide method, Sand Bath method, are used to determine
water content of the soil. Choice (C) 3

10. N = 4 + 7 + 10 = 21
1.5
N value is taken as the sum of blows for the last
300 mm penetration. Ans : 21
2 2
11. Given that soil is in Saturated condition, then void ratio,
e = wG Since 3 m × 4 m rectangle consists of 4 no. of 1.5 m
e = 0.25 × 2.65 = 0.66 × 2 m;
e −e Vertical stress at the corner of 1.5 m × 2 m rectangle =
Relative Density = max × 100
emax − emin 1
4
× (vertical stress at centre of 3 m× 4 m rectangle)
0.92 − 0.66
= × 100 1
0.92 − 0.41 = 4 × 100 = 25 kN/m2 Ans: 25
=50.98 ≈ 51% Choice (B)
15. Maximum depth of Unsupported excavation = Hc
1 2. Given, 4c
D10 = 425 μ = 425 × 10–3 mm = 0.425 mm =
D30 = 2.36 mm γ Ka
D60 = 4.75 mm 1
1−
D302 2.362 1 − sin φ 2
(i) Cc = = = 2.76 Ka = =
D60 × D10 4.75 × 0.425 1 + sin φ 1 + 1
2
D60 4.75 1
(ii) Cu = = \ Ka =
D10 0.425 3
(iii) Permeability; k = 100 D102 4 × 80
⇒ Hc = = 26.40m
It should be noted that ‘D’ is in cm’s here 1
21
⇒ K = 100 (0.425 × 10–1)2 = 0.181 cm/sec 3
K = 1.81 mm/sec Choice (D)
Pa = Ka. γH – 2c Ka
1 3. Quick sand conditions
γsat z – γw h = 0 1 1
  =   × 21 × 26.40 – 2 × 80 ×   = 92.42 kN/m2
Let WT is at x1, initially 3 3
s1 = γsat Z – γw h = 0  Choice (B)
Geotechnical Engineering Test 3 | 3.101

16. 20. Given is a square footing.


D4 So,
qult = 1.3 CNC + gDNq + 0.4 g BNg
Unconfined compressive strength,
D3 qu = 2c
54
⇒ C = = 27 kPa
2
For clay; (Φ = o)
Nc = 5.7, Nq = 1 and Ng = 0
qnetult = (1.3CNc + g DNq) – gD
D1
qnetult = 1.3 CNc + gD Nq – gD
\ qnetult = 1.3 CNc
1.3CN c 1.3 × 27 × 5.7
D2 qnetsafe = = = 80 kPa
F .S 2.5
 Ans: 80 kPa
D2 − D2 282 − 20 2 21.
Area ratio = 2 2 1 × 100 = × 100 = 96%
D1 20 2 100 mm
D − D1
Inside clearance = 3 × 100
D1
22 − 20 100 mm
= × 100 = 10%
20
D2 − D4 28 − 25 Bo
Outside clearance = × 100 = × 100
D4 25
= 12% Choice (D)
1 7. Sheep foot roller – Hearing of earthen down
Pneumatic roller – Casing of earthen down
Smooth heavy roller – Gravel in WBM road LO
Vibratory roller – Dry Sand Choice (B)
(i) Based on individual pile failure mode
18. G = 2.65
Qgi = n [Ab CNc + As aC]
w = 0.30
gw= 10 kN/m3 π 
16  × 0.32 × 5 × 9 + π × 0.3 × 12 × 0.75 × 5
=
wG 4 
e=
sγ = 729.46 t
0.3 × 2.65 (ii) Based on Block failure mode
e= = 0.80 Qgb = AB. CNc + As.C
1
AB = Area of block = Bo. Lo
γ w (G + e ) 10 (2.65 + 0.80 )
gsat = = As = Surface area of block = 2 (Bo .+ Lo) . L
1+ e 1.8 Bo = 3s + d = Lo = 330cm
= 19.14 kN/m = 19 kN/m3
3
\  AB = 3.3 × 3.3 = 10.90 m2
g1 = gsat – gw = 19 – 10 = 9 kN/m3 Similarly,As = 2 (3.3 + 3.3) 12 = 158.4m2
e Qgb = 10.90(5) (9) + (158.4) (5) = 1282 .5t
Porosity, n =
1+ e \  Qg = smaller of Qgi and Qgb = 729.46t
0.8 729.46
n= = 0.45 Choice (B) Qs = = 416.8t = 417t Choice (B)
1.8 1.75
19. Φ = 30° 22. Q = KiA
Unconfined compression test, so
s1 = 2C tan a 720cm3
Q= = 12 cm3/sec
a = 45 + Φ/2 = 60° 60 sec
⇒ 1.5 = 2C tan60° ⇒ 12 = K × 0.6 × 10
\ C = 0.43 kg/cm2 Choice (A) K = 2 cm/sec Choice (D)
3.102 | Geotechnical Engineering Test 3

23. S = c1 + s1 tan Φ1 1200


s1 = 245 – 80 = 165 ∆ σ1 = = 9.74 t/m3
\ S = 12 + (165 × tan 30°) = 107.26 kPa Choice (C)
(3.2 + 7.9)2
24. As more than 50% of soil passes through 4.75mm sieve σ1f = σ10 + ∆ σ1
and retains on 75 µ sieve, it is Sand. =7.944 + 9.74 = 17.684 t/m3
Ip = wL – wp = 30
Cc  σ1f 
Equation of A line Ip = 0.73 (wL – 20) \ ∆H = H log10  1 
= 0.73 (35) = 25.55 1 + e0  σ0 
Falls above the A line so it clayey soil 0.342  17.684 
=
2.8 × log10  = 0.17m
\ Given Soil is Clayey Sand. Choice (D)
1 + 0.95  7.944 
25. Loess is the soil transported by wind. Choice (D)
\ ∆H = 170 mm Choice (B)
26. Given is a Sandy soil,
Cv t
S F  BF ( BP + 0.3) 
3
28. Tv =
=  d2
S P  BP ( BF + 0.3)  Tv and Cv are same for the given soil as % degree con-
SP = 6mm, BF = 2.1m and Bp = 0.3 m solidation is same
t α d2
 2.1(0.6) 
2
2
SF =   × 6 mm = 18.375 = 18.4 mm t1  d1 
 0.3 (2.4 )  ∝
t2  d2 
qF BF 2.1
= qF = × 15 = 105 kPa Choice (C) 25
q p Bp 0.3 d1 = = 0.0125m, d2 =10m
2
27. t1 = 50 × 60 sec = 3000 sec
1200 t
3000  0.0125  2
⇒ = 
t2  10 
\ t2 = 61.728 years Choice (B)
29.
8m
Sand
1.5 m rsat = 1.86 t/m3 H

3.2 m GL
Z
Soft soil
D

2.8 m rsat = 1.76 t/m3


clay
Hard stratum

H+D
Depth factor, DF =
H
∆H c  σ1f 
= c log10  1  For any value of D
H 1 + e0  σ0  DF > 1
Cc = 0.009 (wL – 10) = 0.009 (48 – 10) = 0.342 From the given figure; D > O and H > D
⇒ So; irrespect of H and D values, DF > 1
σ10  = g1 (8) + g1 (2.8/2)
So; Base failure
\ σ10 = 0.86 (8) + 0.76 (1.4) = 7.944 t/m3 DF =1 → Toe failure
D F < 1 → Face failure
1200
∆ σ1 = D F > 1 → Base failure Choice (B)
(3.2 + z )2 3 0. If water table rises, σ and u increases but s1 decreases.
Z = 8 – 1.5 + 1.4 = 7.9m  Choice (C)
Foundation Engineering Test 4
Number of Questions: 25 Time: 60 min.

Directions for questions 1 to 25: Select the correct alterna- 12. Match the following
tive from the given choices.
Type of Boring Usage
1. For toe failure the depth factor DF is
a. Auger boring 1. For drilling holes
(A) < 1 (B) > 1
(C) = 1 (D) None of these b. Rotary drilling 2. Advancing holes in the
ground
2. The maximum value of stability number is
c. Core drilling 3. drilling holes in clay
(A) 0.261 (B) 0.281
d. Percussion drilling 4. Sampling for highways,
(C) 0.241 (D) 0.291 railways etc.
3. Which of the following is true for General shear failure?
(A) ID < 20% (B) e > 0.75 a b c d abcd
(C) N > 30 (D) N < 5 (A) 4 1 3 2 (B) 4 3 1 2
4. The ultimate bearing capacity (qf) and net ultimate (C) 2 3 1 4 (D) 1 2 3 4
bearing capacity (qnf) are connected by relation 13. (i) Cement stabilization is done by using mixture of
(A) qnf = qf + gD (B) qf = qnf + gD soil + cement + water + compaction + curing
(C) qf = qnf/F + gD (D) qf = (qnf + gD)/F (ii) Chemical stabilization is done by using calcium
5. The allowable settlement for isolated foundations as chloride and sodium silicate
per in sand and hard clay is (A) (i) and (ii) are true
(A) 75 mm (B) 50 mm (B) (i) and (ii) are false
(C) 100 mm (D) 80 mm (C) (i) is true and (ii) is false
6. The piles that are provided at an inclination, to resist (D) (i) is false and (ii) is true
lateral forces (or) inclined forces is known as 14. A plate load test is conducted on sand on a 500 mm
(A) Tension piles (B) Anchor piles diameter plate. If the plate settlement is 10 mm at a
(C) Fender piles (D) Batter piles pressure of 200 kPa, the settlement of 0.5 m × 8 m foot-
7. If the actual observed value of standard penetration ing will be
resistance N is greater than 15 in a fine sand layer below (A) 17 mm (B) 20.3 mm
water table, then equivalent penetration resistance will be (C) 23.7 mm (D) 18.6 mm

(A) 15 +
( N + 15) (B) 15 +
( N + 15) Common Data for Questions 15 and 16:
2 2 A group of piles of 20 m length and 0.25 m diameter is
(C) 15 +
( N −15) (D) 15 +
(15 − N ) installed in a 10 m thick stiff clay layer underlain by rock.
2 2 The pile soil adhesion factor is 0.3, average shear strength
8. Cantilever sheet pile is generally suitable for depths of soil on the sides is 200 kPa, undrained shear strength of
(A) d ≤ 5 (B) 5 ≤ d ≤ 10 soil at base is 200 kPa.
(C) d ≥ 10 (D) d ≥ 12 15. The base resistance of a single pile is
9. Geotextiles are used for (A) 88 kN (B) 84.6 kN
(A) compacting loose soils (C) 86.2 kN (D) 88.3 kN
(B) reducing settlements 16. The side friction resistance of single pile is
(C) separation and drainage (A) 92.3 kN (B) 94.2 kN
(D) Improve bearing capacity (C) 98 kN (D) 96.5 kN
10. For determining the index properties of soil the sample
17. A canal having side slopes 1 : 1 is proposed to be con-
should be
structed in a cohesive soil to a depth of 10 m below
(A) disturbed (B) undisturbed
ground surface. The soil properties are ΦU = 20°.
(C) intact (D) None of these
11. In standard penetration test conducted at site the Cu = 25 kPa, e = 1, GS = 2.65
recorded values of blow count for every 15 cm penetra- If taylor’s stability number, Sn is 0.08 and canal is full,
tion at a depth of 45 cm are 5, 10, 15 respectively. The the factor of safety with respect to cohesion against
value of SPT blow count (N) that should be used is failure of canal bank slopes is
(A) 15 (B) 17 (A) 3.85 (B) 3.65
(C) 25 (D) 19 (C) 7.85 (D) 1.7
3.104 | Foundation Engineering Test 4

18. The incorrect statement among the following is (B) lowering of ground water table with resulting
(A) The area ratio should be low ground subsidence
(B) The cutting edge should be thick (C) pile driving operations
(C) The inside clearance should be small (D) All the above
(D) The outside clearance should be small 2 3. When the water table rises to ground level in case
19. The seismic method of soil exploration cannot be used of cohesion less soil, the bearing capacity is reduced
for by – %. If it is cohesive soil, the reduction will be____
(A) sub surface investigation (A) 20%, 50%
(B) hard layers (B) 40%, negligible
(C) clays (C) 50%, negligible
(D) sandy soils (D) negligible, 50%
20. The term mobilized shear strength is referred to as 24. Which of the following is false according to terzaghi’s
(A) shear strength theory?
(B) Maximum shear stress (A) zone I is elastic zone
(C) applied shear stress (B) zone II is radial shear zone
(D) None of the above (C) zone III is Rankine’s passive zone
21. The efficiency of pile group for clays is (D) zone IV is surcharge zone
(A) >100% (B) <100% 25. The cohesion and density of a soil are 4 t/m2 and
(C) =100% (D) None of these 8 t/m2 respectively. For a factor of safety of 2 and stabil-
22. Negative skin frictions developed from ity number 0.1 the safe height of slope is
(A) A cohesive fill placed over cohesion less soil (A) 5 m (B) 50 m
deposit (C) 25 m (D) 2.5 m

Answer Keys
1. C 2. A 3. C 4. B 5. B 6. D 7. C 8. A 9. C 10. A
11. C 12. B 13. A 14. C 15. D 16. B 17. A 18. B 19. B 20. C
21. B 22. D 23. C 24. D 25. D

Hints and Explanations


1. Choice (C) Area of footing = 40 m2
2. Choice (A) Settlement of plate = 10 mm
Settlement of footing = ?
3. Choice (C)
Width of plate = 500 × 10-3 m = 0.5 m
4. Choice (B) Width of footing = 8 m
5. Choice (B)
( ) 
2
Sf  B f B p + 0.3
6. Choice (D) =
7. Choice (C)
(
S p  B p B f + 0.3
 ) 
 8 ( 0.5 + 10.3) 
2
8. Choice (A) Sf
=  
9. Choice (C) 10 × 10 −3  0.5 (8 + 0.3) 
10. Choice (A) Sf = 23.7 mm Choice (C)

11. The SPT value have to be calculated by leaving the 15. Base resistance of single pile in clay = Ab CNC
reading of first 15cm and adding the other penetration π π
= d 2CN c   = × (0.25) × 200 × 9
2

readings. 4 4
i.e., The SPT value = 10 + 15 = 25. Choice (C)  = 88.31 kN Choice (D)
12. Choice (B) 16. Side friction resistance
13. Choice (A) = AS ∝ C
14. Dia of plate = 500 mm = 2 × 3.14 × 0.25 × 0.3 × 200
Area of plate = 0.196 m2 = 94.2 kN. Choice (B)
Foundation Engineering Test 4 | 3.105

c 20. Choice (C)


17. S n =
F × H × γ1 21. Choice (B)
22. Choice (D)
γ w (G + e )
γ1 = 23. Choice (C)
1+ e
24. Choice (D)
9.8 × (2.65 + 1)
  = = 8.08 kN/m 2
1+1 C
25. S n =
25 Fc γH
0.08 =
F × 10 × 8.08 4
⇒ 0.1 =
⇒ F = 3.85 Choice (A) 2×8× H
18. Choice (B) 4
H= = 2.5 m  Choice (D)
19. Choice (B) 2 × 8 × 0.1
This page is intentionally left blank
Unit VIII
Fluid Mechanics
and Hydraulics
This page is intentionally left blank
Fluid Mechanics Test 1
Number of Questions: 25 Time: 60 min.

Directions for questions 1 to 25: Select the correct alterna- (C) decreases with fall in temperature
tive from the given choices. (D) acts in a plane of interface normal to any line in
1. When pressure is increased, the bulk modulus of elas- the surface
ticity ‘K’+ 10. For stability of a floating body
(A) decreases (A) the meta centre ‘M’ should lie between the centre
(B) increases of gravity ‘C’ and centre of buoyancy ‘B’
(C) remains same (B) M should lie above B and C
(D) decreases then increase (C) M should coincide with B and C
2. The viscosity of water and the viscosity of air with (D) M should lie below B and C
increase in temperature
11. An inverted U – tube differential manometer is used to
(A) decrease and increases
measure pressure difference in an inclined water pipe
(B) increases and decreases
as shown in figure. The manometer fluid is oil of spe-
(C) decreases and decreases
cific gravity 0.75
(D) decreases and remains same
3. Pascal’s law for a fluid is not valid if B
(A) fluid is at rest
(B) fluid is at constant rotational velocity in a container
(C) fluid is at constant linear acceleration 0.5 m
(D) None of the above water
4. An inverted U tube manometer is more sensitive than A
an upright manometric because
(A) the height of levels is greater
(B) the manometric fluids are heavier than working 0.8 m oil
fluids 1.2 m
(C) the manometric fluids are lighter than working
fluids 0 0
(D) None of the above
5. A vertical wall is holding a liquid of specific weight 0.7 m
‘w’ and height ‘h’ on one side. The total pressure on the
wall per unit length is
(A) wh (B) wh/2
wh2 2
(C) (D) wh
2 3
Pressure difference between points A and B in N/m2 is
6. If the surface of liquid is concave, then cohesive pres-
(A) 1792 (B) 2882
sure is
(C) 3679 (D) 4216
(A) increased
(B) decreased 12.
(C) absent Carbon tetra chloride
(D) negligible or doesn’t matter
A
7. A body weight 4 kg in air was found to weigh 3.5 kg
when submerged in water. Its specific gravity 3m
(A) 1 (B) 3 Oil
(C) 6 (D) 8 B
8. If density of liquid r = 1000 kg/m3 and area A = 1 m2. 1m
Then flow rate Q at t = 0, (x, y) is (0, 1)
(A) 100 (B) 1000 h
(C) 0 (D) Can’t be known
9. Surface Tension is
(A) also known as capillarity
(B) is a function of curvature of interface Mercury
3.110 | Fluid Mechanics Test 1

Referring to the figure, pipe A contains carbon tet- 15. Mass flow rate of the oil in kg/s is
rachloride of sp gravity 1.59 under a pressure of (A) 2.82 (B) 2.64
105 kN/m2 and pipe B contains oil of sp gravity 0.8 under (C) 2.41 (D) 2.22
pressure 170 kN/m2. Level difference h shown by the 16. Power required to pump oil per 100 m length of pipe is
manometric fluid mercury is (A) 6.12 kw (B) 6.34 kw
(A) 72 mm (B) 83 mm (C) 6.63 kw (D) 6.82 kw
(C) 95 mm (D) 115 mm
17. Water flows through an inclined pipe in which a ven-
13. Water flows through an inclined venturimeter as shown turimeter is installed for discharge measurement. The
in figure. Inlet and throat diameters are 100 mm and inlet and throat sections of the venturimeter have areas
50 mm respectively. Inlet and throat sections have a of cross sections 0.07 m2 and 0.0177 m2 respectively.
level difference of 300 mm. The differential mercury An inverted U-tube manometer is used for measure-
manometer connected across inlet and throat indicates ment of differential pressure head. A liquid of specific
12 cm of mercury level difference at a given flow rate. gravity 0.7 is used in the manometer, which gives a
Coefficient of discharge is 0.99. reading of 250 mm. Inlet and throat sections have a
level difference of 400 mm

200 mm
2

1 300 mm
(1)

12 cm
(2)

The rate of flow in litres/s is Neglecting frictional losses the rate of flow through the
(A) 14.76 (B) 12.85 pipe in m3/s is
(C) 10.91 (D) 8.86 (A) 0.028 (B) 0.022
(C) 0.019 (D) 0.016
14.
18. Vertical distance between the two orifices is
(A) 3.16 m (B) 2.82 m
2
(C) 2.25 m (D) 2.06 m
19. Vertical distance of point of intersection of the jets
from the water level in the tank is
M2
(A) 9.32 m (B) 10.25 m
(C) 10.78 m (D) 11.16 m
1 30°
20. Velocity of kerosene at point B is
(A) 7.8 m/s (B) 6.7 m/s
(C) 5.6 m/s (D) 4.2 m/s
M1 21. Pressure of kerosene at point B is
(A) -29.32 kPa (B) - 21.97 kPa
Water flows through a tapering pipe inclined at 30° to (C) -18.14 kPa (D) - 12.82 kPa
the horizontal. At points 1 and 2 manometers are con- 22. Match List – I with List – II and select the correct
nected. Point 1 is at an elevation of 1 m from ground answer
level and 2 is 3 m from ground level. Diameter at section List – I List – II
1 and 2 are 15 cm and 10 cm respectively. Velocity at (Forces) (Dimensionless number)
1 is 6 m/s. If manometer M2 reads 10 cm of mercury, a. Gravity force 1. Weber number
the reading shown by manometer M1 in cm of mercury
b. Pressure force 2. Mach number
is
(A) 79.5 (B) 65.6 c. Surface tension 3. Froude’s Number
(C) 58.3 (D) 49.4 d. Elastic force 4. Euler’s number
Fluid Mechanics Test 1 | 3.111

Codes: 24. Match List – I with List – II and select the correct
a b c d abcd answer
(A) 1 2 4 3 (B) 4 3 2 1
(C) 3 4 1 2 (D) 2 1 3 4 List – I
List – II
(condition of floating
2 3. A U-tube mercury manometer is used to measure pres- (Result)
bodies)
sure of oil flowing through a pipe at a point. Specific a. M below G 1. Floating body
gravity of oil is 0.8 and the level of mercury is as shown b. M above G 2. Unstable equilibrium
in the figure. The pressure in kPa is
c. M and G coincides 3. Stable equilibrium
d. B below G 4. Neutral equilibrium
[M - Meta centre
G - Centre of gravity
B - centre of buoyancy]
Oil (S.G : 0.8)

A• 150 cm Codes:
a b c d abcd
50 cm
(A) 3 4 1 2 (B) 2 3 4 1
(C) 1 2 3 4 (D) 4 1 2 3
25. A glass tube of 3.7 mm diameter is dipped in water. If
the contact angle at the meniscus is 0° and surface ten-
sion is 0.074 N/m determine the capillary effect in mm.
(Take specific weight of water as 10000 N/m2)
(A) 196.20 (B) 147.15 (A) 4 mm(dep) (B) 4 mm(rise)
(C) 110.36 (D) 73.58 (C) 8 mm(dep) (D) 8 mm (rise)

Answer Keys
1. A 2. A 3. D 4. C 5. C 6. B 7. D 8. B 9. C 10. B
11. C 12. B 13. C 14. A 15. A 16. C 17. B 18. C 19. B 20. D
21. B 22. C 23. A 24. B 25. D

Hints and Explanations


7. Ebg = 4kg (rb – rw) g = 3.5 kg 7
δ = 12.8h + 5.56
Ewg = 0.5 kg 40
ρ h = 0.083 m = 83 mm. Choice (B)
Specific gravity = b = 8  Choice (D)
ρw 13. For manometer,
8. Q = rAV
 p1   p2   sm 
V = (6xt + 3y2) i + (3t + y2) j = j  + z1  −  + z2  −  − 1 hm = h
w w  sw 
Q = 1000 × 1 × 1= 1000 kg/s. Choice (B)
where hm = the manometric level difference
11. Considering pressure balance on the horizontal plane Applying Bernoulli’s equation,
0-0
 v2 − v1
2 2
pA + 0.8 × 9810 = pB + (0.5 + 1.2) × 9810 - 0.7  p1   p2
 + z1 − + z =
× 0.75 × 9810 w   w 2
 2g
pA - pB = 9810[(0.5 + 1.2) - 0.7 × 0.75 -0.8]
   = 9810 × 0.375 = 3679 N/m2. Choice (C) v2 2 − v12
=h
12. Starting from pipe A the manometric equation is 2g
pA + 4 × 1.59 × 9810 + h × 13.6 × 9810 - (h + 1) Also A1v1 = A2v2
× 0.8 × 9810 = pB The discharge equation for venturimeter
4 fL v 2
× = 6.36 + h × 13.6 − 0.8h − 0.8 Cd A1 A2 2gh
D 2g Q= is derived from the above
A12 − A2 2
= 12.8 h + 5.56
3.112 | Fluid Mechanics Test 1

\ Irrespective of inclination h corresponding to \ P = 9.81 × 2.816 × 2.4 = 66.26 W


manometric reading can be directly used in the above power required per 100 m
equation = 6626 W = 6.626 kW. Choice (C)
s   13.6  1 7. Differential pressure head
h = hm  m − 1 = 0.12  − 1 = 1.512 m
 sm   1   s   0.7 
h = hl 1− l  = 0.25 1 − = 0.075 m of water
Cd = 0.99  sw   1.0 
π
A1 = (0.1) = 7.85 × 10
2 −3
A1 A2
4 Q= 2gh
A12 − A2 2
π
A2 = (0.05)2 = 1.96 × 10 −3
4 0.07 × 0.0177 2 × 9.81 × 0.075
  =
0.99 × 7.85 × 1.96 × 10 −6 0.072 − 0.01772
Q= × 2 × 9.31 × 1.512
10 −3 7.582 − 1.962   = 0.022 m3/s. Choice (B)
   =0.01091 m3/s = 10.91 l/s. Choice (C) 18. horizontal distance x = ut
1 4. d1 = 15 cm: z1 = 1 m; v1 = 6 m/s; d2 = 10 cm; z = 3 m 1 2
2 vertical distance z = gt
A1 d  15
2 2
v2 = v1 =  1  v1 =   × 6 = 13.5 m/s
A2  d2   10  u = 2gH
Applying Bernoulli’s theorem From the above,
p1 v12 p v2 x2
+ + z1 = 2 + 2 + z2 z=
w 2g w 2g 4H
where H = height of water level above orifice
p1 v2 2 − v12 p
= + ( z2 − z1 ) + 2 For the upper orifice,
w 2g w
x2 x 2 100
z1 = − = = = 6.25
13.5 − 6
2 2
4 × 4 16 16
= + (3 − 1) + (0.1 × 13.6)
2 × 9.81 For the lower orifice,
10.81 100 100 25
= 7.45 + 2 + 1.36 = 10.81 m of water =
 z2 = = =
13.6 4( 4 + h) 4( 4 + h) ( 4 + h)
= 0.795 m of mercury = 79.5 cm of mercury.
 Choice (A) 25
z1 - z2 = h = 6.25 - or
1 5. m = 1.2 poise ( 4 + h)
   = 0.12 Ns/m2 25
h+  = 6.25
sp. gravity = 0.85 ( 4 + h)
r = 850 kg/m3 D = 0.03 m
(4 + h) h + 25 = 6.25 (4 + h)
Pressure drop/m = 20 kN/m2
4h + h2 + 25 = 25 + 6.25 h
32µuL
∆p= h2 + 4h - 6.25 h = 0
D2 h - 2.25 = 0
32 × 0.12 × u × 1 h = 2.25 m. Choice (C)
20 × 103 =
(0.03) 2
25 25
19. z2 = = =4
u = 4.6875 m/s ( 4 + h) 4 + 2.25
π Vertical distance of water level from point of intersec-
mass flow rate = r A u = (850) ×   × (0.03)2 × 4.6825
4 tion of jets
= 2.816 kg/s. Choice (A) L = 4 + h + z2 = 4 + 2.25 + 4 = 10.25 m. Choice (B)
1 6. Power required per metre 20 Velocity of kerosene at point B = velocity at C
p = w Q hf = g(rQ) hf Applying Bernoulli’s theorem, between oil surface in
where rQ = r A u = mass /s = 2.816 the tank and pipe outlet

p2 − p1 20 × 1000 p1 v12 p v2
hf =
 = = 2.4 m of oil + + z1 = 2 + 2 + z2 + h f (1− 2 )
ρg 850 × 9.81 w 2g w 2g
Fluid Mechanics Test 1 | 3.113

v2 2 23. Equating pressure heads above the line passing


0+0+4=0+ + 1.5 + (0.4 + 1.2) through A
2g
50 150
v2 2 hA + × 0.8 = × 13.6
= 4 − 1.5 − 1.6 = 0.9 m 100 100
2g
\ hA + 0.4 = 20.4
v22 = 0.9 × 2 × 9.81 ⇒ hA = 20 m of water
v2 = 4.2 m/s = vB. Choice (D) Pressure pA = whA = 9810 × 20 N/m2
2 1. Applying Bernoulli’s equation at points 1 and B = 9.81 × 20 kN/m2 = 196.2 kN/m2
= 196.2 kPa. Choice (A)
p (4.2) 2

0+0+4= B + + 5.5 + 0.4


w 2g 4σ cos θ
25. Capillary effect h =
wd
pB 4.22
= 4 − 5.9 − = -2.8 m where s = surface tension, q = contact angle
w 2 × 9.81
w = specific weight
pB = -2.8 × (9810 × 0.8)
   = -21967 N/m2 4 × 0.074 × cos 0°
  =  = 8 mm(rise). Choice (D)
   = -21.97 kPa. Choice (B) 10000 × 3.7 × 10 −3
Fluid Mechanics Test 2
Number of Questions: 30 Time: 75 min.

Directions for questions 1 to 30: Select the correct alterna- 4. A curved surface is submerged in a fluid. Consider the
tive from the given choices. following statements relating to it
1. A metallic cube of side 200 mm and specific weight I. Vertical component of the hydrostatic force acting
26 kN/m3 is suspended by a string in oil and water as on the surface is equal to the weight of the fluid
shown in the figure. Half of the cube is submerged in vertically above the surface
water and the remaining half is submerged in oil. If II. Horizontal component of the force acting on the
specific gravity of the oil is 0.8, determine the tension curved surface is the hydrostatic force acting on
in the string the vertical projection of the curved surface
III. Horizontal component of the force acts through
the centre of gravity of the vertical projection of
the curved surface.
T (A) I and II are correct
(B) I and III are correct
(C) II and III are correct
oil (D) I, II and III are correct
100 mm 5. In a horizontal pipeline as shown in figure, point 2 is a
contraction with reduced area of cross section. At point
1 the pressure head and velocity head are 60 cm and
100 mm

4 cm respectively. If pressure head at point 2 is zero, the


ratio of velocity at point 2 to that at point 1 is
200 mm
water

1 2
(A) 98.2 N (B) 176.6 N
(C) 156.7 N (D) 192.4 N
2. In a 3-dimensional incompressible fluid flow, velocity (A) 2 (B) 3
components in x and y directions are (C) 4 (D) 6
u = x2 + y2z3 6. For a flow, velocity components in the x and y direc-
v = - (xy + yz + zx) tions are given by u = y2, v = -3x
Velocity component in the z direction is Component of rotation about the z-axis is
z2 z2 (A) -(3 + 2y) (B) (3 + 2y)
(A) - xz + + f ( x, y ) (B)
- xz +
2 2 −1 1
(C) (3 + 2 y ) (D) (3 + 2 y )
z2 2 2
(C) xz - + C (D)
-x+z
2 7. The velocity along the centre line of a nozzle of length
1.5 m is given by
3. Match List – I (measuring devices) with List – II 2
(measuring parameter) and select the correct answer  x
v = 2t 1 −  where  = length in m
using the codes given below  2 

List – I List – II v = velocity in m/s,


t = time in seconds
a. Pitot tube 1. Rate of flow measurement
from the commencement of flow and x = distance from
b. Micro manometer 2. Measurement of moderate
inlet. The value of local acceleration at x = 1 m when
pressure
t = 5 seconds is
c. Venturimeter 3. Velocity measurement
(A) 0.67 m/s2 (B) 0.89 m/s2
d. Piezo meter 4. Easier measurement of (C) 1.33 m/s2 (D) 1.67 m/s2
large pressures
8. Differential pressure head measured by a mercury oil
Codes: differential manometer is 9.5 m of oil. If specific grav-
a b c d a b c d ity of oil is 0.68, difference in level of mercury is
(A)
1 3 2 4 (B) 4 2 3 1 (A) 300 mm (B) 400 mm
(C)
2 1 4 3 (D) 3 4 1 2 (C) 500 mm (D) 600 mm
Fluid Mechanics Test 2 | 3.115

9. An orifice meter is calibrated with air in a geometri- Common Data for Questions 15 to 17:
cally similar model. Model to prototype scale ratio is An 80 mm diameter composite solid cylinder consists of a
1 20 mm thick metallic plate and 650 mm long wooden cyl-
4
. The prototype has to carry water. Ratio of kine-
inder of specific gravity 4 and 0.8 respectively. The cylinder
matic viscosity of air to water is 12.5. Dynamically floats in water its axis vertical
similar flow will be obtained when the discharge ratio 15. The position of centre of gravity from bottom is
is (A) 0.3 m (B) 0.35 m
(A) 2.850 (B) 3.125 (C) 0.4 m (D) 0.45 m
(C) 4.540 (D) 4.925
16. Position of centre of buoyancy from bottom is
10. For a flow, the stream function is y. For the flow to be (A) 0.2 m (B) 0.25 m
irrotational, the condition to be satisfied is (C) 0.3 m (D) 0.35 m
∂ψ ∂ψ ∂ψ ∂ψ 17. Metacentric height is
(A) − = 0 (B) + =0
∂x dy ∂x ∂y (A) 0.667 mm (B) 6.67 mm
(C) 3.33 mm (D) 8.63 mm
∂2 ψ ∂2 ψ ∂2 ψ ∂2 ψ
(C) − = 0 (D) + =0 18.
∂x 2 ∂y 2 ∂x 2 ∂y 2 G

EI.18 m
Common Data for Questions 11 and 12: Air
A liquid of viscosity 0.8 and sp. gravity 1.3 flows through a EI.15 m
circular pipe of 100 mm diameter. Maximum shear stress at Liquid Ι
the pipe wall is 220 N/m2 EI.12 m S = 0.7

11. Pressure gradient of the flow in N/m2 per m is A


Wter
(A) - 6800 (B) - 8800
(C) 6800 (D) 8800
EI.8 m
12. Average velocity of flow is Liquid ΙΙ
(A) 2.6 m/s (B) 2.9 m/s S = 1.6
EI.6 m
(C) 3.2 m/s (D) 3.4 m/s
Statement for Linked Answer Questions 13 and 14:

K Viscosity  U
A tank installed at an elevation of 6 m contains a liquid
of specific gravity 1.6, water and another liquid of spe-
h
h–
cific gravity 0.7 space over the liquids contain air. The
Viscosity α
gauge G show a pressure of -17 kN/m2. The elevation
of liquid level in the piezometer A is
(A) 4.73 m (B) 6.73 m
A large thin plate is pulled at a constant velocity U through (C) 8.73 m (D) 10.73 m
a narrow gap of height h. On one side of the plate is filled 19.
with oil of viscosity m and the other side oil of viscosity am Water Water
where a is a constant
A B
13. Total drag force on the plate is Oil
1 α   h−k
(A) AmU  + AmU  k +
(B)
 k h − k   
α 
µU  1 α  0.8 m
µU  h−k
(C)  +  (D)  k +  1m
A  k h−k A α 
0.5 m
0.4 m
0.6 m

14. Value of k such that the drag force is minimum is


h h
(A) (B)
1+ α 1− α
h h
(C) (D)
1− α 1+ α Mercury
3.116 | Fluid Mechanics Test 2

For the compound manometer shown in figure, the 24. The velocity profile of a fully developed laminar flow
pressure difference between points A and B in kN/m2 in a straight circular pipe, as shown in the figure, is
is -------- given by the expression
Given that specific gravity of mercury = 13.6 and spe- 2  1 − r2 
cific gravity of oil = 0.85 u(r) =
−R  ∂p 
 
4 µ  ∂x   2 
(A) 115 (B) 125  R 
(C) 135 (D) 150 ∂p
Where is a constant
Statement for Linked Answer Question 20: ∂x

Open end
R

Air
The average velocity of fluid in the pipe is
− R2  dp  − R2  dp 
(A)   (B)  
0.5 m 8µ  dx  4µ  dx 
Water
− R2  dp  − R2  dp 
(C)   (D)  
2m
2µ  dx  µ  dx 
25. Water having a density of 1000 kg/m3, issues from a
G nozzle with a velocity of 10 m/s and the jet strikes a
Mercury
bucket mounted on a petton wheel. The wheel rotates at
10 rad/s. The mean diameter of the wheel is 1m. The jet
Refer to the figure given above. The tank is filled with water is split into two equal streams by the bucket, such that
upto 2 m from the gauge G. The manometer shows a level each stream is deflected by 120°, as shown in the figure.
difference of 0.5 m as shown. Local atmospheric pressure is Friction in the bucket may be neglected. Magnitude of
750 mm of mercury. the torque exerted by the water on the wheel, per unit
20. Absolute pressure of air above the water surface in the mass flow rate of the incoming jet is.
tank is Deflected jet
(A) 21.68 kN/m2 (B) 33.35 kN/m2 Incoming jet 120°
(C) 38.72 kN/m 2
(D) 42.83 kN/m2
120°
Common Data for Questions 21 and 22: Deflected jet
A propeller turbine is to develop 6250 kW under a head
of 5m, having given that speed ratio ‘ku’ based on outer di- (A) 0 (Nm) / (Kg/s) (B) 1.25 (Nm) / (Kg/s)
ameter = 2.10, flow ratio y = 0.65, diameter of boss = 0.35 (C) 2.5 (Nm) / (Kg/s) (D) 3.75 (Nm) / (Kg/s)
times external diameter of the runner and overall efficiency 2 6. Match the following:
is 85%.
List – I List – II
21. The diameter of the runner in ‘m’ is
P. Compressible flow 1. Nusselt number
(A) 5.81 (B) 4.91
Q. Boundary layer flow 2. Reynold’s number
(C) 5.21 (D) 6.35
R. Pipe flow 3. Skin friction coefficient
22. The speed of the turbine in RPM is
S. Heat convection 4. Mach number
(A) 78.27 (B) 68.37
(C) 58.35 (D) 48.22 Codes:
23. Draft tube is a pipe used in P Q R S P Q R S
(A) reaction turbine for discharge and it has gradually (A) 3 1 4 2 (B) 3 4 2 1
decreasing cross sectional area (C) 4 3 2 1 (D) 2 1 3 4
(B) reaction turbine for discharge and it has gradually Common Data for Questions 27 and 28:
increasing cross sectional area A pelton wheel has to be designed for the following data.
(C) Impulse turbine for discharge and it has gradually Power to be developed = 6000 kW net head available
decreasing cross sectional area = 300 m, speed = 550 RPM, ratio of jet diameter to wheel
(D) Impulse turbine for discharge and it has gradually 1
diameter = and overall efficiency = 85%
increasing cross section area. 10
Fluid Mechanics Test 2 | 3.117

27. Find the no. of jets required 1 − cos θ 1 + cos θ


(A) 1 (B) 2 (A) (B)
2 2
(C) 3 (D) 4 cos θ
(C) (D) None of these
28. The diameter of jet is in mm 2
(A) 135.2 30. Incase of Pelton turbine installed in a hydraulic power
(B) 116.4 plant the gross head available is the vertical distance
(C) 141.2 between
(D) 186.4 (A) forebay and tailrace
29. The maximum hydraulic efficiency for a pelton (B) reservoir level and turbine inlet
wheel is given by the expression (q is the vane angle (C) forebay and turbine inlet
of outlet). (D) reservior level and tail race

Answer Keys
1. B 2. A 3. D 4. A 5. C 6. C 7. B 8. C 9. B 10. D
11. B 12. D 13. A 14. D 15. A 16. C 17. A 18. D 19. C 20. B
21. A 22. B 23. B 24. A 25. D 26. C 27. C 28. B 29. B 30. B

Hints and Explanations


1. Tension in the string T = W - FB 5. Applying Bernoullis theorem,
where W = weight of the cube velocity head + pressure head = constant
FB = Buoyant forces V2 p V 2 p
\ T = (0.2)3 × 26 × 103 - [(0.2)3 × 0.5 × 9810 × 0.8] i.e., 1 + 1 = 2 + 2
2 g ρg 2 g ρg
  = 26 × 8 - 4 × 9.81 × 0.8 = 176.6 N. Choice (B)
V2 2
2. For a three dimensional, incompressible fluid flow the \ 0.04 + 0.6 = +0
continuity equation must be satisfied 2g
∂u ∂v ∂w V2 2
i.e., + + =0 ⇒ = 0.64
∂x ∂y ∂z 2g
u = x2 + y2z3 V2 2
2
∂u 2 g  V2  0.64
= 2x 2
=   = = 16
∂x V1  V1  0.04
v = -(xy + yz + zx) 2g
∂v V2
= − (x + z) ⇒ = 4 . Choice (C)
∂y V1
∂w 6. u = y2, v = -3x
\ 2x - (x + z) + =0
∂z Rotation component about z-axis is
∂w
⇒ = −2 x + x + z = -x + z 1  ∂v ∂u  1 1
∂z ωz =  −  = ( −3 − 2 y ) = - (3 + 2 y )
2  ∂x ∂y  2 2
Integrating with respect to z
 Choice (C)
2
z2  x 
w = -xz + + C where C = f(x, y) 7. V = 2t 1 − 
2  2 
2
z2 ∂V  x
or w = -xz + + f ( x, y ) . Choice (A) Local acceleration = 1 −  × 2
2 ∂t  2 
4. Statements I and II are correct  1 
2
 1
2

= 1 − × 2 = 1 −  × 2
The horizontal component acts at a distance from top  2 × 1.5   3
Ι
surface = x + G where x = the vertical distance of 2
8
Ax  2
=   × 2 = = 0.89 m/s2 Choice (B)
centre of gravity from surface Choice (A)  3 9
3.118 | Fluid Mechanics Test 2

S  du U
8. h = y  m − 1 13. t1 = µ1 =m
 So  dy k
 13.6  du U
9.5 = y  − 1 = y × 19 t2 = m2 = αµ
 0.68  dy h−k
9.5 1 α 
⇒ y = = 0.5 m = 500 mm. Choice (C) Total drag force F = A(t1 + t2) = A m U  + .
19  k h − k 
Lm 1  Choice (A)
9. Scale ratio =
Lp 4 dF
14. Drag force is minimum when =0
Ratio of kinematic viscosity dk
υair υ −1 α
= m = 12.5 + =0
υwater υ p k 2
(h − k )2
For dynamically similar flow, Reynold number will be α 1
same =
(h − k ) 2
k2
i.e., ( Re )m = ( Re) p
α 1
ρVL VL =
Re = = h−k k
µ υ
k α =h-k
Vm Lm Vp Lp
\ = k+k α =h
ϑm ϑp
V Lp υm
(
k 1+ α = h )
⇒ m = . = 4 × 12.5 = 50 k = h/(1+ α ). Choice (D)
V p Lm υ p
2 15.
Q L  V
Discharge ratio m =  m  × m
Qp  L p  Vp
2
 1
  × 50 = 3.125 Choice (B)
=
4 M×
650
10. If Y is stream function
mm
∂ψ ∂ψ G×
= v and = −u
∂x ∂y
∂v ∂u B× h
For irrotational flow, − =0
∂x ∂y 20 mm

∂ψ ∂ψ
2 2
i.e., + = 0 Choice (D) O
∂x 2 ∂y 2
Position of centre of gravity
11. m = 8 poise = 0.8 Ns/m2, sp. gravity = 1.3
D = 100 mm = 0.1 m, to = 220 N/m2   0.65  
A 0.02 × 4 × 0.01 + 0.65 × 0.8  0.02 + 
−∂p R  2  
to = × OG = 
∂x 2 A (0.02 × 4 + 0.65 × 0.8)
−∂p 0.1 where A = area of cross section
220 = ×
∂x 2 × 2 8 × 10 −4 + 0.1794
\ pressure gradient      = = 0.3 m Choice (A)
0.6
∂p
= - 8800 N/m2 per m. Choice (B) 16. Weight of cylinder = buoyancy force
∂x
A (0.02 × 4 + 0.65 × 0.8) = A × h × 1
1 2. Average velocity
2 h = 0.6 m
1 1  −1 ∂p  1  0.1 h
u = umax =  . .R2   =  × 8800 ×   Position of centre of buoyancy OB = = 0.3 m
2 2  4 µ ∂x  8 × 0.8  2 
2
  =3.4375 m/s. Choice (D)  Choice (C)
Fluid Mechanics Test 2 | 3.119

Ι π (0.08) × 4
4
(0.08)2 πDN
17. BM = = = = u=
V 64 π × (0.08)2 × 0.6 16 × 0.6 60
π × 5.81 × N
6.67 × 10–4 m 20.8 =
60
BG = OG – OB = 0.3 – 0.3 = 0
N = 68.37 RPM Choice (B)
MG = BM – BG = 6.67 × 10–4 m
    = 0.667 mm Choice (A) 25.
Deflected
1 8. Air pressure = - 17 kN/m2
Incoming
60° 120°
−17000 v
= -17000 N/m2 = = - 1.733 m of water
9810
Pressure at the bottom of tank in metres of water
= - 1.733 + (15 - 12) × 0.7 + (12-8) × 1 + (8 - 6) × 1.6 f = 60°, K = 1 (No friction)
= 7.567 m of water V = 10 m/s
7.567 1
In metres of liquid II, pressure = = 4.73 m Wheel speed = u = 10 × = 5 m/s
1.6 2
w
Elevation of level in piezometer = 4.73 + 6 Work done per sec by wheel = (v – u) (1 + kcosf) u
= 10.73 m. Choice (D) g
1 9. Starting from point A and forming the manometric mg
Power = (v – u) (1 + kcosf) u
equation, in metres of water g
pA pB P = Tw
+1- 0.6 Sm+ 0.4 So- 0.5 Sm -(0.8- 0.5) =
w w T (v − u ) (1 + k cos φ) u (10 − 5)(1 + cos 60°) 5
=  = 
Where w = sp. weight of water m ω 10
Sm = sp. gravity of mercury    = 3.75 Nm / (kg/s) Choice (D)
So = sp. gravity of oil 2 7. Kv = 0.98, Ku = 0.46
p A − pB
i.e., = (0.6 + 0.5) 13.6 - 0.4 × 0.85 -1 + 0.3 V = kv 2gh = 0.98 2 × 9.8 × 300 = 75.19 m/sec
w
u = ku 2gh = 0.46 2 × 9.8 × 300 = 35.29 m/s
=13.92
pA - pB = 13.92 × 9810 N/m2 = 136.56 kN/m2 p
h0 =

Choice (C) WQH
2 0. Atmospheric pressure 6000 × 103
0.85 =
1 9810 × Q × 300
patm = ρAU × 13.6 × 9810 = 100062 N/m2
2

2 Q = 2.39 m3/s
pair = 0 - 0.5 × 13.6 × 9810 = - 66708 N/m2 πDN π × D × 550
u= =
Absolute pressure of air 60 60
pair (abs) = patm + pair = 100062 - 66708 D = 122.5 m
= 33354 N/m2 = 33.354 kN/m2. Choice (B) d 1
=
D 10
6250 × 103
21. h0 = 0.85 = d = 0.1225 m = 122.5 m
9810 × Q × 5
π
A = (0.1225) = 0.0118 m2
2
Q = 149.91 m3/sec
4
vf
0.65 = Q 2.399
2 × 9.81 × 5 Total jet area required = = = 0.0319 m2
V 75.19
Vf = 6.44 m/s 0.0319
π 2 No. of jets required = = 2.7 = 3 Choice (C)
Q = 149.91 =  D − (0.35 D )2  × 6.44 0.0118
4 
1

D = 5.81 m Choice (A)  2


 0.0319 
u
22. Ku = 2.1 = 2 8. d =   = 0.1164 m = 116.4 mm Choice (B)
2 × 9.81 × 5  3 π  
  4  
u = 20.8 m/s
This page is intentionally left blank
Hydraulics Test 3
Number of Questions: 25 Time: 60 min.

Directions for questions 1 to 25: Select the correct alterna- (A) 182.5 L/s (B) 190.2 L/s
tive from the given choices. (C) 206.8 L/s (D) 242.4 L/s
1. In a fluid the velocity field is given by 7. A moving vane with velocity 20 m/s having an inlet
V = (3x + 2y) i + (2z + 3x2) j + (2t - 3z) k angle zero degree and an outlet angle 25° receives
The velocity at point (1, 1, 1) at time 2 second is water from a jet at a velocity of 48 m/s. Assuming mass
(A) 6.82 units (B) 7.14 units flow rate of 1kg/s, the force acting on the vanes and its
(C) 7.93 units (D) 8.26 units inclination is.
Statement for Linked Answer Questions 2 and 3: (A) 62.16 N, 15°
The velocity along the centre line of a nozzleof length l (B) 58.72N, 13.6°
is given by, (C) 54.68 N, 12.5°
2 (D) 50.25N, 10.4°
 x
  V = 2 t 1 − 
 2l  8. In the model study of a motor boat in a lake, 1 : 25 scale
model is used. It is assumed that viscous resistance due
where V = velocity in m/s, t = time in seconds,
to water and air is negligible compared to the wave
     x = distance from inlet of the nozzle
resistance. If the speed of the model is 2 m/s, deter-
2. Convective acceleration when x = 1 m and l = 1.5 m mine the speed of the prototype for dynamically similar
and t = 5 seconds is conditions
(A) -19.75 m/s2 (B) 19.75 m/s2 (A) 8 m/s (B) 10 m/s
(C) 26.35 m/s (D)
2
-26.35 m/s2 (C) 12 m/s (D) 15 m/s
3. Total acceleration for the same conditions as above is 9. A pipe line through which oil flows has a sudden expan-
(A) 20.64 m/s2 (B) -20.64 m/s2 sion in it such that maximum pressure rise occurs.
(C) -18.86 m/s 2
(D) 18.86 m/s2 Energy loss in the sudden expansion in metres of oil is
4. Velocity distribution in the boundary layer of fluid flow given by (V1 = velocity before expansion)
over a surface is given by V12 V12
(A) (B)
u 3 y 1 y2 4g 6g
= −
U 2 δ 2 δ2
V12 V12
When d = the boundary layer thickness (C) (D)
U = maximum velocity of fluid 8g 2 2g
u = fluid velocity at y 10. Prandtl’s velocity distribution for a boundary layer is
The ratio of displacement thickness to boundary layer given by
thickness is 2
5 7 u  y  y
(A) (B) (A) = 2  −  
12 12 U  δ  δ
5 3 3
(C) (D) u 3  y 1  y
7 7 (B) =  −  
U 2 δ 2 δ
5. A Kaplan turbine develops 20000 kW at an average 3 4
head of 35 m. Assuming a speed ratio of 2, flow ratio u  y  y  y
(C) = 2  − 2  +  
of 0.6 and overall efficiency of 90% and taking boss to U  δ  δ  δ
runner diameter ratio as 0.35, speed of the turbine is
(A) 365 rpm (B) 388 rpm u π y
(D) = sin  
(C) 409 rpm (D) 418 rpm U 2 δ
6. A liquid is flowing between two parallel plates. One
11. Specific speed of a hydraulic pump is the speed of
plate is moving relative to other with a velocity of
geometrically similar pump working against unit head
4 m/s in the negative direction. If pressure gradient
and
∂p
= −100 × 106 N/m3 , viscosity m = 0.4 poise and (A) consuming unit power
∂x (B) having unit velocity of flow
distance between the plates is 1 m, discharge per meter (C) having unit radial velocity
width is (D) delivering unit quantity of water
3.122 | Hydraulics Test 3

12. Match List – I with List – II and select correct answer (A) 80% (B) 77.65%
using the codes given (C) 90% (D) 78.83%
List – I List – II 1 9. In two pelton wheels, the first wheel deflects the water
a. Pelton turbine 1. Mediumdischarge, low through 160° and the other through 170°. The ratio of
(single jet) head the maximum efficiencies of the first and second wheels
b. Francis turbine 2. High discharge, low head is (all other operating parameters being same for both)
(A) 1.023
c. Kaplan turbine 3. medium head
(B) 0.966
4. Low discharge high head
(C) 1.035
a b c abc (D) 0.977
(A) 1 2 3 (B) 4 3 2 Common Data for Questions 20 and 21:
(C) 2 3 4 (D) 3 4 1 A pelton wheel develops 5520 kW under a head of 240 m
13. Efficiency of the turbine is least under part load condi- at an overall efficiency of 80% when revolving at a speed
tion in the case of a of 200 rpm.
(A) Pelton turbine 20. The unit discharge is
(B) Francis turbine (A) 0.172 (B) 0.189
(C) Kaplan turbine (C) 0.135 (D) 0.212
(D) Propeller turbine
21. The unit power is
1 4. Cavitation in a hydraulic turbine is most likely to occur (A) 1.645 (B) 1.581
at the turbine (C) 1.485 (D) 1.321
(A) entry
(B) exit Common Data for Questions 22 and 22:
(C) stator exit A 10 cm diameter water jet having a velocity of 45 m/s
(D) rotor exit strikes a flat plate, the normal of which is inclined at 45° to
1 5. A hydraulic turbine develops 1000 kW power for a the axis of the jet.
head of 40 m. If the turbine is to work under a head of 22. What will be the normal force on the plate if the plate
20 m the power developed in kW is is stationary?
(A) 250 (B) 500 (A) 5.62 kN
500 100 (B) 11.25 kN
(C) (D)
8 8 (C) 15.9 kN
(D) Data insufficient
16. A water turbine delivering 16 MW power is to be
tested with the help of a geometrically similar model of 23. What will be the normal force on the plate if the plate
1 : 4 scale. If the speed of the model is same as that is moving towards the jet with a velocity of 15 m/s
of the prototype then determine, power developed by (A) 5 kN
the model, assuming same efficiencies for model and (B) 10 kN
prototype (C) 20 kN
800 1000 (D) 30 kN
(A) kW (B) kW
32 128 Common Data for Questions 24 and 25:
1000 500 A pipe line 1200 m long supplies water to 3 single jet pelton
(C) kW (D) kW
64 64 wheels. The head above the nozzle is 360 m. The velocity
coefficient for the nozzle is 0.98 and coefficient of friction
17. A jet of water of cross sectional area ‘a’ strikes on a
for the pipeline is 0.02. The turbine efficiency based on the
series of flat plates mounted on a wheel. If the jet veloc-
head at the nozzle is 0.85. The specific speed of each turbine
ity is v and velocity of the plates is u, the force exerted
is 15.3 and the head loss due to friction in the pipeline is
on plate is
12 m of water. If the operating speed of each turbine is 560
(A) ρa (v − u ) (B) ρav 2 (v − u )
2
rpm.
(C) ρa (v − u ) u (D)
ρav (v − u )
2
24. The total power developed by the turbine in kW
(A) 6069 (B) 5059
18. A jet issuing from nozzle with diameter of 180 mm has (C) 7079 (D) 4079
a velocity of 90 m/s and strikes the pelton wheel and
gives a shaft power of 7500 kW. If the coefficient of 25. The diameter of the nozzle in mm
velocity of nozzle is 0.98, the overall efficiency of the (A) 45.7 (B) 95.6
turbine is (C) 76.2 (D) 81.9
Hydraulics Test 3 | 3.123

Answer Keys
1. B 2. A 3. C 4. A 5. C 6. C 7. C 8. B 9. C 10. B
11. D 12. B 13. D 14. D 15. D 16. C 17. D 18. B 19. D 20. B
21. C 22. B 23. C 24. B 25. B

Hints and Explanations


1. The velocity components are 5. u1= ku 2gH = 2 2 × 9.81 × 35 = 52.4 m/s
u = 3x + 2y
vf1= kf 2gH = 0.6 2 × 9.81 × 35 = 15.7 m/s
v = 2z + 3x2
w = 2t – 3z shaft power
= overall h
Substituting, wQH
x = 1, y = 1, z = 1 and t = 2
u=3+2=5 20000 × 103
i.e., = 0.9
v = 2+ 3 = 5 9810 × Q × 35
w=2×2–3=1 Q = 64.72 m3/s
V = u + v + w  = 5 + 5 + 1  =
2 2 2 2 2 2
π
But Q = ( Do − Db ) × v f 1 or
51 2 2

   = 7.14 units. Choice (B) 4


π 2
2. Convective acceleration
2
(
64.72 = Do 1 − 0.35 × 15.7
4
2
)
∂v  x  x  1 
= v = 2t 1 −  × 2t × 2 1 −   −  Do = 2.45 m
∂x  2l   2l   2l 
πDo N
4t 2  x
3 u1 = where N = speed of turbine
= − 1 −  60
l  2l  π × 2.45 × N
i.e., 52.4 =
Given that l = 1.5 m 60
x = 1 and t = 5 seconds N = 409.2 rpm. Choice (C)
−4 × (5)
2 3
 1 
convective acceleration = 1 −  dp
1.5 2 × 1.5  6. U = -3 m/s; = -100 × 106 N/m3
dx
= –19.75 m/s2. Choice (A)
m = 0.4 poise = 0.04 Ns/m2
3. Total acceleration b = 1 mm
= Convective acceleration + local acceleration Discharge per unit width is given by,
2
∂v  x b b 3 ∂p
local acceleration = = 1 −  × 2 q=U − ⋅
∂t  2l  2 12µ ∂x
2



=
1  0.001 (0.001)
1 −  ×2   = (-3) × − × ( −100 × 106)
2 × 1.5  2 12 × 0.04
=0.889 m/s2   = 0.2068 m3/s = 206.8 L/s Choice (C)
Total acceleration = –19.75 + 0.889 = –18.86 m/s2. 7.
 Choice (C)
(V – u)sin 25°
u 3 y 1 y2 V–u
4. = −
U 2 δ 2 δ2 25°
Displacement thickness (V–u)cos 25°
δ δ
 u 3 y 1 y2
d* = ∫ 1 −  dy  = ∫ 1 − +
0
 U 0
2 δ 2 δ2
δ u
 3 y2 1 y3  3 δ 5
    = y − + . 2   = d - δ +  =  δ
 2 2δ 2 3δ 0 4 6 12

δ* 5 δ 5
    = = . Choice (A) V–u
δ 12 δ 12
3.124 | Hydraulics Test 3

u = 20 m/s 13. Blades of a propeller turbine are fixed and not adjusta-
v1x= (v - u) = 48 - 20 = 28 m/s ble under part load condition. So under part load, water
v1y = 0 enters with shock and eddies are formed which reduces
v2x = - (v -u) cos 25 = - 28 cos 25 = - 25.38 m/s its efficiency. Choice (D)
v2y= (v - u) sin 25 = 28 sin 25 = 11.83 m/s 15. P1 = 1000 kW, H1 = 40 m
Fx = 1 x [v1x - v2x] = 28 - (-25.38) = 53.38 m/s H2 = 20 m
Fy = 1 x [v1y - v2y] = 0 - 11.83 = -11.83 m/s For both conditions, unit power is same
3
Fx
P1 P2  H 2
α i.e., Pu = 3
= 3 ⇒ P2 = P1  2 
H 
( H1 ) 2 ( H2 )2 1

3 3
 20  2  1 2
= 1000 ×   = 1000 ×  

 40   2
1000
= kW  Choice (D)
Fy 8
Resultant force 16. Given- Pp = 10 MW
Fx 2 + Fy 2 = 53.38 + ( −11.83) = 54.68 N
2
FR = 2
Dp 1
=
Fy 11.83 Dm 4
tan a = = = 0.2216 Np
Fx 53.38
=1
Nm
a = 12.5°. Choice (C)
8. Since wave resistance is dominant, dynamic similarity hp = hm
will be attained when Froude number is same for model P
Power coefficient is
and prototype N 3 D5
 V   V  Vp Lp same in both cases
i.e.,   =  ⇒V = Lm
= 25 = 5 Pp Pm
 Lg  m  Lg  p m i.e., =
N p 3 D p 5 N m 3 Dm 5
⇒ Vp = 2 × 5 = 10 m/s Choice (B) 3
D 1  N   D 5
9. Maximum pressure rise occurs when 1 = or \ Pm = Pp ×  m  ×  m 
D2 2  Np   D p 
D2 = 2 D1 1
5
1 1
= 16 × 1 ×   = 3 =
 MW
⇒ A2 = 2A1 ∵ A ∝ D2  4 4 64
V1 1000
⇒ V2 = = kW  Choice (C)
2 64
∵ A1V1 = A2V2 = Q 17. Force exerted
Energy (head) loss in sudden expansion = mass/second × change in velocity of the jet
2 = ρav (v − u )  Choice (D)
 V1 
V1 − 
=
(V1 − V2 ) 2

=
2 18. V = Cv 2gH
2g 2g 90 = 0.98 2 × 9.81 × H

V2 1
2
V2 1 V2 H = 429.87 m
= 1 1 −  = 1 × = 1 m of oil

2g  2  2g 4 8g P 7500 × 103
h0 = =  = 0.7766
 Choice (C) wQH 9810 × 2.29 × 429.87
π 2
N Q Q= d ×V
11. Specific speed of hydraulic pump Ns = 3
4
H4 π
=
  (0.18)2 × 90 = 2.29 m3/s
It is the speed when working against unit head and 4
delivering unit quantity of water. Choice (D) h0 = 0.7766 × 100 = 77.66 % Choice (B)
Hydraulics Test 3 | 3.125

1 + cos φ 23. Relative velocity = v +u = 45 + 15 = 60 m/s


19. Maximum efficiency is expressed as hH = Fn = rA (v + u)2 sinq
2
  = 1000 × 7.854 × 10–3(60)2 sin 45°
f1 = 180 – 160 = 20°
  = 19993 N
f2 = 180 – 170 = 10°
1 + cos 20° 19993
hH1 = Fn =  = 20 kN Choice (C)
2 1000
1 + cos 10°
hH2 = 24. Net head available at the base of the nozzle
2
  = 360 – 12 = 348m
ηH 1 (1 + cos 20°) 2 N p
= × Ns =
ηH 1 2 1 + cos 10° 5
H4
1.9397
     = = 0.977 Choice (D) 560 p
1.9848 15.33 = 5
348 4
P
20. h0 = P = 1686.37 kW
wQH Total power = 1686.37 × 3 = 5059.11 kW Choice (B)
5520 × 103
0.8 = P
9810 × Q × 240 25. h0 =
wQH
Q = 2.931 m3/sec
Unit Discharge 1686.37 × 103
Q 2.931 0.85 =
Qu = = = 0.189 Choice (B) 9810 × Q 348
H 240
Q = 0.581 m3/sec
P 5520 Total discharge = 3 × 0.581
21. Unit power Pu = = = 1.485 Choice (C)
3 3
Q = 1.743 m3/s
H 2 (240) 2
V = kv 2gh = 0.98 2 × 9.81 × 348
22. Fn = rAV2 sinq
  = 80.98 m/s
V = 45 m/s
Thus, if the diameter of the nozzle is ‘d’ then
q = 45° = 1000 × 7.8544 × 10–3 × (45)2 sin 45°
  = 11246 N π 2
0.581 = d × 80.98
11246 4
Fn = = 11.25kN Choice (B) D = 0.0956 m = 95.6 mm Choice (B)
1000
This page is intentionally left blank
Unit IX
Water Resources Engineering
This page is intentionally left blank
Hydrology Test 1
Number of Questions: 25 Time: 60 min.

Directions for questions 1 to 25: Select the correct alterna- (A) Perennial stream
tive from the given choices. (B) Influent stream
1. The region where air coming from the pole (cooler (C) Effluent stream
and denser) and the air of the middle cell (warmer and (D) Ephemeral stream
lighter) meet is called ________. 7. A structure with a useful life period of 100 years is
(A) Cold front designed for a 50-year flood. Then the risk in the design
(B) Warm front is given by _______.
(C) Polar front (A) 0.68 (B) 0.71
(D) Occluded front (C) 0.87 (D) 0.99
2. The intensity – duration – frequency Curve from the 8. The peak flow in outflow hydrographs in a channel
following is routing occurs at ______.
(Where a < b < c) (A) Intersection point of inflow and outflow
hydrographs
(B) Before intersection
frequency a (C) After intersection
Intensity

c b
c frequency
Intensity

b (D) Any of the above


a
9. For unconfined aquifers, the storage coefficient
(A) Duration (B) Duration (A) is essentially the same as the specific yield.
(B) does not exist
(C) is essentially the same as the specific retention.
frequency (D) is essentially the same as the porosity.
c
b
a
10. In surface float method, the actual velocity of flow (Va)
Intensity

is equal to ______ times of surface velocity (Vs).


Duration
(A) 0.2 (B) 0.4
(C) (D) None
(C) 0.8 (D) 0.85
3. The rate of evaporation from a water body increases
11. In a river flow which has shallow depth velocity at dif-
directly with increase in
ferent depths at a c/s 0.2d, 0.4d and 0.8d from the bot-
1. Radiation
tom are 0.1, 0.2, 0.5 m/s respectively. Find the mean
2. Wind upto a critical value
velocity at that cross-section.
3. Atmospheric pressure
(A) 0.3 m/s (B) 0.2 m/s
4. Quality of water
(C) 0.4 m/s (D) 0.35 m/s
(A) Only 2, 3, 4 are correct
(B) Both 1 and 2 are correct 12. A sample has a hydraulic conductivity of 12 m/day.
(C) Both 2 and 3 are correct What would be its intrinsic permeability? (indarcys)
(D) Only 1, 2, 4 are correct (A) 12.68 (B) 13.12
4. The infiltration capacity curves which are developed (C) 14.35 (D) 16.89
from infiltrometer tests or the hydrograph analysis 13. During a recuperation test conducted on a open well in
methods are used to estimate ______ from a given a region, the water level in the well was depressed by
storm. 4 m and it was observed to rise by 2 m in 90 minutes.
(A) Infiltration What would be the yield from that well having a diam-
(B) Rainfall eter of 6 m under a depression head of 3 m?
(C) Runoff (A) 35.1 m2/h (B) 39.19 m3/h
(D) All the above (C) 48.32 m /h
3
(D) 51.6 m3/h
5. ______ hydrograph is independent of rainfall duration. 14. Calculate the peak of the outflow hydrograph in a river
(A) Instantaneous unit hydrograph reach using Muskingham method given the following
(B) Synthetic unit hydrograph inflow hydrograph. Take Co = 0.032, C1 = 0.53 and the
(C) Direct runoff hydrograph starting value of outflow hydrograph as 103/s.
(D) Unit hydrograph
Time (hrs) 0 1 2 3 4 5 6 7
6. When the seepage takes place from the stream into the
Inflow (m /s)
3
10 20 30 40 50 40 30 20
ground, it is called ______ stream.
3.130 | Hydrology Test 1

(A) 30.2 m3/s (B) 42.4 m3/s 20. The infiltration rate for excess rain on a small area
(C) 51.6 m3/s (D) 20.8 m3/s was observed to be 5 cm/hr at the beginning of rain
1 5. An urban area has a runoff coefficient of 0.35 and an and decreased exponentially toward an equilibrium of
area of 0.8 km2. The maximum depth of rainfall with a 0.6 cm/hr. A total of 35 cm of water infiltrated during
30 – year return period is as below: 10 hours interval. Determine k of the Horton’s equation.
(A) 0.05/hr
Duration (min) 3 5 10 20 30 40
(B) 0.11/hr
Depth of rainfall (mm) 10 15 20 25 30 35 (C) 0.15/hr
If a culvert for drainage at the outlet of this area is to (D) 0.2/hr
be designed for a time period of 30 years, estimate the 21. Match the following in Group – A with Group – B.
peak flow rate. Take the time of concentration for the Group – A Group – B
drainage area as 20 minutes.
P. Transpiration 1. Phytometer
(A) 5.83 m3/s
Q. Evapotranspiration 2. Rainfall simultator
(B) 15.55 m3/s
(C) 2.78 m3/s R. Evaporation 3. Lysimeter
(D) 10.13 m3/s S. Infiltration 4. Water balance method
Common Data for Questions 16 and 17:
P  Q  R  S P  Q  R  S
The drainage area of water shed is 60 km2. The Φ – index
(A) 2 4 3 1 (B) 1 3 4 2
is 0.4 cm/hr. Base flow at outlet 12 m3/s, 1 hour – UHG of
(C) 3 2 1 4 (D) 4 1 2 3
water shed is triangular in shape with a time base of 10 hour.
Peak ordinate occurs at 5 hours. 22. If 9.2 liters of water is added to an evaporation pan of
1.3m diameter to bring the water surface to the stipu-
16. Peak ordinate of UHG in (m3/s) is ______.
lated level and if a nearby rainguage measured a rain-
(A) 13.33 (B) 23.33
fall of 8.8 mm. what is the evaporation recorded for the
(C) 33.33 (D) 43.33
day?
17. For a storm of depth of 6.4 cm and duration of 1 hr, the (A) 0.6 mm
peak ordinate in m3/s of hydrograph is _______. (B) 1.87 mm
(A) 153 (B) 181 (C) 2.3 mm
(C) 212 (D) 240 (D) 3.2 mm
18. A storm of 3-hour duration occurred over a basin of
Common Data for Questions 23 and 24:
area 555.2 km2. The resulting flow measurement is as
A one-day rainfall of 16 cm in Hyderabad is found to have a
follows:
return period of 100 years.
Time (hr) 0 2 4 6 8 12 15 Calculate the probability that one-day rainfall of this magni-
Q (m3/s) 10 210 310 360 260 60 10 tude or larger magnitude

If Base flow = 10 m3/s, find the depth of runoff over a 23. Will not occur in Hyderabad during the next 50 years.
catchment. (A) 0.01 (B) 0.99
(A) 2.53 cm (C) 0.605 (D) 0.53
(B) 0.81 cm 24. Will occur in next year.
(C) 3.14 cm (A) 0.01 (B) 0.99
(D) 1.43 cm (C) 0.605 (D) 0.53
1 9. The total observed runoff volume during a 7-hour storm 25. Find out the mean precipitation of a catchment which
with a uniform intensity of 2 cm/h is 25 × 106m3. If the is in a triangular shape of side 10 km. Rainguages
area of the basic is 300 km2, find the average infiltration installed at each corner recorded 10 cm, 15 cm, 20 cm
rate for the basin in (mm/hr). respectively.
(A) 2.18 (B) 3.63 (A) 15 cm (B) 12 cm
(C) 5.72 (D) 8.14 (C) 17 cm (D) 14 cm

Answer Keys
1. C 2. A 3. D 4. C 5. A 6. B 7. C 8. D 9. A 10. D
11. B 12. C 13. B 14. B 15. A 16. C 17. C 18. D 19. D 20. C
21. B 22. B 23. C 24. A 25. A
Hydrology Test 1 | 3.131

Hints and Explanations


4. 2.303  4
Rainfall hyetograph    = log  
1.5  2
Volume of runoff
   = 0.462 h–1
Rainfall Q = CAH
intensity Infiltration curve π 
  = 0.462 ×  × 62  × 3
4 
  = 39.19 m3/h. Choice (B)
1 4. C0 = 0.032, C1 = 0.53
time
C2 = 1 – C0 – C1 = 0.438
 Choice (C) Outflow Q2 = C0I2 + C1I1 + C2Q1
= (0.032 × 20) + (0.53 × 10) + (0.438 × 10)
7. Risk = 1 – qn
= 10.32 m3/s
1 1
P = = = 0.02 Similarly
Tr 50
Inflow (m3/s) Outflow (m3/s)
q = 0.98 10 10
∴ Risk = 1 – (0.98)100
20 10.32
  = 0.867 ≅ 0.87 Choice (C)
30 16.08
8. In reservoir routing, the peak of outflow hydrograph is
40 24.22
at the intersection of inflow and outflow, but not for
50 33.41
channel routing. Choice (D)
40 42.41
9. Storage coefficient:
The volume of water that an aquifer releases per unit 30 40.74
surface area of the aquifer per unit change in compo- 20 34.38
nent of head normal to that surface. ∴ Peak discharge = 42.41 m3/s. Choice (B)
In an unconfined aquifer it is equal to specific yield.
(water extracted by force of gravity). Choice (A) 1 5. Area = 0.8 km2 = 80 ha
AIR
10. Va = 0.85 Vs Choice (D) Q=
360
11. For shallow depth, one point method is used
Vm = V0.4 = 0.2 m/s. Choice (B) Duration to be taken = time of concentration = 20
minutes
Kµ 25
12. Permeability k = Depth of rainfall = × 60 = 75 mm/h
ρg 20
k = 12 m/day 80 × 0.35 × 75
Q=
µ = 0.01 gm–cm/s 360
ρ = 1 gm/cc   = 5.83 m3/s. Choice (A)
g = 981 cm/s2 16. Volume = A × depth
12 × 100 × 0.01 = 60 × 106 × 0.01
k=
24 × 60 × 60 × 1 × 981 = 6 × 105m2
  = 1.416 × 10–7cm2 From UHG
  = 1.416 × 10–11m2 1
 × 10 × 60 × 60 × Q = 6 × 105
1 Darcy = 0.987 × 10–12m2 2
1.416 × 10 −11 Q = 33.33 m3/s. Choice (C)
∴ k = = 14.35 Darcy’s. Choice (C)
0.987 × 10 −12
6.4 − R
13. h1 = 4 m 17. Φ = 0.4 cm/hr =
1
h2 = 4 – 2 = 2m
Runoff R = 6 cm
T = 90 minutes = 1.5 h
Peak DRH = 33.33 × 6 = 200 m3/s
k 2.303 h  Peak SHG = DRH + Base flow
C= = log  1 
A T  h2  = 200 + 12 = 212 m3/s. Choice (C)
3.132 | Hydrology Test 1

18. Base flow = B 4.4 − k ×10 4.4 o


Let Os1, Os2, are storm hydrograph ordinates 35 = 6 – e + e
k k
Let OD1, OD2. OD3 ….. are direct runoff hydrograph
4.4
ordinates [1 – e–10k] = 29
OD1 = Os1 – B k
OD2 = Os2 – B e–10k ≃ 0
∴ k = 0.152/hr Choice (C)
OD 0 200 300 350 250 50 0
By Trapezoidal formula:- (9.2 × 10 ) × 1000
−3

22. Evaporation recorded = 8.8 –


Volume π
× 1.32
4
 OD1 + OD7 
= ∆t  + Σ ( remaining ordinates )  = 1.87 mm (1m3 = 1000 liters). Choice (B)
 2 
1 1
0 + 0  23. p = = = 0.01
V = 2 × 60 × 60  + ( 200 + 300 + 350 + 200 + 50)  T 100
 2 
q = 0.99
V = 7.92 × 10 m
6 3
Probability for not occurring in next 50 years.
V 7.92 × 10
6
P = 50C0 p0 q50
Depth = =   = 1 × 1 × (0.99)50
A 555.2 × 106
   = 0.605. Choice (C)
= 0.01426 m
2 4. Will occur next year:
= 1.43 cm. Choice (D)
⇒ at least once in 1 year
19. Total rainfall =
 Intensity of rainfall × duration P = 1C1 p1 q0 = 0.01. Choice (A)
= 2 × 7 = 14 cm
A
Volume of runoff = 25 × 106m3 25. a1 = a2 = a3 = areas represented by rain gauges =
Area of the basin = 300 km2 = 300 × 106m2 3
Volume of runoff Mean precipitation
Depth of runoff =
Area of runoff P1a1 + P2 a2 + P3 a3
P= ↓
A
25 × 106
= = 0.083 m  A  A  A
300 × 106 10 ×  + 15 ×  +  20 × 
3 3 3
= 8.3 cm   =
Total infiltration = rainfall – runoff = 14 – 8.3 = 5.7 cm A
5.7     = 15 cm.
Average infiltration rate = = 0.814 cm/h
7
= 8.14 mm/h. Choice (D) a1
10

(
20. 35 = ∫ 0.6 + (5 − 0.6) e dt
− kt
) a2 a3
0

10
4.4 − kt 
35 = 0.6t – e 
k 0  Choice (A)
Irrigation Test 2
Number of Questions: 25 Time: 60 min.

Directions for questions 1 to 25: Select the correct alterna- 8. Non-modular outlet is the one in which discharge
tive from the given choices. (A) is independent of water levels in the distributing
1. Consumptive use of water for a crop is equal to the channel and water course.
depth of water (B) varies only with water level in the distributing
(A) evaporated by the crop. channel.
(B) transpired by the crop. (C) Varies only with water level in the water course.
(C) transpired and evaporated by the crop (D) depends on difference in water levels in distribut-
(D) used by the crop in transpiration, evaporation and ing channel and water course.
also the quantity of water evaporated from adja- 9. Main causes of water logging are
cent soil. (A) surface runoff
2. The cross-section of weir in order from upstream to (B) steep ground profile
downstream is _______. (C) excessive irrigation
(A) launching Apron → Block protection → sheet (D) All the above
pile → impervious floor → sheet pile → filter → 10. If pH < 8.5, the soil is called
launching apron (A) acidic soil (B) saline soil
(B) block protection → sheet pile → filter → sheet (C) basic soil (D) alkaline soil
pile → impervious floor → launching apron 11. Match the following.
(C) impervious floor → launching apron → sheet
Group – A Group – B
pile → block protection → sheet pile → filter →
launching apron 1. Aqueduct P. Bed of drain well above the
canal F.S.L
(D) filter → impervious floor → sheet pile → block
protection → launching apron → sheet pile → 2. Siphon Aqueduct Q. F.S.L of Canal higher than
bed of drain
launching apron
3. Super passage R. High flood level HFL) of
3. According to Lacey’s method for design of alluvial drain higher than canal bed
channel, the velocity of flow
4. Siphon S. Bed of canal is well above
(A) increases with increase in design discharge the HFL of drain
(B) increases with increase in diameter of silt particle
(C) increases with increase in silt factor 1 2  3 4 1 2  3 4
(D) All the above (A) S, R, P, Q (B) P, Q, R, S
4. Drainage gallery in a dam is used (C) Q, P, R, S (D) R, S, Q, P
(A) to provide drainage of the dam 12. To irrigate a strip of area of size 100 m × m, the time
(B) for post cooling of concrete taken is 50 minutes. Assuming average depth of water
(C) Both a and b is 8 cm and average infiltration is 4 cm/hr. Find the dis-
(D) None of the above charge of stream flow in cumecs?
5. Effective Precipitation for a crop may be defined as (A) 0.04 cumecs
(A) total precipitation from sowing of seeds to cutting (B) 0.0163 cumecs
of crop. (C) 0.023 cumecs
(B) total precipitation minus loss due to evaporation (D) 0.0368 cumecs
and infiltration. 1
1 3. The slope of a channel in alluvium is , mean soil
(C) total precipitation minus runoff. 3036
(D) available water stored in soil within root zone of particle size is 0.5 mm, velocity = 0.618 m/s. Find the
the crop. wetted perimeter of the regime channel in meters.
6. The best method of irrigation for mango trees is (A) 10.62 (B) 9.46
(A) border strip method (C) 11.58 (D) 12.9
(B) basin method 1 4. Find the Exchangeable Sodium Ratio (ESR) where the
(C) checks or Leeves method concentrations of all Sodium, Calcium, Magnesium
(D) furrow method and Potassium are 161, 200, 108, and 156 in ppm
7. _____ are called safety valves of a dam. respectively.
(A) Diversion headwork’s (B) Canal outlets (A) 83.33% (B) 43.63%
(C) Spill ways (D) Drainage Gallery (C) 68.73% (D) 23.33%
3.134 | Irrigation Test 2

15. An area of 1 hectare has a root zone depth of 1.2 m and Weir
available moisture holding capacity is 14 cm per meter
depth. It is irrigated through a stream of 0.04 m3/s. Water
is applied to field when 60% of the available moisture is
7m
depleted. Irrigation period is 9 hours. If the water appli-
cation efficiency is 70% determine the storage efficiency. Floor
(A) 64.3% (B) 72%
(C) 58% (D) 49% 12 m
16. The moisture content of soil in the root zone of an agri- 5m
cultural crop at certain stage is found to be 0.06. The
Field Capacity of the soil is 0.14. The root zone depth
is 1.6 m. The consumption use of crop at this stage
is 3mm/day and there is no precipitation during this Sheet pile
period. Irrigation efficiency is 60%. It is intended to
(A) 1 in 2 (B) 1 in 3
raise the moisture content to the field capacity in 9 days
(C) 1 in 4 (D) 1 in 5
through irrigation. The necessary depth of irrigation in
(mm) is ______. 2 2. Match the following.
(A) 250 (B) 257 Group – A Group – B
(C) 300 (D) 310 1. Bligh’s theory P. UpperBariDoabCanal
17. The transplantation of chilli crop takes 20 days and the 2. Khosla’s theory Q. Regime Channel
total depth of irrigation water required by the crop is 3. Lacey’s theory R. Sheet pile is more useful at
50 cm on field. During this period, useful rainfall on D/S end
field is 10 cm. Find the duty of irrigation water for 4. Kennedy’s theory S. Sheet pile is more useful at
this crop on the field during transplantation period. U/S end
Assuming 30% losses of water in water course, find the 1  2  3  4 1  2  3  4
duty at the head of water course in hectares/cumec. (A) Q S R P (B) R P S Q
(A) 208 (B) 250 (C) P R Q S (D) S R Q P
(C) 302 (D) 350
23. For a channel to be in regime, conditions to be estab-
18. In a gravity dam the friction coefficient is 0.8. Sum of lished are:
vertical forces = 7000t and sum of Horizontal forces (A) Channel should be flowing uniformly in unlimited
= 4000t. Base width is 80m and shear strength is coherent alluvium.
150 t/m2. Find FOS against sliding and shear friction (B) Silt grade and silt charge should be constant.
factor at base? (C) Both a and b
(A) 4.4 and 1.4 (B) 1.4 and 4.4 (D) None of the above
(C) 3.2 and 1.6 (D) 1.6 and 3.2 2 4. Calculate the bed width for an irrigation channel to
19. The limiting height of gravity dam with material carry a discharge of 6 cumecs and side slopes of the
of concrete having specific gravity of 2.5 is equal to 1
100m. Find crushing stress of concrete (in kg/cm2). channel are H : 1V. The critical velocity ratio is 0.9
2
(A) 20 (B) 25 and depth of flow is 0.8 m. Bed slope is 0.3 m/km.
(C) 30 (D) 35 (A) 13.75 m (B) 15.25 m
20. Find the design head (Hd) of the Ogee spill way when a (C) 17.04 m (D) 19.21 m
coordinate (– 10, 5) of the point on the D/s profile with 2 5. Find the spacing of drains in case of closed drains
origin at the crest of the spill way is ______. where the depth of impermeable layer from GL is 10 m
(A) 0.56 m (B) 0.72 m and depth of drain below GL is 2 m. Minimum depth
(C) 0.98 m (D) 1.1 m of drained W.T below GL is 1.5 m. Permeability of
21. A weir on the permeable foundation with downstream soil = 1 cm/s. Discharge through drain is 0.02 m3/s.
sheet pile is shown in figure below. The exit gradient as (A) 14.5 m (B) 16.5 m
per Khosla’s method is ______. (C) 18.5 m (D) 20.5 m

Answer Keys
1. D 2. A 3. D 4. C 5. D 6. B 7. C 8. D 9. C 10. B
11. A 12. B 13. A 14. D 15. A 16. C 17. C 18. B 19. D 20. C
21. B 22. D 23. C 24. C 25. B
Irrigation Test 2 | 3.135

Hints and Explanations


2. Q
Launching weir 1.517 =
Launching 1
apron apron Q−
D/S 180
U/S
0.517Q = 8.43 × 10–3
Q = 0.0163 m3/s. Choice (B)
13. Silt factor f = 1.76 d = 1.76 0.5 = 1.244
filter
Block 1
protection Sheet pile  Qf 2  6
Velocity V = 
C/S of Weir  140 
1
 Choice (A)  Q × 1.244 2  6
0.618 = 
1  140 
 Qf 2  6 2
3. V =   = fR Q = 5 m3/s
 140  5 (OR)
But f = 1.76 d ⇒ f ∝ d Longitudinal slope
5
1
f3
∴ V ∝ Q 6 S= 1

1 3340 Q 6
   ∝ f 3
5
1 (1.244)3
1
= 1
   ∝ d .6
Choice (D) 3036
3340Q 6
4. Drainage gallery is used Q = 5 m3/s
1. To provide drainage of the dam. Perimeter P = 4.75 Q
2. For drilling and grouting of foundation
3. Post cooling of concrete =
4.75 5 = 10.62 m Choice (A)
4. To lay mechanical equipment for operation of out- 14. Exchangeable Sodium Ratio (ESR)
let gates and spill way gates. Choice (C)
Na +
5. Effective precipitation can mean rainfall and losses. =
Ca + + + Mg + + + Na + + K +
But for a crop it means only available water within root
zone that can be easily extracted by plants. All concentrations are in meq/lt
 Choice (D) 161
Na (meq/lt) = =7
6. A basin in created surrounding each tree and field 23
channel supplies water to the basin used for irrigation 200
of orchards (fruit trees) Choice (B) Ca (meq/lt) = = 10
40
9. Causes of water logging 2
→ Excessive rainfall irrigation
108
→ Seepage from canals and lands beside Mg (meq/lt) = =9
24
→ Flat ground profile
→ Improper drainage of surface runoff Choice (C) 2

12. The time required to irrigate the land 156


K (meq/lt) = =4
39
y  Q 
t= log e   7
I  Q − IA  ∴ ESR = × 100 = 23.33% Choice (D)
7 + 10 + 9 + 4
  15. Depth of root zone = 1.2 m
0.8  Q  Total available depth of water
50 × 60 = log e  
0.04 0.04 cm
 Q − × 500 
 y = 14 × 1.2 m = 16.8 cm
60 × 60 60 × 60 m
3.136 | Irrigation Test 2

Water consumed = (100 – 60%) y = 6.72 cm 19. Critical Height of dam


Water needed to raise upto Field capacity = 16.8 – 6.72 fC
= 10.28 cm HC =
w  S + 1
Volume of water supplied = 0.04 × 9 × 60 × 60
= (1296 cm3) fC
100 =
Wroot zone depth 1000 2.5 + 1
Application efficiency ηa = 60% =
1296 fc = 3.5 × 105 kg/m2
Water in root zone = 778 m3 Crushing stress fc = 35 kg/cm2 Choice (D)
778 2 0. Equation of Ogee spill way
Depth of water stored = = 0.06483 m
1.2 × 10 4 y  x 
1.85

= –0.5 
= 6.483 cm Hd  H d 
water stored in root zone (x, y) = (5, – 10)
ηstorage =
water needed to raise the w/c to F .C −10  5 
1.85

=  –0.5
6.483 Hd  H 
∴ ηstorage = = 0.643 = 64.3%. Choice (A) d
10.08
H d1.85
= 0.982
16. d = 1.6m, OM = 0.06 Hd
FC = 0.14, Cu = 3 mm/day
H d0.85 = 0.982
Trying with S = 1
Depth of water in root zone dw = d.s[F.C – OM] Hd = 0.978 m. Choice (C)
= 1.6 × 1[0.14 – 0.06] H
2 1. Exit Gradient GE = 
= 0.128 m = 128 mm dπ λ
Total depth of water to be kept in root zone
b 12
= 128 + Cu × time ∝= = = 2.4
= 128 + (3 × 9) d 5
= 155 mm 1 + 1+ ∝2 1 + 1 + 2.4 2
155 155 λ= = = 1.8
Total depth to be released at head canal = = 2 2
ηa 0.6
7
= 258 mm GE = = 0.332
5 π 1.8
Answer must be > 258 mm (assumed S = 1)
∴ depth of irrigation = 300 mm (>258 mm) ⇒ 1 in 3.01. Choice (B)
 Choice (C) 22. Bligh’s Theory:
17. Irrigation depth of water h
tmin =
∆ = 50 – 10 = 40 cm = 0.4 m (GC −1)
Base period, B = 20 days
To minimize floor thickness sheetpile is more useful
8.64 B 8.64 × 20 at U/S end
DF, duty at field = =
∆ 0.4 Khosla’s theory:
= 432 hectares/cumec H
Exit Gradient, GE =
Duty at head of canal = DF × ηC dπ λ
= 432 × [1 – 0.3] [∵ losses = 30%]
H = head causing flow
= 302.4 hectares/cumec. Choice (C)
To avoid piping Khosla’s proposed sheet pile @ D/s
end.
µΣV 0.8 (7000 ) Kennedy made his observations on Upper Basi Drab
18. Sliding FOS =  = = 1.4
ΣH 4000 Canal
µΣV + (b × 1 × q ) Lacey’s equations are applicable for regime channel.
Shear Friction Factor (SFF) =  Choice (D)
ΣH
2 3. For a channel in regime conditions
0.8 (7000 ) + (80 × 150) 1. Channel should be flowing uniformly.
=
4000 2. Silt grade and silt charge should be constant
 = 4.4. Choice (B) 3. Discharge should be constant Choice (C)
Irrigation Test 2 | 3.137

24. Q = 6 cumecs GL
Critical velocity ratio. M = 0.9
D = 0.8 m 2m 1.5
VK = 0.55 m D0.64
   = 0.55 (0.9) (0.8)0.64 = 0.43
A = (B + 0.5D)D = 0.8 B + 0.32
Q = A.VK
b = 8.5m
6 = (0.8 B + 0.32) 0.43 a = 8m
B = 17.04 m Choice (C)
25. Spacing of drains Impervious layer

4 k (b − a
2 2
)
L=
Qo 4 × 0.01(8.52 − 82 )
L= = 16.5 m
b = 10 – 1.5 = 8.5 0.02
a = 10 – 2 = 8 ∴ Spacing of drains = 16.5 m Choice (B)
Water Resource Engineering Test 3
Number of Questions: 30 Time: 75 min.

Directions for questions 1 to 30: Select the correct alterna- 9. As a result of the construction of a diversion structure
tive from the given choices. across a river, there will be a rise in the flood level on
1. The slope of the rainfall mass curve is zero, when it the upstream side of the structure and it is called as
is horizontal. This happens when the intensity for that (A) Free board (B) uplift
period is (C) aggradation (D) afflux
(A) constant (B) Increasing 10. Coefficient of discharge of Ogee spillway is
(C) Decreasing (D) Zero (A) 2 (B) 1.8
2. Which of the following is not a continuous random (C) 2.2 (D) 2.4
variable? 11. A steady groundwater flow is running through an
(A) Annual peak flood unconfined aquifer of coarse sand under laid by a hori-
(B) Annual runoff zontal impervious formation as shown in figure. The
(C) Number of cyclones in a year depths of water table below the ground surface in the
(D) Wind velocity two observation wells, fixed at a spacing of 300 m along
3. The following rainfall chart is from which type of rain the direction of flow are recorded as 6 m and 6.5m. The
gauge. sand layer is 30 m thick and its coefficient of permea-
bility is 5 × 10–3 cm/sec. Determine the rate of flow in
Rainfall m3/day/m width of the quifer.
(mm) (A) 0.17 (B) 0.23
(C) 0.38 (D) 0.42
12. Match the parameters in Group - A with Group - B.
12 14 16 18 20 24 2 4 6 Group – A Group – B
Time (hours) P. Isonif 1. Line joining points having equal
rainfall.
(A) Weighing bucket type
Q. Isohyte 2. Line joining points having equal
(B) Float type Raingauge snowfall.
(C) Tipping bucket type
R. Isopleth 3. A line joining points having equal
(D) Both A and B depths of rainfall of particular dura-
4. The average pan coefficient of ISI standard pan is tion with particular return period.
(A) 0.95 (B) 0.8 S. Isopluvial 4. Line joining points having equal
(C) 0.7 (D) 0.6 depth of evapotranspiration

5. Storage in the channel is equal to P Q R S P Q R S


(A) Prism storage (B) Wedge storage (A)
4 3 2 1 (B) 2 1 4 3
(C) Largest of A and B (D) (A + B) (C)
4 1 3 2 (D) 3 1 4 2
6. Which of the following formations does not contain
13. Sodium dichromate solution with a concentration
any ground water?
of 30mg/cc is introduced into a stream at a rate of
(A) aquifee (B) acquifuge
1.7 litres/minute. The samples collected at a down-
(C) aquitard (D) aquiclude
stream section sufficiently far away indicated on equi-
7. Moisture equivalent is equal to librium concentration of 0.001 ppm. Determine the
(A) Field capacity discharge in the stream (Assume no initial concentra-
(B) Saturation capacity tion of sodium dichromate in the stream)
(C) Available Moisture (A) 424.5 m3/s (B) 380.6 m3/s
(D) Ultimate wilting (C) 298.3 m /s
3
(D) 484.1 m3/s
8. Indentify the incorrect statement related to the adverse
effects of soil salinity or alkalinity, from the following: Common Data for Questions 14 and 15:
(A) Causing low yields of crops Ordinates of 1 hr UGH at 1 hr intervals are 6, 8, 11, 14, 12,
(B) Limiting of the type of crops 2, 1 m3/s.
(C) Rendering the quality of folder poor 14. Calculate the equilibrium discharge of S-curve in m3/s.
(D) Causing high infiltration, resulting in damage to (A) 53 (B) 54
crops (C) 55 (D) 56
Water Resource Engineering Test 3 | 3.139

15. Calculate the maximum ordinate of 3 hr UHG in m3/s. 20. What would be the runoff volume produced by the
(A) 9.33 (B) 8.33 storm (in hr.m)
(C) 12.33 (D) 13.33 (A) 2400 (B) 2000
(C) 3000 (D) 2800
Common Data for Questions 16 and 17:
A 4 hour rain of average intensity 1.6 cm/hr falls over the 21. What would be the area that can be irrigated, if the
catchment as shown below: above runoff is utilized without wastage to irrigate a
land where the depth of irrigation required is 20 cm.
2
3 1 (A) 12000 ha
(B) 10000 ha
Zone II Zone I Discharge (C) 15000 ha
tc = 3 hr

Zone III
tc = 2 hr

tc = 1 hr
40 ha sight (D) 14000 ha
200 ha 30 ha
22. The following data pertains to the healthy growth of
a crop. Field capacity of soil = 40%. Permanent wit-
3 1 ting percentage = 11%. Density of soil = 1400 kg/m3.
2
Effective depth of root zone = 800 mm. Daily
The time of concentration from the lines 11, 22, 33 are 1hr, consumptive use of water for the given crop = 15 mm.
2hr, and 3hr respectively to the discharge measuring site. For healthy growth moisture content must not fall below
The values of run-coefficients are 0.4, 0.5, and 0.6 for 1st, 25% of the water holding capacity between the field
2nd, 3rd hours of rainfall respectively and there after attains a capacity and the permanent witting point. Determine
constant value of 0.65. the watering interval in days.
(A) 13 (B) 14
16. The discharge at the end of 3 hours period, at the meas-
(C) 15 (D) 16
uring site is
(A) 1.03 m3/s (B) 1.51 m3/s 23. Match List - A with List - B
(C) 1.77 m3/s (D) 2.04 m3/s
List – A List – B
17. The discharge at the end of 5 hours period, at the meas-
uring site, is Cu + Re + w
P. CIR 1.
(A) 0.58 m3/s (B) 0.93 m3/s ηa
(C) 1.21 m /s
3
(D) 1.53 m3/s
Cu − Re + w
18. The isohyets drawn for a storm which occurred over a Q. NIR 2.
drainage basin of area 950 km2 yielded the following ηa .ηc
information: R. FIR 3. Cu – Re

95-85 85-75 75-65 65-55 S. GIR 4. Cu – Re + w


Isohyet interval
in (mm)
CIR – Consumptive Irrigation Requirement
Area between 126 198 224 175
isohyets in km2
NIR – Net Irrigation Requirement
FIR – Field Irrigation Requirement
Determine the average depth of rainfall over the basin. GIR – Gross Irrigation Requirement
(A) 60.11 mm (B) 68.21 mm Cu – Consumptive Use
(C) 73.8 mm (D) 80.64 mm Re – Effective Rainfall
w – Water lost in deep percolation
19. Normal annual precipitation of 5 Rain Gauge stations ηa – water application efficiency
P, Q, R, S, T are 125, 102, 76, 113 and 137 cm. During ηc – water conveyance efficiency
a particular storm the precipitation recorded by stations
P Q R S
P, Q, R, S are 13.2, 9.2, 6.8 and 10.2 cm. Station T was
(A) 4 3 1 2
not working. Estimate rainfall during this storm at T.
(B) 3 4 1 2
(A) 10.21 cm (B) 12.86 cm
(C) 3 4 2 1
(C) 13.43 cm (D) 7.89 cm
(D) 4 3 2 1
Common Data for Questions 20 and 21: 24. The given figure gives the profile of a gravity dam with
A 600 sq.km watershed received a 8 hr storm which pro- reservoir level as shown. If the coefficient of friction
duced hourly intensities of 4, 10, 16, 20, 11, 2, 13, 4 mm/hr. is 0.75, Find the F.O.S against sliding and if the dam
If the initial abstractions are estimated to be 14 mm and is safe? (∫concrete = 2.4 tonnes/m3) (Neglect uplift
Φ – index is 5 mm/hr. pressure)
3.140 | Water Resource Engineering Test 3

7m
(D) Silt charge and silt grade have not been properly
defined.
12 m 2 7. Match the terms of List - A with List - B
List – A List – B
P. Nappe (sheet of water) 1. Canal regulation work
Q. Aqueduct 2. Spillways
R. Rigid module 3. Cross drainage work
H = 95m

S. Canal drop 4. Canal outlet


90 m
P Q R S P Q R S
(A) 1 2 3 4 (B) 2 3 1 4
(C) 4 1 3 2 (D) 2 3 4 1
70 m
28. Find the discharge over an ogee weir with coefficient of
(A) 1.1 and not safe (B) 1.42 and safe discharge equal to 2.4 at a head of 3m. The length of
(C) 2.1 and not safe (D) 1.8 and safe spillway is 100m. Crest of weir is 10m above the bot-
2 5. A weir across an alluvial river has a horizontal floor tom of the approach channel having the same width as
of length 70m and retains 7m of water under full pond that of the spillway
condition. If the downstream sheet pile is driven to a (A) 1247 m3/s (B) 1276 m3/s
depth of 6m below the average bed level, calculate the (C) 1301 m3/s (D) 1348 m3/s
exit gradient if porosity is 30% and the relative den- Common Data for Questions 29 and 30:
sity of soil particles as 2.7. Estimate the vertical exit For border strip method of irrigation, discharge in an area
gradient. from tube well was 0.01 cumecs. The infiltration capacity of
(A) GE = 0.181 and iC = 1.48 the soil may be taken as 6 cm/hour and the average depth of
(B) GE = 0.147 and iC = 2.12 the flow on the field as 10 cm.
(C) GE = 0.147 and iC = 1.19 29. Determine the time required to irrigate strip of land of
(D) GE = 0.181 and iC = 2.12 0.05 hectares.
26. Which of the incorrect statement among the drawbacks (A) 0.48 hours (B) 48 minutes
in Lacey’s theory. (C) 2.98 hours (D) 298 minutes
(A) Silt transportation is incorporated in a single 30. Determine the maximum area that can be irrigated
factor. from this tube well.
(B) Equations are empirical (A) 0.001 ha (B) 0.02 ha
(C) Regime conditions are only theoretical (C) 0.06 ha (D) 0.2 ha

Answer Keys
1. D 2. C 3. B 4. B 5. D 6. B 7. A 8. D 9. D 10. C
11. A 12. B 13. A 14. B 15. C 16. D 17. A 18. C 19. B 20. A
21. A 22. D 23. B 24. B 25. C 26. A 27. D 28. B 29. C 30. C

Hints and Explanations


11. Q = k I A 1.7 × 103
QT = 1.7 litres/minute =
=(5 × 10–3 × 10–2) × 60
 6.5 − 6  24 + 23.5 =
2.83 × 10–5 m3/s
 ×1×
 300  2
=1.979 × 10–6 m3/s/m QT CT + QRC R
Cmin =
=0.171 m3/day/m. Choice (A) QT + QR
13. Co = 0
CT = 30 mg/cc = 0.03 µg/cc (2.83 × 10 −5 × 0.03) + 0
2 × 10–9 =
C = 0.002 ppm 2.83 × 10 −5 + QR
0.002
  =
  6 = 2 × 10–9 µg/cc
10 QR = 424.5 m3/s. Choice (A)
Water Resource Engineering Test 3 | 3.141

14.
S A − SB × 1
1 hr S Curve
Time SA SB 3 20
UH addition
3 Hr UH i
mm/hr 16
0  0 –  0 –    0 13
1  6  0  6 –    2 11
10
2  8  6 14 – 14/3 φ – Index
3 11 14 25  0 25/3 4 4
4 14 25 39  6    11 2

5 12 39 51 14 37/3
1 2 3 4 5 6 7 8
6  2 51 53 25 28/3
Time (hr)
7  1 53 54 39    5
8  0 54 54 51     1 4
Runoff volume = × 600 × 106
54 54 53   1/3 100
54    0 =
24 × 106 m3 = 2400 ha.m. Choice (A)
Equilibrium discharge QC = 54 m3/s. Choice (B) Volume 2400
21. Area = = = 12000 ha. Choice (A)
1 5. Maximum ordinate = 37/3 = 12.33 m3/s. Choice (C) depth 0.2
A1 I 3 R1 A2 I 2 R2 A3 I1 R3 22. Water holding capacity above wilting point = FC – wc =
16. Q3hr = + +
360 360 360 40 – 11 = 29%
Optimum moisture
1.6 × 10
[(30 × 0.6) + (40 × 0.5) + (20 × 0.4)] mo = 40 – 0.75 × 29 = 18.25%
360 dw = S.d[FC – mo]
= 2.04 m3/s. Choice (D) 1400
  = × 800[0.4 – 0.1825] = 243.6 mm
A3 I 3 R 20 × 0.65 × (1.6 × 10) 1000
17. Q5hr = =
360 360 Daily consumptive use Cu = 15 mm
=
0.58 m3/s. Choice (A) dw 243.6
∴ frequency of watering = =
Cu 15
Σ (avg of isohyte ) × Area
18. d =
ΣArea =
16.24 @ 16 days. Choice (D)
(90 × 126) + (80 × 198) + ( 70 × 224) + (60 × 175) 24.
  = 7m
(126 + 198 + 224 + 175)
12 m
  = 73.8 mm. Choice (C)
19. Nx ± 10% Nx = 137 ± (10% 137) = 123.3 to 150.7
Some normal precipations are out of these ranges. 90 m
H = 95 m
Using Normal ratio method.
P W1
Nx  P1 P2 Pm 
Px =
m  N + N + ... N  W2
 1 2 m 

137 13.2 9.2 6.8 10.2 


   = + + + WH 70 m
4  125 102 76 113 
Horizontal water pressure P
   =12.86 cm. Choice (B)
wH 2 1000
20. Initial abstractions = losses = 14 mm = 4 + 10 = = × (95)2 × 10–3
2 2
\ (first 2 hour rainfall not considered)
= 4512.5 tonnes/m
Φindex = 5 mm/hr
Selfweight W1 = 7 × 102 × 2.4 = 1713.6 tonnes/m
[(16 + 20 + 11 + 13) × 1 − Runoff ] 1
= W2 = × (70 – 7) × 90 × 2.4
4 2
Runoff = 40 mm = 4 cm =
6804 tonnes/m
3.142 | Water Resource Engineering Test 3

Total self wt W = W1 + W2 = 8517.6 tonnes/m Approach velocity


µW 8517.6 Q
F.O.S against sliding =  = 0.75 × Va =
P 4512.5 Height × Width of channel
= 1.416 > 1 1247.07
∴ safe against sliding. Choice (B)   =
(10 + 3) × 100
2 5. b = 70m   =0.959 m/s
Hs = 7m
Va2 ( 0.959)
2
d = 6m
Ha = = = 0.047 m
b 70 35 2 g 2 × 9.81
∴ =  = =
d 6 3 He = H + ha = 3 + 0.047 = 3.047 m
2
 35  3
1+ 1+  
 3 Q = 2.4 × 100 × (3.047) 2
1 1+ ∝ 2

λ= = = 6.35   =1276.49 m3/s. Choice (B)


2 2
Exit hydraulic 29. Q = 0.01 cumecs = 0.01 × 3600 m3/hr
Hs 1 7 1   =0.0036 hectare-m/hr
Gradient GE = × = × = 0.147 y = 10cm = 0.1m
d π λ 6 π 6.35
I = 6cm/hour = 0.06 m/hr
Critical hydraulic Gradient A = 0.05 hectare
G −1 y  Q 
iC = t = 2.303 log10 
1+ e I  Q − IA 
n 0.3 2.303 × 0.1 0.0036
e= = = 0.429  
 = log10 
1 − n 1 − 0.3 0.06  0.0036 − 0.06 × 0.05 
2.7 − 1   =2.986 hours. Choice (C)
iC = = 1.19. Choice (C)
1 + 0.429
Q 0.0036
28. Neglecting approach velocity 30. Maximum area that can be irrigated = =
3 3
I 0.06
Q = Cd LH 2 = 2.4 × 100 (3) 2 = 1247.07 m3/s = 0.06 ha. Choice (C)
Unit X
Environmental Engineering
This page is intentionally left blank
Water Supply Engineering Test 1
Number of Questions: 25 Time: 60 min.

Directions for questions 1 to 25: Select the correct alterna- 10. The valve which allows flow only in one direction is
tive from the given choices. (A) Reflux valve (B) Sluice valve
1. Baylisturbidimeter is generally used to measure turbid- (C) Blow off valve (D) Air valve
ities in the range of 11. Match Group – A with Group – B
(A) 0 – 10 mg/l (B) 0 – 5 mg/l
Group – A Group – B
(C) 5 – 15 mg/l (D) 5 – 10 mg/l
P. Radial system 1. More valves are used
2. Water supply projects, under normal circumstances,
Q. Grid iron system 2. Design calculations are
may be designed for a period of (from the completion
simple
of project)
R. Dead end system 3. Distribution area divided
(A) 10 years (B) 30 years into rectangular circular
(C) 25 years (D) 25 years blocks.
3. The presence of silver, (Ag) in drinking water causes S. Ring system 4. Suitable for direct pumping
(A) argyria (B) hypertension and gravity system
(C) Anaemia (D) Blue baby disease
P  Q  R  S P  Q  R  S
4. Ferric Chloride (FeCl3) is used as a coagulant if the pH (A) 3 4 1 2 (B) 2 3 4 1
range is (C) 1 2 3 4 (D) 4 1 2 3
(A) > 8.5 (B) < 6.5 and >8.5 12. Chlorine gas is used for disinfection combined with water
(C) < 7 (D) None to form hypochlorous acid (HOCl). The HOCl ionizes to
5. Design a circular sedimentation (diameter (D) and form hypochlorite (OCl–) in a reversible reaction: HOCl
depth (d) of tank) to capture flocculants particles after ↔ H+ + OCl– (k = 3 × 10–8 at 20°C), the equilibrium of
coagulant with a surface loading rate of 30 m3/day/m2. which is governed by pH. The sum of HOCl and OCl is
Detention time = 3 hrs to treat 15 MLD of water known as free chlorine residual and HOCl is the more
(A) D = 20.3 m and d = 5.6 m effective disinfectant. The 95% fraction of HOCl in the
(B) D = 25.2 m and d = 5.6 m free chlorine residual is available at a pH value
(C) D = 20.3 m and d = 3.12 m (A) 5.38 (B) 6.24
(D) D = 25.2 m and d = 3.12 m (C) 7.82 (D) 8.38
6. Plain sedimentation follows Common Data for Questions 13 and 14:
(A) Type – I settling In a rapid sand filter, the time for reaching particle break-
(B) Type – II settling through (TB) is defined as the time elapsed from start of fil-
(C) Type – III settling ter run to the time at which the turbidity of the effluent from
(D) Both B and C the filter is greater than 2.5 NTU. The time for reaching ter-
7. Filtration helps in removing ___ from water. minal head loss (TH) is defined as the time elapsed from the
(A) color and odour only start of the filter run to the time when head loss across the
(B) turbidity filter is greater than 3 m.
(C) Some pathogenic bacteria 13. The effect of increasing the porosity (while keeping all
(D) All the above other conditions same) on TB and TH is
(A) TB increases and TH decreases
8. In water treatment, slow sand filters when compared to
(B) TB decreases and TH increases
rapid sand filters produce
(C) Both TB and TH increase
(A) lesser contaminated effluent
(D) Both TB and TH decrease
(B) More contaminated effluent
(C) Equally contaminated effluent 14. The effect of increasing the concentration of impurities
(D) Cannot be judged (while keeping all other conditions same) on TB and
TH is
9. An air bubble is present in water. If concentration of a
(A) TB increases and TH decreases
gas (Ct) in water is more than saturated concentration
(B) TB decreases and TH increases
(Cs), then
(C) Both TB and TH increase
(A) absorption first, desorption next
(D) Both TB and TH decrease
(B) absorption takes place
(C) desorption takes place 15. A coagulation treatment plant with a flow of 0.7 m3/s
(D) desorption first, absorption next is dosing alum at 25 mg/L. No other chemicals are
3.146 | Water Supply Engineering Test 1

added. The raw water suspended solid concentration is Common Data for Questions 21 and 22:
40 mg/L. The effluent suspended solids concentration is Following chemical species were reported for water sample
measured as 15 mg/L. Specific gravity of sludge solids from a well:
is 3.01. Find the volume of sludge solids produced in m3/ Concentration (milli
day? Species
equivalent/L)
(A) 0.53 (B) 0.41 Chloride (Cl–) 10
(C) 0.62 (D) 0.86
20
Sulphate ( SO2−
4 )
16. In a continuous flow settling tank 3.5 m deep and 60
m long, what flow velocity of water would you recom-  7
Carbonate ( CO2−
3
)
mend for effective removal of 0.026 mm particles at 25
Bicarbonate ( HCO3− )
25°C. The specific gravity of particles is 2.5 and kin-
ematic viscosity for water is 0.01 cm2/sec. (take free Calcium (Ca2+) 15
board as 0.5 m) Magnesium (Mg ) 2+
20
(A) 0.8 cm/s (B) 1.1 cm/s pH  9
(C) 2.0 cm/s (D) 1.6 cm/s
21. Total hardness in mg/L as CaCO3 is
1 7. Match the type of settling Group – A with where it
(A) 120.8 (B) 32.16
occurs Group – B
(C) 1600 (D) 1750
Group – A Group – B
22. Carbonate hardness (mg/L × as CaCO3) present in the
P. Type – Ι settling 1. Secondary settling tank above water sample is
Q. Type – ΙΙ settling 2. Sedimentation with (A) 32.16 (B) 1600
coagulation (C) 88.64 (D) 1750
R. Type – ΙΙΙ settling 3. Waste water treatment with 23. Match the Characteristics of water in Group – A with
excessive solid concentration
corresponding test used for measuring in Group – B.
S. Type – IV settling 4. Plain sedimentation
Group – A Group – B
P  Q  R  S P  Q  R  S P. Color 1. Nephelometer
(A) 4 2 1 3 (B) 2 4 3 1
Q. Turbidity 2. EDTA
(C) 1 3 2 4 (D) 3 1 4 2
R. pH 3. Tintometer
18. Medium amount of coagulant dosage is used when tur-
S. Hardness 4. Potentiometer
bidity (T) and Alkalinity (A) are
(A) T is high and A is low P  Q  R  S P  Q  R  S
(B) T is low and A is high (A) 3 1 4 2 (B) 4 3 1 2
(C) Both T and A are high (C) 1 4 3 2 (D) 2 1 3 4
(D) Both T and A are low 2 4. Determine the future population of Mumbai town by
1 9. In order to test filtration process, clear water is made to Geometric increase method for the year 2021, given the
pass through a bed of uniform sand at a filtering veloc- following data.
ity of 4 m/hour. Sand bed has the following properties.
Depth of bed : 0.8 m Year 1951 1961 1971 1981 … 2021
Sand grain size : 0.6 mm Population 95 121 135 164 … ?
Sand specific gravity : 2.65 in thousand
Sand shape factor Φ : 0.9
(A) 4,51,622 (B) 3,80,460
Porosity of sand bed n : 0.5
(C) 3,28,323 (D) 4,89,671
Kinematic viscosity r : 1 × 10–6m2/s
Calculate the loss of head in filtration in (cm). 2 5. To treat 6 MLD of water 1 mg/L of bleaching power
(A) 7.86 (B) 8.43 is added to water to have free chlorine residual of
(C) 10.61 (D) 9.45 0.1 mg/l. Find out Cl demand of water in mg/l and
amount of bleaching powder/month in kgs if the availa-
20. A water sample has pH of 10.3. The concentration of
ble Cl in bleaching power is 40%.
hydroxyl ions in the water sample is
(A) 0.9 mg/l and 180 kg/month
(A) 10–10.3 moles/L
(B) 0.9 mg/l and 72 kg/month
(B) 10–37 moles/L
(C) 0.3 mg/l and 72 kg/month
(C) 3.39 mg/L
(D) 0.3 mg/l and 180 kg/month
(D) 0.339 mg/L
Water Supply Engineering Test 1 | 3.147

Answer Keys
1. B 2. B 3. A 4. B 5. D 6. A 7. D 8. A 9. C 10. A
11. D 12. B 13. B 14. D 15. C 16. B 17. A 18. B 19. D 20. C
21. D 22. B 23. A 24. C 25. D

Hints and Explanations


3. Argyria (blue-gray) discoloration of skin and also toxic
to aquatic organisms. Choice (A) hf∝
(1 − n) ∝
1
−3 n 3 n2
Q 15 × 10 × 10 m day
6 3

5. Area = = = 500 m2 n = porosity


vo 30 m3 day m 2
1 1
Volume = Q × DT ∝ hf ∝ 2
TH n
15 × 106 × 10 −3
× 2.5 ∴ As porosity increases, head loss reduces and time
24
for terminal head loss increases.
= 1562.5 m3 ∴ TH increases. Choice (B)
Volume
Depth = = 3.125 m 14. As concentration of impurities increase, as all voids get
Area
filled by impurities shearing off starts early.
π ∴ TB (time for turbidity) reduces.
A = d 2 = 500
4 As concentration of impurities increase, more particles
d = 25.2 m Choice (D) are clogged, more resistance to flow
6. Type I settling i.e., settling under gravity because spe- ⇒ more friction
cific gravity of impurities is greater than water. ∴ More head loss in less time
 Choice (A) ∴ TH decrease Choice (D)
9. 15. Al2 (SO4)3.18H2O + 3 Ca(OH)2
(Alum)
water
→ CaSO4 + 2 Al(OH)3 ↓ + 18 H2O
air
1 mole of Alum → 2 moles of Al (OH)3
666 gm of Alum → 2 × 78 gm of Al(OH3)
Ct > cs 2 × 78
1 gm of alum = = 0.24 gm of Al(OH)3
666
desorption
∴ 25gm of alum will produce
Gas tries to reach to saturated concentration. = 25 × 0.24 = 6 mg/l of solid sludge  → (1)
 Choice (C) Suspended solids (turbidity) removed
= 40 – 15 = 25 mg/l  → (2)
12.
[ HOCl ] × 100 =
1
× 100 = % of HOCl Total dry sludge (solids) removed
[ HOCl ] + OCl −  k
1+ + = (1) + (2) = 31 mg/l
H Discharge of plant = 0.7 m3/s = 0.7 × 24 × 60 × 60
1 Q = 60480 m3/day
0.95 =
1+
(3 ×10−8 ) Total dry sludge solids produced per day
= 31 mg/l × 60480 m3/day
H+
= 31 gm/m3 × 60480 m3/day
[H+] = 5.7 × 10–7 = 1.87 × 106gm/day = 1.87 t/day
 1   1  Volume of solid sludge produced
pH = –log10 [H+] = log10  +  = log10  −7 
 
H  5 .7 × 10  1.87 1.87 t day
= =
pH = 6.24 Choice (B) Density of sludge solids 3.01 t m3
13. As porosity increases, more particles are allowed to = 0.62 m3/day Choice (C)
pass through voids and turbidity reaches early
∴ TB decreases 16. Settling velocity for particles 0.026 mm (<0.1 mm) dia

hf =
fLV (1 − n)
2
Vs =
g d2
× 3 (S – 1)
gd nφ 18 ϑ
3.148 | Water Supply Engineering Test 1

981 (0.0026) 2
 60 
[CO32–] = 7 ×   = 210 mg/l
Vs = (2.5 – 1) = 0.055 cm/sec
18 0.01 2
In settling tank,  61 
[ HCO3− ] = 25 ×   = 1525 mg/l
VS H 1
=
VH L  50   50 
TH in mg/l as CaCO3 = Ca2+ ×   + Mg2+ ×  
VS 30  20   12 
= [assuming freeboard = 0.5m,
VH 60  50   50 
= 300 ×   + 240 ×  
H = 3.5 – 0.5 = 3.0m]  20   12 
 = 1750 mg/l as CaCO3 Choice (D)
0.055 3
= 22. Alkalinity in mg/l as
VH 60
 50   50 
CaCO3 = CO32 − ×   + HCO3− ×  
VH = 1.1 cm/s  30   61 
∴ Flow velocity = 1.1 cm/s Choice (B)
 50   50 
ρ Vd Vd = 210 ×   + 1525 ×  
1 9. Re = w × φ = ×φ  30   61 
µ γ
= 1600 mg/l as CaCO3
 4   0.6 × 10 −3  Alkalinity <TH
=  × −6 
× 0.9 = 0.6
 60 × 60   1 × 10  Carbonate hardness = least of both
150 (1 − n) = 1600 mg/l as CaCO3 Choice (B)
Friction factor f = + 1.75 24.
Re
150 (1 − 0.5) Population
Increase in % increase in
= + 1.75 = 126.75 Year in
Population population = growth
0.6 in
rate = Col ( 3)  × 100
Thousand
hf =
fLV 2
(1− n) Thousands
Col ( 2)
gd n3 φ (1) (2) (3) (4)
2 1951 95
126.75 × 0.8 ×  4   1 − 0.5 
= × 
9.81 × 0.6 × 10 −3  60 × 60   (0.5)3 × 0.9  1961 121 26
26
 × 100 = 27.37
95
= 0.0945 m = 9.45 cm. Choice (D) 1971 135 14 11.57
2 0. p[H] + p[OH] = 14 1981 164 29 21.48
p[OH] = 14 – 10.3 = 3.7
–log10[OH] = 3.7 Constant growth rate assumed for future
Concentration of [OH] r = geometric mean of past growth rates

= 10–3.7 moles/L = 10–3.7 ×


[17]   = 3 27.37 × 11.57 × 21.48
1   = 18.95% per decade
[∵ 17 is eqwt of OH] The population after n decades
= 3.39 × 10–3gm/L = 3.39 mg/l Choice (C) n
 r 
Pn = Po 1 +
2 1. TH in mg/l as CaCO3  100 
 eq wt of Ca CO3  P2021 = Population 4 decades from 1981
= Ca++ in mg/l ×  2+  4
 eq wt of Ca   18.95 
= P1981 1 +  = 164000 × (1.1895)4
 eq wt of CaCO3   100 
+Mg++ in mg/l ×  2+ 
 eq wt of Mg  = 3, 28,323 Choice (C)
[Ca2+] = 15 milli equivalent/L 25. Cl2 dose = 1 mg/l × 40% Cl2 = 0.4 mg/l
[Ca2+] = 15 × 10–3 × 20 gm/L Cl demand = Cl dose added – residual
= 0.3 gm/L = 300 mg/l = 0.4 – 0.1 = 0.3 mg/l
[Mg2+] = 20 mill equivalent/L Total bleaching powder added
= Q × dosage of bleach = 6 × 1 mg/l
 24  = 6 kg/day = (6 × 30) kg/month
= 20 ×   mg/l = 240 mg/l
2 = 180 kg/month Choice (D)
Waste Water Engineering Test 2
Number of Questions: 25 Time: 60 min.

Directions for questions 1 to 25: Select the correct alterna- (C) Medium sized cities
tive from the given choices. (D) All of them
1. Maximum runoff will be obtained from the rain having 9. Prechlorination of sewage before it enters the sedimen-
a duration equal to time of concentration, and this is tation tank may help in
called the 1. Controlling odour
(A) Point rainfall intensity 2. Prevent flies in trickling filter
(B) Time of equilibrium 3. Assist in removal of grease
(C) One hour rainfall 4. Reduce bacterial count
(D) Critical rainfall duration (A) 1 and 4 (B) 1, 2, 3 only
2. A well oxidized sewage will contain (C) 2, 3, 4 only (D) All of the above
(A) Nitrites and sulphur 10. In the final stage of sludge digestion, more resistant
(B) More Ammonia and H2S but less nitrates and materials like proteins and organic acids are attacked
sulphates and broken up by anaerobic bacteria called
(C) Less Ammonia and H2S but more nitrates and (A) Acid formers (B) Suspended culture
sulphates (C) Mesophilic organisms (D) Methane formers
(D) H2S, CO2 and water 11. A river with a flow of 0.25 m3/s receives waste water
3. Particles whose size is greater than 1 µm are at a rate of 0.1 m3/s. River has 8 mg/l of DO with no
(A) Colloidal solids (B) Suspended solids BOD. The waste water with BOD 30 mg/l having no
(C) Settleable solids (D) Dissolved solids DO disposed into it. If deoxygenating and reoxygena-
tion constant are 0.36 d–1 and 0.7 d–1 respectively. Find
4. In determination of Chemical Oxygen Demand, sew-
the critical deficit (DC) and distance at which it occurs
age is titrated with ferrous ammonium sulphate using
on D/s of disposal of waste water?
potassium dichromate as oxidant in the presence of
Assume (DO)saturation = 9 mg/l and velocity of flow in
silver sulphate and mercuric sulphate as catalyst and
river = 0.4 m/s.
inhibitor respectively. In this the excess amount of
(A) DC = 3.825 mg/l at 30.96 km
_______ left in sample after digestion is found.
(B) DC = 3.4 mg/l at 5 km
(A) Potassium dichromate
(C) DC = 3.29 mg/l at 30 km
(B) Ferrous Ammonium sulphate
(D) DC = 2.8 mg/l at 15 km
(C) Mercuric Sulphate
(D) Oxygen 12. In a community of 1500 people water is supplied at
200 l/head/day. If BOD produced is 40 g/head/day and
5. The degree and amount of treatment given to raw sew-
BOD loading rate for oxidation pond is 20 kg/ha/day.
age before disposing off into river stream will depend
(Assume depth of pond, d = 2 m and efficiency of pond
upon
as 80%).
1. Quantity of raw sewage only
Find the BOD of the effluent and whether it can be used
2. Self purification capacity of river stream
for irrigation?
3. Intended use of its water
(A) 100 mg/l and suitable for irrigation
(A) 1, 3 (B) 2 only
(B) 150 mg/l and not suitable for irrigation
(C) 1, 2, 3 (D) 1 only
(C) 75 mg/l and not suitable for irrigation
6. Preliminary treatment reduce the BOD of the waste (D) 50 mg/l and suitable for irrigation
water by
13. Following data pertaining to Activated sludge process
(A) 5 – 10% (B) 10 – 20%
is given below:
(C) 15 – 30% (D) 25 – 40%
Flow rate Q = 12,000 m3/day
7. The girt chambers of a sewage treatment plant nor- BOD of influent yi = 280 mg/l
mally need cleaning BOD of effluent ye = 30 mg/l
(A) Every hour (B) Every day X: MLSS concentration = 3000 mg/l
(C) Every fortnight (D) Every year Xu: MLSS in underflow = 10,000 mg/l
8. Activated sludge treatment plants are normally pre- Xe: MLSS in effluent = 0
ferred for F
If = 0.3 day–1 and θC = 12 days. Find mass of solids
(A) Large sized cities M
(B) Towns and smaller cities wasted in kg/day?
3.150 | Waste Water Engineering Test 2

(A) 1000 (B) 1120 249


(C) 1500 (D) 800 218
14. Find out the Theoretical Oxygen Demand (in mg/l) for 196
glucose of 300 mg/lpresent in waste waters if the fol- BOD 181
lowing chemical reaction is given (mg/l)
C6H12O6 + 6O2 → 6CO2 + 6H2O
(A) 320 (B) 420 10 15 20 25
(C) 120 (D) 220 (B) (T° C)
1 5. A population of 20,000 is residing in a town having an
area of 60 hectares. Water supply per capita is 120 lt/day. 306
If the average runoff coefficient of the area is 0.5 and 274
time of concentration of design rain is 30 minutes, 239
Calculate the discharge for which the sewers of a pro-
BOD 206
posed combined system will be designed for the town?
(mg/l)
[Use rational method for runoff. Assume 80% of water
supply as wastage and maximum sewage discharge as 10 15 20 25
3 × (Average discharge)] (T° C)
(C)
(A) 1.67 m3/s (B) 2.1 m3/s
(C) 1.74 m /s
3
(D) 0.06 m3/s 306
16. Match List – Ι with List – ΙΙ and select the correct 274
answer using the code given below.
239
List – I List – II
BOD 206
(Process) (Biological agent)
(mg/l)
a. Oxidation ditch 1. Facultative bacteria
b. Waste water stabilization 2. Anaerobic bacteria 10 15 20 25
c. Imhoff tank 3. Suspended Culture (D) (T° C)
(Aerobic bacteria)
d. Rotating Biological 4. Attached Culture Common Data for Questions 19 and 20:
Contactors (RBC) (Aerobic bacteria) A completely mixed activated sludge process is used to
treat a waste water flow of 2 MLD having a BOD at 5 days
a b c d abcd as 250 g/l. The biomass concentration of aeration tank is
(A) 4 1 2 3 (B) 3 1 2 4 2100 mg/l and the concentration of the net biomass leaving
(C) 1 2 3 4 (D) 3 4 1 2 the system is 50 mg/l. The aeration tank has a volume of
17. The sewage is flowing at 5 MLD from a primary 250 m3.
clarifer to a standard rate trickling filter. The y520°C  = 19. What is the hydraulic retention time of the waste water
180 mg/l. The value of the adopted organic loading is to in aeration tank? (in hours)
be 150 gm/m3/day and surface loading 1500 l/m2/day. (A) 1 (B) 3
Calculate the efficiency of this filter unit. (C) 9 (D) 12
(A) 85.44% (B) 78.51%
20. What is the average time for which the biomass stays in
(C) 80.03% (D) 69.91%
the system?
18. The 3 day 20°C BOD of a sample of sewage is (A) 3.18 hours (B) 4.61 days
200 mg/l. Draw a graph of 5 day BOD as a function of (C) 5.25 days (D) 8.21 hours
temperature in the range, 10°C to 25°C in steps of 5°C.
21. Match Column – Ι with Column – ΙΙ
Assume KD at 20°C = 0.1
Column – I Column – II
249
P. Grit Chamber 1. Zone settling
218
Q. Secondary settling tank 2. Stoke’s law
196 R. Activated sludge process 3. Aerobic process
BOD 181 S. Trickling filter 4. Contact stabilization
(mg/l)
P  Q  R  S P  Q  R  S
10 15 20 25 (A) 1 2 3 4 (B) 2 1 3 4
(A) (T° C) (C) 1 2 4 3 (D) 2 1 4 3
Waste Water Engineering Test 2 | 3.151

22. Statement Ι: Nitrates are non-objectionable end prod- standard BOD of domestic sewage is 0.09 kg/day/per-
ucts in aerobic treatment of sewage. son. Find the population equivalent of sewage
Statement ΙΙ: Nitrates > 45 ppm cause nitrate poisoning (A) 27,000 (B) 20,000
in infants. (C) 22,000 (D) 25,000
(A) Ι is true and ΙΙ is false
(B) Ι is false and ΙΙ is true 25. Match List – Ι with List – ΙΙ
(C) Both Ι and ΙΙ are true List – I List – II
(D) Both Ιand ΙΙ are false (Empirical Formula) (Catchments)
23. A sewer has a diameter of 250 mm and slope of 1 in P. Inglis formula 1. North India
500. While running full it has a mean velocity of 0.8 Q. Ryve’s formula 2. South India
m/s. If both diameter and slope are doubled, what will R. Nawab Jung Bahadur 3. Old Bombay State
be the changed mean velocity when running half full? formula
(Use Manning’s formula) S. Burge’s formula 4. Hyderabad Deccan
(A) 1.0 m/s (B) 1.2 m/s
5. Based on Indian records
(C) 1.6 m/s (D) 1.8 m/s
24. A combined sewer is serving 30,000 persons having P  Q  R  S P  Q  R  S
BOD 80 gm per capita for day and 60,000 liters indus- (A) 2 1 3 4 (B) 1 3 4 5
trial effluent per day having BOD 500 mg/l. If average (C) 2 3 1 5 (D) 3 2 4 5

Answer Keys
1. D 2. C 3. B 4. A 5. C 6. C 7. C 8. A 9. B 10. D
11. A 12. D 13. B 14. A 15. C 16. B 17. A 18. D 19. B 20. C
21. B 22. C 23. D 24. A 25. D

Hints and Explanations


2. Free ammonia, H2s and nitrites indicate initial stages K1 = 0.36 d–1
of decomposition. Nitrites and sulphates indicates fully K2 = 0.7 d–1
oxidized sewage Choice (C)
ymix =
(0 + 0.1 × 30) = 8.57 mg/l
(0.25 + 0.1)
 Pariticlessize > 10 µ m ⇒ settleable solids. 
  0.25 × 8
> 1 µ m ⇒ suspended solids. (DO)mix = = 5.71 mg/l
3.   0.25 + 0.1
 1 − 10 −3 µ m ⇒ Colloidals solids.
  DO = (DO)sat – (DO)mix
 < 10 −3 µ m ⇒ dissolved solids.  = 9 – 5.71 = 3.29 mg/l
 Choice (B) ymix = Lo 1 − e − k1 t 
7. Grit chambers of a sewage treatment plant are cleaned
8.57 = Lo 1 − e −0.36 × 5 
every fort night Choice (C)
8. Activated sludge treatment plants are prepared large ⇒ Lo = 10.27 mg/l
sized cities as they are costly. Choice (A) Critical time
9. Prechlorination helps for first three whereas Post chlo- 1 K  K − K1  
tc = ln  2 1 − Do . 2 
rination helps for reducing bacterial count and BOD. K 2 − K1  K1  K1 L0  
 Choice (B)
10. In Acid Fermentation (1st stage) sewage is acted upon   =
1  0.7 
ln 
(0.7 − 0.36)  
1 − 3.29 × 
by acid formers. In Alkaline Fermentation (3rd stage) 0.7 − 0.36  0.36  0.36 × 10.27  
 
sewage is acted upon by methane formers. Choice (D)
  = 0.896 days
11. K1
Critical deficit: DC = Lo e − k1tc
River Waste water K2
QR: 0.25 m3/s QW = 0.1 m3/s
0.36
(DO)R: 8 mg/l (DO)W = 0 = × 10.27 e–0.36 × 0.896
0.7
yR : 0 yw : 30 mg/l
= 3.825 mg/l
3.152 | Waste Water Engineering Test 2

Velocity flow in river V = 0.4 m/s Quantity of sewage produced per second
Distance at which critical flow occurs 1920
= = 0.022 cumecs
= 0.4 × 0.896 × 24 × 60 × 60 24 × 60 × 60
= 30.96 × 103 m
Maximum sewage discharge = 3 × 0.022
= 30.96 km. Choice (A)
Qs = 0.066 cumecs
12. Population = 1500 Storm water discharge by Rational Method
Water supplied = 200 l/head/day KPC A
80% of water is converted to waste water QR =
36
Qw = 1500 × 200 × 0.8 = 24 × 104 l/day
   = 240 m3/day = 0.24 MLD  a 
Total BOD produced = 1500 × 40  PC = t + b cm/hr 
 c

= 60 × 103g/day as tc ≥ 20 minutes 
= 60 kg/day  
 a = 100; b = 20 
Total BOD applied = Qyi
60 × 106mg/day = 0.24 × 106l/day × yi  
yi = 250 mg/l 100
PC = = 2cm/hr
η = 80% 30 + 20
250 − ye 1
80 =  × 100 QR = × 0.5 × 2 × 60
250 36
ye = 50 mg/l ≤ 100 mg/l which is the limitation value = 1.67 cumecs
for irrigation Total peak discharge = Qs + QR = 0.066 + 1.67
∴ Effluent water is suitable for Irrigation. Choice (D) = 1.736 cumecs. Choice (C)
13. We know, 100
1 7. Efficiency η = 
F Qyi 1 + 0.0044 u
=
M VX
u = organic loading in kg/ha–m/day
12, 000 × 280 u = 150 gm/m3/day
0.25 =
V × 3000 1 hectare = 104m2
V = 4480 m3 u = 150 × 104gm/ha–m/day
  = 1500 kg/ha–m/day
Mass of solids in aeration 100
θc =
Mass of solids wasted/day η=
1 + 0.0044 1500
VX
12 =   = 85.44%. Choice (A)
Qw X u + Qe X e
= L 1 − (10 ) 
o − KD × t
4480 × 103 × 3000 18. y320 C

QwXu = [∵ 1m3 = 1000 L and Xe = 0]
12 200 = L[1 – (10)–0.1×3]
= 1.12 × 109 mg/day L = 400.95 mg/l
= 1120 kg/day. Choice (B)   K D 10 o C = K D 20 o C [1.047]
T − 20 o C
( ) ( )
1 4. Mol. Wt of C6H12O6 (Glucose)
= (6 × 12) + (12 × 1) + (6 × 16) = 180 = 0.1[1.047]10–20
Mol. wt of 6O2 = 6[2 × 16] = 192 = 0.063
180 parts of glucose demand: 192 parts of O2 K D 15o C = 0.1[1.047]15–20 = 0.079
192 ( )
1 part glucose demand = parts of O2 K D 25 o C = 0.1[1.047]25–20 = 0.1258
180 ( )
192 10 o C
300 mg/l of glucose demand = × 300 y = 400.95[1 – 10–0.063×5]
180 5
= 206.82 mg/l
= 320 mg/l of O2 o

∴ TOD = 320 mg/l. Choice (A) y515 C


= 400.95[1 – 10–0.079×5]
1 5. Quantity of sanitary sewage produced per day = 239.48 mg/l
o

 80   × 120 × 20000 = 1.92 × 106ltrs. y520 C = 400.95[1 – 10–0.1×5]s


=
 100  =
274.16 mg/l
Waste Water Engineering Test 2 | 3.153

o
y525 C = 400.95 [1 – 10–0.1258×5] 1 23 12
23. V = R S
N
=
306.74 mg/l
D
306.74 Hydraulic mean depth R =
4
274.16 23 12
V1 R1 S1
239.48 = 23 12
V2 R2 S2
206.82
D1
BOD R1 =
(mg/l) Choice (D) 4
0 10 15 20 25 D2 2 D1 D
R2 = = [though running half/full R = ]
(T° C) 4 4 4
1
 Choice (D) S1 =
500
1 9. Q = 2 × 106 lit/day
1
yi = 250 mg/l S2 =
X = 2100 mg/l 250
12
Xe = 50 mg/l  1 
V = 250 m3 = 250 × 103 lit  
0.8 R2 3 500 
= 1 23 ×
V 250 × 10
3
V2 ( 2 R1 )  1 
12

HRT = =  
Q 2 × 106 250
= 0.125 days = 3 hours. Choice (B) V2 = 1.795 m/s
≃1.8 m/s. Choice (D)
VX 2 4. (i) BOD of sewage produced per day
20. θC = 
Qw X w + Qe X e = 30000 × 80 = 2400 × 103 gm/day
Neglecting Xw (being small and not given) = 2400 kg/day
(ii) BOD of industrial effluent per day
250 × 103 × 2100 = 60000 × 500
QC = = 30 × 106mg/day
2 × 106 × 50
= 30 kg/day
  = 5.25 days. Choice (C) Total BOD entering the sewer
22. Statement Ι and ΙΙ seems to be contradicting. = (i) + (ii)
In infants there exists lower acidity in intestines which = 2430 kg/day
permits growth of nitrate reducing bacteria, which con- Population equivalent
vert nitrate to nitrites. Nitrates’ having great affinity for Total BOD
haemoglobin than oxygen and cause suffocation and =
Standard BOD of domestic sewage
body turns blue called blue baby disease or mathaemo-
globinemia. This is only in children <6 months. 2430
= = 27000. Choice (A)
 Choice (C) 0.09
Solid Waste Management and Pollution Test 3
Number of Questions: 25 Time: 60 min.

Directions for questions 1 to 25: Select the correct alterna- 11. The sound pressure level for a jet plane on the ground
tive from the given choices. with sound pressure of 200 µ bar should be
1. Which of the following factors is primarily responsible (A) 40 dB (B) 80 dB
for causing air pollution in modern days is (C) 100 dB (D) 120 dB
(A) Dust storms 12. Express 350 µg/m of SO2 in ppm at STP?
3

(B) Forest fires (A) 0.122 (B) 0.142


(C) Industries and automobiles (C) 0.20 (D) 0.31
(D) None 13. An air parcel having 50°C temperature moves from
2. The most hazardous gaseous air pollutant for human ground level to 800 m elevation in dry air following
health is: the adiabatic lapse rate. The resulting temperature of
(A) Nitrogen air parcel at 800 m elevation will be?
(B) Carbondioxide (A) 40°C (B) 35°C
(C) Oxygen (C) 42°C (D) 50°C
(D) Sulphurdioxide 14. The maximum dispersion of pollutants in atmosphere
3. Among the given choices below; which of the follow- occur when
ing is not a primary air pollutant? (A) Environmental lapse rate is equal to adiabatic
(A) PAN lapse rate
(B) Volatile organic compounds like hydrocarbon (B) Environmental lapse rate is less than adiabatic
(C) Suspended particulate matter lapse rate
(D) Oxides of nitrogen (C) Environmental lapse rate is greater than adiabatic
4. During temperature inversion in atmosphere, air pollut- lapse rate
ants tend to. (D) None
(A) Accumulate above inversion layer 15. Elevation and temperature data for a place are tabulated
(B) Accumulate below inversion layer below:
(C) Disperse laterally
Elevation, m Temperature °C
(D) Disperse vertically
10 m 25.25°C
5. For ambient air quality standard, the permissible
250 m 15.70°C
SO2 for residential and industrial areas in India is
(A) 40 µg/m3 (B) 50 µg/m3 Based on the above data, lapse rate can be referred as:
(C) 65 µg/m 3
(D) 35 µg/m3 (A) Sub-adiabatic (B) Super adiabatic
6. The frequency of sound is measured in (units): (C) Neutral (D) inversion
(A) Hertz (B) Doboson unit (DU) 16. If energy content of solid waste as discarded is 14,700
(C) Decibel (dB) (D) None kJ/kg. Find energy content on dry basis if moisture
7. As per IS:4954 – 1964; An acceptable noise level for content of solid waste is 20%
residential and business urban areas is (A) 18375 kJ/kg (B) 18000 kJ/kg
(A) 40-50 dB (B) 30-40 dB (C) 14,700 kJ/kg (D) 15,000 kJ/kg
(C) 15-25 dB (D) 50-60 dB 1 7. The composition of certain MSW sample and specific
8. Noise is measured in units of: weight of its various components are given below:
(A) Bacqueral (B) Doboson
Percent by Specific weight
(C) Hertz (D) Decibel Component
weight (kg/m3)
9. The method of refuse disposal, involving burial in Food waste 70 300
trenches, is called Dirt ash 20 500
(A) Incineration (B) Composting
Plastics 10  65
(C) Polverisation (D) None
10. The quantity of refuse produced in an average Indian The specific weight (kg/m3) of the MSW sample is
city or a town is of the order of: (A) 209 (B) 319
1 1 (C) 217 (D) 234
(A) − t/c day (B) 4 – 6 t/c day
4 5 1 8. A waste water stream (flow = 3m3/s) with ultimate
(D) 2 – 4 t/c day (D) 1 – 2 t/c day BOD 110 mg/ is joining a small river (flow = 10m3/s),
Solid Waste Management and Pollution Test 3 | 3.155

ultimate BOD = 5 mg/. Both water streams get mixed (A) Leather and tin cans
up simultaneously at where cross sectional area of the (B) Plastics and wood
river is 70 m2.Assuming K = 0.25 / day, the BOD (in (C) Cardboard and glass
mg/) of the river water, 10 km downstream of the mix- (D) Food waste and garden trimmings
ing point is 23. Two electrostatic precipitators are in series. The frac-
(A) 8.832 gm/m3 (B) 24 gm/m3 tional efficiencies of the upstream and downstream
(C) 15 gm/m3 (D) none ESPS for size dp are 90% and 80% respectively. What
19. The reference pressure used in determination of sound is the overall efficiency of the system for same dp?
pressure level is: (A) 93% (B) 95%
(A) 20 db (B) 10 µPa (C) 98% (D) 90%
(C) 20 µPa (D) 10 db 2 4. SO2 and CO adversely effects
20. Which one of the following is the correct sound inten- (A) Oxygen carrying capacity of blood and function-
sity expression with usual notations. ing of lungs respectively.
(A) dB = 10 log10(I/I0)2 (B) Functioning of the respiratory system and brain
(B) dB = 10 log10 (I/I0) respectively.
(C) dB = 10 log10(I – I0)2 (C) Functioning of the respiratory system and oxygen
(D) dB = 10 log10 (I – Io) carrying capacity of blood respectively.
21. A source emitting 40 dB, 70 dB, 110 dB of different (D) Functioning of air passage and chess respectively.
times in a day. What is average noise produces by 25. Two air pollution control devices that are usually used
source in a day? to remove very fine particles from the flue gas are:
(A) 220 dB (B) 80 dB (A) Cyclone and venture scrubber
(C) 74 dB (D) 100 dB (B) Cyclone and packed scrubber
22. Which among the following are two biodegradable (C) Electrostatic precipitator and fabric filter
components of municipal solid waste (D) Settling chamber and tray scrubber

Answer Keys
1. C 2. D 3. A 4. B 5. B 6. A 7. A 8. D 9. B 10. A
11. D 12. A 13. C 14. C 15. B 16. A 17. D 18. A 19. C 20. A
21. D 22. D 23. C 24. C 25. C

Hints and Explanations


1. Industries and automobile is primarily responsible for 20 × 106 µ. Pa
causing air pollution in modern days. Choice (C) dB = 20 log10
20 µ. Pa
2. Sulphur dioxide is the most hazardous gaseous air pol- = 20 × 6 = 120 dB Choice (D)
lutant. Choice (D)
3. PAN is not a primary air pollutant. Choice (A) 12. At STP
5. The permissible SO2 for residential and Industrial area M
1PPM = × 103µglm3
in India is 50 µg/m3. Choice (B) 22.4
6. Frequency of sound is measured in Hertz. Choice (A) 22.4
1µg/m3 = PPM
7. An acceptable noise level of 40–50 dB in residential M × 103
area is allowed. Choice (A)
22.4
8. Decibel is the unit to measure noise. Choice (D) 350 µg/m3 = × 350 PPM
M × 103
11. Sound pressure level in (dB)
Where M = molecular weight of SO2
 Prms 
= 20 log10  = 32 + (2 × 16) = 64
 20 
22.4 × 350
Prms = 200 µ bar =
64 × 103
= 200 × 105µPa  [1 bar = 100kPa= 105Pa]
= 20 × 106µ.Pa = 0.122 PPM. Choice (A)
3.156 | Solid Waste Management and Pollution Test 3

13. For adiabatic lapse rate: There is a decrease in temper- 100 70 20 10


  = + +
ature of 1°C for every 100 m specific weight of MSW 300 500 65
dT
i.e., = –1°C/100m Specific weight of MSW (kg/m3) = 234. Choice (D)
dZ
100m → –1°C Q1 × ( BOD)1 + Q2 × ( BOD)2
800m → ? 18. BOD of mixture = s
Q1 + Q2
800 ( −1 °C)
100
= − 8 °C
=
(3)(110) + (10 × 5)
∴ The resulting temperature 3 + 10
= 50 – 8 = 42°C. Choice (C) = 29.23 gm/m3
dT 25.25 − 15.70 (BOD) of mixture at 10 km downstream of mixing
1 5. Lapse rate = = point
dZ 250 − 10
y10km = y0 (1 – 10–K t)
9.55
= = 0.039 Time taken to reach 10 km,
240
distance
−3.97 °C t= ;
= velocity
100 m
Q1 + Q2 10 + 3
−1 °C velocity of flow = =
Adiabatic lapse rate = A 70
100 m
V = 0.185 m/sec
∴ Lapse rate > ALR
⇒ Super adiabatic. Choice (B) 10 × 1000 1
∴ t = × ,
1 6. Energy content on dry basis 0.185 24 × 60 × 60
 100  t = 0.625 day
= Energy content as discarded ×  
100 − mc  y10 = 29.23 [1 – 10–0.25 × 0.625]
where mc - moisture content. gm
   = 8.832  Choice (A)
100 m3
= 14,1700 ×
100 − 20
1
∑ (10 ) N / 20
L
= 18375 KJ/kg. Choice (A) 21. LP  = 20 log10
N
100
17. = 1  2040 70 110

specific weight of MSW =
20 log10 10 + 10 20
+ 10 20
 
3 
%Foodwaste
specific weight of foodwaste =
100.54dB. Choice (D)
% dirt ash 23. Particles removed by ESP1 = 90%
+ Only 10 % particles remains left and flows through
specific weight of dirt ash
ESP 2.
%plastics So, Particles removed by ESP2 = 80 % of 10% = 8%
+
specific weight of plastics ∴ Overall efficiency = 90 + 8 = 98% Choice (C)
Environmental Engineering Test 4
Number of Questions: 30 Time: 75 min.

Directions for questions 1 to 30: Select the correct alterna-


tive from the given choices.
Volume b
1. SOx in atmosphere is measured by (m3)
(A) Non-dispersive infrared analyzer
(B) West and Gack method a
(C) Sodium arsenate method
(D) Gas Chromatography
2. Algae dies out, though fish life may survive, in a river time (hrs)
zone, known as (A) a – b
(A) Zone of degradation (B) a + b
(B) Zone of active decomposition (C) a × b
(C) Zone of recovery (D) larger of a or b
(D) None of these
9. Which of the following are removed by rapid sand filter
3. Man hole covers are circular in shape to from water?
(A) strengthen the cover 1. Dissolved solids
(B) make entry convenient 2. Suspended solids
(C) for architectural reasons 3. Bacteria
(D) prevent falling of cover into manhole 4. Helminths
4. The factors which affect the sludge digestion are (A) 1 and 2
1. Temperature (B) 2 and 3
2. pH (C) 1 and 3
3. Mixing and stirring of raw sludge (D) 2, 3 and 4
4. Seeding of sludge 10. The yield of a well depends upon
(A) 1 and 2 (B) 2 and 3 (A) permeability
(C) 2, 3, 4 only (D) All the above (B) area of aquifer opening into the wells
5. Primary Clarifier used in waste water treatment follows (C) actual flow velocity
______ type of settling. (D) All of the above
(A) Type I settling 11. Match List - A (lapse rate) with List - B (different types
(B) Type II settling of plumes) at stack level.
(C) Type III settling
List – A (Lape rate)
(D) Type IV settling
P. ALR
6. If the Cl demand of water is 0.5 mg/l to have a residual
Cl of 0.1 mg/l. What dose of bleaching powder is added Altitude ELR
to the water if bleaching powder contains 32% of avail-
able chlorine.
(A) 1.875 mg/l (T°C)
(B) 1.25 mg/l
(C) 1.562 mg/l Q.
ELR
ALR
(D) 2.188 mg/l Altitude
7. If the methyl orange alkalinity of water equals or
exceeds total hardness, all of the hardness is
(A) Non-Carbonate hardness
(T°C)
(B) Carbonate hardness
(C) Pseudo hardness R. ALR
ELR
(D) Negative non-carbonate hardness Altitude
8. The given figure shows roughly the daily mass curves
of supply and demand from an elevated reservoir.
The minimum required capacity of the reservoir is (T°C)
given by
3.158 | Environmental Engineering Test 4

S. ELR (A) 18641 MLD (B) 19094 MLD


ALR (C) 20986 MLD (D) 22782 MLD
Altitude
14. A standard multiple tube fermentation test was con-
ducted on a sample of water from a stream. The results
of the analysis of the confirmed test are given below.
(T°C)
No. of positive No. of negative
Sample
results out of results out of
List – B size (ml)
5 tubes 5 tubes
1.
10 2 3
1 0 5
0.1 2 3

Fumigation plume
MPN Index and 95% confidence limits for combination
of positive results when 5 tubes used per dilution (1ml,
2.
0.1ml, 0.01ml)

Combination MPN Index 95% Confidence limit


of positives per 100 ml Lower Upper
1–3–5 280 120 360
Trapping plume 2–0–2 330 160 380
3. 3–5–3 140 100 210

Using the above MPN Index table, the Most Probable


number (MPN) of the sample is
(A) 25 (B) 14
(C) 28 (D) 33
Looping plume 1 5. 12 mg of copperas is consumed with lime at a coagu-
lation basin per litre water. Determine the quantity of
4.
copperas and the quick lime required to treat 20 MLD.
[Fe = 56, S = 32, O = 16, Ca = 40, H = 1]
(A) 240 and 16 kg/day
(B) 240 and 48.4 kg/day
(C) 180 and 16 kg/day
Coning plume (D) 180 and 48.4 kg/day
P Q R S P Q R S Common Data for Questions 16 and 17:
(A) 3 4 1 2 (B) 3 1 2 4 In a slow sand filter, water is supplied at the rate of
(C) 1 4 2 3 (D) 4 2 1 3 24 million litres per day and rate of filtration is 5 m3/hr/m2.
12. Find the equivalent noise level for fluctuating noise If Backwashing is done for 15 minutes at the rate of 5 times
level of 95 minutes. The one with 80 dB lasted for of rate of filtration for every 24 hours (Let L : B = 2:1)
10 minutes, followed by 60 dB for 80 minutes followed 16. Find the volume of water filtered between backwashing
by 100 dB for 5 minutes. in m3.
(A) 68 dB (B) 74 dB (A) 23750 m3 (B) 24000 m3
(C) 87 dB (D) 96 dB (C) 24,860 m3 (D) 25,390 m3
1 3. In Delhi it has been decided to provide 200 Litre per 17. Volume of water used in backwashing in m3.
head per day in 2020. Estimate the domestic water (A) 250 m3 (B) 500 m3
requirements of this city in 2020 by projecting the pop- (C) 750 m 3
(D) 1250 m3
ulation of the town by incremental increase method. 18. Calculate the requirement of soda ash or softening
Year Population 2 MLD of water formed to have the following chemical
1970 2, 37, 98, 624 composition.
1980 4, 50, 78, 325
CO2 = 39.6 mg/l
Ca2+ = 44 mg/l
1990 5, 53, 86, 432
Mg2+ = 18 mg/l
2000 6, 91, 87, 241 HCO3 = 122 mg/l
Environmental Engineering Test 4 | 3.159

(A) 168.4 kg/day (B) 180.2 kg/day (A) 78.43m3 and 80 tonnes
(C) 230.1 kg/day (D) 242.8 kg/day (B) 66.67m3 and 68 tonnes
19. Which among the following are dechlorinating agents. (C) 50.98m3 and 52 tonnes
1. Sulphur dioxide gas (D) 44.12m3 and 45 tonnes
2. Ammonia
3. Sodium sulphate 25. Match List - I with List - II
4. Sodium sulphite
List – I List – II
(A) 1 and 2 (B) 2, 3 and 4 (Treatment units) (Types of Processes)
(C) 1, 2 and 4 (D) All the above
a. Trickling filter 1. Symbiotic
20. A town with a population of one lakh is to be supplied b. Activated sludge 2. Extended aeration
with water, daily at 120 l/head. The variation in demand
c.. Oxidation ditch 3. Suspended growth
is as follows.
d. Oxidation pond 4. Attached growth
6am – 10am – 50% of total demand
10am – 12pm – 10% of total demand a b c d
12pm – 6pm – 5% of total demand (A) 3 4 2 1
6pm – 12am – 30% of total demand (B) 4 3 1 2
12am – 6am – 5% of total demand (C) 3 4 1 2
Determine the capacity of service reservoir assuming (D) 4 3 2 1
pumping to be at uniform rate and the period of pump-
ing from 6am – 10am and6pm – 10pm (Neglect fire Common Data for Questions 26 and 27:
demand) The sewage is flowing at 6 MLD from a primary clarifier to
(A) 1.2 ML (B) 2.4 ML a standard rate tricking filter. The 5 day BOD of the influ-
(C) 3.6 ML (D) 4.8 ML ent is 150 mg/l. The adopted organic loading is to be 160
gm/m3/day and surface loading 2000 l/m2/day.
21. A 30 cm diameter sewer with an invert slope of 1 in 400
is running full. Calculate the rate of flow in the sewer. 26. Determine the volume of the trickling filter.
(use mannings equation and N = 0.015) (A) 4532 m3 (B) 5138 m3
(A) 0.02 m3/s (B) 0.03 m3/s (C) 5625 m
3
(D) 6100 m3
(C) 0.04 m /s
3
(D) 0.05 m3/s 27. Calculate the efficiency of the filter
22. If the percapita contribution of suspended solids and (A) 85% (B) 74%
B.O.D is 100 gm and 60 gm, find the population equiv- (C) 61% (D) 91%
alent of 50,000 liters daily of industrial waste water 28. The composition of a certain MSW sample and specific
containing 1800 mg/l of suspended solids. weight of its various components are given below.
(A) 800 (B) 900
(C) 1000 (D) 1100 Component % by weight
Specific weight
(kg/m3)
23. The following data are given for a channel type grit
chamber of length 8 m. Food waste 60 250

1. Flow through velocity = 0.4 m/s Dirt and Ash 20 400


2. Depth of waste water at peak flow in the channel = Plastics 10  70
1m Wood and Yard waste 10 130
3. Specific Gravity of inorganic particles = 2.6
4. g = 9.8 m/s2, µ = 1.002 × 10–3 N–s/m2 at 20°C, Specific weight (kg/m3) of the MSW sample is
ρw = 1000 kg/m3 (A) 120 (B) 143
Assuming that the stokes law is valid, the largest diam- (C) 196 (D) 219
eter particle that would be removed with 100%. effi- 29. In a stream flowing at 6 m3/s has no concentration of
ciency is. chemical. Industrial water is released into stream at
(A) 0.08 mm (B) 0.12 mm 25 MLD with chemical concentration as 30 mg/l. If rate
(C) 0.18 mm (D) 0.24 mm of dissipation of impurities in stream is 0.13 mg/l/hr.
24. A sedimentation tank is treating 5 million litres of sew- Calculate the distance at which the chemical is removed
age per day containing 300 ppm of suspended solids. completely from stream (velocity of stream is 0.3 m/s)
The tank removes 60% of suspended solids. Calculate (A) 9.72 km
the quantity of sludge produced per day is bulk and (B) 11.47 km
weight respectively if moisture content of sludge is (C) 8.61 km
98% (assume specific gravity of wet sludge = 1.02) (D) 12.43 km
3.160 | Environmental Engineering Test 4

30. A sewage containing 300 mg/l of suspended solids is digestion of the sludge from 2000 m3 of sewage are in
passed through primary settling tank where 60% of sus- 3:2 ratio. If the fuel value of methane is 36000 kJ/m3.
pended solids are removed. 70% of suspended solids Find the fuel value of gas produced (Assume gas pro-
are volatile. The solids from the primary settling tank duction is at the rate of 0.9 m3/kg of volatile) in MKJ.
are digested to recover the gas where volatile matter (A) 26.3 (B) 31.4
is reduced by 70%. Methane and CO2 produced in the (C) 48.5 (D) 34.8

Answer Keys
1. B 2. A 3. B 4. D 5. C 6. A 7. B 8. B 9. D 10. A
11. A 12. C 13. D 14. D 15. B 16. A 17. D 18. B 19. C 20. B
21. C 22. B 23. D 24. D 25. D 26. C 27. A 28. C 29. B 30. D

Hints and Explanations


1. SOx are measured by West and Gack method. 9. Rapid sand filters are less efficient in removing bacteria
 Choice (B) (80 to 90%) but efficient in removing color and sus-
pended solids.
2. In zone of degradation, degradation just begins and all
Helminths are worms, which naturally would get
aquatic organisms disappear except fish. Choice (A)
removed in rapid sand filters. These are larger than
3. Circular man hole covers are convenient to easy entry bacteria. Choice (D)
at times of repair. Choice (B) 10. Discharge Q = KIA
4. Factors affecting sludge digestion are I = Hydraulic gradient
1. Temperature K = permeability coefficient
2. pH A = c/s area of well. Choice (A)
3. Seeding of sludge Li
4. Mixing and stirring of raw sludge. Choice (D) 12. Leq = 10 log10 Σ1010 × Ti
5. Primary clarifies is aimed to remove settleable organic ti
Ti =
solids of large size in waste water. These organic sol- Σ ti
ids are similar to flocculent particles and influence 10
other particles surrounding them and all of them set- ∴ T2 = = 0.105
10 + 80 + 5
tle together. This is called hindered/Type-III settling.
 Choice (C) 80
T2 = = 0.842
95
6. Cl2 added = 0.5 + 0.1 = 0.6 mg/l
5
Cl 2 added T3 = = 0.053
Bleaching powder = 95
% of Cl 2
 10 80
  60 
0.6 10 × 0.10.5 + 10 10 × 0.842 
= = 1.875 mg/l. Choice (A)    
0.32 Leq = 10 log10  
 100

 + 10 10 × 0.053 
   
 
 Alkalinity 
7. Carbonate hardness = least of  ( or ) 
=
87.33 dB. Choice (C)
 
 Total hardness
13.
∴ When Alkalinity > Total hardness
Increase in Incremental
Carbonate Hardness = Total Hardness Year Population
Population Increase
∴ “Methyl Orange Alkalinity” indicates use of methyl
1970 2,37,98,624 2,12,79,701 (+) 1,09,71,594
orange indicator is titration, adopted for determin-
1980 4,50,78,325 1,03,08,107 (+) 34,92,702
ing Alkalinity which changes its color from orange 1990 5,53,86,432 1,38,00,809
at pH = 4.6 to pink at pH = 4. 2000  6,91,87,241
 Choice (B) Total 4,53,88,617 1,44,64,296
8. Minimum/Balancing storage capacity of reservoir Average
x = 1,51,29,539 y = 48,21,432
= maximum surplus + maximum deficit = b + a. per
decade
 Choice (B)
Environmental Engineering Test 4 | 3.161

Expected population at the end of year 2020 (after 16. Volume of water filtered between backwashing
2 decades from 2000) = R.O.F × time of filtration × area of filter
2 × (2 + 1)  15 
P = Po + 2 x + .y = 5 ×  24 −  × 200 = 23750 m3. Choice (A)
2  60 
2×3 17. Volume of water used in backwashing
  = 69187241 + 2(15129539) + × (4821432)
2 = (Rate of backwashing × time of backwashing × area
  = 11,39,10,615 of filter)
∴ Water requirement in 2020 @ 200 l/head/day 15
= (5 × 5) × × 200 = 1250 m3. Choice (D)
200 × 11,39,10,615 60
= MLD
106
18. Concentration of soda ash required
= 22,782 MLD. Choice (D)
Na2 CO3 = Ca2+ + Mg2+ – Alkalinity
1 4. Since the volume of samples is 10 times more than  (m.eq/lt) , (m.eq/lt)
specified sample size, the MPN from table is divided
by 10 to get actual MPN of the sample 44
Ca2+ = = 2.2 m.eq/lt
330 40 2
MPN = = 33 . Choice (D)
10 18
Mg2+ = = 1.5 m.eq/lt
15. The equations involved are 24 2
Fe SO4.7H2O + Ca (OH)2 122
(hydrated lime)
HCO3 = = 2 m.eq/lt
61 1
→ Ca SO4 + Fe(OH)2 + 7H2O
Na2 CO3 = 2.2 + 1.5 – 2 = 1.7 m.eq/lt
Ca(OH)2 → Ca O + H2 O
Soda ash = soda ash in m.eq/lt × eq. wt of Na2 CO3
Q = 20 MLD
Dosage of copperas = 12 mg/l 2 ( 23) + 12 + 3 (16)
= 1.7 × = 90.1 mg/l
Fe SO4.7H2O = 56 + 32 + 64 + 7(2 + 16) = 278 2
Ca O = 56 Total soda ash required
Total quantity of ferrous sulphate consumed = 2 × 90.1 = 180.2 kg/day. Choice (B)
= 20 × 12 = 240 kg/day
278 parts of copperas = 56 parts of Ca O 19. Sodium sulphate is not a dechlorinating agent whereas
56 sodium thisulphate, sodium meta bisulphate, sodium
1 part of copperas = Ca O sulphite, sodium bisulphate, ammonia, sulphur dioxide
278
gas are dechlorinating agents. Choice (C)
56
12 mg/l of copperas =
 × 12 as Ca O 20. Total demand = 1,00,000 × 120 = 12 MLD
278
Volume of water demand in a day = 12 ML
= 2.42 mg/l as Ca O
∴ Ca O required =
 2.42 × 20 12 3
Rate of supply = = ML/hr
= 48.4 kg/day. Choice (B) 8 2

Demand % of Demand Supply Cumulative Cumulative Deficit Surplus


Time
total (ML) (ML) demand (ML) supply (ML) (ML) (ML)

3
6am-10am 50 6 ×4=6 6 6 – –
2
10am-12pm 10 1.2 – 7.2 6 1.2 –

12pm-6pm 5 0.6 7.8 6 1.8 –
3
6pm-12am 30 3.6 ×4=6 11.4 12 – 0.6
2
12am-6am 5 0.6 – 12 12 – –

∴ storage capacity
= Maximum surplus + Maximum deficit
= 0.6 + 1.8 = 2.4 ML. Choice (B)
3.162 | Environmental Engineering Test 4

21. D = diameter of server = 30 cm = 0.3 m 24. Volume of sewage treated


Π 2 Π =5 M-liters/day
Area of sewer A =  D   = × (0.3)2 = 0.07m2 Mass of suspended solids in sewage = 300 × 5
4 4
π =1500 kg/day
.D As 60% solids are removed in sedimentation tank.
A 4 D 0.3 3
R= = = = = Mass of solids removed in sedimentation tank
P π.D 4 4 40
60
1 = 1500 ×
S= 100
400
= 900 kg/day
N = 0.015 When moisture content is 98%
Use Mannings equation 2kg of solids → 100kg of wet sludge
1 23 12 100
V= R S 900 kg of solids → × 900
N 2
2 1
1  3 3  1 2 = 45000 kg
  =     Wet sludge produced per day = 45000 kg = 45 tonnes
0.015  40   40 
Sp.gravity of wet sludge
  = 0.59 m/s = 1.02 × 1 t/m3
Q=A×V = 1.02 t/m3
  =0.07 × 0.59 Volume of wet sludge per day
  =0.0413 m3/s. Choice (C)
45
2 2. 1 liter of industrial waste water contains 1800 mg = = 44.12 m3. Choice (D)
1.02
of suspended solids 50,000 liters of industrial water
1800 × 50000 26. y5 = 150 mg/l
produce = gm of suspended solids =
103 Total BOD = 150 × 6 = 900 kg/day
= 9,00,000 gm/day
90000 gm
Volume of filter media required
Standard percapita solid contribution of suspended sol-
ids = 100 gm Total BOD
=
90000 Organic loading rate
∴ Population equivalent = = 900.
100 900000
= = 5625 m3. Choice (C)
 Choice (B) 160
2 3. L = 8 m 27. Organic loading u = 160 gm/m3/day
VH = 0.4 m/s 160
H=1m = × 10 4 kg/ha-m/day
1000
µ
Kinematic viscosity J =  = 1600 kg/ha-m/day
ρ
100
1.002 × 10 −3 η=
= = 1.002 × 10–6 m2/sec 1 + 0.0044 1600
1000
100
η = 100%     = = 85.03%. Choice (A)
1 + 0.176
L H
∴ = 100 % A % B %C
VH VS 28. = + +
SD SA SB SC
8 1
=
0.4 VS 100 60 20 10 10
= + + +
S D 250 400 70 130
VS = 0.05 m/s
g d2 SD = 196.16 kg/m3 Choice (C)
VS = ( S − 1)
18 ϑ QR + C R + QW CW
29. Cmix =
9.80 d2 QR + QW
0.05 = (2.6 – 1) ×
18 1.002 × 10 −6 0 + (25 × 106 × 30 )
= = 1.38 mg/l
d = 2.398 × 10–4 m [25 × 10 + (6 × 1000 × 60
6 2
× 24 )]
  = 0.24 mm. Choice (D)
Environmental Engineering Test 4 | 3.163

Rate of dissipation = Rate of removal = 0.13 mg/l/hr Volatile matter reduced in 20000 m3 of sewage
1.38 ( 20000 × 1000)
Time to remove chemical from water = = 10.62 hr = 89.6 ×
0.13 106
Distance travelled by river in 10.62 hr = 0.3 × 10.62 × = 1792 kg
602 = 11469.6 m = 11.47 km Choice (B) 1kg volatile matter = 0.9 m3 G as 1792 kg volatile
3 0. Total suspended solids in sewage = 300 mg/l matter = 0.9 × 1792 = 1612.8 m3
Suspended solids removed = 60% (300) = 180 mg/l 3
Volatile solids removed as sludge = 70% (180) Methane produce = × 1612.8 = 967.68 m3
3+ 2
= 128 mg/l
Volatile solids are reduced by 70% in digestion tank Fuel value of gas = 36000 × 967.68 = 34.8 MKJ
= 70% (128) = 89.6 mg/l  Choice (D)
This page is intentionally left blank
Unit XI
Transportation Engineering
This page is intentionally left blank
Transportation Infrastructure Test 1
Number of Questions: 25 Time: 60 min.

Directions for questions 1 to 25: Select the correct alterna- 10. The sight distance available to a driver travelling on the
tive from the given choices. road at any instance depends on the following, factors.
1. Indian Road Congress was established in the year ____. I. height of drivers eyes above the road surface
(A) 1932 (B) 1930 II. height of object above the road surface
(C) 1934 (D) 1936 (A) Both I and II are true
(B) I is false and II is true
2. The maximum grade compensation necessary to ease
(C) Both I and II are false
the gradients at horizontal curves is ______.
(D) I is true and II is false
60 30 + R
(A) (B) 11. Which of the following relation is true
R R
(A) Ruling gradient < limiting gradients < exceptional
75 40 + R gradient
(C) (D)
R R (B) limiting gradient > ruling gradient > exceptional
3. The length of the summit curve for a stopping sight gradient
distance of 100 m with an upward gradient of 1% and (C) Exceptional gradient < limiting gradient < Ruling
downward gradient of 2% is ______. gradient
Take the height of the eye of the driver to be 1.2 m (D) Ruling gradient > limiting gradients > exceptional
and the height of the object above the roadways to be gradient
0.15 m. 12. Calculate the safe stopping sight distance on a level
(A) 62 m (B) 65 m road, stretch for design speed of 40 kmph for a two
(C) 73 m (D) 69 m way traffic on a single lane road assuming coefficient
4. The extra widening necessary on a two lane pavement of friction as 0.37 and reaction time of driver as 2.5
for a curve of radius 80 m and wheel base 6 m with a seconds.
design speed of 65 kmph is ______. (A) 44.79 m (B) 45.68 m
(A) 1.23 m (B) 1.43 m (C) 89.59 m (D) 88.63 m
(C) 1.17 m (D) 1.13 m Common Data for Questions 13 and 14:
5. Width of roadway of two lane national highway in The radius of horizontal circular curve is 400 m. The design
mountainous and steep terrain is speed is 80 kmph and coefficient of lateral friction is 0.1.
(A) 6.25 m (B) 8.8 m 13. Calculate the super elevation required (if the lateral
(C) 4.75 m (D) 9 m friction is assumed to develop).
6. An aggregate is said to be exceptionally strong when (A) 0.58 (B) 0.025
the aggregate impact value is (C) 0.007 (D) 0.48
(A) less than 10. (B) 10 to 20. 14. Coefficient of friction needed if no super elevation is
(C) 20 to 30. (D) above 35. provided is ______.
7. Bitumen of viscosity 80/100 penetration grade is used (A) 0.15 (B) 0.19
for (C) 0.125 (D) 0.087
(A) spraying applications Common Data for Questions 15 and 16:
(B) paving applications in cold regions A vertical summit curve is to be designed when two grades
(C) paving applications in hot regions 1 1
+ and – meet on the highway. The stopping sight
(D) high stress areas like toll plaza, terminals etc. 20 50
distance and over taking sight distance required are 120 m
8. Length of the transition curve for comfort condition
and 600 m respectively. The length of vertical the curve is
is ______.
L restricted to a maximum value of 500 m.
V3
(A) LS = (B) LS = 15. The length of the summit curve needed to fulfill the
CR 2
requirements of stopping sight distance is ______.
NV 3 (A) 228 m (B) 229 m
(C) LS = (D) All of the above
C (C) 230 m (D) 222 m
9. The minimum length of overtaking zone should be 16. The length of summit curve needed to fulfill the
(d1 + d2 + d3 = OSD) requirements of overtaking sight distance is ______.
(A) 2(d1 + d2 + d3) (B) 5(d1 + d2 + d3) (A) 1052.7 m (B) 1048.9 m
(C) 3(d1 + d2 + d3) (D) 4(d1 + d2 + d3) (C) 1062.8 m (D) 1073.9 m
3.168 | Transportation Infrastructure Test 1

Common Data for Questions 17 and 18: (A) (i) is true and (ii) is false
The wheel load of 5100 kg is applied on a pavement of (B) (i) is false and (ii) is true
thickness 18 cm, modulus of elasticity 3 × 105 kg/cm2, radi- (C) Both (i) and (ii) are true
us of contact area 15 cm and Poisson’s ratio = 0.15. modulus (D) Both (i) and (ii) are false
of subgrade reaction = 6kg/cm3. 21. The number of commercial vehicles in the year of com-
17. The stress at the corner of the slab is __. pleting overlay construction is estimated to be 2850 per
(A) 24.27 kg/cm2 day with an average growth rate of 6.5 percent per year,
(B) 22.56 kg/cm2 if the average VDF value is 5.4 and the traffic design
(C) 25.96 kg/cm2 period is 10 years. The design traffic volume if the
(D) 26.5 kg/cm2 LDF = 0.75 is ______.
18. The location where a crack develops due to corner load (A) 110 msa (B) 98 msa
is ______. (C) 83 msa (D) 57 msa
(A) 85 cm (B) 84.5 cm 22. The following data are related to a horizontal curved
(C) 84 cm (D) 84.3 cm portion of a two lane highway: length of curve = 300 m,
19. According to the bituminous mix design by Marshall radius of curve = 600 m, width of pavement = 9.5 m. In
method, match the following correctly (Binder content order to provide an SSD of 80 m the set back distance
% on x – axis) of the inner lane of the pavement is
(A) 3.71 m
A B (B) 2.35 m
(i) Marshall stability 1. (C) 4.62 m
Vs (D) 2.97 m
Binder content% 23. The speeds of overtaking and overtaken vehicles
on the highways are 70 kmph and 65 kmph respec-
tively. Assuming acceleration of overtaking vehicle as
(ii) Flow value 2.
2.5 kmph/sec and speed of vehicle in opposite direc-
Vs
Binder content% tion as 65 kmph, the overtaking sight distance needed
for two way traffic is (assume reaction time of driver
= 2sec) ______.
(iii) Voids% 3. (A) 451 m (B) 326 m
Vs (C) 564 m (D) 275 m.
Binder content%
24. In a concrete pavement
(i) Temperature stress is tensile at bottom during day
time
(iv) Voids filled with Bitumen 4.
Vs
(ii) Load stress is compressive at the bottom.
Binder content % Identify the correct choice from the following.
(A) Both the statement (A) and (B) are correct
(B) Statement (A) is correct and (B) is incorrect.
(C) Statement (B) is incorrect.and (A) is orrect
(A) (i – 2), (ii – 3), (iii – 4), (iv – 1) (D) Both statements (A) and (B) are incorrect
(B) (i – 4), (ii – 2), (iii – 1), (iv – 3)
25. A valley curve is formed by a descending grade of 1 in
(C) (i – 3), (ii – 4), (iii – 1), (iv – 2)
35 meeting an ascending grade of 1 in 30. Assuming
(D) (i – 2), (ii – 1), (iii – 4), (iv – 3)
allowable rate of change of centrifugal acceleration is
20. (i) Viscosity test on bitumen is done to assess the 0.6 m/sec3, design speed is 50 kmph, then the length of
ability to be sprayed through jets. the valley curve to fulfill comfort condition is ______.
(ii) Coating test on bitumen is done to assess the sta- (A) 128 m (B) 33 m
bility in presence of fines in aggregates. (C) 73 m (D) 94 m

Answer Keys
1. C 2. C 3. D 4. C 5. B 6. A 7. A 8. D 9. C 10. A
11. A 12. C 13. B 14. C 15. B 16. C 17. A 18. C 19. D 20. A
21. D 22. A 23. A 24. A 25. B
Transportation Infrastructure Test 1 | 3.169

Hints and Explanations

NS 2 (0.01 + 0.02) × 100 × 100 3P   a 2   3 × 5100   15 2  


0.6 0.6
3. L =  = = 69 m. SC =  2 × 1 −  = 1 − 
4.4 4.4 h      182   70.6  
 Choice (D)    
n ×  2 0.1 × V   = 24.27 kg/cm .
2
Choice (A)
4. We =  + .
2R R 18. X = 2.58 aL = 2.58  (15 × 70.6)
Where   =83.96 @ 84 cm. Choice (C)
V = design speed of road in kmph 20. Coating test is assess coating of stone aggregates.
n = no of lanes  Choice (A)
R = Radius of the curve 2 1. N = 2650
2 × (6)2 0.1 × 65 r = 6.5%
We = + = 1.17 m Choice (C) VDF = 5.4
2 × 80 80
LDF = 0.75
V3 L NV 3 n = 10 years
8. LS = = = . Choice (D)
CR 2 C  [1 + r )n − 1] 
Design traffic = 365  N × VDF × LDF × 
v2  r 
12. Stopping sight distance (SSD) = vt + ,
2 gf 365 × 2850 × 5.4 × 0.75 × 1.06510 − 1 
5 =
where V = 40 × = 11.11 m/s 0.065
18  = 56.8 × 106 @ 57 msa. Choice (D)
(11.11)2  ∝
SSD = (11.11 × 2.5) + 22. Set back distance = R – (R – d) cos  
(2 × 9.8 × 0.37)  2
⇒ SSD = 44.79 m. 9.5 SSD × 180 80 × 180
SSD when there is a single lane 2 way = 2 SSD d=   ∝=  =  = 7.64°
4 Π × ( R − d ) π ×  600 − 9.5 
= 2 × 44.79 = 89.59 m. Choice (C)  
4
V2 V2 80 2
13. e + f = = ⇒ e + 0.1 =  9.5   7.66° 
gR 127 R 127 × 400 \ Set back distance = 600 –  600 −   × cos 
 4   2 
⇒ 0.025. Choice (B) ⇒ set back distance = 3.71 m. Choice (A)
14. If no super elevation is provided e = 0 2 3. Vb = 65 kmph
V2 a = 2.5 kmph/sec = 0.694 m/sec2
⇒ e + f = 5
127 R 
S = 0.7 Vb + 6 = 0.7 ×  65 ×   + 6 = 18.64 m
 18 
80 2
f = = 0.125.  [∵ e = 0] Choice (C)
27 × 400 4S 4 × 18.6
T=  = 
15. Let L > SSD, a 0.694
+1  −1 7   = 10.36 sec (reaction time of driver t = 2 sec.)
Deviation angle, N = −    =
20  50  100 OSD = 0.28Vb + 0.28VbT +2 s + 0.28 VT.
NS 2 7 × (120 )
2 = (0.28 × 65 × 2) + (0.28 × 65 × 10.36)
L= = = 229 m . Choice (B) + (2 × 18.64) + (0.28 × 65 × 10.36)
4.4 4.4 × 100
= 450.78 m ≃ 451 m. Choice (A)
16. Length of the curve restricted to 500 m, 1
L < OSD (500 < 600)  Nv 3  2 5
9.6 9.6 × 100 25. Ls = 2   v = 50 × = 13.88
 C  18
L = 2S – = 2 × (600) –  = 1062.8 m.
N 7
 Choice (C) 1 1 −13
N =  −  = 
17. Radius of relative stiffness −35 30 210
1 1 1
 Eh3  4  3 × 105 × 153  4  13 13.883  2
= =  = 70.6 cm.   Ls = 2  ×  = 33.2 m. Choice (B)
2 
12k (1 − m ) 
2 
12 × 6 × (1 − 0.15 )   210 0.6 
Traffic Engineering Test 2
Number of Questions: 25 Time: 60 min.

Directions for questions 1 to 25: Select the correct alterna-


tive from the given choices.
1. The vehicle speed affects the design of Speed
(A) sight distance (B)
(B) super elevation
(C) length of transition curve
(D) All of the above
Volume, q
2. The wheel base of the vehicle is 8.5 m. The off-
tracking while negotiating curved path with mean
radius of 32 m is
(A) 1.18 m (B) 1.13 m
(C) Speed (v)
(C) 1.12 m (D) 1.15 m
3. The speed at which (or) below which 85 percent
of vehicles are passing the point on highway can be Volume, q
assessed is known as
(A) 85th percentile speed
(B) 20th percentile speed Speed (v)
(C) 15th percentile speed (D)
(D) 25th percentile speed
4. The method of origin and destination studies in which Volume, q
the car is struck with a pre-coded card as it enters the
area under study is 8. The free mean speed on a roadway is found to be
(A) Road side interview method 100 kmph under stopped condition the average spacing
(B) Home interview method between vehicles is 8.9m. The Jam density of flow is
(C) License plate method (A) 113 vehicles/km
(D) Tag on car method (B) 118 vehicles/km
(C) 145 vehicles/km
5. A vehicle a of weight 2 tonne skids through a distance (D) 148 vehicles/km
equal to 50 m before colliding with other parked vehi-
9. The maximum number of vehicles that can pass a given
cle of weight 1 tonne. After collision both the vehicles
point on a lane or roadway during one hour under pre-
together skid through a distance of 10 m before stop-
vailing traffic conditions is known as
ping. The initial speed of moving vehicle is (Assume
(A) Basic capacity
coefficient of friction f = 0.5).
(B) Practical capacity
(A) 9.89 m/s (B) 8.93 m/s
(C) Possible capacity
(C) 7.96 m/s (D) 9.82 m/s
(D) Highway capacity
6. When a vehicle moves obliquely across the path of 10.
another vehicle moving in same direction at small
angle of crossing is termed as
(A) Merging (B) Weaving manoeuvre
(C) Crossing manoeuvre (D) Diverging
7. Which of the following graph represents relation
between speed and volume?
The above sign is categorized under
(A) Warning signs
(B) Regulatory signs
(C) Informatory signs
(A) Speed (v)
(D) None of these
1 1. The theoretical capacity of traffic lane with one way
Volume, q traffic flow at a stream speed of 80 kmph. The average
Traffic Engineering Test 2 | 3.171

space gap Sg = 0.278 Vt and average length of vehicles 4.  Full clover leaf d.
= 8m
(A) 3450 veh/hr/lane (B) 3395 veh/hr/lane
(C) 3530 veh/hr/lane (D) 3834 veh/hr/lane
12. The average normal flow of traffic on cross roads A and
B during design period are 400 and 250 PCU per hour,
the saturation flow values on these roads is estimated to
be 1350 and 1200 PCU per hour respectively. The all-
red time required for pedestrian crossing is 15 sec. The 1 2 3 4 1 2 3 4
total cycle time using Webster’s method is (A) a b c d (B) a d c b
(A) 62.4 sec (B) 63.3 sec (C) d c b a (D) c b d a
(C) 65.53 sec (D) 67.5 sec 1 6. The no parking symbol is represented by
(A) triangle (B) circle
13. (i) At intersection the area of conflict should be as
(C) octagonal (D) hexagonal
small as possible
(ii) Sudden change of path should be avoided 17. The practical capacity of a rotary is given by the
(A) (i) and (ii) are true (B) (ii) and (iii) are false formula
(C) (i) is true (ii) is false (D) (i) is false (ii) is true  p
280W (1 + W ) 1 − 
 3
14. In a street light system (A) QP = 
Street width = 20m  W
1 + 
Mounting height = 10m L
Lamp size = 5000 lumen  w  P 
Luminaire type – II 280 w 1 +  1 − 
 L  3
Coefficient of utilization = 0.36 (B) QP  =
 e 
Spacing between lighting units if average lighting 1 + 
w
intensity is 6 Lux
(assume maintenance factor = 0.8)  e  w 
280 w 1 +  1 + 
(A) 16 m (B) 18 m  w  L 
(C) 15 m (D) 12 m (C) QP = 
 P
1 − 
15. Match the following 3
(D) None of the above
1. Diamond a.
18. The average width of entry e1 is 150 m and average
width of exit is 200 m. The width of the rotary roadway
is
(A) 182 m (B) 178 m
(C) 176 m (D) 179 m
1 9. Match the following
Type of Marking Areas
2. Rotary b.
1. Markings at a. slow & stop
intersections
2. Carriage way b. speed change lanes and
marking stop lines
3. object markings c. no parking zones and
traffic lanes
4. word messages d. kerb markings and objects
within the carriage way
3.  Partial clover leaf c.
Codes:
1 2 3 4
(A) c a b d
(B) d c b a
(C) b c d a
(D) a d c b
3.172 | Traffic Engineering Test 2

20. Desired lines are used in 3. Crossing right c.


(A) speed and delay studies
(B) origin and destination study
(C) spot speed study 4. merging left d.
(D) None of these
21. The spacing between the vehicles allowed by the driver
1 2 3 4 1 2 3 4
of the following vehicle does not depends on
(A) b a c d (B) c a b d
(A) speed of leading vehicle
(C) d c a b (D) c d b a
(B) Average length and width of vehicle class
(C) tyre and characteristics of two vehicles 24. The charts showing the volume variations over a period
(D) driver characteristics of following vehicle of years are known as
22. The speed of road is 100 kmph and average center to (A) Trend charts
center spacing of vehicles (or) space headway is 10m. (B) Variation charts
The capacity of single lane is (C) Traffic flow maps
(A) 103 veh/hr/lane (B) 105 veh/hr/lane (D) Volume flow diagram
(C) 10 veh/hr/lane
4
(D) 102 veh/hr/lane 25. A vehicle moving at 40 kmph speed was stopped by
23. Match the following applying brakes and length of skid mark is 13.2 m.
Average skid resistance of pavement is 0.5. The brake
Traffic Man oeuvres Figures
efficiency of test vehicle is
1. diverging left a.
(A) 64.6% (B) 73.7%
(C) 86.9% (D) 95.4%
2. diverging right b.

Answer Keys
1. D 2. C 3. A 4. D 5. A 6. B 7. C 8. A 9. C 10. B
11. B 12. D 13. A 14. D 15. B 16. B 17. A 18. D 19. C 20. B
21. B 22. C 23. B 24. A 25. D

Hints and Explanations


7. Choice (C)
1. Choice (D)
8. Spacing between vehicles s = 8.9 m
L2 8.52
2. Off tracking = = = 1.12m Choice (C) 1000 1000
2 R 2 × 32 Jam density = = = 112.3
s 8.9
3. Choice (A) KMax ≃ 113 vehicles/km (per lane) Choice (A)
4. Choice (D)
9. Choice (C)
5. Loss of kinetic energy of both vehicles together = work
done against frictional force 10. It is a give way sign categorized as Regulatory sign
 Choice (B)
 wa + wb  2
 2 g  (V2 − V1 ) = (wa + wb)f S2
2

11. Reaction time of ideal driver t = 0.7 sec


V −V
2 2
S = 0.278 Vt + L
= 2
= fS2
1
      
2g   = (0.278 × 80 × 0.7) + 8
But V4 = 0   = 23.568m
1000V 1000 × 80
V32 ∴ Theoretical capacity =  =
= 0.5 × 10 S 23.568
2g
= 3394.4
V3 = 0.5 × 10 × 2 × 9.8 = 9.89 m/s Choice (A)
≅ 3395 veh/hr/lane
6. Choice (B)  Choice (B)
Traffic Engineering Test 2 | 3.173

qa 400 15. Choice (B)


12. ya = = = 0.296
S a 1350 16. Choice (B)
17. Choice (A)
qb 250
yb = = = 0.208 18. Average width of rotary is
Sb 1200
e + e 
Y = ya + yb = 0.296 + 0.208 = 0.504 =  1 2  + 3.5
 2 
Lost time = 2n + R = (2 × 2) + 15 = 19 sec
150 + 200
1.5 L + 5 = + 3.5 = 178.5m ≅ 179 m. Choice (D)
Optimum cycle time Co = 2
1− Y
19. Choice (C)
  =
(1.5 ×19) + 5 = 67.54 sec 20. Choice (B)
1 − 0.504 21. Choice (B)
ya 1000V
Ga = (Co – L) 22. Capacity of single lane is =
Y S
0.296 1000 × 100
  = (67.54 – 19) = 28.5 sec = (V in ‘kmph’ S in ‘m’)
0.504 10
yb 0.208  = 104veh/hr/lane Choice (C)
Gb = (Co – L) = (67.54 – 19) = 20 sec 23. Choice (B)
Y 0.504
Total cycle time = 28.5 + 20 + (15 + 2 + 2) 2 4. Choice (A)
= 67.5 sec Choice (D) 100 f 1
2 5. Brake efficiency =
1 3. Choice (A) f
v2
Average skid resistance developed, f 1 =
14. Spacing 2 gL
v 40
lamp lumem × coefficient of utilization V= = = 11.11 m/s
3.6 3.6
× Maintenance factor
 = 11.112
Average lux × width of road f 1 = = 0.477.
2 × 9.81 × 13.2
5000 × 0.36 × 0.8 100 × 0.47
= = 12m Choice (D) Brake efficiency = = 95.4% Choice (D)
6 × 20 0.5
Transportation Engineering Test 3
Number of Questions: 30 Time: 75 min.

Directions for questions 1 to 30: Select the correct alterna- 9. The mix design of concrete pavement is based on
tive from the given choices. (A) Flexural strength
1. The factors affecting the highway alignment are (B) Compressive strength
(A) Traffic (B) Geometric design (C) Shear strength
(C) Economy (D) All the above. (D) Bond strength
2. A test car of mass 1400 N is travelling at a speed of 10. The speed at which greatest number of vehicles travel
85 kmph, when it is suddenly braked the wheels. The is called
average vehicles comes to a stop in a distance of 50 m. (A) Medium speed (B) Model speed
Skid resisting force is (C) 15th percentile speed (D) 98th percentile speed
(A) 7862 N (B) 7928 N 11. A circular curve of radius 300 m, coefficient of lateral
(C) 7804.7 N (D) 7642 N friction of 0.15 and the design speed is 40 kmph. The
3. The height and width of the pavement are as given super elevation at which equal pressure is distributed
below figure on inner and outer wheel would be
(A) 0.02 (B) 0.06
(C) 0.05 (D) 0.04
5m
12. What will be the non passing sight distance on a high-
way for a design speed of 100 kmph when its ascend-
ing gradient is 2%. Assuming coefficient of friction as
0.7 and brake efficiency is 50%
25 m (A) 176.2 m (B) 174.5 m
(C) 172.3 m (D) 175.05 m
If f = 0.15 and ruling design speed is 60 kmph. Find the
13. A summit curve is formed at the intersection of 3%
absolute minimum radius on the curve in ‘m’ is _____.
upgrade and 5% downgrade. What is the length of the
(A) 8.94 m (B) 9.34 m
summit curve in order to provide a stopping distance of
(C) 6.62 m (D) 7.34 m
128 m is
4. The turning angle of the curve is 30° and tractive force (A) 223 m (B) 248 m
on the vehicle is 300 N. Then the loss of tractive force (C) 298 m (D) 300 m
due to turning of vehicle in horizontal curve is
14. Match List - I (traffic survey) with List - II and select
(A) 38 N (B) 40 N
the correct answer using the codes given below
(C) 41 N (D) None of these
5. If width of the vehicle is 6 m and height of the vehicle List – I List – II
is 10 m and coefficient of friction 0.15 then a. Spot speed 1. By video tape
(A) Vehicle overturns prior to skidding b. Traffic volume 2. By road side interview
(B) Vehicle skids prior to overtaking c. O-D survey 3. By Doppler radar
(C) Overturning is avoided
d. Parking survey 4. By pneumatic tube
(D) Skid is avoided.
6. The design speed of a road is 40 kmph and the radius of a b c d a b c d
curve is 200 m. Then find the length of transition curve (A) 3 1 2 4 (B) 2 4 3 1
for the road of plain and rolling terrain. (C) 3 4 2 1 (D) 4 2 1 3
(A) 21.6 m (B) 26.1 m 15. What will be the initial traffic after construction, in the
(C) 16.2 m (D) 24.2 m Commercial Vehicles per day (CVpd) for following
7. A summit curve is to be designed with two gradients data? Rate of traffic growth per annum = 7%. The road
+2% and –6%. The rate of change of gradient is 1% per is proposed to be completed in 3 years and present traf-
100 m length. The minimum radius of curve is. fic existing is 400 CVpd
(A) 100 m (B) 1000 m (A) 50 (B) 449
(C) 200 m (D) 300 m (C) 490 (D) 421
8. The material obtained by the destructive distillation of 1 6. What is the deflection at the surface of a flexible pave-
wood is _________. ment due to a wheel load of 40 kN and a tyre pressure
(A) Bitumen (B) Cutback of 0.5 MPa? The value of Є for pavement and sub grade
(C) Emulsion (D) Tar is 20 MPa.
Transportation Engineering Test 3 | 3.175

(A) 15 mm (B) 11 mm (A) Lateral skid occurs first


(C) 9 mm (D) 6 mm (B) Overturning occurs first
17. Which one of the following is the set of physical (C) Neither lateral skid nor overturning
requirements of coarse aggregates for construction of (D) Both simultaneously
WBM roads as per IRC recommendation? 25. Find minimum sight distance to avoid head on colli-
LAV(%) AIV(%) FI(%) sion of two cars approaching at 90 kmph and 60 kmph.
(A)   < 50 < 40 < 15 Used reaction time of driver t = 2.5 sec, coefficient of
(B)   < 50 < 30 < 15 longitude friction, f = 0.7 and brake efficiency of 50%
(C)   < 40 < 30 < 20 in either case is.
(D)   < 40 < 30 < 15 (A) 235.8 m
1 8. In 500 gm sample of course aggregate are 100 gm of (B) 243.2 m
flaky particles and 80 gm elongated particles. What are (C) 256.8 m
the flakiness and elongation particles as per Is. (D) 292.3 m
(A) 40% (B) 3.6% 2 6. The last time due to starting delay on a traffic signal
(C) 18% (D) 4% approach is noted to be 3 seconds, the actual green time
19. The design speed of a traffic lane is 70 kmph. What is is 20 seconds and amber time is 3 seconds. How much
the theoretical capacity per hour taking the total reac- is the effective green time.
tion time to be 2 seconds and average length of vehicles (A) 19 sec (B) 22 sec
as 8 m. (C) 27 sec (D) 31 sec
(A) 828 veh/m/day (B) 735 veh/m/day 27. The free mean speed on a road wing is found to be
(C) 628 veh/m/day (D) 428 veh/m/day 60kmph under stopped condition the average spacing
20. Consider following factors between vehicle is 6 m. The capacity of flow, assuming
1. Length of the vehicle linear speed density relation is
2. Width of the vehicle (A) 2333 veh/hr
3. Approach speed (B) 3333 veh/hr
4. Stopping time for approaching vehicle (C) 2870 veh/hr
5. Passing sight distance (D) 3838 veh/hr
Which of these factors are taken into consideration for 28. A road 10 m wide is to deflect through and angle of 65°
determing yellow time of traffic signal at intersection? with the centreline radius 350 m. A transition curve is
(A) 1, 2 and 5 (B) 2, 3 and 4 to be used at each end of a circular curve of such a
(C) 1, 3 and 5 (D) 1, 3 and 4 length that the rate of gain of radial acceleration
0.4 m 2 / sec , when speed is 60 kmph. Find the shift of
21. On a road the free speed was 65 kmph and the space
the transition curve.
headway at jam density was 6.25 m. What is the maxi-
(A) 0.13 m (B) 3.12 m
mum flow which could be expected on this road?
(C) 0.18 m (D) 3.42 m
(A) 2600 vph (B) 1625 vph
(C) 1300 vph (D) 406 vph 29. If the lamp lumen is 30 lux coefficient of utilization is
0.3, maintenance factor is 0.25, average lux on road is
22. In marshall method of mix design, the course aggre-
15 and width of road is 7.5
gates, fine aggregates, filler material and bitumen,
(A) 2 m (B) 3 m
having respective specific gravities of 2.62, 2.72, 2.70
(C) 2.5 m (D) 3.5 m
and 1.02 are mixed in the ratio of 55, 34.6, 4.8 and 5.6
percent respectively. The theoretical specific gravity of 30. Consider the following statements with reference to
mix would be. pavements
(A) 2.36 (B) 2.4 (1) Flexible pavement are more suitable an rigid
(C) 2.44 (D) 2.5 pavements in regions. Where sub grade strength is
uneven
23. Compute the equalent radius of resisting section of
(2) Load carrying capacity of rigid pavements
15 cm slab, if ratio of radius of wheel load distribution
depends more on properties of concrete than
to thickness of slab is 0.5
strength of sub grade
(A) 7.82 cm (B) 7.93 cm
(3) Compared to flexible pavements, rigid pavements
(C) 7.48 cm (D) 7.62 cm
are more affected by temperature variations
24. The centrifugal ratio of a vehicle is 0.25, width of vehi- Which of these statements are correct.
cle is 2.4 m, height of vehicle to its C.G is 4.2 m, lateral (A) 1 and 2 (B) 1 and 3
friction is 0.15, assuming no superlevation. (C) 2 and 3 (D) 3 alone
3.176 | Transportation Engineering Test 3

Answer Keys
1. D 2. C 3. A 4. B 5. B 6. A 7. A 8. D 9. A 10. B
11. B 12. D 13. C 14. B 15. C 16. D 17. D 18. A 19. B 20. D
21. A 22. D 23. D 24. A 25. A 26. B 27. B 28. A 29. A 30. C

Hints and Explanations


1 Total change of grade 10 × 100
2. mv 2 = (wf) × d L= = = 1000
2 rate of change of grade 1
1000
        R= = 100 m  Choice (A)
10
Skid resistance force
1 mv 2 V2
.·. skid resistance force = 11. e + f =
2 d gR
2
(1400) ×  85 × 5  For equilibrium super elevation (equal pressure on two
1  18  wheels) f = 0

2 50 V2
e=
=
7804.7 kg Choice (C) gR
2
Vmin 5
3. e + f = V = 40 kmph = 40 × = 11.11 m/s
gRmin 18
E 1 (11.11)2
e= = = 0.04 \ e = = 0.06 Choice (B)
B 5 9.8 × 200
5
2 v2
 12. Non passing sight distance = vt +
 60 ×  2 g ( f + n)
18
0.04 + 0.15 = 5
9.8 × Rmin
V = 100 kmph = 100 × = 27.7 m/s
2 18
 5
 60 ×  Non passing sight distance
18
Rmin =
9.8 ( 0.04 + 0.15) ( 27.7)2
=
(27.7 × 2.5) +
16.66 2 × 9.8 ( 0.7 × 0.5 + 0.02)
     = = 8.94 m Choice (A)
9.8 × 0.19 = 69.25 + 105.8 = 175.05 m Choice (D)
13. Deviation angle
4. Loss of tractive force T (1 − cos θ)
N = 3– (–5) = 8% =0.8
= 300(1-cos 30°) = 40.1 N Choice (B) Assuming L > S
5. b = 6 m
NS 2 0.08 × (128)
2
h = 10 m L= = = 298 m Choice (C)
f = 0.15 4.4 4.4
b 6  r 
= = 0.3 15. A = P 1 + n
2h 2 × 10  100 
b P = 400 commercial veh/day
F = 0.15 <
2h r = rate of traffic growth = 7%
.·. Vehicle skids prior to overtaking Choice (B) n = 3 years completion time
6. V = 40 kmph  7 
3

A = 400 1 + = 490 cvd Choice (C)


R = 200 m  100 
2.7 × 402
16. Deflection,
L= = 21.6 m Choice (A)
200
1.5pa
∆= (for flexible plate)
7. R = L/N ES
N = g1 − g 2 = 2–(–6) Where,
10 % p = contact pressure due to wheel load = 0.5MPa
Transportation Engineering Test 3 | 3.177

a = radius of contact area 22. Theoretical specific gravity


Wheel load 40 × 103 N 100 100
Contact area = = Gt = =
tyre pressure 0.5 N/mm 2 w1 w2 w3 w4 55 34.6 4.8 5.6
+ + + + + +
G1 G2 G3 G4 2.02 2.72 2.7 1.02
= 80 × 103 mm 2
And area      = 2.49 ≃ 2.5 Choice (D)
πa2 = 80 × 103 2 3. h = 15 cm, a/h = 0.5
a = 0.5 ×15 = 7.5 cm
80 × 103
A= = 159.615 mm b= (1.6a2 + h2 ) − 0.675h
π
Es = 250 MPa   = (1.6 (7.5)2 + 152 ) − 0.675 (15)
1.5 × 0.5 × 159.617   = 7.62 cm Choice (D)
So, ∆ = = 6 mm  Choice (D)
20 24. P/w = 0.25
b = 2.4 m
17. For WBM the prescribed value of LAV, AIV and FI for h = 4.2 m
Base course are < 50%, < 40%, and < 15% respec- f = 0.15
tively and Surface course are < 40%, < 30% and < 15%
respectively P b v2
= =
Where, W 2h gR
LAV is los-angles abrasion value b P
AIV is aggregate Impact value To avoid overturn >
2h w
FI is flakiness index. Choice (D) 2.4
100 ⇒ > 0.25
1 8. Flakiness index = × 100 = 20% 2 × ( 4.2)
500
0.285 > 0.25
80 P
Elongated index = × 100 = 20% To avoid lateral skid f >
400 w
Total = 20 + 20 = 40% Choice (A) ⇒ 0.5 > 0.25
b
19. Theoretical capacity F < ⇒ 0.15 < 0.285
2h
1000V
C= ⇒ vehicle skids prior to over turning
S
b
( 0.278v )2 < f ⇒ 0.285 < 0.15 C
S = SSD = L = 0.278 vt + +L 2h
2 fg
⇒ vehicle overturns prior to skidding Choice (A)
( 0.278 × 70) 2
25. v1 = 90 km/ hr
  = (0.278× 70× 2) + + 8 = 95.1736 m
2 × 0.4 × 9.81
v2 = 60 km/hr
1000v 1000 × 70 t = 2.5sec
Capacity = =
S 95.136 Brake efficiency = 50%
= 735.5 veh/m/day. Choice (B) f = 0.7
SSD1 for first car is = 153.6 m
1000 SSD2 for second car is = 82.2 m
21. Jam density =
space head way (H) SSD = SSD1 + SSD2 = 235.8 m Choice (A)
1000 26. Effective green time
= = 160 veh/ km
6.25 Ge = G + A − L
jam density × free speed G = 25 sec
Maximum flow =
4 A = 3 sec
L = intinal lost time ( Li ) + final lost time ( L f )
160 × 65
= = 2600 vph Choice (A) Ge = 25 + 3 − (3 + 3) = 22 sec Choice (B)
4
3.178 | Transportation Engineering Test 3

1000 v 3 16.673
27. Jam density= = 166.67Veh/km LS = = = 33.07 m
6 cR 2
Maximum flow (or) capacity L2 (33.07)2
S= = = 0.13 m  Choice (A)
 80   166.67  24 R ( 24 × 350)
=   = 3333.33 veh / hr Choice (B)
 2   2  100 × 0.3 × 0.25
29. Spacing of lamps = =2m
28. R = 350 m 15 × 7.5
V = 60 kmph = 16.67 m/s  Choice (A)
∝ = 0.4 m3, /sec 3 0. 2 and 3 Choice (C)
Highway Engineering Test 4
Number of Questions: 25 Time: 60 min.

Directions for questions 1 to 25: Select the correct alterna- 8. Match the following
tive from the given choices. Types of Test Property
1. The mean weight of aggregates in the cylinder is 100 a. Impact 1. Resistance to weather
and weight of water required to fill the cylinder is 60 g
b. Soundness 2. Hardness
and specific gravity of aggregate is 2.6. Then the angu-
c. Crushing 3. Toughness
larity number is ________.
d. Abrasion 4. Strength
2. The grade of tar which is generally used for surface
pointing and renewal coats of road is Codes:
(A) RT – 1 (B) RT – 2 a b c d abcd
(C) RT – 3 (D) RT – 4 (A) 4 3 2 1 (B) 2 1 4 3
3. The lowest point temperature at which a material gets (C) 3 4 1 2 (D) 3 1 4 2
ignited and burns under specified conditions of test 9. The design traffic of a flexible pavement is based on
(A) Fire point 7 day 24 hours traffic count as per
(B) Flash point (A) IRC : 39.2001 (B) IRC : 58 – 2002
(C) Triple point (C) IRC : 9 (D) IRC : 7
(D) None of these 10. The parameters required to get the value of radius of
4. The elastic modulus of granular layer is 5.5 MPa and relative stiffness of CC pavement is (conventions are as
thickness of granular layer is 3mm. The compos- per IRC)
ite elastic modulus of granular sub base and base in (A) E, k, r, u (B) E, h, k, u
MPa is. (C) h, k, u, r (D) P, h, k, u
(A) 1.4 (B) 1.8 11. A bitumen concrete mix has average specific gravity of
(C) 2.4 (D) 1.3 2.325 and theoretical specific gravity as 2.41. The den-
5. If the wheel loads stress at edge is 40kg /cm2, warping sity of bitumen is 1.03 g/cc with 4.5% bitumen content
stresses at edge is 9kg/cm2 and frictional stress is 5kg/ by weight in mix. The VFB of mix is
cm2 and then the critical stress in edge region for sum- (A) 32.8% (B) 30.8%
mer mid day is. (C) 20.73% (D) 10.15%
(A) 48 kg/cm2 12. A subgrade soil sample was tested using standard CBR
(B) 42 kg/cm2 apparatus and the observations are given below:
(C) 44 kg/cm2
Load, kg Penetration, mm
(D) 40 kg/cm2
40.8 2.5
6.
60.5 5.0

The CBR value of the sample is


(A) 3.9% (B) 2.9%
V
(C) 4.8% (D) 12.2%
13. The width of expansion joint is 30 mm in a cement
x = % bitumen concrete pavement. The laying temperature is 10°C
and the maximum slab temperature in summer is 50°C.
In the above graph v = ? The coefficient of termal expansion of concrete is
(A) marshal stability 10 × 10–6/°c and the joint filler compresses up to 50%
(B) unit weight of the thickness. The spacing between expansion joints
(C) flow value should be ____ (in m).
(D) percentage voids of total mix 1 4. What is the deflection at the surface of flexible pave-
7. In a dual wheel assembly if (p) is equal to each wheel ment due a wheel load of 30 kN and a tyre pressure of
load, ‘s’ is centre to centre spacing of dual wheels and 0.6 Mpa. Take E = 20 MPa
‘d’ is the clear distance between wheels, then the equiv- (A) 5.8mm (B) 5.6mm
alent single wheel load for depth between d/2 and 2s is (C) 5.4mm (D) 5.2mm
(A) p (B) 2p 1 5. A plain cement concrete pavement is constructed at
(C) between P and 2p (D) None temperature of 20°c. The peak summer temperature is
3.180 | Highway Engineering Test 4

40°c. The coefficient of thermal expansion of concrete (C) > 1.0 for tyre pressure less than 7 kg /cm2
is 8 × 10-6 per °c, the gap at expansion joint is 3.5 cm. (D) all the above are correct
The spacing of expansion joint is _______ (in m). 21. A combined value of flakiness and elongation index
16. In marshall method of mix design, the coarse aggre- are to be determined for a sample of aggregates. The
gates, fine aggregates, filler material and bitumen, hav- sequence in which two tests are conducted is
ing respective specific gravities of 2.82, 2.42, 2.68, and (A) elongation index test followed by flakiness index
1.02 are mixed in ratio of 52, 34.8, 4.2 and 5.4 percent test
respectively. The theoretical specific gravity of the mix (B) flakiness index test followed by elongation index
would be ___ test
1 7. In case of governing equations for calculating wheel (C) flakiness index test followed by elongation index
loads stress using westergaard’s approach, the follow- on non flaxy aggregates
ing statements are made. (D) elongation index test followed by flakiness index
(I) load stress are inversely proportional to wheel load. test on non elongated aggregates
(II) Modulus of sub grade reaction is useful for load 2 2. Compute the equivalent radius of resisting section of
stress calculation. 20 cm slab, if ratio of radius of wheel load distribution
(A) Both statements are true to thickness of slab is 0.6.
(B) I is true and II is false (A) 14.22 cm (B) 14.02 cm
(C) Both statements are false (C) 14.8 cm (D) 14.7 cm
(D) I is false and II is true 2 3. Match the following
1 8. Match the following Type of test Purpose
Type of test property a. Penetration test 1. Design of bitumen
Types of cut backs Materials concrete mix

1. Rapid curing (RC) a. Bitumen fluxed with b. Marshal test 2. Overlay design
kerosene c. Ring and ball test 3. Gradation of Asphalts
2. Medium curing (MC) b. Bitumen blended with d. Benkelman beam test 4. Determination of
high boiling point gas oil softening point
3. Slow curing (BC) c. Bitumen fluxed with Codes:
naphtha (or) gasoline
a b c d abcd
Codes: (A) 3 2 4 1 (B) 3 1 4 2
1 2 3 123 (C) 2 3 4 1 (D) 4 2 3 1
(A) c a b (B) bca 24. If 6.5 cm bituminous concrete surface with EC = 1000
(C) a b c (D) cab kg/cm2 is equivalent to thickness tb of base course, then
19. Which of the following are the purposes for use of steel equivalent thickness tb of base course having tb = 400
bar reinforcement in cement concrete pavements. The kg/cm2 will be.
correct answer using the code is (A) 12.42 cm (B) 9.36 cm
(1) To increase the flexural strength of concrete (C) 10.48 cm (D) 8.82 cm
(2) To prevent the onset of cracks to allow wider spac- 2 5. In the revised CBR design method recommended by
ing of joints. IRC for design of flexible pavement total thickness
(3) To allow wider spacing of joints depends upon
(A) 1 and 2 only (B) 2 and 3 only (A) CBR value of soil only
(C) 1 and 3 only (D) 1, 2 and 3 (B) CBR value of soil and magnitude of wheel load
2 0. Variation of tyre pressure is (C) CBR value of soil and number of commercial
(A) Equal to 1.0 for an average tyre pressure of vehicle per day
7 kg/cm2 (D) CBR value of soil and cumulative standard axle
(B) < 1.0 for tyre pressure greater than 7 kg /cm2 loads

Answer Keys
1. 2.85 – 2.95 2. C 3. A 4. 1.8 5. C 6. C 7. C 8. D 9. C
10. B 11. A 12. B 13. 37.5 m 14. B 15. 21.2 m 16. 2.50 – 2.52
17. B 18. A 19. D 20. D 21. B 22. A 23. B 24. D 25. D
Highway Engineering Test 4 | 3.181

Hints and Explanations


1. Angularity number = 67 -% solid volume δ1 δ
13. Le = =
α (t2 − t1 ) 2 (α ) (t2 − t1 )
 100 
= 67 –  × 100
 60 × 2.6  30
  = = 37500 mm
2 x 10 x 10 −6 x ( 40)
= 2.89 = 3 Ans: 2.85 – 2.95
  = 37.5 m Ans: 37.5 m
2. RT – 3 Choice (C)
1.5 pa
3. Fire point Choice (A) 14. ∆ =
Es
4. E1 = E2 × 0.2 × h0.45 wheel load 30 × 103
= 5.5 × 0.2 × (3)0.45 Contact area = =
Tyre Pressure 0.6 n/mm
= 5.5 × 0.2 × (3)0.45
π × a2 = 50 × 103

= 1.8 MPa. Ans: 1.8
50 × 103
5. σ critical σ e + σ w (e) − σ f a=
π
= 40 + 9 – 5 = 44 kg/cm2 Choice (C) a = 126.1mm
6. Flow value Choice (C) 1.5 × 0.6 × 126.1
∆= = 5.6 mm Choice (B)
20
7. In between P and 2P Choice (C)
δ
8. Choice (D) 15. Le =
α (t2 − t1 )
9. IRC – 9 Choice (C) δ = 3.5 cm
10. E, h, k, u Choice (B) α = 8 × 10–6per°c
t2 = 40°c
t1 = 20°c
vb ×100
11. VFB = 3.5
VMA Le = = 21.2 m Ans: 21.2 m
8 × 10 ( 40 − 20 )
−6

Va + Vb
VMA =  × 100 100
Vt 16. Gt =
W1 W2 W3 W4
Gm xWb 2.325 × 4.5 + + +
Vb= = = 10.15 G1 G2 G3 G4
Gb 1.03
W1 = 52 G1 = 2.82
G − Gm W2 = 34.8 G2 = 2.42
Va = t ×100
Gt W3 = 4.2 G3 = 2.62
W4 = 5.4 G4 = 1.02
2.41 − 2.325
  = × 100 = 20.73 100
241 Gt =
52 34.8 4.2 5.4
10.15 + 20.73 + + +
VMA = × 100 = 30.88 2.82 2.42 2.62 1.02
1
  = 2.517 Ans: 2.50 – 2.52
10.15 × 100
VFB = = 32.8% Choice (A) 17. Choice (B)
30.88
18. Choice (A)
load at 2.5mm
12. CBR = × 100 19. Choice (D)
standard load
20. Choice (D)
40.8
At 2.5 mm = × 100 = 2.97% Choice (B) 21. Choice (B)
1370
3.182 | Highway Engineering Test 4

22. h = 20 cm, a/h = 0.6 1


tb  Ec  3
a = 0.6 × 20 = 12 cm 24. =
tc  Eb 
b= (1.6a 2
+ h2 ) – 0.675h 1
tb  1000  3
= 
1.6 (12) + (20 ) – (0.675 × 20)
2 2
 = 6.5  400 
1
  = 14.22 cm Choice (A)
 1000  3
tb = 6.5  = 8.82 cm Choice (D)
 400 
23. Choice (B)
25. Choice (D)
Unit XII
Geomatics Engineering
This page is intentionally left blank
Geomatics Engineering Test 1
Number of Questions: 25 Time: 60 min.

Directions for questions 1 to 25: Select the correct alterna- (C) both whole circle bearing and quadrantal bearing
tive from the given choices. (D) None of the above
1. What is the principle of surveying? 10. Small scale representation of map is known as
(A) Working from part to whole (A) scale (B) plan
(B) Working from whole to part (C) grid (D) survey map
(C) Working from higher to lower 1 1. The sag correction for a 30 m steel tape under a pull
(D) Working from lower to higher of 200 N in four equal spans of 8 m each. The weight
2. An offset is measured with an accuracy of 1 in 40. If of one cubic cm of steel = 0.078 N and area of c/s of
the scale of plotting is 1 cm = 30 m, the limiting length tape = 0.08 sqm.
of offset, so that the displacement of point on paper (A) 3.9N (B) 4.2N
from both sources of error may not exceed 0.025 mm, (C) 4.99N (D) 4.6N
is _______. 12.
(A) 21.3 m (B) 21.4 m
Co-ordinates
(C) 21.2 m (D) 21.8 m
Point N E
3. The whole circle bearing of S 31° 361E is _______ A O O
(A) 138° 241 (B) 158° 241
(C) 128° 241 (D) 148° 241 B 3014 256
C 1764 1398
4. If the image formed by the objective lens is not in the
D – –
same plane with cross hairs, then it is known as
(A) focusing of eye piece A straight tunnel is to be run between two points A and
(B) focusing of objective B whose co-ordinates are as given above.
(C) parallax The length and bearing of CD if D is the midpoint of
(D) aberration AB, are _______.
5. The automatic check for leveling in case of height of (A) 74° 381, 1380.2 m
instrument method is (B) 75° 391, 1278.4 m
(A) ΣB.S – ΣF.S = Last RL – First R.L. (C) 78° 341, 1295.7 m
(B) ΣB.S – ΣF.S = ΣRise – Σfall. (D) 76° 321, 1287.6 m
(C) ΣRise – Σfall = last 1 – First RL. 13. In leveling between two points A and B on opposite
(D) None of the above. banks of a river, the staff readings at A and B were 1.295
6. A line lying throughout the surface of the ground and and 2.960 m respectively. The level was then removed
preserving a constant inclination to the horizontal is and set up near B and the readings on A and B were 0.56
(A) contour gradient. and 2.42. The true difference of levels between A and B
(B) horizontal equivalent. is __.
(C) contour interval. (A) 3.486 m (B) 3.525 m
(D) vertical control. (C) 3.538 m (D) 3.624 m
7. Surveyor’s chain consists of 100 links along a length 14. The perpendicular offsets at 20 m intervals from sur-
of vey line to an irregular boundary lines are 3.25, 5.6,
(A) 100 ft long (B) 33 ft long 4.2, 6.65, 8.75, 6.2, 3.25, 4.2, 5.65. The area enclosed
(C) 66 ft long (D) 50 ft long between survey line by the application of trapezoidal
8. Correction for temperature in a chain when the tem- rule is _____.
perature at field is more than the standard temperature (A) 820 m2 (B) 833 m2
is _____ (C) 860 m 2
(D) 866 m2
(A) additive 15. An observation with a theodilite given staff readings of
(B) negative 1.052 and 2.502 for angles of elevation gave 8% and
(C) constant 6% respectively. The vertical angle was 5.25%. The
(D) None of the these horizontal distance of staff station if the instrument ele-
9. In surveyor’s compass the graduation are in vation is 942.5 m is _____
(A) whole circle bearing (A) 48 m (B) 52 m
(B) quadrantal bearing (C) 56 m (D) 50 m
3.186 | Geomatics Engineering Test 1

16. The following are bearings in closed traverse. (ii) The variation which has a yearly period is known
as annual variation.
Line FB BB
(A) (i) is true.
AB 80° 101 259° 01 (B) (ii) is true.
BC 120° 201 301° 501 (C) Both (i) and (ii) are true.
CD 170° 50 1
350° 501 (D) Both (i) and (ii) are false.
DE 230° 101 49° 301 2 1. Match the following
Purpose of survey Scale
EA 310° 201 130° 151
1. Building site i. 1 cm = 50 mts 200 m
The interior angles between the bearings are ________.
2. Location survey ii. 1 cm = 10 m or less
(A) 50°, 65.3°, 230.5°, 100°, 123°
3. Town planning iii. 1 cm = 50 m to 100 m
(B) 50.5°, 138.40°, 131°, 120.40°, 99.10°
(C) 53°, 48.4°, 123.2°, 121°, 100.10° (A) (1 – ii), (2 – iii), (3 – i)
(D) 58°, 62.5°, 73o, 148°, 133° (B) (1 – iii), (2 – ii), (3 – i)
17. In order to find the difference in elevation between two (C) (1 – ii), (2 – iii), (3 – i)
points P and Q, a level was set upon line PQ; 40 meters (D) (1 – i), (2 – ii), (3 – iii)
from P and 1.280 m from Q. The readings obtained on 22. Two straight lines intersect at an angle of 50°. The
shaft kept at P and Q were 0.525 m and 4.92 m. respec- radius of curve joining the two straight lines is 400 m.
tively. The true difference in elevation between P and Q The length of long chord and mid ordinates in meters
is ______. is. _____.
(A) 3.87 m (B) 4.82 m (A) 337.8 m, 36.8 m (B) 340.6 m, 38.3 m
(C) 4.39 m (D) 4.31 m (C) 338.09 m, 37.47 m (D) 360.7 m, 35.6 m
1 8. Match the following. 23. During the leveling work along a falling gradient using
a dumpy level and staff of 3 m length, the following
1. Vertical axis i. Axis about which telescope
successive readings are taken:
rotates in vertical plane
1.623, 2.789, 0.260, 1.520.
2. Turnion axis ii. Line passing through intersec-
tion of horizontal and vertical
What will be the correct order of booking these four
cross hairs and optical center readings in level book?
3. Line collination iii. Axis about which instrument
(A) BS, FS, BS, FS (B) BS, IS, FS, FS
rotates in horizontal plane (C) BS, IS, IS, FS (D) BS, IS, BS, FS
4. Axis of level tube iv. Line tangential to longitudinal 24. The bench mark with reduced level (RL) = 156.305 m
curve of level tube at its center has been established at the floor of a room. It is required
to find the R.L of the underside of root. Back slight
(A) (1 – ii), (2 – iii), (3 – i), (4 – iv)
(BS) is 1.8 m whereas the Foresight (FS) is 0.675, The
(B) (1 – iv), (2 – i), (3 – iii), (4 – ii)
R.L will be ______.
(C) (1 – iii), (2 – i), (3 – ii), (4 – iv)
(A) 152.3 m (B) 159.8 m
(D) (1 – iii), (2 – iv), (3 – ii), (4 – i)
(C) 158.7 m (D) 153.3 m
1 9. A surveyor measured the distance between two points 25. A:  Radiation method of plane table survey is
on plan drawn to a scale of 1 cm = 30 m and the result employed for locating the details.
was 500 m. Later it was discovered that 1 cm = 10 m R: Radiation method is suitable when distances are
scale was used. The true distance between the points small.
would be _____. (A) A and R are true and R is the correct explanation
(A) 168.3 m (B) 165.6 m of A.
(C) 162.3 m (D) 166.6 m (B) A and R are true R is not the correct explanation
20. (i) The durinal variation is the departure of the des- of A.
tination from its mean value a during a period of (C) A is true and R is false.
24 hrs. (D) A is false and R is true.

Answer Keys
1. B 2. C 3. D 4. C 5. A 6. A 7. C 8. A 9. B 10. B
11. C 12. C 13. B 14. D 15. D 16. B 17. A 18. C 19. D 20. C
21. A 22. C 23. A 24. C 25. B
Geomatics Engineering Test 1 | 3.187

Hints and Explanations


2L 14. According to trapezoidal rule
2. Total displacement of paper = cm
rs  O + On 
 ∆ =  o + O1 + O2 + O3 + .... On −1  × d
2L  2 
= 0.025
rs  3.25 + 5.65 
⇒  + 5.6 + 4.2 + 6.65 + 8.75 + 6.2 + 3.25 + 4.2
0.025  2 
L= × 40 × 30 = 21.21 m. Choice (C)
2 × 20 = 866 m2. Choice (D)
3. 15. tan∝1 = 0.06, tan∝2 = 0.08.
N
Horizontal distance
S 2.502 − 1.502
D=  =
tan ∝1 − tan ∝2 0.08 − 0.06
  = 50 m. Choice (D)
W E
16. ∠A = Bearing of AE – Bearing of AB
= 130° 151 – 80° 101 = 50° 51.
31° 361
∠B = Bearing of BA – Bearing of BC
= 259 – 120° 201. = 138° 401
S ∠C = Bearing of CB – Bearing of CD
= 301° 501 – 170° 501 = 131° 01
WCB = 180° – R.B ∠D = Bearing of CD – Bearing of DE
= 180° – 31° 361 = 350° 501 – 230° 101 = 120° 401.
= 148° 241 Choice (D) ∠E = Bearing of ED – Bearing of EA
11. Volume of tape/m = 0.08 × 200 = 16 m .
3
= 49° 301 – 310° 201 + 360° = 99° 101. Choice (B)
Weight of tape/m = 16 × 0.078 = 0.624N. 1 7. Combined correction for Q
Total weight of tape between two supports = 0.624 × 8 = 0.067828 (1.280)2 = 0.110m (subtractive)
= 4.992N. Choice (C) Correct staff reading at Q
1 2. Since D is midway between A and B, its co-ordinates = 4.92 – 0.525 = 4.395 m
are 1507 and 128 Difference in elevation between P and Q
Latitude of AD = 1507. = 4.395 – 0.525 = 3.87 m. Choice (A)
Departure of AD = 128. 19. Measured length = 500 m.
Latitude of AC = 1764. 1 1
R.F of wrong scale = = .
Departure of AC = 1398. 30 × 100 3000
Latitude of DC = 1764 – 1507 = 257.
1 1
Departure of DC = 1398 – 128 = 1270. RF of correct scale = = .
∴ Latitude of CD(L) = –257. 10 × 1000 1000
Departure of CD(D) = –1270.  1 
 
D 1270
= ∴ Correct length =  3000  × 500
Bearing of CD = tan θ = 1
L 257  
1000 
⇒ θ = 78° 341.
= 166.6 m. Choice (D)
Length of CD  = D 2 + L2
22. ∆ = 50, R = 400 m,
= 1270 2 + 2572 ∆
Length of long chord = 2RS in
2
= 1295.7 m.
  Choice (C) 50
= 2 × 400 × sin = 338.09 m.
13. When instrument is at A. 2
2.96 – 1.295 = 1.665 m.  ∆
When instrument is at B. Length of midordinate = R 1 − cos 
 2
2.42 – 0.56 = 1.86 m.
1.665 + 1.86  50 
True difference in elevation = = 400 1 − cos  = 37.47 m. Choice (C)
2  2
=
3.525 m. Choice (B) 24. RL = 156.305 + 0.675 + 1.8 = 158.7 m. Choice (C)
Geomatics Engineering Test 2
Number of Questions: 30 Time: 75 min.

Directions for questions 1 to 30: Select the correct alterna- (A) 172.4 m (B) 173.2 m
tive from the given choices. (C) 174.8 m (D) 170.8 m
1. A closed contour line with one or more higher contour 10. The survey which is done for fixing the property lines
lines inside represents is known as
(A) cliff (B) hill (A) Topographical survey
(C) valley (D) cave (B) Cadastral survey
2. Any convenient direction towards a permanent and (C) City survey
prominent mark is called (D) Astronomical survey
(A) True meridian (B) Magnetic meridian 11. The volume of 130 m long road of formation width
(C) Arbitrary meridian (D) None of the above 10 m, side slopes 1 : 1, average depth of cutting along
3. Theodilite is a measuring device which is included center of line is 5m and slopes of ground in cross-sec-
under category of tion is 10 to 1 is
(A) First order measurements (A) 9880 m3 (B) 9723 m3
(B) Second order measurements (C) 9624 m3 (D) 9892 m3
(C) Third order measurements 12. In a region with magnetic declination of 4°E, the mag-
(D) Fourth order measurements netic fore bearing (FB) of a line AB was measured as N
4. A tape is standardized at 100N pull. If the load applied 82° 501 E. There was a local attraction at A. To deter-
is 120N the sag correction mine the correct magnetic bearing of the line, a point O
(A) L1 W12 (1.12 × 10–6) was selected at which there was no local attraction. The
magnetic FB of line AO and OA were observed to be
(B) L1 W12 (1.273 × 10–6) S 42° 401 E and N 48° 201 W respectively. What is the
(C) L1 w12 (1.73 × 10–6) true FB of line AB?
(A) N 81° 50’ (B) N 82° 10’ E
(D) L1 W12 (1.53 × 10–6)
(C) N 79° 30’ E (D) N 84° 10’ E
5. The long and short sides of a rectangle measure 13. The length of the tape is 30 m and the sag is 30.35 cm
9.32 m and 4.82 m, with errors ± 5 mm. Express the at the mid span under a tension of 100 N, the weight of
area of correct number of significant figures the tape is
(A) 44.98 m2 (B) 44.96 m2 (A) 0.248 N/m (B) 0.269 N/m
(C) 44.92 m 2
(D) 44.85 m2 (C) 0.326 N/m (D) 0.459 N/m
6. An offset is laid out 3° from its true direction of the 14. Two distances of 30 and 100 meters were accurately
field. If the scale of plotting is 10 m to 1 cm, find the measured out and the intercepts on the staff between the
maximum length of offset so that the displacement of outer stadia webs were 0.176 m at the former distance
the point on paper may not exceed 0.25mm. and 0.892 m at the latter. The tacheometric constant K
(A) 4.43 (B) 4.58 is
(C) 4.23 (D) 4.77 (A) 100 (B) 97.7
7. The combined correction of curvature and refraction (C) 95 (D) 96.2
for distance of 1.29 km is
15. A flag staff of 2 m height was erected on topic of hill
(A) 0.234 m (B) 0.121 m
(Q) and the observations were made from two stations
(C) 0.112 m (D) 0.187 m
P and R, 50 meters apart. The horizontal angle meas-
8. The observation ray between two triangulation stations ured at P between R and top of flag staff was 50° 30’
A and B just grazes the sea. If the heights of A and B are and that measured at R between the top of the flag-staff
6000 m and 2000 m respectively, the distance AB is P was 50° 18’. Angle of elevation to top of staff was
(Let radius of earth R = 6440 km) measured to be 10° 121 at P. The angle of elevation
(A) 432.4 km (B) 438.3 km to top of flag staff and was measured to be 10° 48’ at
(C) 450.2 km (D) 442.4 km R. Staff readings on B.M when the instrument was at
9. The constant for an instrument is 750, the value of P = 1.826 m and that with the instrument at R = 2.285
C = 0.3 m and intercept S = 2 m. The distance from m. The elevation of the top of hill Q if the BM was
instrument to the staff when the micrometer readings 485.065 m is
are 4.326 and 4.283 and the line of sight is at +8° 361 (A) 485.36 m (B) 488.32 m
when the staff was held vertical is (C) 494.22 m (D) 498.32 m
Geomatics Engineering Test 2 | 3.189

16. Match the following Common Data for Questions for 22 and 23:
In reciprocal levelling, the following readings are taken.
List – I List – I

Correction for Instrument Staff reading


L.c
i. a. Ca = station
standard length l A B
A 1.286 2.768 Distance AB = 1150 m
ii. Correction for tension b. Cp =
(P − PO ) L
AE B 1.292 2.432 RL of A = 100 m

iii. Sag correction c. Ct = a(Tm – To)L Collimation error = 0.003


150m
Correction for W 2L
iv. d. Csa =
Temperature 24P2 22. The correction for collimation is
(A) 0.021 (B) 0.048
i ii iii iv i ii iii iv (C) 0.023 (D) 0.032
(A)
d a b c (B) a b d c
(C)
d c b a (D) c a b d 23. Correction for refraction is
(A) 1.33m (B) 1.34m
17. (i) Optical square is better than prism square. (C) 1.38m (D) 1.29m
(ii) In both optical and prism squares, the principle of
24. The following observations were taken during testing
operation is same.
of a dumpy level s
(A) only (i) is correct
(B) only (ii) is correct Staff readings at
Instrument at
(C) Both (i) and (ii) are correct A B
(D) None of the above A 1.342 2.125
18. Match the following B 1.485 1.683

List – I List – II If A and B are 100 m apart the angle of inclination of


i. Vertical cliff a. contour lines of different eleva- line of collimation is
tions unite to form one line (A) 14’ 15.50’ (B) 12’ 23.30’
ii. Steep slope b. contour lines of different eleva- (C) 10’ 2.29’ (D) 131 3.40’
tions cross one another
25. The distance between two points A and B by tachometer
iii. Hill c. Contour lines are closely spaced
fitted with anallactic lens which made a vertical angle
iv. Overhanging d. closed contour lines with higher of +10° 46’ and staff intercept of 1.763 m. Later on the
cliff values inside them
constants of instrument were changed to 100 and 0.5.
i ii iii iv The percentage error is computed horizontal distance is
(A) d c a b (A) 0.287% (B) 0.321%
(B) a c d b (C) 0.262% (D) 0.213%
(C) a b d c 26. In a quadrilateral ABCD, the coordinates of points
(D) d b a c are as follows
19. A Circular curve has 300 m radius and 55° deflection Point East North
angle, then the apex distance is A 0 0
(A) 38.23m (B) 38.21m
(C) 39.23m (D) 40.24m B 0 –842.8

20. The length of mid ordinate in the above question is C   600.1   742.8
(A) 37.23m (B) 38.62m D 1023.4   659.3
(C) 33.89m (D) 32.43m
The area of the figure is
21. Find the area between line AB and the stream taken at a (A) 4.68 hectares (B) 7.06 hectares
regular interval of 30 m along line AB, using simpson’s (C) 9.12 hectares (D) 12.51 hectares
rule.
27. The incorrect statement among the following is
Distance 0 30 40 90 120 150 180 210 240 (A) The direction of magnetic meridian is variable
Offset length 23 40 42 30 32 60 10 14 22 (B) The direction of true meridian is invariable
(C) The magnetic bearing of line varies with time
(A) 7980 m2 (B) 6352 m2 (D) Magnetic meridian through various stations are
(C) 5652 m2 (D) 4734 m2 not parallel but converge at poles
3.190 | Geomatics Engineering Test 2

28. After surveying an area with plane table at a station it (A) 747 m, 620.4 m (B) 827 m, 580.3 m
was detected that the centering of the instrument was (C) 800 m, 565.6 m (D) 852 m, 648.3 m
not done accurately. The displacement of the plotted
30. Match the following related to Electromagnetic dis-
point at right angles to the ray was 20 cm. The scale
tance measurement (EDM)
used was 1 cm = 20 m. Determine the error in the plot-
ted position of the point (in cm) i. short range a. less than 3km
(A) 0.03 (B) 0.04 ii. Medium range b. less than 100km
(C) 0.01 (D) 0.02 iii. long range c. less than 25km
29. To continue a survey line AB past an obstacle, a 400
m long line BC was set out perpendicular to AB and i ii iii
from C angles BCD and BCE were set out 60° and (A) b c a
45° respectively. Determine the lengths which must be (B) a c b
chained off along CD and CE in order that ED may be (C) c a b
in line with AB produced. (D) a b c

Answer Keys
1. B 2. C 3. B 4. B 5. C 6. D 7. C 8. B 9. D 10. B
11. A 12. C 13. B 14. B 15. C 16. B 17. C 18. B 19. B 20. C
21. D 22. C 23. A 24. C 25. A 26. B 27. D 28. C 29. C 30. B

Hints and Explanations


1. A closed contour line with one or more higher ones 6. Displacement of point on the paper
inside represents a hill. Choice (B) L sin ∝ L sin 3o
= = cm
2. Any convenient direction towards a permanent and S 10
prominent mark is called Arbitrary meridian. This should not exceed 0.025 cm
 Choice (C)
3. The odilite is included under second order measure- L sin 3o
= 0.025
ments. Choice (B) 10
4. Total sag correction = Sag correction for 100N pull – L sin3° = 0.025 × 10
Sag correction for 120 N pull L = 4.77 m. Choice (D)
2 2 7. Combined correction = 0.06728 (1.29)2 = 0.1196 m.
LW LW
= 1 1
− 1 1
 Choice (C)
24 (100) 24 (120)
2 2

8. C1 = A A = 6 km
1
2
LW
1 1  1 1  Radius of earth (R) = 6440 km
= −
24 100 2 120 2  C2= B1 B = 2 km
Correction of curvature
 = LW
1 1 (1.273 × 10 ).
2 –6
Choice (B)
d2
C1 = 1
5. A = 9.32 × 4.82 = 44.92 m2 2R
Maximum error in individual measurements = 0.005 m Distance d1 = 2 × R × C1
∆L 0.005 1
∴ error ratios are = = = and d1 = 2 × 6440 × 6 = 277.9 km
L 9.32 1864
∆b 0.005 1 A O B
= = C’ d1 d2 C2
b 4.82 964
A’ B’
 1 1 
δA = 44.92  +  = ± 0.07 m
2
 1864 964
Hence the area limits can be 44.92 + 0.07 = 44.99 m2
44.92 – 0.07 = 44.85 m2
The compatible area is 44.92 m2. Choice (C) O’
Geomatics Engineering Test 2 | 3.191

Distance d2 = 2 × R × C2 = 2 × 6440 × 2 wL12


∴ 30.35 =
= 160.49 km = OA + OB = OA’ + OB’ 8P
Distance AB = d1 + d2 = 438.3 km. Choice (B) 0.3035 × 8 × 100
⇒ w =
9. The sum of micrometer reading = 4.326 + 4.283 = 30 × 30
8.609 =
0.269 N/m. Choice (B)
K .S 14. In the first observation
D= cos2θ + cosθ ≯ 1 (one) 30 = K × S + C
n
∴ 30 = K × (0.176) + C –––––– (1)
 750 × 2  In the second observation
  =
8.609
( )
× cos 2 8o 361  + [0.3 × cos(8° 361)] 100 = K × (0.892) + C –––––– (2)
 
Subtracting equations (2) – (1)
  = 170.6 m. Choice (D) K(0.892 – 0.176) = 100 – 30
10. Cadastral survey is the survey in which fixing of K = 97.7 Choice (B)
property lines is done. Choice (B)
15.
11.
Q
1 in m

α2
h
1 in n Q’
R θ α1 Q”
2
1 in A b/2 b Q’
A
K θ1
b/2 D
s
p
2
 b
n + m2 (bh + nh2 ) b = 50 m, q1 = 58° 18’
 2
A = q2 = 50°30’
m 2 − n2 a1 = 10°12’
Where n = 1, m = 10, h = 5, b = 10 a2 = 10°48’
2 S = staff readings.
 10 
1 + 10 2 (10 × 5 + 1 × 52 ) b sin θ2
 2 PQ1 = D = 
\ A = = 76 m2 sin (θ1 + θ2 )
10 2 − 12
V = A × L = 76 × 130 = 9880 m3. Choice (A) 50 × sin (50 o 30 ′ )
  = = 40.75 m
1 2. Declination (D) = 2° E sin (50 o 30 ′ + 58o18′ )
Magnetic F.B. of AB = N82° 50′ E = 82° 50′
h1 = D tana1 = (40.75) tan(10° 121) = 7.33m
Correct F.B of OA = N48° 20′ W = 311° 40′
RL of Q = (R.L of instrument axis at P) + h1
\ Correct B.B of OA = 131° 40′ = True variation (as
=(R.L of BM + s) + h1
O is free from local attraction) In general, F.B. of
=485.065 + 1.826 + 7.33
AO = B.B. of OA.
=494.22 m. Choice (C)
But F.B. of AO = S42°40′ E = 137°20′. ≠ 131°40′.
As A has local attraction,  ∆ 
\ Magnetic variation = 137°20′ (F.B. of AO) 19. The apex distance = R  sec − 1
 2 
\ Error = Magnetic variation – True Variation
= 137°20′ – 131°40′ = 540°.  55 
= 300  sec − 1 = 38.21 m. Choice (B)
Therefore Correction = –5°40′  2 
True F.B. of line AB = 2°50′+2°–5°40′
= 79°10′ = N79°10′E. Choice (C)  ∆
20. Length of mid ordinate = R 1 − cos 
 2
wL1d1
13. For sag h = 55 
8P 
=
300 1 − cos 
Here L1 = d1 (Considering approximately) as not given  2
h = 30.35 cm = 0.3035 m =
33.89 m. Choice (C)
3.192 | Geomatics Engineering Test 2

25. Case 1:– K = 100, C = 0


d (01 + 0 n ) + 4 (0 2 + 0 4 + .... + 0 n −1 )
21. A =   D = KS cos2θ + Ccosθ
3  +2 (03 + 05 + ....0 n − 2 )    =[100 × 1.763 × cos2 (10° 461)] + [0 × cos(10° 461]
30   =170.14 m
= [(23 + 22) + 4 ( 40 + 30 + 60 + 14) + 2 ( 42 + 32 + 10)] Case 2:– K = 100, C = 0.5
5
D = KScos2θ + C cosθ
= 4734 m2. Choice (D)   =[100 × 1.763 × cos2 (10° 46’)]
+ [0.5 × cos(10° 46’)] = 170.63 m
0.003
22. Collimation error =
 × 1150 = 0.023 m 170.63 − 170.14
150 Percentage error = × 100
170.14
Correction for collimation = – 0.023 m. Choice (C)
= 0.287%. Choice (A)
23. Correction for refraction 26. Area of quadrilateral ABCD
Error due to curvature (Cc) = 0.0785d2 = 0.0785(1.15)2 Let y → North, x → East Area
Cc = 0.104 m
Correction for curvature Cc = – 0.104 m 1  y1 ( x2 − x4 ) + y2 ( x3 − x1 ) 
Area A =  
Combined correction for curvature and refraction 2  + y3 ( x4 − x2 ) + y4 ( x1 − x3 )
= – 0.104 + CR
Total correction = –0.023 – 0.104 + CR 1 0 ( 0 − 10234) − 8428 × (600.1 − 0) 
 
Corrected staff reading at B 2  +7428 (10234 − 1) + 659.3 ( 0.600.1) 
= 2.768 – 0.023 – 0.104 + CR
= 2.641 + CR = −70614.345 m 2
Incorrect level difference between A and B at A = 7.0614 × 104m2 = 7.06 hectars Choice (B)
= 2.768 – 1.286 = 1.482 M 27. Magnetic meridian through various stations are not
Incorrect level difference between A and B at B parallel, so donot converge at poles. Choice (D)
= 2.432 – 1.292 = 1.14 m
2 8. Scale is 1 cm = 20 cm
True difference of level between A and B 1
1.482 + 1.14 Representative fraction K =
= 1.311 m 2000
2
Displacement of the plotted position
2.641 + CR = 1.311 1
CR = – 1.33 m. Choice (A) e = 20 ×  = 0.01 cm < 0.025 cm
2000
24. If instrument is at A As the permissible error in plane table survey is 0.025
Apparent difference of level = 2.125 – 1.342 = (0.783) cm. The above error is not much significant.
If instrument is at B  Choice (C)
Apparent difference of level = 1.683 – 1.485 = (0.198)
29.
0.783 + 0.198 C
True level difference = = 0.490 45°
2
Collimation error when instrument is at B 200 m
60°
Correct reading on B = 1.683
Correct reading on A = 1.683 – 0.490 = 1.193m
The amount of inclination = 1.485 – 1.193 = 0.292m
A B E D
Inclination of line of collimation
From ∆BCD
0.292
tanq = = 2.93 × 103 CD = BC sec60° = 400 sec60° = 800 m
100 From ∆BCE
q = 10’ 2.29”. Choice (C) CE = BC sec45° = 400 sec 45° = 565.6 m Choice (C)
Part IV  Mock tests

Mock Test 1������������������������������������������������������������������������������������������������������������������������������������ 4.3


Section – I General Aptitude�������������������������������������������������������������������������������������������������� 4.3
Section – II Civil Engineering������������������������������������������������������������������������������������������������ 4.4
Mock Test 2���������������������������������������������������������������������������������������������������������������������������������� 4.16
Section – I: General Aptitude����������������������������������������������������������������������������������������������� 4.16
Section – II: Civil Engineering��������������������������������������������������������������������������������������������� 4.17
Mock Test 3���������������������������������������������������������������������������������������������������������������������������������� 4.29
Section – I: General Aptitude����������������������������������������������������������������������������������������������� 4.29
Section – II: Civil Engineering��������������������������������������������������������������������������������������������� 4.30
Mock Test 4���������������������������������������������������������������������������������������������������������������������������������� 4.42
Section – I: General Aptitude����������������������������������������������������������������������������������������������� 4.42
Section – II: Civil Engineering��������������������������������������������������������������������������������������������� 4.43
Mock Test 5���������������������������������������������������������������������������������������������������������������������������������� 4.54
Section – I: General Aptitude���������������������������������������������������������������������������������������������� 4.54
Section – II: Civil Engineering��������������������������������������������������������������������������������������������� 4.55
This page is intentionally left blank
Mock Test 1
Number of Questions: 65 Total Marks: 100

Section – I: General Aptitude

Questions 1 to 5 carry One Mark each. 7.


Directions for questions 1 and 2: Select the correct alterna- P S
tive from the given choices.
1. P = {4, 6, 8, 10}
Q = {8, 10, 12, 14, 16) T
8m 10 m
Two integers were randomly selected such that one was
selected from set P and the other was selected from set
Q. What is the probability of the sum of these integers
not being equal to 18?
(A) 0.67 (B) 0.70 Q U R
(C) 0.75 (D) 0.80
In the figure above, angle PQR = angle QRS = angle
2. In a certain code language, if the word SPRING is TUR = 90°, PQ = 8 m and SR = 10 m. Find TU (in m).
coded as TSWPWR, then how is the word ELASTIC 29 31
coded in that language? (A) (B)
9 9
(A) FMBTUJD
40 34
(B) FOFZEVT (C) (D)
(C) FOFZTCZ 9 9
(D) FOFZCTP
8. There are few pens in one box, few pencils in another
Directions for question 3 and 4: Choose the word from the box and a mixture of pens and pencils in a third box. But
options given below, that is most nearly similar in meaning each of these boxes is wrongly labeled and the labels on
to the given word. the boxes are ‘Pens’ ‘Pencils’ and ‘Mixture’. You are
3. REIMBURSE allowed to select only one box and only one item from
(A) Offset (B) Outshine that box. Which labeled box would you select to deter-
(C) Emanate (D) Indemnify mine what each box contains?
4. RAUCOUS (A) Pens
(A) Hoarse (B) Risky (B) Pencils
(C) Phonetic (D) Bitter (C) Mixture
(D) Any one of the above
Directions for question 5: Fill in the blank with the correct
idiom/phrase. Directions for question 9: Choose the appropriate word/
phrase, out of the four options given below, to complete the
5. His association with bad companions got him _______. following sentence.
(A) a piece of cake
(B) into hot water 9. The child had a _____ birth defect in the form of a cleft
(C) back to the drawing board palate.
(D) beating around the bush (A) hereditary (B) compulsive
(C) congenial (D) congenital
Questions 6 to 10 carry Two Marks each.
10. Mr. Dutta, a politician, said that he had visited England
Directions for questions 6 to 8: Select the correct alterna- and he wanted to float the first green party in India. He
tive from the given choices. said his greens were going to replace red in Bengal.
6. A right angled triangle GHI is to be formed in a rectan- Which of the following can be inferred from the sen-
gular coordinate system such that the triangle is right tence above?
angled at H and HI is parallel to the y-axis. It is also I. There is a green party in England.
intended for the x-and the y-coordinates of G, H and II. Green party is not the name of a party.
I to satisfy the inequalities -8 ≤ x ≤ 8 and 7 ≤ y ≤ 15. III. The green party is going to be floated in Bengal.
How many triangles can be formed such that the above (A) All follow
conditions are satisfied? (B) Only I and III follow
(A) 78 (B) 1224 (C) Only III follows
(C) 19584 (D) 20566 (D) Only II and III follow
4.4 | Mock Test 1

Section – II: Civil Engineering


Direction for questions 11 to 65: Select the correct alterna- (A) 0.402 (B) 0.483
tive form the given choices. (C) 0.409 (D) 0.408
Questions 11 to 35 carry One Mark each. 20. MPN index is a measure of ______.
11. The length of the curve x = y y from (0, 0) to (8, 4) (A) Hardness. (B) Turbidity.
(C) Coliform bacteria. (D) Sludge.
is ______
21.
(A) 8.06 (B) 9.07
(C) 10.07 (D) 7.08 50 KN

12. If 2, – 4 and 7 are the eigenvalues of a 3 × 3 square D C


matrix A, then the rank of A is _______
(A) 3 (B) 2 F
(C) 1 (D) None of these
13. If the inverse Laplace transform of a function F(s) is
L–1[F(s)] = t2 + 3t – 2 sint, then L–1[s F(s)] is _________. A E B
2
(A) t + 3 - sin t
t
3 The number of zero force members in the given truss
t 3 2
(B) + t + 2 cos t is ______.
3 2 (A) 3 (B) 4
(C) t3 + 3t2 – 2t sin t (C) 5 (D) 6
(D) 2t + 3 – 2 cost
22. Shear force in a beam under load is
14. For larger values of the degrees of freedom υ , x 2 – (A) Rate of change of loading
distribution can be approximated to _____ (B) Rate of change of bending moment
(A) Poisson distribution (C) Rate of change of slope
(B) Normal distribution (D) Rate of change of deflection
(C) F – distribution 2 3. In a two dimensional incompressible flow, x-compo-
(D) Exponential distribution nent of velocity u = y2 + 4xy. If y component of velocity
15. The order and degree of the partial differential equation v = 0 at y = 0 expression for v is
∂2 u  ∂2 u 
2
∂3 u ∂u (A) 2xy (B) -2y2
− 3  2
+ 4 − 6 + 4u 2 = 0 respectively (C) 2y 2
(D) 4y
∂t 2
 ∂x  ∂x ∂t 2
∂x
24. The target mean strength of concrete mix is give by
are ________ (follow usual notations)
(A) 2 and 1 (B) 2 and 2 (A) fck + s (B) k.fck + s
(C) 3 and 1 (D) 3 and 2 (C) fck + ks (D) k(fck + s)
16. The shear test that is more suitable in the field
25. Match the List-I with List-II
is ______.
(A) Unconfined compression test. List–I (Type of tran-
List–II (Characteristics)
(B) Tri axial shear test. sition curves)
(C) Box shear test. a. Glover’s spiral 1. An autogenous
(D) Vane shear test. curve of automobile
17. The camber provided on a sloping road is 1 in 28. b. Cubic spiral 2. Radius of curve at any
Which of the following is ruling gradient? point varies inversely as
(A) 1 in 15 (B) 1 in 14 the distance from the
(C) 1 in 41 (D) 1 in 28 beginning of curve.
18. EMD in contracts refer to c. Froude’s transition 3. sin f = f
(A) Earlier Money Draft curve
(B) Earnest Money Draft d. Bernoulli’s 4. X = L, cos f=1
(C) Emergency Money Deposit lemniscates
(D) Earnest Money Deposit Codes:
19. The return period for annual maximum flood of a given a b c d a b c d
magnitude is 10 years. The probability that this flood (A) 3 4 2 1 (B) 2 3 4 1
will be exceeded once during next 5 years is (C) 4 3 2 1 (D) 1 2 3 4
Mock Test 1 | 4.5

26. A column is fixed at the bottom and free at the top end. 35. The length of the runway under standard conditions is
If L is the length of the column, EI is the flexural rigid- 1480. The airport site has on elevation of 260 m. Its
ity, Euler’s critical load for the column is reference temperature is 32°C. Assuming all over con-
p 2 EI p 2 EI ditions to be under standard level then correction for
(A) (B) elevation is ______.
4 L2 L2
(A) 1400 m (B) 1470 m
2p 2 E 4p 2 EI
(C) 2
(D) (C) 1500 m (D) 1570 m
L L2
27. The minimum area of tension reinforcement in a beam Questions 36 to 65 carry Two Marks each.
shall be greater than which of the following for the  3 
∫  xydy − 2 y dx , C being the boundary
2
grade of Fe415 steel 36. The value of
C
(A) 0.002 bd (B) 0.0002 bd
(C) 0.003 bd (D) 0.0003 bd of the region enclosed by the circle x2 + y2 = 9 and the
28. The expression for specific speeds of a hydraulic tur- coordinate axes in the first quadrant is _____
bine and a pump respectively are (A) 9 (B) 18
N Q N P (C) 27 (D) 36
N P N Q
(A) 5
, 5 (B) 3
, 5 3 7. If the characteristic equation of a 3 × 3 matrix A is
H 4
H 4
H 4
H4 λ3 – 3 λ2 + λ – 3 = 0, then the determinant of the matrix
N P N Q N Q N P B = 2A7 – 6A6 + 2A5 – 6A4 – 3A3 + 7A2 – 3A + 9I3
(C) 5
, 3 (D) 3
, 5 is _______
H 4
H 4
H 4
H4 (A) 72 (B) –72
29. The instrument used to measure contour interval is (C) 36 (D) –36
called 38. The Taylor’s series expansion of
(A) Roughometer f(x) = x3 – 5x2 + 6x – 2 about x = –1 is _______
(B) Gradienmeter (A) (x – 1)3 – 8(x – 1)2 + 19(x – 1) – 14
(C) Auto level (B) (x + 1)3 + 16(x + 1)2 – 19(x + 1) + 14
(D) Thacheometer (C) (x + 1)3 – 8(x + 1)2 + 19(x + 1) – 14
30. The phenomenon of salts coming in solution and form- (D) (x – 1)3 + 16(x – 1)2 – 19(x – 1) + 14
ing a thin crust on surface of soil after evaporating of
water is called ______. 39. In a binomial experiment, the chances of getting suc-
(A) white alkali cess and failure are in the ratio 1:2, then the probability
(B) saline soil of getting the 5th success in the 8th trial is ______
(C) black alkali 280 140
(A) (B)
(D) red alkali 2187 6561
31. The ultimate BOD of the waste water, whose 5 – day 280 140
(C) (D)
BOD and rate constant (base e) are respectively 6561 2187
180 mg/l and 0.23/day, is
(A) 164 mg/l (B) 204 mg/l 40. Consider the following table of values of x and f(x)
(C) 224 mg/l (D) 264 mg/l
x –1 0 3 6 7
32. In case of passive earth pressure, what would be the
f(x) 3 –6 39 822 1611
angle (in degrees) of failure plan with respect to hori-
zontal plane (Take f of the soil as 30°) ______. The 4th divided difference value is ______
(A) 30° (B) 45° (A) 1 (B) 13
(C) 60° (D) 90° (C) 132 (D) 789
33. For the soil having specific gravity 2.65 and void ratio 41. A simply supported pre stressed concrete beam of 6m
as 0.4; the critical gradient would be ______. long, 300mm wide and 600mm of depth is pre stressed
(A) 1.89 (B) 1.18 by horizontal cable tensioned at eccentricity of 100mm.
(C) 1.03 (D) 1.65 If the pre stressing tensile force in the cable tendon is
34. Generally, fatigue life of welded steel structure to given as 1000 KN, what will be the maximum and min-
fatigue life of riveted steel structure ratio is imum normal stress developed in beam at transfer?
(A) smaller than 1 (A) 11.11Mpa, 5.55Mpa
(B) equal to 1 (B) 11.11Mpa, 0
(C) greater than 1 (C) 0, 5.5Mpa
(D) greater than 2.1 (D) 11.11Mpa, 11.11Mpa.
4.6 | Mock Test 1

42. The following are the rates of rainfall of successive 47. A U-tube mercury manometer is used to measure pres-
20min period of a 150min storm 5.0, 4.0, 11.0, 8.5, sure of oil flowing through a pipe at a point. Specific
1.45, 1.85, 7.0 cm/hr. Taking the value of f - index as gravity of oil is 0.8 and the level of mercury is as shown
4.5 cm/hr, find the net runoff in cm. in the figure. The pressure in kPa is
(A) 2.3 cm (B) 6.4 cm
(C) 4.82 cm (D) 4.51 cm
43. Chose the correct pairs from the following
Group–I Group–II Oil (S.G : 0.8)
1. HC a. Primary air pollutant 150 cm
A•
2. O3 b. Secondary pollutant
3. PAN
50 cm
4. NOx

(A) 1 – b, 2 – b, 3 – a, 4 – a
(B) 1 – b, 2 – a, 3 – a, 4 – b
(C) 1 – a, 2 – b, 3 – b, 4 – a
(D) 1 – a, 2 – a, 3 – b, 4 – b
44. A soil has a bulk density of 2.05 g/cc and water content (A) 196.20
of 21%. If G = 2.64, the degree of saturation of the soil (B) 147.15
is ______. (C) 110.36
(A) 50% (B) 75% (D) 73.58
(C) 100% (D) 0% 4 8. The development length of a deformed reinforced bar
45.  f�ss 
under compression can be expressed as K.  .
 4tbd 
20 kN/m

40 kN 40 kN − m
From the IS 456:2000, the value of K can be calculated
A E as ______.
B C D
(A) 2 (B) 0.75
(C) 0.5 (D) 0.125
3m 1m 2m 2m
49.
4 2
5 6
A simply supported beam is loaded as shown above. 2
2
Maximum bending at point D is 3
(A) 25 kNm 2
1
(B) 35 kNm
4
(C) 65 kNm
(D) 45 kNm 3 4
3
4 6. Match List-I with List-II and select correct answer
using the codes given Critical path of the give network is ______.
List–I List–II
(A) 1 - 2 - 4 – 5- 6
(B) 1 - 2 - 5 - 6
a. Pelton turbine 1. Medium discharge, low head
(single jet)
(C) 1 - 3 - 4 - 5 - 6
(D) None of the above
b. F r a n c i s 2. High discharge, low head
turbine 5 0. During a leveling work along a falling gradient using a
c. Kaplan turbine 3. Medium discharge, medium head dumpy level and a staff of 3m length, following succes-
sive readings were taken:-
4. Low discharge, high head
1.745, 2.765, 0.243, 1.432. What will be the correct
Codes: order of booking these four readings in level book (BS:
a b c Back Sight, IS: Intermediate Sight, FS: Fore Sight).
(A) 1 2 3 (A) BS, FS, BS, FS
(B) 4 3 2 (B) BS, IS, FS, FS
(C) 2 3 4 (C) BS, IS, IS, FS
(D) 3 4 1 (D) BS, IS, BS, FS.
Mock Test 1 | 4.7

51. 57. In a certain situation, waste water is discharged into


260 MPa a river mixes with the river water instantaneously and
completely. Following is the data available.
60 MPa Waste water DO = 2mg/l
Discharge rate = 1.5 m3/sec
River water DO = 8.4 mg/l
100 MPa 100 MPa Flow rate = 8.8 m3/sec
Temperature = 20°C
What will be the initial amount of DO in the mixture of
60 MPa waste and river water?
260 MPa (A) 7.39 mg/l
(B) 7.47 mg/l
Figure shows state of stress at a point in a stressed
(C) 5.29 mg/s
body. Radius of Mohr’s circle representing the state of
(D) 7.29 mg/l
stress is
(A) 60 (B) 80 58. For a given beam AB, ILD for vertical reaction at fixed
(C) 120 (D) 100 end A is ______.
52. A single lane unidirectional highway has design speed
A B
of 55 kmph. The perception brake reaction time of driv-
er’s is 2.5 seconds and average length of vehicles is 6m.
The capacity of this road in terms of kmph is.
1
(A) 604 veh/hr/lane (B) 743 veh/hr/lane (A)
(C) 402 veh/hr/lane (D) 516 veh/hr/lane
53. If the actual observed value of standard penetration 1
resistance, N in a fine sand layer below water table is (B)
25. What will be the equivalent penetration resistance
after correction ______. 1
(A) 10 (B) 15 (C)
(C) 20 (D) 25
54. A water treatment plant of capacity 5 m3/s has filter 1
boxes of dimensions 8m × 15m. Loading rate to the (D)
filter is 240 m3/day/ m2. When three of the filters are
out of service for back washing, the loading rate (in m3/ 59.
day/ m2) is ______.
(A) 200 (B) 300
(C) 400 (D) 500
55. The static and kinematic indeterminacy of the frame
shown below is ______. x
x
O

Centroid of the shaded area shown in figure, is at a dis-


tance of _________ from O
(A) 3 and 3 (B) 6 and 9
(C) 6 and 7 (D) 3 and 7 d d
(A) (B)
5 6. If the wheel base of a train moving on broad gauge 8 6
track is 5.5m, the diameter of wheel base is 1.4m and d d
(C) (D)
depth of flanges below the top of rail is 3.2cm. The 3 2
extra width of provided gauge, if radius of curve is 60. Figure shows the geometry of a strip footing support-
100m is ______. ing the load bearing walls of a multi storied building
(A) 4.6 cm (B) 5.6 cm and the properties of clay layer.
(C) 6.6 cm (D) 7.2 cm (Take gw = 10 KN/m3)
4.8 | Mock Test 1

Q1
of diameter d is Q2. If d = 0.5 D, the ratio of is
Q2
γ bulk = 18 KN/m3 equal to
1.5m
(A) 3.462 (B) 4.683
150 cm (C) 5.657 (D) 6.448
1.8m cc = 0.08 6 4. Identify the most effective but joint (with double cover
6m Clay layer eo = 0.45 plates) for a plate in tension from the patterns (plan
γ sat = 20.KN/m3 view) shown below, each comprising 6 identical bolts
with the same pitch and gauge:
Hard stratum

If the pressure acting on the footing is 40 Kpa, the con-


solidation settlement of the footing will be ______
(A) 7.64 mm
(B) 76.4 mm
(C) 54.38 mm Common Elevation
(D) 5.43 mm (all plates have same thickness)

61. The gross commanded area for a distributary is 8000


hectares, 70% of which culturable irrigable. The inten-
(A)
sity of irrigation for karif season is 35% and that for
rabi season is 60%. If average duty of head of distrib-
utary is 800 hectares/cumec for karif season and 1500
hectares/cumec for rabi season, find the discharge
required for design at head of distributary. (B)
(A) 2.24 m3/sec
(B) 2.45 m3/sec
(C) 2.52 m3/sec
(D) 2.25 m3/sec
(C)
62. A non-homogeneous soil deposit consists of a clay
layer sandwiched between a fine sand layer at top and a
silt layer below. Permeability of the silt layer is given as
1
five times the permeability of clay layer and th of (D)
10
the sand layer. Thickness of sand layer is 2 times the
thickness of the silt layer and half of the thickness of
clay layer. Then, what will be the ratio of equivalent 65. The following data pertains to number of commercial
horizontal and vertical permeability of the deposit ____ vehicles per day for the design flexible pavement for
(Round up to nearest integer) national highway as per IRC: 37-1984.
(A) 5 (B) 10 Vehicles con- Vehicles
Type of commercial no. of
(C) 15 (D) 20 vehicles per day
sidering no. damaging
of lanes factor
63.
Two axle trucks 3000 7
Q1, D
Tanden axle trucks 400 8

Q Assuming a traffic growth factor of 7.5% per annum


for both the types of vehicles, the cumulative number
of standard axle load repetitions for a design life of ten
Q2, d years is
L (A) 13.42 msa
(B) 14.26 msa
Two pipes of equal length L are connected in parallel. (C) 12.49 msa
Rate of flow through pipe of diameter D is Q1 and pipe (D) 15.3 msa
Mock Test 1 | 4.9

Answer Keys
1. D 2. D 3. D 4. A 5. B 6. C 7. C 8. C 9. D 10. A
11. B 12. A 13. D 14. B 15. C 16. D 17. B 18. D 19. C 20. C
21. B 22. B 23. B 24. C 25. B 26. A 27. A 28. C 29. B 30. A
31. D 32. A 33. B 34. A 35. D 36. D 37. B 38. C 39. C 40. A
41. B 42. D 43. C 44. C 45. C 46. B 47. A 48. C 49. C 50. C
51. D 52. B 53. C 54. B 55. D 56. A 57. B 58. C 59. B 60. B
61. B 62. B 63. C 64. A 65. C

Hints and Explanations


1. Probability (The sum not being equal to 18) = 1 – \ HI has 9 × 8 = 72 possibilities.
Probability (The sum being equal to 18) As explained above, it similarly follows that when HI
The number of ways of selecting two integers one from is the line x = 7 or x = 6 or ….. x = –8, HI has 72 possi-
each set is 4 × 5 = 20. bilities in each case.
The sum can be 18 when the integers selected are 4 and Total number of possibilities for HI = (72) (17) = 1224
14, 6 and 12, 8 and 10, 10 and 8. The triangle is right angled at H.
4 \ The y-coordinate of G must be the same as that of
\ Probability that the sum being 18 =
20 H and its x-coordinate can be any possible value other
4 than that of H.
\ Required probability = 1 − = 0�8
\ The x-coordinate of G has 17 – 1 = 16 possibilities.
4×5
\ G has 16 possible positions.
 Choice (D)
From (1) and (2)
2. Word: S P R I N G \ The triangle GHI has 1224 × 16 = 19584 possible
Logic: +1 +3 +5 +7 +9 + 11 arrangements i.e., 19584 triangles can be formed satis-
Code: T S W P W R fying the given conditions.
Similarly, Alternate Solution:
Word: E L A S T I C Given –8≤x≤8 and 7≤y≤15 i.e. there are 17 vertical
Logic: +1 +3 +5 +7 +9 +11 +13 lines and 9 horizontal lines. The number of rectangles
Code: F O F Z C T P Choice (D) formed with these lines is 17 C2 × 9 C2 .
3. To ‘reimburse’ is to pay back the money spent. To
We know that one rectangle gives 4 right angled
‘indemnify’ is also to pay back money (for some loss
triangles.
or damage). ‘Offset’ is an amount that diminishes or
\ Total number of right angled triangles formed is
balances the effect of an opposite one. It does not mean
4 × 17 C2 × 9 C2 = 19584. Choice (C)
a guarantee, as does ‘indemnity’. Choice (D)
4. ‘Raucous’ is harsh or ‘hoarse’. Other synonyms are, 7. In DTUR and DPQR,
grating, discordant, jarring or strident. Choice (A) ∠R is common.
∠TUR = ∠PQR = 90°
5. When we get into bad company we get into hot water The above conclusions mean the third pair of angles
or get into trouble. Other idioms do not work in the of both D’s must be equal. The third angle of each D =
context. ‘A piece of cake’ is a job/task that is very 180° − (sum of the other two of its angles).
easy while ‘back to the drawing board’ means a failed \ DTUR ∼ DPQR.
attempt that has to be started again. To ‘beat around
the bush’ is to avoid speaking openly/directly about an TU UR
\ =  ---------- (1)
issue. Choice (B) PQ QR
6. HI is parallel to the y-axis. Any line parallel to the Similarly DTUQ ≡ DSRQ.
y-axis must have its equation of the form x = a constant.
TU QU
\ HI can be any of the lines x = –8 or x = –7 or …. \ =  --------- (2)
x = –1 or x = 0 or x = 1 or ….. x = 8. (∵ –8 ≤ x ≤ 8, SR QR
7 ≤ y ≤ 15). (1) SR UR
⇒ =
Suppose HI is the line x = 8. Then H can be any of (2) PQ QU
(8, 7), (8, 8), …… (8, 15). \ H has 9 possible posi-
tions. For each of these possible positions I can have QU 8 4
\ = =
any of the remaining 8 possible positions. UR 10 5
4.10 | Mock Test 1

QU 4 4
 9 
⇒ 1 + = 1+      =∫ 1 + y  dy
UR 5 4 
0

UR + QU 9 3
⇒ = 2 9  2 4
4
UR 5      = 1 + y × 
3 4  9 0
QR 9
⇒ = 3 3
UR 5 8  9 2 8  9 2
     =  1 + × 4 −  1 + × 0
UR 5 27  4  27  4 
=
QR 9      = 9.0734. Choice (B)
TU 5 12. Since none of the eigenvalues of A are zero, A is a
From (1) ⇒ = non-singular matrix.
8 9
\ The rank of A = The order of A = 3. Choice (A)
40 1 3. Given L–1[F(s)] = t2 + 3t – 2sint = f(t), (say)
TU = m Choice (C)
9 \ f(0) = 02 + 3 × 0 – 2 sin 0 = 0
8. We have to select the box that is labeled as mixture. We know that, if L–1[F(s)] = f(t) and
Now if we get a pen, as the box cannot have a mix- f(0) = 0, then L–1[s F(s)] = f1(t)
ture, it has pens. Now the box which is labeled as pens d 2
\  L–1[s F(s)] = f1(t) = [t + 3t – 2 sin t] = 2t + 3 – 2
cannot have mixture. [∵If it happens then the box with dt
label pencil must contain pencils] cos t. Choice (D)
⇒ The box with label pens has pencils and that with
pencils has a mixture of them. Choice (C) 14. Standard Result. Choice (B)
9. Birth defects are congenital (present from birth) and not 1 5. For the partial differential equation
2
hereditary, compulsive, or congenial (affable; friendly). ∂2 u  ∂2 u  ∂3 u ∂u
 Choice (D) − 3  2
+ 4 −6 + 4 u2 = 0
∂t 2
∂x  ∂ x ∂t 2
∂x
10. Mr. Dutta has decided to float the first green party in
Order = order of the highest ordered partial derivative
India after visiting England. It is clear that there is a
=3
green party in England and Mr. Dutta is impressed with
Degree = Degree (power) of the highest ordered partial
the party. Assume the case where Mr. Dutta learnt about
derivative = 1. Choice (C)
the green party while in England but there is no green
party in England; in such a case, there would not be any 1 1
1 7. G = 2C = 2 × =  Choice (B)
relevance to the visit to England. Hence I follows. 28 14
According to the statement, Mr. Dutta wants to float 19. p1 = 1 – qn
the first green party. It implies that there can be sev- 1 1
eral green parties. There cannot be more than one party p= = = 0.1
T 10
with the same name. Hence, II follows.
Mr. Dutta says his green would replace the reds in q = 1 – 0.1 = 0.9
p = 1 − (0.9)  = 0.409
5
Bengal. From this, it can be concluded that the green Choice (C)
party in going to be floated in Bengal as well. Hence,
21. Zero force members are:
III too follows.
DC, BC, EF, DE Choice (B)
Therefore all follow. Choice (A)
3 23. Continuity equation must be satisfied for the flow
11. Given curve is x = y y = y 2 ∂u ∂v
i.e., + =0
dx 3 12 ∂x ∂y
⇒ = y
dy 2 u = y2 + 4xy
\ The length of the curve x = y y from (0, 0) to ∂u
= 4y
4   dx  2  ∂x
(8, 4) = ∫
y=0
1 +    dy
  dy  
To satisfy continuity equation
∂v
= −4 y
4   3 1 2 ∂y
     = ∫
y=0
1 +  y 2   dy
 2   On integration, v = -
4 y2
+ C = -2y2 + C
2
Mock Test 1 | 4.11

It is given that v = 0 at y = 0 By Green’s theorem, we know that


\ C = 0  ∂N ∂M 
\ The expression for v is v = -2y2 Choice (B) ∫ Mdx + Ndy = ∫∫  ∂x −
C R
∂y 
dxdy  → (1)

p 2 EI
26. Euler’s critical load Pcr = −3 2
L2e Here M =  y and N = xy
2
where Le = effective length
∂M ∂N
For the given case, Le = 2L \ = –3y and =y
∂y ∂x
p 2 EI
\ PCr =  Choice (A) In the region R,
4 L2
y varies from y = 0 to y = 9 − x and x varies from
2

As 0�85
27. = x = 0 to x = 3
bd fy
3
∫ xydy − 2 y dx = ∫ M dx + N dy
2
0�85 \
\ As = × bd = 0.002 bd Choice (A) C C
415
 ∂N ∂M 
31. yt = y0(1–e–kt) = ∫∫  − dx dy  [From (1)]
R
 ∂x ∂y 
(
y5 = y0 1 − e −(0�23× 5) ) = ∫∫ ( y − ( −3 y )) dx dy
R
180 = y0(0.683)
\ y0 263.4 mg/l 3  9 − x2 
y0 = 263.4 mg/l Choice (D) = ∫
x =0



y =0
4 y dy  dx

32. Passive earth pressure case,
3
45 + f/2 w.r.t. major principal plane (vertical) 9 − x2

45 – f/2 w.r.t. minor principal plane (horizontal)


= ∫
x=0
2 y 2  y = 0 dx
 Choice (A) 3

33. G = 2.65 and e = 0.4 = ∫ 2 (9 − x 2 ) dx


0
G − 1 1�65
ic = = = 1�18  Choice (B) 3
1+ e 1�4 2 3
= 18x – x = 36. Choice (D)
3  0
35. Correction for elevation
7 260 37. Given the characteristic equation of a
= × 1480 × = 89�78m
100 300 3 × 3 matrix A is λ3 – 3 λ2 + λ – 3 = 0
\ By Cayley – Hamilton theorem, we have A3 – 3A2
Corrected length = 1480 + 89.78m = 1569.7m
+ A – 3I3 = O → (1)
 Choice (D)
Consider B = 2A7 – 6A6 + 2A5 – 6A4 – 3A3 + 7A2 – 3A +
 3 2  9I3 = 2A4(A3 – 3A2 + A – 3I3) – 3(A3 – 3A2 + A – 3I3) –
∫C  xy dy − 2 y dx
36. We have  2A2 = 2A4 × 0 – 3 × 0 – 2A2. (From (1))
\ B = –2A2
 −3  Now Det(B) = Det(–2A2)
= ∫  2
C
y 2 dx + xydy 
 = (–2)3 |A2| (∵ |ka| = kn |A|, where n = order of A)
= –8|A|2
Y (∵ |Am| = |A|m for any positive integer m)
\ |B| = –8|A|2  → (2)
We know that
|A| = Product of the eigenvalues of A = (–1)n × con-
stant term in the characteristic equation of A (where
C
n = order of A)
R
= (–1)3 × (–3)
X \ |A| = 3
O (3, 0)
Hence from (2), |B| = –8 × 32 = –72. Choice (B)
4.12 | Mock Test 1

38. We have f(x) = x3 – 5x2 + 6x – 2 Alternative solution:


We know that the Taylor’s series expansion of f(x) about We know that the Taylor’s series expansion of f(x) about
x = a is x = –1 is same as that of the Taylor’s series expansion of
( x − a) f11(a) + ( x − a)
2 3
f(x) in powers of x + 1.
f(x) = f(a) + (x – a) f1(a) + \ f(x) = x3 – 5x2 + 6x – 2
2! 3!
= [(x + 1) – 1]3 – 5[(x + 1) – 1]2 + 6[(x + 1) – 1] – 2
f (a) + …… ∞ → (1)
111
= [(x + 1)3 – 3(x + 1)2 + 3(x + 1) – 1] – 5[(x + 1)2 –
Here a = –1 2(x + 1) + 1] + 6(x + 1) – 6 – 2
f(x) = x3 – 5x2 + 6x – 2 ⇒ f(–1) = –14 = (x + 1)3 – 8(x + 1)2 + 19(x + 1) – 14. Choice (C)
f1(x) = 3x2 – 10x + 6 ⇒ f1(–1) = 19
f11(x) = 6x – 10 ⇒ f11(–1) = –16 39. Given the ratio of success and failure in a binomial
f111(x) = 6 ⇒ f111(–1) = 6 experiment is 1 : 2
And f(IV)(x) = f(V) (x) = …. = 0 1 1
\ The probability of success = p = =
\  From (1), the Taylor’s series expansion of f(x) 1+ 2 3
about x = –1 is 2
The probability of failure = q = 1 – p =
( x − (−1))
2
3
f(x) = f(-1) + (x – (-1)) f1(-1) + f11(-1) +
2! Probability of getting the 5th success in the 8th trial =
( x − (−1)) Probability of getting 4 successes in first 7 trails and a
3

f111(–1) + …… ∞ success in the 8th trial.


3!
  1  2   1
4 3

( x + 1) ( x + 1) =  7C4      × 3
2 3

= –14 + (x + 1) × 19 + × (–16) + ×6+  3 3 


2! 3!
0 + 0 + …….. 280 280
= = . Choice (C)
\ f(x)=(x + 1)3 – 8(x + 1)2 + 19(x + 1) – 14 38 6561

40. Given pairs of values of x and f(x) are

x -1 0 3 6 7
f(x) 3 -6 39 822 1611

The divided difference table for this data is as shown below.

x f(x) 1st divided differences 2nd divided differences 3rd divided differences 4th divided differences
-1 3
−6 − 3
− −9
0 − ( −1)
0 -6 15 − ( −9)
=6
3 − ( −1)
39 − ( −6) 41 − 6
= 15 =5
3− 0 6 − ( −1)
3 39 261 − 15 13 − 5
= 41 =1
6−0 7 − ( −1)
822 − 39 132 − 41
= 261 = 13
6−3 7−0
6 822 789 − 261
= 132
7−3
1611 − 822
= 789
7−6
7 1,611

\ The 4th divided difference value for the given data is 1. Choice (A)
Mock Test 1 | 4.13

P P�e wG = 0�21 × 2�64 = 0�55


41. s = ±
A z Sg = 100%
bd 2 (or)
z=
6 g 2�05
gd = = = 1.70
300 × 600 2 1 + w 1 + 0�21
   =
6
gwG
  = 18 × 106 mm3 gd =
1+ e
1000 × 103 1000 × 103 × 100
s= ± Mpa 1 × 2�64
300 × 600 18 × 106 1�70 =
  = 5�55 ± 5�55 1+ e
300 mm e = 0.55
wG 0�21 × 2�64
e= ⇒ sg =
sg 0�55
600 mm
\ Sg = 100% Choice (C)

20 × 3
45. RA + RE = + 40 = 70 kN
\ s max = 11�11 Mpa and 2
smin = 0 Mpa  Choice (B) Taking moment about E
42. The total rainfall is 20 × 3
RA × 8 - × 6 - 40 × 4 + 40 = 0
20 2
[5 + 4 + 11 + 8�5 + 1�45 + 1�85 + 7] × = 12�93cm
RA × 8 = 180 + 160 - 40 =300
60
f = 4.5cm/hr RA = 37.5 kN
20 RE = 70 - 37.5 = 32.5kN
Pe = [5 + 11 + 8�5 + 7] × = 10.5cm Minimum bending moment at D
60
= RE × 2 - 40 = 32.5 × 2 - 40 = 25 kNm
Pe − R
f index = Maximum bending moment at D
te
= RE × 2 = 32.5 × 2 = 65 kNm. Choice (C)
4 × 20
te = = 1�33 47. Equating pressure heads above the line passing through
60
10�5 − R A
4.5 = 50 150
1�33 hA + × 0�8 = × 13�6
100 100
R = 4.515 cm Choice (D)
\ hA + 0.4 = 20.4
43. Primary pollutants: NOx, HC
⇒ hA = 20 m of water
Secondary pollutants: O3, PAN Choice (C)
Pressure pA = whA = 9810 × 20 N/m2
4 4. g = 2.05 g/cc
w = 21% = 9.81 × 20 kN/m2 = 196.2 kN/m2
G = 2.64 = 196.2 kPa Choice (A)

g =
(
g w G + e�sg ) φ�σ s
48. Ld =
1+ e 4 τ bd

2�05 =
(
1 2�64 + e�sg ) For the deformed bars tbd can be increased by 60%
1+ e and for bars under compression, tbd can further
increased by 25% as per IS 456 : 2000.
wG
e= So; tbd becomes (1�6 × 1�25) tbd
sg
1(2�64 + wG ) φ σs
⇒ 2�05 = \ Ld =
wG 4 2τ bd
1+
sg ⇒ k = 0.5 Choice (C)
4.14 | Mock Test 1

49. 1800
11/11
No. of filters = = 15
2/4 4 120
2
2 5 6 But, Given that 3 are out of service
2
3 43200 m3 / day
2 \ Surface loading rate =
0/0 12 × 120 m 2
1
4 =
300m3/day/m2 Choice (B)
3 7/7
55.
3 4

3/3

\ Critical path = 1 – 3 – 4 – 5 – 6 Choice (C)


5 0. BS = 1.745
IS = 2.765
IS = 0.243
FS = 1.432 Choice (C) Ds = (3 + 2 + 1) – 3 = 3
2
 px − p y  Dk = (0 + 1 + 2 + 3 + 3 + 3) – m
 2  + q
2
51. Radius of Mohr’s circle is =   = 12 – 5 = 7 Choice (D)
5 6. h = 3.2cm
2
 260 − 100  D = 140cm
 + 60
2
= 
2 B = 5.5m
R = 100m
= 80 2 + 60 2
L= h2 + D.h = 0.02 (3.2)2 + (140 × 3.2) = 0.428m
= 6400 + 3600
Extra width of gauge (d)
= 1000 = 100.  Choice (D)
13 ( B + L ) 13 (5.5 + 0.428)
2 2

d= =
CS R 100
52. V =
1000   = 4.568cm Choice (A)
v 2
QW .DOW + QR .DOR
S = vt + 57. DOmix =
2 gf QW + QR

(0.278 × 55)2 1.5 (2) + 8.8 (8.4 )


  = (0.278 × 55 × 2.5) +    = = 7.47 mg/l Choice (B)
2 × 9.8 × 0.4 1.5 + 8.8
  = 68.04m = 68.04 + 6 = 74.04 m 58. According to Muller-Breslau principle to find reaction
1000 × 55 at A, remove the constraint at A & apply unit force in
C= = 743 veh/hr/lane Choice (B) the direction of reaction. But, here the other end B is
(68.01 + 6) not hinged, it is a free end. So, end B also lifted up
53. When NR > 15 and fine sand present below the water along with end A when we apply unit reaction at A.
table; diletancy correction should be applied.  Choice (C)
A1 x1 − A2 x2
 N − 15  59. x =
Equivalent N = 15 +  R ( A1 − A2 )
 2 
 25 − 15   p ( 2 d )2  pd 2 d
\ N eq = 15 +  = 20 Choice (C)
 2   × 0 − ×
=  4  4 2
54. Plant capacity = 5 m3/s = 5 × 24 × 60 × 60 m3/day/ p ( 2d ) pd
2 2

= 43200 m3/day −
4 4
Area of filter = 8 × 15 = 120 m 2
pd 3
Surface loading rate = 240/m3/day/m2 −
= 8 = − d = d to left of O. Choice (B)
432000 m3 / day 3pd 2 6 6
⇒ Area required = =1800m2
240 m3 / day / m 2 4
Mock Test 1 | 4.15

cc  sf  H1
60. ∆H = H � log   ⇒ H 2 = 2 H1 , H 3 =
1 + e0  s0  2
K H + K2 H 2 + K3 H 3
At the middle of the clay layer, kH = 1 1
H1 + H 2 + H 3
s0 = (1�5 × 18) + (3 × (20 − 10 ))
 H1 
  = 27 + 30 = 57 Kpa (10 K3 ) H1+ K (2 H )+ K 
3  2 
3
1
sf = 57 + 40 = 97 Kpa = 5
H
6000 × 0�08  97  H1 + 2 H1 + 1
∆H = log10   2
1 + 0�45  57 
=
3.114K3
\ ∆H = 76�43mm  Choice (B) H1
6 1. GCA = 8000 Ha H1 + 2 H1 +
KV = 2
70 H1 2 H1 H
CCA = 8000 × = 5600ha + + 1
100 10 K 3  K 3  K 3
 
35 5
Area under kharif = × 5600 = 1960ha
100 =0.315 K3
60 K H : K V = 9�87 ≅ 10  Choice (B)
Area under rabi = × 5600 = 3360ha
100
63. When pipes are connected in parallel, head lost is same
A in both pipes
Discharge required for kharif Q =
D 4 fL V12 4 fl V22
1960 \ h = = =
= = 2.45 m3/sec D 2g d 2g
800
A flQ12 fLQ22 Q2 Q2
Discharge required rabi Q = ⇒ 5
= 5
⇒ 15 = 25
D 3D 3d D d
3360 2 5
= = 2�24 m3 /sec  Q1   D
⇒   =   = (2) = 32
5
1500
 Q2  d
Design discharge = higher of both the discharge
= 2.45m3/sec Choice (B) Q1
⇒ = 32 = 5.657 Choice (C)
Q2
62.
64. The most common type of rivet patterns are chain riv-
1 Sand eting and diamond riveting. Staggered pattern in option
(a) yields more net area of the section and because
2 Clay
of this reason this pattern is most suitable for tension
members. Staggered and diamond pattern better as
3 Silt
compared to the chain pattern. Choice (A)
1 65. A1 = 3000 F1 = 7 r = 0.075
K3 = 5K2 = K A2 = 400 F2 = 8 n = 10yrs
10 1
K 365 (1 + 0�075) − 1 (3000 × 7) + ( 400 × 8)
10
⇒ K1 = 10K3, K2 = 3
5 N=
0�075
H2
H1 = 2H3 =   = 12�49msa  Choice (C)
2
Mock Test 2
Number of Questions: 65 Total Marks: 100

Section – I: General Aptitude


Questions 1 to 5 carry One Mark each. then select the pair of words which has a similar relation-
Directions for question 1: Select the correct alternative ship to the capitalized words and mark the number of that
from the given choices. pair as your answer.
1 1 5. ACRID : BITTER

1. If  x 2 + 2  = 79, then find x3 + 3 . (A) Mythical : Shallow
 x  x
(B) Suave : Urbane
(A) ±692 (B) ±702 (C) Clairvoyant : Disgraceful
(C) ±712 (D) ±756 (D) Diminutive : Gargantuan
2. The following pie-chart gives the split up of number of Questions 6 to 10 carry Two Marks each.
students having different hobbies in a college of 1800
Directions for questions 6 to 8: Select the correct alterna-
students.
tive from the given choices.
Music Cricket 120° 6. Pointing at a photograph Ms. Sudha told Mr. Bhujbal,
90° “the person in the photograph is your father’s wife’s
mother-in-law’s only son and my mother’s brother’s
only brother-in-law”. How is Bujbal related to Sudha?
(A) Brother (B) Cousin
(C) Uncle (D) Brother-in-law
Dance
60° Singing 45° 7. log cot1° + log cot2° + log cot3° + . . . + log cot
89° = ______.
Basket ball (A) 1 (B) – 1
Reading
30° 15° (C) 0 (D) None of these
8. It is high time that we address the problem of obesity
Every student has exactly one of the above mentioned with more seriousness as the ill effects of obesity is
hobbies. The difference in the number of students with more diverse and more harmful than it was thought
cricket and music as their hobbies is _______ earlier.
Directions for question 3: The question given below gives Which of the following can be inferred from the above
a word followed by four choices. From the choices, select statement?
the most suitable synonym (word which means the same) (A) Obesity was not considered a disease earlier.
for the question word and mark its number as the answer. (B) Obesity is the reason for all health related
problems.
3. ADJOURN (C) An obese person cannot be free from a disease.
(A) prevent (D) Measures to address the problem of obesity has
(B) withhold been initiated in the past.
(C) postpone
(D) abrogate Directions for question 9: In the question below, four dif-
ferent ways of writing a sentence are indicated. Choose the
Directions for question 4: The question given below has a best way of writing the sentence.
sentence with two blanks, followed by four pairs of words as
9. A.  The report discusses growth stories in sectors
choices. From the choices, select the pair of words that can
like IT and pharma where Indian companies have
best complete the given sentence.
made a mark and describes how domestic indus-
4. Reason accepts the ______ of spontaneous intuition but try has profitably restructured through the past ten
asks that we check our intuition and ______ whether years as economic reforms saw tariff wall fall and
they be correct. the license permit raj dismantled.
(A) problem . . . evaluate B. The report discusses growth stories in sectors like
(B) tranquility . . . promulgate IT and pharma where Indian companies have made
(C) challenge . . . question a mark and describes how domestic industry has
(D) existence . . . ascertain profitably restructured over the past ten years into
Directions for question 5: In the question below, determine economic reforms seeing tariff walls fall and the
the relationship between the pair of capitalised words and license permit raj dismantled.
Mock Test 2 | 4.17

C. The report discusses growth stories in sectors laws were recently passed to reduce the punishment for
like IT and pharma where Indian companies have crimes by lessening prison terms. Clearly the new laws
made a mark and describes how domestic industry have been solely responsible for the sudden spurt in
has profitably restructured over the past ten years crimes reported.
as economic reforms saw tariff walls fall and the Which of the following can be logically inferred from
license permit raj dismantled. argument above?
D. The report discusses growth stories in sectors like (A) As a part of the recent measures undertaken by
IT and pharma where Indian companies have made the state, almost 20% of the entire police force has
a mark and describes how domestic industry has been laid off.
restructured profitably through the past ten years (B) There has been no appreciable increase in crimes
when economic reforms have seen tariff walls fall reported in other countries where similar laws
and the license permit raj dismantling. were passed.
Directions for question 10: Choose the best answer for the (C) The crime rate is still 10% lower than that in any
following question: other country of comparable size.
(D) The state has recently increased the strength of
10. Media watchers (and readers) have found that there has
judges in the courts from 10 to 20%.
been a sudden increase of 25% in crimes reported after

Section – II: Civil Engineering


Directions for questions 1 to 55: Select the correct alterna- 9 + x7 − 9 − x7
tive form the given choices. 15. The value of Lim is ______
x→0 x7
Questions 11 to 35 carry One Mark each. (A) 7 (B) 3
11. Which of the following statements is ALWAYS true 1 1
with reference to the LU – decomposition A = LU of a (C) (D)
7 3
square matrix A with L and U being the lower triangular
and upper triangular matrices respectively? 16. Two steel plates each of 10 mm thickness, are con-
(A) All the principal diagonal elements will be equal nected by a double cover butt joint by bolts as shown in
to 1 for only L. figure. If bolt diameter is 20 mm and steel is of grade
(B) All the principal diagonal elements will be equal 410, which of the following section is the most critical
to 1 for any U. section for a main plate?
(C) All the principal diagonal elements will be equal 3 2 1
to 1 for both L and U.
(D) All the principal diagonal elements will be equal
to 1 for any one of L and U.

 x
12. If u(x, y) = tan−1   , then the ratio of the first partial
 y
derivatives of u(x, y) with respect to x and y is same as 3 2 1
the ratio of ______
(A) x and y, (A) section 1 – 1
(B) −y and x (B) section 2 – 2
(C) x2 and y2 (C) section 3 – 3
(D) −y2 and x2 (D) section 1 – 1 and 3 – 3
13. In the process of finding the reciprocal value of 23 by 17.
the Newton – Raphson method with 0.08 as the initial 30
3
guess, the value of the reciprocal of 23, after the first 6
5 2
iteration will be _______ 10 20
4
40 60
4
3
 p + x; −p < x < 0 3
1 4. If f(x) =  and f(x + 2p) = f(x), then
0; 0≤ x≤p 50 70
6
the Fourier series expansion of f(x) at x = 0 converges
to ______. For the network shown in figure, find the critical path?
4.18 | Mock Test 2

(A) 10 – 20 – 30 – 60 - 70 (A) 100.1 m, 1.9 m (B) 100.1 m, 2.2 m


(B) 10 – 20 – 30 – 40 – 50 – 70 (C) 101.173 m, 2.2 m (D) 101.175 m, 1.9 m
(C) 10 – 20 – 40 – 60 – 70 27. If the magnetic bearing = 40°301 and magnetic declina-
(D) 10 – 20 – 30 – 40 – 60 – 70 tion is 6° 301 E then true bearing is
18. The following constituent imparts plasticity to brick (A) 47°01 (B) 34°
earth ___ (C) 42° (D) 50°
(A) Alumina (B) Silica 28. If 10 ml of water sample is diluted with 190 ml distilled
(C) Lime (D) Magnesia water at which odour is just detectable, then TON of
1 9. The contribution compounds of cement in decreasing sample is
order of rate of hydration are _____ (A) 10 (B) 20
(A) C3S, C3A, and C2S (C) 40 (D) 30
(B) C2S, C3S, and C3A 29. A source emitting 40 dB, 20 dB and 90 dB at differ-
(C) C3A, C2S, C3S ent times in a day. What is average noise produced by
(D) C3A, C3S, C2S source in a day
20. The critical section considered for the shear design, if (A) 96.2 dB (B) 96.4 dB
the end of beam is subjected to tension due to support (C) 94 dB (D) None of the above
reaction? 30. The concentration of dissolved oxygen (DO) may fall
(A) section at a distance of effective depth (d) from the down to zero, causing anaerobic conditions in river
support zone known as
(B) section at a face of the support (A) zone of degradation
d (B) zone of active decomposition
(C) section at a distance from the support
2 (C) zone of recovery
(D) section at a distance 2d from the support (D) zone of clear water
21. The equation of A line is ____ 31. Determine the amount of lime required for treatment of
(A) Ip = 0.83 (WL – 20) (B) Ip = 0.63 (WL – 20) one million liters of water per day if raw water contains
(C) Ip = 0.53 (WL – 20) (D) Ip = 0.73 (WL – 20) 200 mg/L of total hardness is
(A) 110 kg/day (B) 140 kg/day
22. For a dense packing of regular spheres at the maximum
(C) 120 kg/day (D) 112 kg/day
density the void ratio is _____
(A) 0.91 (B) 0.81 32. Find the width of grit chamber to handle a waste water
(C) 0.65 (D) 0.35 flow of 20 MLD with VH = 0.3 m/sec, detention time =
1 min, depth = 0.8 m is
23. In case of soil mechanics the following head is
(A) 0.8 m (B) 1 m
neglected
(C) 0.6 m (D) 0.4 m
(A) Pressure head (B) Velocity head
(C) Elevation head (D) Total head 33. A wire of radius r is stretched by a force P. If another
wire of radius 2r of same material is stretched by the
24. Consolidation of soil is due to load which is same load, its modulus of elasticity will
(A) Static and short term (A) be doubled (B) be halved
(B) Dynamic and short term (C) become 4 times (D) not be changed
(C) Dynamic and long term
(D) Static and long term 34.
P
25. A fully compensated raft foundation for a building
is ____ M
(A) Designed as a completely flexible footing
(B) Designed as a completely Rigid footing L
(C) Designed such that weight of excavated soil is
equal to the load due to building
(D) None of these A cantilever beam of span L is subjected to a concen-
trated load P and a moment M as shown in the figure.
26. The missing data (X1 and X2 respectively) from the table Deflection at the free end is given by
given below is
PL3 ML2 PL2 ML3
(A) + (B) +
Station BS FS HI RL 3 EI 2 EI 2 EI 3 EI
BM 1.175 X1 100 PL3 ML2 PL3 ML2
A X2 98.975
(C) − (D) −
3 EI 2 EI 2 EI 3 EI
Mock Test 2 | 4.19

35. Diameter of an air bubble in water is 0.01 mm. If the z2 z2


surface tension at air water interface is 0.07 N/m, the (A) -xz + + f ( x, y ) (B) -xz +
2 2
pressure difference between inside and outside of the
air bubble is z2
(C) xz - + C (D) -x + z
(A) 14 kPa (B) 28 kPa 2
(C) 56 kPa (D) 62 kPa 1
4 3. A fixed beam subjected to load w at rd span as shown
Questions 36 to 65 carry Two Mark each. 3
36. The least possible value of the determinant of a 3 × 3 in figure. Estimate the collapse load?
matrix A with all entries being either 0 or 1 is ______.
(A) 0 (B) −1 L/3
(C) −2 (D) −3
C
37. The value of the line integral ∫ F .d r where F = xy i + A
c 2MP B
2yz j − z k and the curve c is given by x = t, y = t ,
2 2
L/2 L/2
z = 3t, 0 ≤ t ≤ 2 is ______.
Mp Mp
3 8. Rajeev goes to office either by car, scooter, bus or train (A) 12 (B) 9
L L
with various possibilities as shown in the table below.
Mp Mp
Mode of Percentage Chance of reaching (C) 15 (D) 6
L L
transport chance of using the office lately
4 4. For a particular activity of a project, time estimates
Car 20% 1 out of 25
received from engineers x, y and z are as follows
Scooter 40% 2 out of 25
Optimistic Most likely Pessimistic
Bus 30% 1 out of 10
time time time
Train 10% 1 out of 5
Engineer x 4 6 8
If Rajeev reached the office lately, then the probability Engineer y 3 5 8
that he travelled by scooter is ______. Engineer z 5 7 10
1 8
(A) (B) Who is more certain about time of completion of job?
3 9
14 16 (A) Engineer x (B) Engineer y
(C) (D) (C) Engineer z (D) Can’t say
27 45
45. A concrete beam of 200 mm × 400 mm is pre-stressed
4
with a force of 600 kN at an eccentricity of 120 mm.
x
39. The value of the definite integral ∫ 1 + 3x
0
2
dx obtained The maximum and minimum stresses developed
are ____
by the Simpson’s rule with step size h = 1 is ______. (A) 21 MPa and 6 Mpa (B) 15 MPa and 9 MPa
(A) 0.6059 (B) 0.6486 (C) 6 MPa and 1.5 MPa (D) 12 MPa and 21 MPa
(C) 0.6687 (D) 0.6984 4 6. Identify the correct relation with respect to “shape fac-
4 0. The value of y at x = 3, if y satisfies the differential tor” for different shapes in increasing order
dy
equation x − 2y = x3 and y = 0 at x = 1 is ______
dx
(A) 9 (B) 18
(C) 27 (D) 36 (i) (ii)
41. Velocity potential function for a flow is given by
f = x2 - y2. The stream function for the flow is
(A) 2xy + f(y) (B) 2xy + f(x)
(C) 2xy + constant (D) 2x - 2y + constant (iii) (iv)
(A) iii, iv, ii, i (B) ii, iii, iv, i
42. In a 3 dimensional incompressible fluid flow, velocity (C) iv, iii, i, ii (D) iv, i, ii, iii
components in x and y directions are
u = x2 + y2z3 47. A saturated clay stratum draining both at the top and
v = -(xy + yz + zx) bottom undergoes 50 percent consolidation in 24 years
Velocity component in the z direction is under an applied load. If an additional drainage layer
4.20 | Mock Test 2

were present at the middle of the clay stratum, 50 per- 54. Find the extra widening necessary for horizontal curves
cent consolidation would occur in ______ years. having R = 100 m, wheel base 6.1 m, pavement width =
48. An unconfined compression test yielded a strength of 7 m, Design speed, V = 70km/hr
0.2 N/mm2. If the failure plane is inclined at 60° to (A) 1.112 m (B) 1.14 m
the horizontal, what are the values of shear strength (C) 1.11 m (D) 1.32 m
parameters? 55. A design speed of traffic lane is 80 kmph. Average
(A) 0.057 N/mm2; 30° length of vehicle is 6m. The capacity of the road when
(B) 0.35 N/mm2; 60° reaction time of driver is 2 sec and coefficient of longi-
(C) 0.15 N/mm2; 30° tudinal friction is 0.35 is
(D) 0.4 N/mm2; 60° (A) 523veh/hr
4 9. A 6 m high retaining wall is supporting a saturated (B) 656veh/hr
sand (saturated due to capillary action) of bulk density (C) 832veh/hr
21 KN/m3 and angle of shearing resistance 30°. The (D) 932veh/hr
change in magnitude of earth pressure at the base due 56. The rate of super elevation on a highway in plain ter-
to rise in ground water table from the base of footing to rain with design speed of 100 kmph with radius of
the ground surface shall (gw = 10 kN/m3) curve 500 m under mixed traffic condition is
(A) increases by 20 kN/m2 (A) 7% (B) 9.2%
(B) increases by 40 kN/m2 (C) 8.8% (D) 15.72%
(C) decreases by 20 kN/m2
57. Consider
(D) decrease by 40 kN/m2
1. Creation of central road fund
5 0. A bed of sand consists of three horizontal layers of 2. National highway act
equal thickness. The value of Darcy’s K for the upper 3. Formation of Indian Road congress
and lowest layers is 1 × 10–1 cm/sec and that for the 4. Creation of highway research board
middle layer is 1 × 10–2 cm/sec. The ratio of permea- The correct chronological order of the events is
bility of the bed in the horizontal direction to that in (A) 4, 3, 2, 1 (B) 2, 1, 3, 4
vertical direction is _____. (C) 1, 3, 2, 4 (D) 2, 3, 1, 4
51. If a square footing of size 4m × 4m is resting on the 58. Following test results were obtained by a CBR test on a
surface of a deposit of soft clay having cohesion of 13.5 subgrade soil?
kPa, the ultimate bearing capacity of the footing (as per
Terzahi’s equation) is _____. Penetration in “mm” 0 2.5 5.0

52. Find the capacity of digester from the following data Load (kg) 0 55 78
Q = 20 ML, SS in waste water flow 200 mg/L
Area of plunger is 19.6 cm2
% of SS removed from clarifier= 70
The aggregates pressures at 2.5 mm and 5mm are 70
Moisture content in fresh sludge P1 = 98%
kg/cm2 and 105 kg/cm2 respectively. The CBR of the
Moisture content of digested sludge P2 = 90%
soil is?
Specific gravity of fresh sludge = 1.06
(A) 3.8% (B) 3.5%
Specific gravity of digested sludge = 1.02
(C) 4% (D) 4.4%
Digestion period = 60 days
53. The specific gravities and weight proportions are given 59. From the analysis of rainfall data at a particular sta-
as under for the preparation of marshall mould. The tion it was found that a rainfall of 400mm had a return
volume and weight of marshall mould are 475 cc and period of 20 years. What is the probability of rainfall
1100 g respectively equal to or greater than 400 mm occurring at least once
in 10 successive years is
Specimen Weight (g) Specific gravity (A) 50.43 (B) 40.12
A1 825 2.63 (C) 20.32 (D) 26.23
A2 1200 2.51 60. Route the following flood hydrograph through a river
A3 325 2.46 reach for which Muskingum coefficient k = 8 and
A4 150 2.43
x = 0.25
Bitumen 100 1.05 Time (h) 0
Inflow cm3/s 8
The % voids in mineral aggregates is
(A) 15.25 (B) 16.85 If the initial out flow discharge from the reach is 8 m3/s.
(C) 12.49 (D) 13.42 Find the final outflow
Mock Test 2 | 4.21

(A) 10 m3/s (B) 8 m3/s 64.


(C) 6 m3/s (D) 4 m3/s
80 MPa
61. In an irrigated plot the net irrigation requirement of
crop is found to be 14.9 cm, the application efficiency
is 80 % and the water conveyance efficiency is 70%.
What is the gross irrigation requirement?
20 MPa 20 MPa
(A) 29.8 cm (B) 25.2 cm
(C) 31.3 cm (D) 26.60 cm
62. The area of the irrigation channel in alluvial soil
according to lacey’s sit theory with the following data,
discharge is 50m3/sec lacey’s silt factor = 1, side slope
= 0.5H : 1V 80 MPa
(A) 56.4 m2 (B) 62.3 m2
(C) 64.3 m 2
(D) 59.38m2 For the state of stress at a point in a stressed body,
63. shown in figure radius of the Mohr’s circle is
(A) 80 MPa (B) 60 MPa
P
(C) 50 MPa (D) 40 MPa
C D 65.
L L L 100 N
A B
3 3 3 P
A 500 mm 500 mm B

Figure shows shear force diagram of a beam ABCD 10 Nm


of length L. Bending moment at the centre of the beam
is
A simply supported beam of 1 m length is loaded at the
PL
(A) 0 (B) mid span as shown in the figure. Maximum bending
3 moment is
PL 2 PL (A) 30 Nm (B) 35 Nm
(C) (D)
2 3 (C) 40 Nm (D) 45 Nm

Answer Keys
1. B 2. 150 3. C 4. D 5. B 6. A 7. C 8. D 9. C 10. A
11. D 12. B 13.  0.011 to 0.014 14.  1.55 to 1.59 15. D 16. C 17. B 18. A
19. D 20. B 21. D 22. D 23. B 24. D 25. C 26. C 27. A 28. B
29. D 30. B 31. D 32. B 33. D 34. C 35. B 36. C 37. 8.7 to 8.9
38. D 39. A 40. B 41. C 42. A 43. C 44. A 45. A 46. C
47. 6 years 48. A 49. B 50. 2.8 51. 100 – 102 Kpa 52. 478.35 m3 53. C
54. C 55. B 56. A 57. C 58. C 59. B 60. B 61. D 62. D 63. B
64. C 65. A

Hints and Explanations


1 1  1
3
1 1
1. Given x + = 79
2

x2 x3 + =  x +  − 3 x�  x + 
x3  x x x
1 ( ± 9) − 3 ( ± 9)
3
x2 + + 2 = 81 =
x2
± [729 − 27] = ± 702 .
= Choice (B)
2
 1
 x +  = 81
x (120 − 90)
2. Required difference = × 1800 = 150.
1 360
⇒ x + =± 9
x  Ans: 150
4.22 | Mock Test 2

3. ‘Adjourn’ means ‘put off’ as in “The meeting was  x


adjourned until December 4”. So ‘postpone’ is apt. 12. Given u(x, y) = tan−1  
 y
‘Abrogate’ means override / revoke or cancel.
 Choice (C) ∂u 1  1 y
⇒ = .  = s 2
4. The second blank needs a word synonymous with ∂x  x
2
 y x + y2
the word “check” since the sentence talks about how 1+ 
 y
one needs to check whether one’s intuition is correct.
∂u 1 −x −x
Hence, it could be “ascertain”. Though evaluate also and = .  2  = 2
suits, the former suits better in this context. The first ∂y  x  y 
2
x + y2
1+ 
blank can take “existence”. Tranquility means peace.  y
 Choice (D) −x
∂u ∂u y
5. ‘Suave’ means sophisticated, polished, ‘urbane’. The \ : = 2 2 : 2 2 = −y : x
∂x ∂y x + y x + y
question words ‘acrid’ and ‘bitter’ are synonyms too.
 Choice (B)  Choice (B)
6. Bhujbal’s father’s wife’s mother-in-law is Bhujbal’s 1 3. We have to find x such that
father’s mother, whose only child is Bhujbal’s father. 1 1
x= ⇒ = 23
Hence, the person in the photograph is Bhujbal’s father. 23 x
Bhujbal’s father is Sudha’s mother’s brother’s only
1
brother-in-law, i.e. Sudha is Bhujbal’s father’s daughter. Let f(x) = − 23 = 0
\ Bhujbal is Sudha’s brother. Choice (A) x
7. log cot1° + log cot 2° + . . . log cot 89° −1
⇒ f’(x) =
= log (cot1° . cot2° . . . cot 89°) x2
Now cot 89° = cot (90° – 1) = tan 1° Given the initial guess = x0 = 0.08
cot 88° = cot (90° – 2) = tan 2° 1
∴ log (cot 1° cot 2° . . . .cot 88° cot 89°) \ f(x0) = − 23 = −10.5
0.08
= log (cot 1° cot 2° . . . .tan 2° tan 1°)
= log 1 = 0 (∵ cot 1° tan 1° = 1 −1
cot 2° tan 2° = 1) Choice (C) and f’(x0) = = −156.25
( 0.08)2
8. From the statement it is clear that certain action is being By Newton – Raphson method, the reciprocal value of
taken to address obesity. Hence, (D) can be inferred 23 after first iteration is given by
from the statement. Choice (A) is in contradiction to
f ( x0 )
the statement. From the statement we know that obesity x1 = x0 −
leads to certain problems. But the information does not f ′ ( x0 )
allow us to infer that every health problem is caused by
( −10.5)
obesity. Hence, (C) cannot be inferred. Hence, (D) can   =0.08 −
be inferred. Choice (D) ( −156.25)

9. In (A) and (D) the usage of ‘through the past ten years’ \ x1 = 0.0128.
is incorrect. Further the usage of ‘wall’ in (A) is incor-  Ans: 0.011 to 0.014
rect. Also in (D) ‘dismantling’ is awkward. In (B) the  p + x; − p < x < 0
usage of the ‘ing’ form of the verb ‘see’ is incorrect. 14. Given f(x) = 
0; 0≤ x<p
Choice (C) is correct in all ways. Choice (C)
and f(x + 2p) = f(x)
10. Only (A) can be logically inferred from the sentences.
Clearly, f(x) is discontinuous at x = 0,
If the laws have been changed to reduce punishment 1
and prison terms, obviously it follows that some of the \ Fourier series of f(x) at x = 0 converges to
2
police staff will not be needed as there will be fewer
 f (0 + ) + f (0 −) 
prisoners. The other statements (B) and (C) are outside
the scope of the text as what happens in other countries where f(0+) = Lt f(x) = Lt 0 = 0
is beyond our scope of understanding. Choice (D) only x→0+ x→0+

weakens / goes against the arguments in the text. and f(0 –) = Lt f(x) = Lt (p + x) = p
x→0− x→0−
 Choice (A)
Hence the Fourier series of f(x) at x = 0 converges to
11. From the procedure of LU – decomposition
 Choice (D) 1 p
[0 + p] = = 1.57  Ans: 1.55 to 1.59
2 2
Mock Test 2 | 4.23

31. Molecular weight of lime (CaO) = 56 mol.wt


9 + x7 − 9 − x7
15. We have Lt 100mg of TH requires 56mg of quick lime
x→0 x7 200mg of TH requires 56mg of quick lime
 9 + x7 − 9 − x7   9 + x7 + 9 − x7  200 × 56
= Lim  ×  ⇒ = 112mg/lit
x→0  x7   9 + x 7 + 9 − x 7 100

(9 + x 7 ) − (9 − x 7 ) ⇒ Total lime dosage required = Q × dosage
= Lt =1 × 112 = 112kg/day. Choice (D)
x→0
9 + x7 + 9 − x7
3 2. Volume of grit chamber = Q1 × DT
2 1
= xLt =  Choice (D) 20 × 106
3 =3 × 1 = 13.86m3
9+ x + 9− x (10 × 24 × 60)
→0 7 7

16. For cover plate, 1 – 1 is critical and for main plate 3 – 3


Length of grit chamber = VH × DT
is critical section. Choice (C)
=0.3 × 1 × 60 = 18 m
17. C volume 13�86
8/8 = = = 0.771m
30 S length 18
0/0 5/5 3
6 0�771
5
10 20
4
2 \ Width = = 0.96m ⋍ 1 m. Choice (B)
10/10 14/16 0�8
40 60
4 35. s = 0.07 N/m, d = 0.01 mm
3 3
4s 4 × 0�07
13/13 DP = =
50
6
70 d 0�01 × 10 −3
19/19
   = 28 × 103 N/m2 = 28 kN/m2= 28 kPa. Choice (B)
Critical path: 10 – 20 – 30 – 40 – 50 – 70 (or) Critical
 a1 b1 c1 
path is the path along which duration is maximum.
3 6. Let A =  a2 b2 c2  be a 3 × 3 matrix with all entries
From options, the path 10 – 20 – 30 – 40 – 50 – 70 has  
maximum duration. Choice (B)  a3 b3 c3 
18. Alumina imparts plasticity to brick earth. Choice (A) being either 0 or 1.
19. C3A > C3S > C2S. Choice (D)  a1 b1 c1 
20. If end of beam is subjected to tension; critical section \ Det A =  a2 b2 c2 
is at face of the support. If end of beam is subjected to  
 a3 b3 c3 
compression; critical section is at a distance of d, effec-
tive depth from the support. Choice (B) = a1(b2c3 – b3c2) − b1(a2c3 − a3c2) + c1(a2b3 − a3b2)
21. Ip= 0.73 (WL – 20). Choice (D) \ Det A = a1b2c3 + a3b1c2 + a2b3c1 – a1b3c2
– a2b1c3 − a3b2c1 → (1)
22. Void ratio for diagonal packing (for densest condition)
As the entries of A being 0 or 1 only and from (1), we
is 0.35 where as for loose packing (cubical array) void
can observe that the least possible value for |A| cannot
ratio is 0.91. Choice (D)
be less than −3.
23. Velocity head is neglected in soil mechanics. Also, from (1), we can observe that for no possible set
 Choice (B) of values of ai, bj and ck (i = 1 to 3, j = 1 to 3 and k = 1
24. Consolidation is due to static and long term loading. to 3), determinant of A can be −3.
 Choice (D)
1 1 0 
X1 = RL + BS = 100 + 1.175 = 101.175m
And let A = 1 0 1 ,
26.
HI – X2 = 98.975  
X2 = HI – 98.975 = X1 – 98.975 0 1 1
  = 101.175 – 98.975 = 2.2m. Choice (C)
1 1 0
27. Forbearing = 40°30 + 6°30 = 47°01.
1 1
Choice (A)
Det A = 1 0 1 = −2
A + B 10 + 190
28. TON = = = 20 Choice (B) 0 1 1
A 10
\ The least possible value of determinant of A = −2
1  40 20 90
  Choice (C)
29. L = 20log10 10 20 + 10 20 + 10 20 
3   3 7. Given F = xy i + 2yz j − z2 k
  = 96.69dB. Choice (D) and C is x = t, y = t2, z = 3t; 0 ≤ t ≤ 2
4.24 | Mock Test 2

⇒ dx = dt, dy = 2tdt and dz = 3dt 1


= [(0 + 0.0816) + 4(0.25 + 0.1071) + 2(0.1538)]
∫ F �dr = ∫ ( xyi
2
\ + 2 yz j − z k )�( dxi + dy j + dzk ) 3
c c
= 0.6059 Choice (A)
2

∫ (t i + 6t j − )� (dti + 2tdt j + 3dtk )


3 3 4 0. Given differential equation is
=
t =0
dy
x – 2y = x3 → (1)
2 dx
∫ (12t + t − 27t ) dt
4 3 2
= dy 2 y
⇒ − = x2 → (2)
t =0 dx x
2
t5 t4 t3  Clearly, (2) is a linear equation of first order in y of the
12 + − 27 
= dy
5 4 3 0 form + P(x)y = Q(x)
dx
32 16 −2
=
12 × + −9×8 where P(x) = and Q(x) = x2
5 4 x
−2
384 44 ∫ dx −2

= + 4 – 72 = = 8.8. Ans: 8.7 to 8.9 \ Integrating factor = I.F. = e x


= e In x
5 5
1
IF =
38. Let B1, B2, B3 and B4 denote the events of Rajeev going x2
to office by car, scooter, bus and train respectively. \ The general solution of (1) is
Let A be the event of Rajeev reaching the office lately.
\ P(B1) = 0.2, P(B2) = 0.4, P(B3) = 0.3, P(B4) = 0.1 y × (I.F) = ∫ [Q ( x ) × I�F] dx + c

 A 1  A 2 1  1
P   = = 0.04; P   = = 0.08 ⇒ y × = ∫  x 2 × 2  dx + c
 B1  25  B2  25 x 2
 x 
1  A 1 y
P   = = 0.1; P   = = 0.2
A
x2 ∫
⇒ = dx + c
 B3  10  B4  5
y
\ Probability that Rajeev reached the office lately ⇒ 2 = x + c  → (3)
given that he travelled by scooter. x
Given y = 0 at x = 1
 A
P ( B2 ) P   \ From (3),
B   B2 
P 2 = 4
= 0
 A  A =1+c
∑i =1
P ( Bi ) P  
 Bi 
12
⇒ c = −1
(By Bayes’ theorem) Substituting the value of c in (3), we have
0�4 × 0�08 y
= = x − 1 ⇒ y = x3 – x2 → (4)
0�2 × 0�04 + 0�4 × 0�08 + 0�3 × 0�1 + 0�1 × 0�2 x2
0�032 32 16 From (4), at x = 3, we have
= = =  Choice (D) y = 33 – 32 = 18 Choice (B)
0�09 90 45
4
4 1. Potential function f = x2 - y2
x
39. We have to evaluate ∫ 1 + 3x 2 dx by Simpson’s rule. ∂φ
= u and
∂φ
=v
0
∂x ∂y
x
Here y = f(x) = ; h = 1; a = 0 and b = 4 \ 2x = u or u = 2x
1 + 3x 2
and -2y = v or v = -2y
x 0 1 2 3 4 For stream function y
y = f(x) 0 0.25 0.1538 0.1071 0.0816 ∂ψ
= −v  -----(1)
∂x
\ By Simpson’s rule, we have
∂ψ
4
x
b
h = u ----(2)
 ∫0 1 + 3x 2 dx = ∫ ydx = 3 ∂y
a
From (1)
[(y0 + y4) + 4(y1 + y3) + 2y2] ∂y = -v dx = (2y)dx
Mock Test 2 | 4.25

L
y= ∫ 2 ydx = 2xy + f(y) 5Mpq = W.q.
3
∂y
= 2x + f ' ( y) 15 M p
∂y \ W =
L
Equating with (2) Case II: Two plastic hinges at supports and one under
u = 2x + f′(y) concentrated load
or 2x = 2x + f′(y)
W
⇒ f′y = 0
Or f(y) = C(constant) A C
\ y = 2xy + C Choice (C)  

42. For a three dimensional, incompressible fluid flow the


continuity equation must be satisfied
 
∂u ∂v ∂w
i.e., + + =0 ∆ ∆
∂x ∂y ∂z q1 = and q =
L 2L
u = x2 + y2z3
3 3
∂u L 2L
= 2x
∂x D = q1 × =q×
3 3
v = -(xy + yz + zx)
⇒ q1 = 2q
∂v
= − (x + z) Internal work done = 2Mpq1 + 2Mp(q + q1) + MP.q
∂y =4Mpq + 6Mpq + Mpq = 11 Mpq
∂w 2L
\ 2x - (x + z) + =0
∂z Mpq = W. q
3
∂w
⇒ = −2 x + x + z = -x + z 16�5 M p
∂z \ W =
L
Integrating with respect to z
The collapse load will be minimum of above two values
z2
w = -xz + + C where C = f(x, y) 15 M p
2 So; collapse load = . Choice (C)
L
z2
or w = -xz + + f ( x, y ) . Choice (A)
2 44. The degree of uncertainty is indicated by the variance
of time estimates
43. No of plastic hinges required = DS + 1 = 2 + 1 = 3 2
 t p − to 
Case I: Two plastic hinges at supports, and one at point Variance = 
where cross section changes  6 
W Variance of time estimates given by
2
 8 − 4
x: s x 2 =  = 0.4356
 6 
A C
θ θ 2
 10 − 5 
Similarly: s y 2 =  = 0.69 and s z 2 = 0.69
 6 
θ θ Greater than variance; greater the uncertainty. So,
Engineer X’s time estimates have less variance, hence
more certain. Choice (A)
Internal work done = 2Mpq + Mp(q + q) + Mpq
External work done = W.D P P�e
⇒ 5Mpq = W.D 45. s = ±
A z
∆ Given: P = 600 KN
tanq =
L A = 200 × 400 = 80 × 103mm2
3 E = 120mm
L bd 2 200 × 400 2
D = q. Z= = = 5.33 × 106mm3
3 6
6
4.26 | Mock Test 2

600 × 103 600 × 103 × 120 1 − sin 30 1


\ s = ± = 7.5 ± 13.5 49. Ka = =
80 × 103 5�33 × 106 1 + sin 30 3
smax = 21 Mpa Case I: W.T at base
smin = – 6 Mpa. Choice (A) 1
(Po)at base = Ka gH = × 21 × 6
4 6. Shape factor different shapes are 3
(Pa)at base = 42 kN/m2
1.5 2.34 Case 2: W. T at ground surface
(Pa)at base = kag1H + gwH
1
= × (21 – 10) × 6 + 10 × 6 = 22 + 60
3
(Pa)atbase = 82KN/m2
1.7 2
\  From case (1) and case (2), Pa increased by
40 KN/m2 Choice (B)
 Choice (C) K1 Z1 + K 2 Z 2 + K 3 Z3
50. KH =
4 7. For same degree of consolidation Z1 + Z 2 + Z3
Tv is same
Cv t
(1 × 10 )1 + (1 × 10 )1+ (1 ×10 )1
−1 −2 −1

= constant     =
d2 1+1+1
\ t & d2 KH = 0.07 cm/sec
H Z1 + Z 2 + Z3
In, 1st case d = KV =
2 Z1 Z 2 Z3
+ +
In 2nd case; an additional drainage layer is provided K1 K 2 K 3
1+1+1
   =
1 1 1
1 −1
+ −2
+
H/2 1 × 10 1 × 10 1 × 10 −1
Kv= 0.025
H/2 KH 0�07
2 = = 2.8 Ans: 2.8
P
KV 0�025

51. Qu = 1.3 CNc + gDNq + 0.4gBNr


So, d should be taken as higher of (1) and (2) For pure clay; (f = 0)
H Nc = 5.7; Nq = 1 and Nr = 0
\ d2 =
4 \ qu = 1.3 × 13.5 × 5.7 + g (0) (1) + 0
 H
2
qu = 100.kPa Ans: 100 – 102 Kpa
t1  2 70
\ = 
t2 H 52. M = 20 × 200 × = 280 kg/day
  100
4
ef = Sf × ew = 1.06 × 1000 = 1060 kg/m3
24
= 22 ed = Sd × ew = 1.02 × 1000 = 1020 kg/m3
t2
100 M
T2 = 6yrs Ans: 6 years Vf = ×
100 − p1 ef
4 8. qu = 0.2 N/mm2
100 280
qu = 2c tana   = ×
100 − 98 1060
qu 0�2
C= =   = 13.2 m2/day
2 tan a 2 × tan 60
100 M
C = 0.057 N/mm2 Vd = ×
100 − P2 ed
φ
45 + = 60
2 100 280
  = × = 2.745m3/day
q = 30°. Choice (A) 100 − 90 1020
Mock Test 2 | 4.27

 V f + Vd  S = SSD + L = 116 + 6 = 122m


Capacity of digester =  ×t 1000 V 1000 × 80
 2  C= = = 656veh/hr. Choice (B)
S 122
13�2 + 2�745 
=
  × 60 = 478.35m
3
2 (0�75v )2
56. e =
 Ans : 478.35m3 gR
va + vb (0�75 × 27�77)2
53. VMA = × 100 = = 0.08 = 8% is greater than 7% as
vt 9�81 × 500
Gt − Gm per IRC in plain terrain. Choice (A)
Va = × 100
Gt load at 2�5 mm penetration
58. CBR =
100 standard load at 2�5 mm penetration
Gt =
W1 W2 W3 W4
+ + + 55
G1 G2 G3 G4    = = 4.0%
(19�6 × 70)
W = w1 + w2 + w3 + w4 + w5 = 825 + 1200 + 325 + 150
load at 5�0 mm penetration
+ 100 = 2600 CBR (%) =
825 standard load
W1 = × 100 = 31.7%
2600 78 × 100
= = 3.79%
1200 19�6 × 105
W2 = × 100 = 46.1%
2600 CBR (%) = 4.0% Choice (C)
325 5 9. T = 20 years
W3 = × 100 = 12.5% 1 1
2600
P= = = 0.05
150 T 20
W4 = × 100 = 5.76%
260 N = 10 years
100 Q = 1 – p = 1 – 0.05 = 0.95
W5 = × 100 = 3.84% Probability of occuring at least once = 1 – qn
2600
= 1 – (0.95)10
100 = 0.4012 = 40.12%. Choice (B)
Gt = = 2.405,
31�7 46�1 12�5 5�76 3�84
+ + + + − kx + 0�5∆t ( −8 × 0�25) + (0�5 × 4)
2�63 2�51 2�46 2�43 1�05 60. C0 = = =0
k − kx + 0�5∆t 8 − (8 × 0�25) + (0�5 × 4)
1100
Gm = = 2.315 kx + 0�5∆t (8 × 0�25) + (0�5 × 4)
475 C1 = = =0.5
k − kx + 0�5∆t 8 − (8 × 0�25) + (0�5 × 4)
2�405 − 2�315
Va = × 100 = 3.74 %, C2 = 1 – C0 – C1 = 1 – 0 – 0.5 = 0.5
2�405
Q2 = COI2 + C1I1 + C2Q1
Gm × Wb 2�315 × 3�84   = 0 + (0.5 × 8) + (0.5 × 8)
Vb = = = 8.479%
Gb 1�05   = 8m3/sec. Choice (B)
VMA = 3.74 + 8.749 = 12.491. Choice (C) 61. ha = 0.8, hc = 0.7
5 4. Total extra widening on the road Net irrigation requirement, NIR = 14.9
7m wide road has two lanes as per IRC NIR 14�9
FIR = = = 18.65cm
nl 2 V na 0�8
We = +
2R 9�5 R FIR 18�625
GIR = = = 26.607cm. Choice (D)
2 × (6�1) 2
70 nc 0�7
   = + = 1.11m. Choice (C)
2 × 100 9�5 100 62. Q = 50m3/sec. f = 1, x = 0.5
1 1
v2  Qf 2  6  50 × 12  6
5 5. SSD = vt + V=  = = 0.842m/s
2 gf  140   140 
22�222 50
   =22.2 (2) + = 116m A= = 59.38m2. Choice (D)
2 × 9�81 × 0�35 0�842
4.28 | Mock Test 2

63. 65.
P P 100 N

L L L 500 mm 500 mm B
P P A
3 3 3

Space diagram is as shown in the figure. 10 Nm


Bending moment at the centre RA RB
L L  1 1  PL
= P × − P × = PL  −  =  Choice (B) RA + RB = 100 N
2 6  2 6 3
Taking moment about A,
64. As there is no shear stress the stresses shown may be
taken as principal stresses RB× 1 + 10 - 100 × 0.5 = 0
p1 = 80 MPa; p2 = -20 MPa ⇒ RB = 50 - 10 = 40 N
p1 − p2 80 + 20 ⇒ RA = 100 - 40 = 60 N
Radius of Mohr’s circle = = Maximum bending moment = 60 × 0.5 = 30 Nm
2 2
 Choice (A)
= 50 MPa Choice (C)
Mock Test 3
Number of Questions: 65 Total Marks: 100

Section – I: General Aptitude


Questions 1 to 5 carry One Mark each. 5. In this special meeting, many individuals
Directions for question 1: Select the correct alternative  (A) (B)
from the given choices. as well as some groups has participated.
  (C)   (D)
1. In a UG and PG College with a total strength of 1000,
26% of the students are girls. There are 520 PGs and 38 Questions 6 to 10 carry Two Marks each.
more male UGs than female UGs. What percent of the Directions for questions 6 to 8: Select the correct alterna-
males are UGs? tive from the given choices.
(A) 45%
(B) 65% 6. The terrorist attacks in Mumbai has raised security
(C) 26% concerns. These attacks have raised serious doubts
(D) 35% about the safety of the residents of Mumbai.
Which of the following can be an inference drawn
Directions for question 2: In the question below, there is a from the above statements? (An inference is something
main statement followed by four statements a, b, c and d. which is not directly stated but can be inferred from the
From the choices, choose the ordered pair where the first given facts)
statement implies the second statement and the two are log- (A) Terrorists attacked common citizens.
ically consistent with the main statement. (B) Terrorist attacked a province of high importance.
2. Unless the Indian government seals the borders illegal (C) The residents of Mumbai are shifting to other
migration in India will not stop. cities.
a. Indian government sealed the border. (D) None of these
b. Illegal migration in India stopped.
7. If , m and n are distinct and a = mb = nc = ℓmn, then
c. Indian government had not sealed the borders.
d. Illegal migration in India will not stop. ab + bc + ca =
(A) ( a + b + c ) (B)
2
(A) cd (B) ab a (b + c)
(C) cb (D) ad (C) abc (D) abc ( a + b + c )
Directions for question 3: In the question below, determine 1
the relationship between the pair of capitalised words and 8. A can complete a job 2 times as big as job X in 25
2
then select the pair of words which has a similar relation- 1
ship to the capitalized words and mark the number of that hours. B can complete a job 3 times as big as X in 42
2
pair as your answer.
1
3. BEAVER : INDUSTRIOUS hours. C can complete a job 4 times as big as X in 49
2
(A) Dog : Watchful
(B) Hyena : Frightening 1 1
hours. D can complete the job 5 times as big as X
(C) Vixen : Waiting 2 2
(D) Fox : Cunning 9
in 54 hours. Who is the most efficient among the
20
Directions for question 4: In the given sentence select the
correct alternative that best explains the meaning of the four?
idiom from the given choices. (A) D (B)
C
(C) B (D)
A
4. The leader of the opposition party had an axe to grind
against the CM ever since he lost the elections. Directions for question 9: In the following question state-
Here ‘axe to grind’ means ments 1 and 6 are respectively the first and the last sentences
(A) a damaging condemnation. of a paragraph and statements (2), (3), (4) and (5) come
(B) an authoritative air. in between them. Rearrange (2), (3), (4) and (5) in such
(C) having a selfish motive for doing something. a way that they make a coherent paragraph together with
(D) an official disgrace. statements 1 and 6. Select the correct order from the given
Directions for question 5: Identify the underlined part that choices and mark its number as your answer.
has an error (of grammar usage, word choice or idiom) and 9. (1) strology is considered to be one of the six limbs of
write the number of that underlined part as your answer. the Vedas.
4.30 | Mock Test 3

(2) Astrology’s inclusion as a subject in universities is Directions for question 10: Select the correct alternative
not a retrograde step and certainly doesn’t deserve from the given choices.
to be derided. 10. Cases of food poisoning have been reported from vil-
(3) It will create a sensation in the West like yoga – lage ‘X’. After a dinner party arranged for 100 peo-
ridiculed by pseudo secularists here but assimi- ple, 68 have been admitted to the hospital, 36 cases
lated by the Europeans and Americans – did. are reported to be out of danger. The food, which was
(4) Astrology attempts to interpret the influence of cooked and stored in an open space for almost 12 hours
heavenly bodies on human affairs. earlier was served after reheating it. Investigation is
(5) It is, in fact, a pragmatic measure considering that going on. A news report.
our society is misguided by some unscrupulous Which of the following can be hypothesized from the
elements in the garb of astrologers who have no above information?
basic understanding of the subject. (A) Late night dinner parties for large number of peo-
(6) Incidentally, when a dead subject like Marxism ple result into food poisoning.
can continue to be taught, why not astrology in (B) Stale food is likely to be the cause of food
which a vast majority of Indians have faith? poisoning.
(A) 2453 (C) Cases of food poisoning need to be handled
(B) 4253 carefully.
(C) 2345 (D) Food poisoning is a matter of chance and no pre-
(D) 4352 ventive measure can be suggested.

Section – II: Civil Engineering


Directions for questions 11 to 65: Select the correct alter- 14. If the trace of a 3 × 3 matrix A with all of its eigenvalues
native form the given choices. 15
being one and the same is , then the determinant of
Questions 11 to 35 carry One Mark each. 2
11. By changing the order of integration, the double inte- A is ______.
1 1− x
125 25
gral ∫ ∫
−1 x −1
f ( x, y )dxdy changes to ______. (A)
4
(B)
4
1− y
125 25
1
(C) (D)
(A) ∫ ∫
−1 1+ y
f ( x, y )dxdy 8 8

0 1− y
15. The partial differential equation of z = eyf(x2 − y2)
is ______.
(B) ∫ ∫
−1 1+ y
f ( x, y )dxdy
(A) zx = py + qx (B) zy = px + qy
1 1− y (C) zx = py − qx (D) zy = px − qy
(C) ∫ ∫
0 1+ y
f ( x, y )dxdy 16. A rapid test to indicate the intensity of pollution in river
water is______
1+ y 1 1− y
0
(A) BOD (B) DO
(D) ∫ ∫
−1 0
f ( x, y )dxdy + ∫
0 0
∫ f ( x, y )dxdy (C) MPN (D) TDS
17. The unit in which both sedimentation and digestion
dy 2x
12. The solution of the differential equation + = 0; processes of sludge takes place simultaneously is ____
dx y (A) Skimming tank (B) Imhoff tank
y(0) = 2 at x = 0.1 obtained by the Runge-Kutta method (C) Deritus tank (D) Digestion tank
of second order with step size 0.1 is ______. 18. Modulus of elasticity of concrete is _____
(A) 1.995 (B) 2.005 1. Tangent Modulus
(C) 1.905 (D) 2.01 2. Secant Modulus
−2
20 3. Proportional to fck
1 3. The value of ∫ 6 dx is ______.
−∞ x 4. Proportional
1
1 1 fck
(A) 4 (B)
8
Which of these statements are correct?
1 7 (A) 1 and 3 only (B) 1 and 4 only
(C) (D)
16 8 (C) 2 and 3 only (D) 2 and 4 only
Mock Test 3 | 4.31

19. The timber preservative “Creosote” belongs to group of 27. Consumptive use refers to loss of water as a result of
(A) Water soluble salts (B) Inorganic solvent (A) Evaporation and transpiration.
(C) Tar oil (D) Organic solvent (B) Crop water requirement.
20. Minimum clear cover (in mm) to the main steel in slab, (C) Evaporation and Infiltration.
beam, column are respectively _______ (D) Evaporation and transpiration from cropped area.
(A) 15,25 and 40 (B) 20, 25 and 30 28. Garret’s diagrams are used to
(C) 20,35 and 50 (D) 10,15 and 25 (A) Separate base flow from total run off.
21. If optimistic, pessimistic and expected times of an (B) Correct inconsistency in rainfall data.
activity are given as 8, 16 and 12 days respectively, find (C) Determine reservoir capacity.
the most probable time of the given activity? (D) Design channels.
(A) 16 days (B) 12 days 29. “Isogonic lines” passes through points of
(C) 8 days (D) 10 days (A) Zero declination (B) Equal declination
22. The maximum longitudinal pitch in bolted joints, sub- (C) Equal dip (D) None of above
jected to tensile forces, (Where t – thickness of the 30. Which of the following closely represents the shape of
plate and D – diameter of the bolt) the earth.
(A) 32D (B) 16D (A) Spheroid (B) Ellipsoid
(C) 32t (D) 16t (C) Oblate spheroid (D) Oblate ellipsoid
23. If the gross bearing capacity of a strip footing of 31. The consistency and flow resistance of bitumen can be
1.2 m wide located at a depth of 2 m in a clay is 340 determined from the following
kN/m2, its net bearing capacity for g = 20 kN/m3 is_____ (A) Ductility test (B) Penetration test
(in kN/m2) (C) Softening point test (D) Viscosity test
(A) 250 (B) 300 32. A velocity potential function is given by f = 3xy. x and
(C) 350 (D) 400 y components of velocity at point (1, 2) is
24. Which of the following statements below are correct? (A) 3, 6 (B) 6, 3
(i) Vibratory rollers are best suited for compacting (C) -3, -6 (D) -6, -3
sandy soil. 33. In a pipe of 300 mm diameter and 700 m length an oil
(ii) Relative compaction is the same as relative of specific gravity 0.8 is flowing at the rate of 0.45 m3/s.
density. If coefficient of friction is 0.00498, head loss due to
(iii) Zero air void line and 100% saturation line are friction (in m) of oil is
identical. (A) 96 (B) 115
(A) (i), (ii), (iii) are correct (C) 124 (D) 136
(B) (i) and (iii) are correct 34. For viscous flow through pipe, Darcy-Weisbach fric-
(C) (i) and (ii) are correct tion factor in terms of Reynold’s number Re is given by
(D) (ii) and (iii) are correct 16 32
(A) (B)
25. A concentrated load of 500 kN acts vertically at a point Re Re
on the soil surface. According to Boussinesq’s equation 48 64
the ratio of the vertical stresses at a depth of 2 m and 4 (C) (D)
Re Re
m is________
(A) 0.5 (B) 0.25 35. State of stress at a point in a stressed body are, sx = 900
(C) 2 (D) 4 MPa, sy = 300 MPa, txy = 400 MPa. Radius of the circle
representing this state of stress is
26. The static indeterminacy of the truss shown below
(A) 300 MPa (B) 400 MPa
is___
(C) 500 MPa (D) 600 MPa

Questions 36 to 65 carry Two Marks each.


36. The directional derivative of f(x, y, z) = xnz − y2 + 40
at (−1, 2, 1) in the direction of the normal to the surface
4x2y + z3 = 4 at (1, −1, 2) is ______.
7 7
(A) (B)
2 2
(A) 0 (B) 1 7 −7
(C) − (D)
(C) 2 (D) 3 2 2
4.32 | Mock Test 3

37. If A and B are two events of the sample space S such kg and 6000 kg respectively; then the number of rivets
3 1 required is _____
that P(B) = and P(A’∩B’) = (where E’ denote the
5 6 (A) 12 (B) 14
 A (C) 16 (D) 17
complement of an event E), then P   is ______. 1
 B′ 
43. For a fixed beam with a concentrated load W at 4 th of
7 2
(A) (B) span from an end; the collapse load is _____
12 3
3 16 Mp 4 Mp
5 (A) (B)
(C) 4 (D) 3L L
12
32 Mp 6 Mp
38. Which of the following functions is/are continuous at (C) (D)
x = 2? (Here [x] denotes the greatest integer less than 3L L
or equal to x). 44. Consider the singly reinforced beam section with fol-
3 x − [ x ]; if x < 2 lowing parameters. Calculate the moment of resistance
 by Limit state method?
I. f(x) = 5 ; if x = 2
 x + 3 ; if x > 2 Grade of concrete used: M30
 Reinforcement: 5 No’s of Fe 415 having dia of 16 mm
Cross section: 200 mm × 350 mm
 x2 + 2x − 8
 2 ; if x ≠ 2 Effective cover: 25 mm
II. g(x) =  x + 3 x − 10 (A) 73 KN-m (B) 87 KN-m
5 ; if x = 2 (C) 93 Kw-m (D) 112 KN-m
 7
45. Results of a water sample analysis are as follows.
(A) I only (B) II only
(C) Both I and II (D) Neither I nor II Concentration
Cation Equivalent weight
(mg/l)
39. If A is a real matrix of order 3 × 3 with trace – 1 and
Na+ 40 23
−2 − 5i as one of the eigenvalues, then the determinant
of A is _____. Mg +2
10 12.2
(A) 63 (B) −63 Ca+2 55 20
(C) 87 (D) −87 K+
12 39
4 0. The Laplace transform of the solution of the initial
Hardness of the water sample in mg/L as caco3 is ___
value problem
(A) 44.8 (B) 89.5
d2x dx  dx  (C) 179 (D) 358
+ 5 − 14x = 0; x(0) = 1 and   =2
dt 2
dt  dt  at t = 0 46. A landfill is to be designed to serve a population of
is ______. 2,00,000 for a period of 20 years. The solid waste (SW)
generated is 2.5 Kg/person/day. The density of un-com-
1 1 pacted SW is 100 Kg/m3 and a compaction ratio of 4 is
(A) (B)
s−2 s+7 recommended. The ratio of compacted fill (i.e; SW +
s+2 s−7 cover) to compacted SW is 1.5. The landfill volume (in
(C) (D) million m3) required is _____
s + 5s − 14
2
s 2
+ 5s − 14
(A) 10.6 (B) 11.6
(C) 12.7 (D) 13.7
41. A slender column is fixed at one end and the other end
is hinged. Euler’s buckling load and effective length 4 7. Two Electro static precipitators (ESPS) are in series.
respectively are. The fractional efficiencies of the up stream and down
stream ESPS for a size dp are 80% and 65%, respec-
π2 EI 2π 2 E I 
(A) 2
,  (B) , tively. What is the overall efficiency of the system for
 2 2 the same dp?
4π 2 E I  π 2 EI (A) 100% (B) 93%
(C) , (D) , 2 (C) 80% (D) 65%
2 2 4 2
4 8. A sample of dry soil weighs 50 gm. Find the volume of
42. A mild steel flat subjected to tensile force of 84 tonnes voids (in ml) if the total volume of the sample is 50 ml
is connected to a gusset plate using rivets. If the force and the specific gravity of solids is 2.65.
required to sheer a single rivet, to crush the rivet and (A) 31.5 (B) 25.2
to tear the plate per pitch length are 5000 Kg, 8000 (C) 15.4 (D) 42.1
Mock Test 3 | 4.33

49. The consistency limits of a soil sample are; 54. Distribution factor for DE in given figure is
Liquid limit = 40% B
Plastic limit = 25%
Shrinkage limit = 5%.
If the specimen of this soil shrinks from a volume of
4L
7.5 cm3 at liquid limit to 4.5 cm3 at the shrinkage limit,
the specific gravity of soil solids is _______
(A) 2.25 (B) 2.75 E
A C
(C) 2.10 (D) 2.02 3L 3L

50. In a consolidation test on a soil, the void ratio of the 4L


sample decreased from 1.25 to 1.10 when the pressure
is increased from 250 kN/m2 to 450 kN/m2. The coeffi-
cient of consolidation of soil if the coefficient of per- D
meability is 8 × 10–8 cm/sec is ____ (m2/year). 12 3
(A) 6.7 (B) 7.7 (A) (B)
49 13
(C) 8.7 (D) 9.7
49
51. A shear vane, 7.5 cm dia and 11.25 cm long, was (C) 1 (D)
48
pressed into soft clay at the bottom of a bore hole. The
shear strength of clay if the torque required for failure 55. The number of Zero force members in the truss shown
was 50 N-m is____(in kN/m2) below is____
(A) 25 (B) 40 P
(C) 50 (D) 65
52. A long natural slope in an over consolidated clay
(C’ = 12 kN/m2, f’ = 30°, γsat = 21 kN/m3) is inclined
at 10° to the horizontal. The water table is at the sur-
face and the seepage is parallel to the slope. The factor
of safety of slope at a depth of 5 m below the slope
is_____ (take γw = 10 kN/m3). (A) 3 (B) 4
(A) 2.12 (B) 2.38 (C) 5 (D) 6
(C) 3.72 (D) 4.24 5 6. The following rainfall data refers to station A and B
53. The retaining wall with backfill conditions are shown which are equidistant from station X
in below figure. The total passive force at bottom of
Long term A x B
the wall is____ (Take water table at level of B and
Normal Annual rainfall in mm 200 250 300
γw = 10 kN/m3).
Annul rainfall in mm for the year 1940 140 P 270
A
The value of P will be.
Ι (A) 250 (B) 220
φ '= 28° (C) 205 (D) 200
C' = 0 3m 57. A direct runoff hydrograph due to isolated storm was
γ = 18 kN/m 3
triangular in shape with a base of 80 h and peak of
B 200 m3/s. If the catchment area is 1440 km2, the effec-
tive rainfall of the storm is
ΙΙ (A) 20 cm (B) 10 cm
φ ' = 22°, (C) 5cm (D) 2 cm
3m
C'= 10 kN/m 2, 5 8. What is the maximum length of an offset so that the
γ = 20 kN/m 3 displacement of a point on the paper should not exceed
C 0.25 mm, gives that maximum error expected is 2.5°
from its true direction and the scale is 1 : 2000
(A) 815 kN (A) 11.46 m (B) 1.48 m
(B) 755 kN (C) 14.32 m (D) 9.82 m
(C) 635 kN 5 9. It is required to set up points on a sloping down ground
(D) 952 kN of line 50 m at every 20 m contour interval. If the staff
4.34 | Mock Test 3

reading over first point is 0.55 m. The staff reading over


next point B is E1
(A) 0.95 m (B) 0.15 m
C1
(C) 1.05 m (D) 0.25 m b
d
60. Sliding considerations for stopped vehicle on super ele- D1
vation. Horizontal curve provide the following bound
on amount of super elevation. (A) 68% and 60% respectively
(A) e > coefficient of rolling friction (B) 80% and 72% respectively
(B) e > coefficient of side friction. (C) 70% and 60% respectively
(C) e > coefficient of rolling friction. (D) 40% and 80% respectively
(D) e > coefficient of side friction.
64. In a smooth inclined pipe of 250 mm diameter carrying
61. A vertical summit curve is formed at the intersection water, point 2 is at a higher elevation of 2 m from point
two gradients +3% and –6%. Design sped of two lane 1. Velocity through the pipe is 1.25 m/s. Pressures at
two way is 80 kmph. Driver reaction time is 2.5 sec. points 1 and 2 are 50 kPa and 20 kPa respectively. Rate
Over the curve to stop the vehicle frictional coefficient of flow (in m3/s) through the pipe and direction of flow
between road and tyres is 0.35. Then the length of the are
summit curve based on SSD is (A) 0.0614, upwards
(A) 296 m (B) 1182m (B) 0.0614, downwards
(C) 73.6 m (D) 1472 m (C) 0.0307, upwards
(D) 0.0307, downwards
62. Consider the following factors:
1. Reaction time 65.
2. Speed A B C
3. Coefficient of longitudinal fraction D
4. Gradient
Which of these factors are taken into account for com- 40 mm 40 kN 30 mmφ
puting braking distance? 30 kN
(A) 1 and 3 (B) 1, 2 and 4
(C) 2, 3 and 4 (D) 2 and 3
500 mm
63. The uplift pressure at points E and D of a straight hori- 250 mm 250 mm
zontal floor of negligible thickness with a sheet pile at In the composite bar as shown above portion AC is
down stream end are 30% and 40% respectively. If the made of steel and portion CD is made of copper. The
sheet pile is at upstream end of floor the uplift pressure two materials are rigidly joined at C. Modulus of elas-
at point D1 and C1 ticities of steel and copper are 2 × 105 N/mm2 and
1.1 × 105 N/mm2 respectively. If the bar is loaded as
shown in the figure, total extension produced in mm is
______.
E C (A) 0.2 mm
b d (B) 0.3 mm
(C) 0.4 mm
D (D) 0.5 mm

Answer Keys
1. D 2. A 3. D 4. C 5. D 6. D 7. C 8. A 9. B 10. B
11. D 12. A 13. B 14. C 15. A 16. D 17. B 18. C 19. C 20. A
21. B 22. D 23. B 24. B 25. D 26. B 27. D 28. D 29. B 30. C
31. D 32. D 33. A 34. D 35. C 36. C 37. A 38. A 39. C 40. A
41. B 42. D 43. C 44. B 45. C 46. D 47. B 48. A 49. C 50. B
51. B 52. B 53. A 54. A 55. B 56. D 57. D 58. A 59. A 60. B
61. B 62. C 63. C 64. A 65. B
Mock Test 3 | 4.35

Hints and Explanations


1. The data is tabulated below 25 25 42 42
= = 10 ; = which is > 10.
Girls Boys Total
1 2�5 1 3�5
2 3
PG  39 481  520
2 2
UG 221 259  480
1 9
49 54
Similarly it follows that 2 > 10 and 20 < 10
Total 260 740 1000
1 1
4 5
Percentage of males who are UG’s is 2 2
259 \ D would take the least time to complete the job.
× 100 = 35% . Choice (D)
740 \ D is the most efficient. Choice (A)
2.   p 9. It would be more appropriate for statement (4) to fol-
Unless The Indian government seals the borders, low statement 1 because it explains the function of
“astrology” which has been introduced in the previous
q sentence. (4) cannot be followed by (3) because of the
illegal migration will not stop� future tense in ‘it will’. Choice (B)
The implications of the above statement will 10. The report says that 68 out of 100 people were admitted
(i) ~p ⇒ q and to the hospital because of the food poisoning. The most
(ii) ~q ⇒ p. appropriate cause, which is stated in the statements, is
It can be CD or BA. Choice (A) stale food. Choice (B)
3. A ‘beaver’s’ nature is to be ‘industrious’ (hard work- 11. Given double integral is <<Eqn3767.eps>>= I, (say)
ing). Among all the options, D is the correct one since Here y varies from y = x – 1 to y = 1 – x
‘cunning’ is the nature of a fox. A dog is ‘faithful’ and And x varies from x = 0 to x = 1
being watchful is its secondary trait. A Hyena is not \ The region of integration is the triangle ABC as
necessarily frightening. ‘Vixen’ (a female fox) and shown in the figure.
‘waiting’ are unconnected. Choice (D) By changing the order of integration, the given integral
becomes,
4. When someone has ‘an axe to grind’ against someone
or something, it means he/she has a ‘false grievance’ or
a selfish motive for doing something. Choice (C)
I= ∫∫
OAB
f ( x, y )dxdy + ∫∫
OBC
f ( x, y )dxdy  → (I)

5. The reference is the ‘some groups’ (in the plural).


Hence, ‘has’ is erroneous and should be replaced with Y
‘have’ as the subject is a plural one. Choice (D)
6. No information is given whom the terrorists targeted.
C (0,1)
Hence, neither (A) nor (B) can be inferred. It cannot be
inferred that the residents are moving out of Mumbai. y = 1–x
X
Hence (C) cannot be inferred. Hence, none can be O B (1,0)
A (0,–1)
inferred. Choice (D)
7. Let  a = mb = nc = lmn = λ y=x–1
1 1 1
 = λ , m = λ and n = λ
a b c

1 1 1
mn = ( λ ) a
+ +
b c Where in triangle OAB, x varies from x= 0 to x = 1 + y
ab + bc + ca and y varies from y = −1 to y = 0
λ1 = λ abc And in triangle OBC,
ab + bc + ca x varies from x = 0 to x = 1 – y
⇒ = 1 ⇒ ab + bc + ca = abc. and y varies from y = 0 to y = 1
abc
 Choice (C) \ By changing the order of integration, we have
1 1− x 0 1+ y
8. Times in which A, B, C and D can complete ‘X’ (in
1 9
I= ∫∫
0 x −1
f ( x, y )dxdy = ∫ ∫ f ( x, y )dxdy
−1 0
49 54
25 42 2 20
hours) are , , and respectively. 1 1− y
1 1 1 1 +∫
2 3 4 5
2
∫ f ( x, y )dxdy  Choice (D)
2 2 2 0 0
4.36 | Mock Test 3

dy 2x Differentiating (2) w.r.t x partially,


12. Given differential equation is + = 0; y(0) = 2 1 ∂z
dx y f’(x2 − y2).2x = y
dy −2x e ∂x
⇒ = p
dx y ⇒ 2x. f’(x2 − y2) = y  → (3)
e
−2x
\ f(x, y) = ; h = 0.1; Differentiating (2) w.r.t y partially
y
∂z
x0 = 0; y0 = y(0) = 2 ey − ze y
∂ y
By Runge-Kutta method of second order, we have f’(x2 − y2) (−2y) =
1
(e y )2
y1 = y0 + [k1 + k2] → (1) q− z
2 ⇒ −2y f’(x2 − y2) = y  → (4)
where k1 = hf(x0, y0) e
p
 −2x0  −2 × 0  2 xf ( x 2 − y 2 )
= (0.1) 
(3) e y
=
h  ⇒ =
 0 
y  2  (4) −2 yf ( x − y ) ( q − z )
2 2

\ k1 = 0 ey
and k2 = hf(x0 + h, y0 + k1) −x p
⇒ =
y q−z
 −2 ( x0 + h)   −2(0 + 0�1) 
=
h  = (0.1)   ⇒ −qx + zx = py
 y0 + k1   2+0 
⇒ zx = py + qx
k2 = −0.01 \ The partial differential equation of (1) is
\ Substituting these in (1), we get zx = py + qx Choice (A)
1 16. Total dissolved solids, TDS indicates the intensity of
y1 = y(0.1) = 2 + [0 + (−0.01)]
2 the pollution in a rapid manner. Choice (D)
= 1.995 Choice (A) 1 7. Imhoff tank Choice (B)
−2
20
−2
20 1 8. Ec = 5000 fck and it is secant modulus
13. We have ∫−∞ x 6 dx = Lim
A→∞ ∫ x 6
−A
dx
 Choice (C)
−2 19. Creosote belongs to group of “ Tar oil” Choice (C)
20 
= Lim 2 0. Min clear cover for slabs, beams, columns, and footings
A→∞ −5 x 5  − A
are 15mm, 25mm, 40mm and 50mm respectively.
 20 20  Choice (A)
= Lim  −
A→∞ −5 ( −2 ) −5 ( − A) 
5 5
 2 1. tO = 8 days
tp = 16 days
1 4  1 1
= Lim  − 5  = − 0 =  Choice (B) te = 12 days
A→∞  8 A  8 8
t 0 + 4t m + t p 8 + 4 (tm ) + 16
14. Let l be an eigenvalue of A. te = = = 12
15 6 6
\ Trace of A = \ tm = 12 days. Choice (B)
2
15 22. The maximum longitudinal pitch in tension 16t or
⇒  l+l+l= (∵ Trace of A = Sum of the eigen- 200mm, whichever is less and in compression is 12t or
2
200mm which ever is less. Choice (D)
values of A)
5 23. qn = qu − γ � D = 340 − 20 × 2 = 300 kN/m 2
⇒ l =
2  Choice (B)
\ The determinant of A = Product of the eigenvalues 2 4. Statement (i) and (iii) are correct. Choice (B)
of A = l × l × l = l3
3 (σ z )1 ( Z2 )2 42 4 × 4
 5 125 25. = = = = 4�  Choice (D)
=  =
 2 8
 Choice (C) (σ z ) 2 ( Z1 )2 22 4

1 5. Given z = eyf(x2 − y2) → (1) 26. Ds = ( m + r ) − 2 j   = (9 + 4 ) − 2 × 6   = 13 − 12


z
⇒ f(x2 − y2) = y  → (2) Ds = 1  Choice (B)
e
Mock Test 3 | 4.37

32. For potential function \ Normal to f (x, y, z) = 0 at (1, −1, 2) is


−∂φ ∇fat(1, −1, 2) = – 8 i + 4 j + 12 k
u = and
∂x
∇φ −8i + 4 j + 12k
−∂φ \ N̂ = =
v = ∇φ ( −8)2 + 4 2 + 122
∂y
−∂ (3xy ) 4( −2i + 4 j + 3 k )
\ u = = -3y   =
∂x 4 4 +1+ 9
−∂ (3xy ) −2 1 3
and v = = -3x N̂ = i+ j + k
∂y 14 14 14
At point (1, 2) \ The directional derivative of f(x, y, z) in the direc-
u = -3 × 2 = -6 tion of the normal to the surface f (x, y, z) = 0 is
v = -3 × 1 = -3 ∇f. N̂
i.e., x component u = -6  −2 1 3 
y component v = -3 Choice (D) = (−4 j − k ).  i+ j+ k
 14 14 14 
3 3. D = 300 mm
−4 3 7
  =0.3 m = − =−  Choice (C)
L= 7 m 14 14 2
Q = 0.45 m3/s
f = 0.00498 3 1
37. Given P(B) = and P(A’∩B’) =
4 fL v 2 f LQ 2 5 6
hf = =
d 2g 3D 5  A  P ( A ∩ B ')
P  =
 B′  P ( B ')
0�00498 × 700 × (0�45)
2

  = P ( A ∪ B) − P ( B)
3 × (0�3)
5
=
P ( B ')
  =96.83 m Choice (A)
16 1 − P ( A '∩ B ') − P ( B)
3 4. Friction coefficient f = =
Re 1 − P ( B)
64 1 3 7
Friction factor = 4 f =  Choice (D) 1− −
Re = 6 5 = 30 = 7
3 2 12
35. Radius of the Mohr’s circle 1−
5 5
= Maximum shear stress
2 S
 σx − σ y  A
=   + τ 2xy B
2 
A∩B
2
 900 − 300 
 + 400
2
=  A’∩Β’
2
= 500 MPa Choice (C)
 Choice (A)
36. Given f(x, y, z) = xnz − y + 40
2
3 x − [ x ] ; if x < 2

∂f ∂f ∂f x 38. (I) Given f(x) = 5 ; if x = 2
\ ∇f = i+ j+ k = nz i − 2y j + k
∂x ∂y ∂z z  x + 3 ; if x > 2

\ ∇fat(−1, 2,1) = −4 j − k
Left Limit = Lt f(x) = Lt (3x – [x]) = 6 − 1 = 5
x→2− x→2−
Let f (x, y, z) = 4x2y + z3 − 4 = 0
\ Normal to the surface f(x, y, z) = 0 is Right Limit = Lt f(x) = Lt (x + 3) = 2 + 3 = 5 and
x→2+ x→2+

∂φ ∂φ ∂φ f(2) = 5
∇φ= i+ j+ k
∂x ∂y ∂x \ Lt f(x) = Lt f(x) = f(2)
x→2− x→2+

    = 8xy i + 4x2 j + 3z2 k Hence f(x) is continuous at x = 2


4.38 | Mock Test 3

\ (I) is true → (1) 42. Rivet value = Minimum of {5000, 8000, 6000} kg
 x2 + 2x − 8 = 5000 kg.
 x 2 + 3 x − 10 ; if x ≠ 2 84 × 1000
Number of rivets = = 16.8 @17 No.
(II) Given g(x) =  5000
5 ; if x = 2
 7  Choice (D)
43.
x2 + 2x − 8
Lt g(x) = Lt 2 W
→ → x + 3 x − 10
( x + 4)( x −2)
= Lt L/4 3L/4
x → 2 ( x + 5)( x −2)

x+4 6 W
= Lt =
x→2 x + 5 7
MP MP
6 θ1 θ2
And g(2) =
7
\ Lt g(x) ≠ g(2) MP
Mp
x→2

Hence g(x) is not continuous at x = 2


W. ∆ = 2 Mpθ1 + 2Mpθ2
\ (II) is not true → (2).
∆ ∆
 Choice (A) θ1 = and θ2 =
L 3L
39. Given −2 – 5i is an eigenvalue of a 3 × 3 real matrix A.
4 4
\ −2 + 5i is also an eigenvalue of A.
Let l be the third eigenvalue of A L 3L
∆ = θ1 . = θ2.
Given the trace of A = −1 4 4
i.e., (−2 – 5i) + (−2 + 5i) + l = −1 \ θ1 = 3 θ2
⇒ −4 + l = −1 ⇒ l = 3 W ∆ = 2Mp (3 θ2 +θ2)
\ The determinant of A = The product of the eigen- 3L
values of A. W. . θ2 = 2Mp .4 θ2
4
= ((−2 – 5i)(−2 + 5i) 3
= 87 Choice (C) 32 M p
\ Wc= . Choice (C)
d2x dx 3L
4 0. Given initial value problem is 2 + 5 − 14x = 0 44.
dt dt
 → (1) 200 mm

 dx 
where x(0) = 1 and   =2  → (2)
 dt  at t = 0 325 mm
Applying Laplace transform on both sides of (1),
 d2x   dx  25 mm
we have L  2  + 5L   − 14L[x] = L[0]
 dt   dt t = 0
b = 200 mm and d = 325 mm
⇒ s2 x − sx(0) − x’(0) + 5(s x − x(0)) − 14 x = 0 π
where x =L[x] Ast = 5 × × 162 = 1005.3 mm2
4
⇒ (s2 + 5s −14) x − s × 1 − 2 − 5 × 1 = 0 Depth of neutral Axis,
⇒ (s2 + 5s −14) x = s + 7 0�87 f y Ast 0�87 × 415 × 1005�3
zu = =
( s + 7) 0�36 fck �b 0�36 × 30 × 200
⇒ x =
( s + 7)( s −2) = 167.988 mm
1 xu , max = 0.48 d = 0.48 × 325 = 156 mm
⇒ x =
s−2 xu > xu max ⇒ over reinforced section
\ The Laplace transform of the solution is So; Mu, lim= 0.36 fck b xu,max(d – 0.42 xu,max)
1 = 0.36 × 30 × 200 × 156(325 – 0.42(156))
x=  Choice (A)
s−2 = 87.43 × 106 N-mm = 87.43 kN-m. Choice (B)
Mock Test 3 | 4.39

 50   +2 50  SR = 1.904%
45. Total hardness =  Mg +2 ×  +  Ca ×  SR is nothing but (Gm)dry
 12�2   20
 1 1
 50   50  \ Ws =  −  × 100
=
10 ×  +  55 × 
 ( m )dry
12�2 20 G G

= 179 mg/l Choice (C)
 1 1
46. Solid waste generated per day = (2 × 105 ) × 2�5 kg/day 5= −  × 100
1�904 G 
= 5,00,000 kg/day G = 2�10  Choice (C)
Total solid waste generated in 20 years = 5 × 105 × 20 ×
365 = 365 × 107 kg K
Volume before compaction 50. Cv =
Compaction ratio = 4 = mv γ w
Volume after compaction
⇒ Volume before compaction, av ∆e
mv = =
365 × 10 7 1 + e (1 + e ) ∆σ1
V = = 365 ×105 m3
100
SW + Cover Cover =
(1�25 − 1�10)
= 1�5 ⇒ 1 + = 1�5

SW SW (1 + 1�25) (200)
Cover volume = 0.5 ×91.25 × 105 = 45.625 × 105 m3 mv = 3�33 × 10 −4 m 2 /kN
\ Total volume = SW + Cover
8 × 10 −8 × 10 −2 m/sec
= (91�25 × 105 ) + (45�625 × 105 ) Cv =
3�33 × 10 −4 m 2 /KN × 9�81 kN/m
=
13.7 × 10 m = 13.7 million m
6 3 3
Choice (D)   = 2�446 × 10 −5 m 2 /sec
47. Given that;
Cv = 7�72 m 2 /year  Choice (B)
η1 = 80% and η2 = 65%
Since, efficiency of up stream ESP is 80%; only 20% of
 H d
particulates are not removed. 51. T = πd 2 Cu  + 
The remaining 20% particulates will face down stream  2 6
ESP, which has an efficiency of 65% T 50 × 100 N − cm
So; particulates removed by down stream ESP = 65% Cu = =
 H d   11�25 7�5 
of 20% πd 2  +  π × (7�5)2 ×  +
 2 6  2 6 
65
= 20 × = 13% N
100 Cu = 4�11
Since they are connected in series, overall efficiency cm 2
= 80% + 13% = 93% Choice (B)    =4.11 × 104 N/m2
Wd 50 Cu = 41.1kN/m2 Choice (B)
48. γ d = = = 1 gm/cc
V 50 52. Seepage parallel to the slope,
γ ws C 1 + γ 1 z cos 2 i tan φ1
G= s = F=
γ w vs γ w γ sat Z cos i sin i
ws 50
⇒ vs = = 12 + ( 21 − 10) × 5 cos 2 10 0 tan 30 0
G γ w 2�65 × 1 = = 2.38.
21 × 5 × cos10 × sin10
Vs = 18.868 cm 3

V = Vv + Vs  Choice (B)
Vv = V – Vs = 50 – 18.868 1 + sin φ1
1 + sin 28
Vv = 31.132 ml Choice (A)
53. (k )p 1 =
1 − sin φ 1
=
1 − sin 28
= 2�77

  VL − VS   1 + sin 22
  7�5 − 4�5  

 V S

   4�5   (k )p 2 =
1 − sin 22
= 2�197
49. SR =   × 100 =   × 100
WL − WS  40 − 5  At top
   
   
  Z =0;
4.40 | Mock Test 3

σv = γ z = 0 55. The Zero force members are marked below:

( P ) = ( Kp ) σ
p A 1 v + 2C1 ( Kp)1
P

   = 0 + 2 × 0 × 2�77 = 0
At B
Just above:
 Choice (B)
Kp1 = 2�7
250  140 270 
56. P = + = 200 mm  Choice (D)
(σ v ) = ( γ )z = 18 × 3 = 54 kN/m2 2  200 300 

 ( )  = 2�77 × 54 + 2 × 0 ×
 Pp
B at top
2�77 57.
200m3/s
( P ) = 149�58 kN/m
p B
2

Just below Q
( Kp)2 = 2�197 ; C2 = 10 kN/m2 80 h
T
(σ v ) = ( γ ) × z = 18 × 3 = 54 kN/m2 time


 Pp( ) B  below = (2�197) × 54 + 2 × 10 × 2�197
 Total runoff = Area of triangle
1
      = 148�28 kN/m 2 = × 80 × 3600 × 200
2
At C(i.e; at Bottom): = 28�8 × 106 m3
(σ v ) = 18 × 3 + (20 − 10) × 3 = 84 kN/m2 Area of catchment = 1440 cm2

 ( )
 Pp  = (2�197) × 84 + 2 × 10 2�197 + 3 × 10
c
\ Effective rainfall
Total runoff 28�8 × 106
     = 244�192 = = = 0�02 m
Area of Catchment 1440 × 106
o
=2 cm Choice (D)
3m
5 8. S = 2000
148.28 149.58 Displacement of field = L sinα
3m L sin α
244.192
∴ Displacement on the paper =
S
Pressure diagram L sin (2�5) 0�25 × 2000
 0�25 = ⇒L= = 11462 mm
Total Passive force 2000 sin 25
1 148�28 + 244�192    = 11�462 m  Choice (A)
= × 149�58 × 3 +   ×3
2  2  59. Difference between two consecutive points is
= 813.078 kN. Choice (A) 1
× 20 = 0�4 m
54. 50

Relative K Staff reading at A = 0.55 m staff reading at B should be


Joint Members ∑K D.F =
stiffness, K ΣK 0.4 m lower than A (staff reading increases by 0.4 m)
EB i.e; = 0.55 + 0.4 = 0.95 m. Choice (A)
E 3Ι 3Ι 49 I Ι × 48 L 12
= = 6 1. Design speed V = 80 kmph =22.22m/s
4 ( 4 L ) 16 L 48 L 4 L × 49 Ι 49
v2 (22�22)2

EC Ι SSD = vt + = ( 22�22 × 2�5) +


2 gf 2 × 9�81 × 0�35
3L
   =127.45 m
ED Ι Safe SSD = 2 × SSD = 2 × 127.5 = 255 m
4L deviation angle N= 3 – (-5) = 8 %
length of summit curve
EA
3Ι 3Ι
=  8 
4 (3 L ) 12 L NS 2  100 
(255)2
L= = = 1182 m  Choice (B)
 Choice (A) 4�4 4�4
Mock Test 3 | 4.41

63. At Point E : hE = 100 – 30 = 70% Force acting = 30 kN (Tensile).


At Point D : hD = 100 – 40 = 60%. Choice (C) Portion BC (consider section xx)
64. x
(2) C
D
30 kN
30 kN
2m

(1) x

D = 250 mm = 0.25 m Force acting = 30 k N (Tensile).


Z2 - Z1 = 2 m Portion AB (consider section xx)
V1 = V2 = 1.25 m/s x
p1 = 50 k Pa B C
p2 = 20 k Pa D
70 kN
π × (0�25)
2 30 kN
πD 2 40 kN
A1 = A2 = = = 0.0491 m2
4 4
Rate of flow = A1V1 x
= 0.0491 × 1.25
= 0.0614 m3/s Force acting = 70 kN (Tensile)
PL
Total energy difference Extension produced = ∑
P − P V 2 − V22 AE
E1 - E2 = 1 2 + 1 + ( Z1 − Z 2 )
w 2g 30 × 103 × 500 30 × 103 × 250
= + +
π π
(50 − 20)103 (30)2 × 1�1 × 105 (40)2 × 2 × 105
= +0−2 4 4
9810
=1.0581 m 70 × 103 × 250
As E1 > E2, the flow is upwards Choice (A) π
(40)2 × 2 × 105
6 5. Portion CD 4
C D
30 kN 4 × 250  30 × 2 30 × 1 70 × 1 
30 kN = 2 
+ + 2 
π × 10 1�1 × (30 ) 2 × ( 40 )
2 2
2 × ( 40 ) 
Force acting = 30 kN (Tensile). = 0.2924 mm Choice (B)
Mock Test 4
Number of Questions: 65 Total Marks: 100

Section – I: General Aptitude


Questions 1 to 5 carry One Mark each. Questions 6 to 10 carry Two Marks each.
Directions for questions 1 to 3: Select the correct alterna- Directions for questions 6 to 7: Select the correct alterna-
tive from the given choices. tive from the given choices.
1. A bag contains a total of 12 kg of wheat, which is a 6. ▪ ▪ ▪ ▪ ▪ ▪
mixture of three varieties of wheat x, y, z. If 4 kg of ▪▪▪▪▪▪
wheat comprising x and y in the ratio 3 : 1 and 12 ▪▪▪▪▪▪
kg of wheat comprising y and z in the ratio 1 : 3 are ▪▪▪▪▪▪
poured into the bag, the resulting wheat mixture in the ▪▪▪▪▪▪
bag will have x, y and z in the ratio 3 : 4 : 9. Find the ▪▪▪▪▪▪
ratio of x, y and z in the wheat mixture presently in the The number of triangles can be formed, whose vertices
bag. lie on the dots shown in the above pattern is ________
(A) 4 : 6 : 11 7. Most scientists now believe that the near extinction of
(B) 3 : 5 : 13 the cobra in the North Indian forests was due to the
(C) 3 : 4 : 9 extensive hunting down of the poor creature by the tri-
(D) Cannot be determined bals of the forest. During the 40s and the 50s cobra was
in great demand for its medicinal use which the tribals
2. Mohan and Sohan can complete a job in 12 days and exploited greatly, the scientists say.
15 days respectively. With the help of Rohan, they com- Which of the following, weakens the above argument?
pleted the job in 3 days. The wage paid to Rohan for (A) During the 40s and the 50s claws, antlers, horns,
completing his part of the job was ` 320 more than the tusks, and other animal parts were in great demand
total wage paid to the other two for completing their for their medicinal uses which prompted most tri-
parts of the job. Find Rohan’s wage. bals to kill animals.
(A) ` 2340 (B) ` 1760 (B) During 40s and the 50s, the government built a
(C) ` 2520 (D) ` 1540 dam in the vicinity of North Indian forests.
(C) The extensive government sponsored logging
3. Six wrestlers – A, B, C, D, E, and F participated in a
operations of the 40s and the 50s in the North
competition called “who is stronger” and at end of the
Indian forests led to the immediate extinction of
competition they were ranked based on their strength,
rodents, a major prey of snakes.
C and B are less stronger than D and they have atleast
(D) The government undertook afforestation to revive
one wrestlers who is weaker than them. F is ranked
the forests to prevent extinction of snakes in the
second and E is ranked third in strength. How many
North India but failed miserably.
wrestlers are stronger than A?
Directions for question 8: In the following question, a part of
(A) 2 (B) 3
a sentence is in bold. Indicate which among the answer choices
(C) 1 (D) 5
given below the sentence can best replace the bold part to make
it grammatically and logically correct. If the sentence is correct
Directions for question 4: The question has a word followed
by four choices. From the choices, identify the one which as it is, mark (D), i.e., ‘No correction required’ as the answer.
is opposite in meaning (antonym) to the question word and 8. The next batch in which I am placed, will start 9 o’
mark its number as your answer. clock in the mornings.
(A) would start at 9 o’ clock in the mornings
4. RISIBLE
(B) starts at 9 o’clock in the morning
(A) Grave (B) Funny (C) will have started at 9 o’clock in the morning
(C) Jovial (D) Insecure (D) No correction required
Directions for question 9: In the following question given
Directions for question 5: Fill in the blank with the most
below, statements 1 and 6 are respectively the first and the
suitable options from the choices given.
last sentences of a paragraph and statements (2), (3), (4) and
5. In places, the trunks of the fallen trees have been (5) come in between them. Rearrange (1), (2), (3) and (4)
______ white and ghostly by the strong sunlight. in such a way that they make a coherent paragraph together
(A) bleached (B) stained with statements 1 and 6. Select the correct order from the
(C) considered (D) furnished given choices and mark its number as your answer.
Mock Test 4 | 4.43

9. (1) With the decline of feudalism in ancient society, Directions for question 10: Select the correct alternative
some important developments like merchant cap- from the given choices.
ital, emergence of wage labour, putting out sys- 10. The killings of Dalits in Uttar Pradesh proves that
tem and enclosure movement set the stage for the things have not improved over the years and the plight
industrial revolution. of Dalits is the same everywhere. The tragedy in UP is
(2) A new cyclical pattern of growth took place. that despite the state having a Dalit Chief Minister who
(3) Increased agricultural production and new tech- is seen by many as a future Prime Minister, the atroci-
nology further contributed to its growth. ties against Dalits continue.
(4) Though industrial development was marked by Which of the following can be inferred from the above
periods of depression, gradually this problem was statement?
overcome. (A) A Dalit Chief Minister understands the agony of
(5) With industrial development, the composition of Dalits better.
capital also underwent changes. (B) The chances of a Dalit Chief Minister becoming a
Prime Minister will not be paralysed due to these
(6) Soon the increased industrial production gave rise
events.
to capitalism and new social classes.
(C) Even a Dalit Chief Minister cannot prevent Dalits
(A) 2345 (B) 4532
from being derogated.
(C) 3524 (D) 5234
(D) None of the above.

Section – II: Civil Engineering


Directions for questions 11 to 65: Select the correct alter- 16. The contact pressure for a rigid footing resting on clay,
native form the given choices. at the centre and the edges are respectively _____.
Questions 11 to 35 carry One Mark each. (A) Maximum and zero
(B) Maximum and Minimum
11. Which among the following types of matrices is always
(C) Zero and Maximum
non – singular?
(D) Minimum and Maximum
(A) Symmetric matrix (B) Idempotent matrix
(C) Involutory matrix (D) Nilpotent matrix 17. The width and depth of a footing are 2 and 1.5 m respec-
tively. The water table at the site is at a depth of 2.5 m
12. If the Laplace transform of f(t) is F(s), then which of below the ground level. The water table correction factor
d
the following is necessary for L [ sF ( s) ] = ( f (t ) ) ?
−1
for calculation of bearing capacity will be _____
dt (A) 1.0 (B) 0.75
(A) F(0) = 0 (B) f(0) = 0 (C) 0.50 (D) 0.25
 dF   df  18. The creep is caused due to _____.
(C)  (D)
 ds  at s = 0  
dt at t = 0 (A) Initial Consolidation
(B) Primary Consolidation
13. The greatest value of C such that the partial differential (C) Secondary Consolidation
equation (D) Tertiary Consolidation
∂2 u ∂2 u ∂2 u ∂u ∂u 19. When water table rises _____
3 −6 + C 2 − 7x + 5x2 y + 3u = 0
∂x 2
∂x ∂y ∂y ∂x ∂y (A) Total stress increases, effective and pore pressures
is not hyperbolic is _____. decreases.
(B) Effective stress decreases, total and pore pressures
14. Which of the following is a sufficient condition for the increases
cross product of two vector point functions F and G (C) Pore pressure decreases, total and effective pres-
to be solenoidal? sure increases
(A) Both F and G have to be solenoidal. (D) Total stress decreases, effective and pore pressure
(B) F has to be solenoidal and G has to be increases.
irrotational. 20. If a R.C. beam fails in bond, its bond strength can be
(C) Both F and G have to be irrotational. increased by _____
(A) Increasing the depth of beam
(D) F has to be irrotational and G has to be solenoidal.
(B) Using thinner bars but more in numbers
15. The life time of bulbs manufactured by a company is (C) Using thicker bars but less in numbers
normally distributed with mean 450 hours and standard (D) Providing vertical stirrups
deviation 30 hours. If a random sample of 36 bulbs has 21. A 2 – way RCC slab is simply supported on all its edges
an average life time of 440 hours, then the test statistic with corners not free to lift. The area of steel in shorter
in the process of testing the average life of bulbs is 450 span is 812 mm2. The torsional reinforcement required
hours or not is _____. at corner is _____.
4.44 | Mock Test 4

(A) 0 mm2 (B) 609 mm2 Using this data available, identify the correct statement
(C) 406 mm2 (D) 203 mm2 from the following.
22. The moisture content in a well seasoned timber (A) The given fine grained soil is inactive soil.
is _____. (B) The given fine grained soil is normal active soil.
(A) 4 to 6% (B) 10 to 12% (C) The given fine grained soil is active soil.
(C) 15 to 20% (D) 50% (D) Data insufficient.
23. The most appropriate method to specify the concrete 3 4. For a sample of water with ionic composition, the car-
mix is by _____. bonate and non – carbonate hardness concentrations (in
(A) Nominal mix ratio (B) Design mix ratio mg/L as CaCO3) respectively are :
(C) Degree of control (D) Grade of concrete meq/L 0 4 5 7
24. A combined sewer is the one which transports
Ca2+ Mg2+ Na2+
(A) domestic sewage and storm water
(B) domestic sewage and industrial wastes HCO3– SO42–
(C) domestic sewage and over head flow
(D) domestic sewage, industrial wastes and storm water. 0 3.5 7
meq/L
25. The main constituents of gas generated during anaero- (A) 200 and 50 (B) 175 and 75
bic digestion of sewage sludge are (C) 75 and 175 (D) 50 and 200
(A) carbon dioxide and methane
3 5. A wastewater sample has an initial BOD of 222 mg/l.
(B) methane and ethane
(C) carbon dioxide and carbon monoxide The first order BOD decay coefficient is 0.4/day. The
(D) carbon monoxide and nitrogen BOD consumed (in mgu) in 5 days is
(A) 150 (B) 192
26. The type of surveying in which the curvature of the (C) 30 (D) 50
earth is taken into account is called.
(A) Geodetic surveying Questions 36 to 65 carry Two Marks each.
(B) Plane surveying 36. If g(x) = (x + 5) (x + 2) (x – 1) (x – 6) and f(x) = 
(C) Preliminary surveying ∫ g ( x)dx, then the number of local maxima for f(x)
(D) Topographical surveying is _____.
27. A lysimeter is used to measure 37. If the system of linear equations
(A) Infiltration (B) Evaporation a1 x1 + b1 x2 + c1 x3 = 0
(C) Evapotranspiration (D) Radiation a2 x1 + b2 x2 + c2 x3 = 0
28. The standard project flood is a3 x1 + b3 x2 + c3 x3 = 0
(A) same as the probable maximum flood (where ai, bi and ci, i = 1, 2, 3 are all constants) has only
(B) same as the design flood a trivial solution, then the number of solutions of the
(C) smaller than the probable maximum flood system of linear equations
(D) larger than the probable maximum flood by a fac- a1 x1 + b1 x2 + c1 x3 = 5
tor implying safety factor a2 x1 + b2 x2 + c2 x3 = 3
a3 x1 + b3 x2 + c3 x3 = 2 is _____.
29. Stopping slight distance and frictional coefficients are
(A) 0 (B) 1
(A) directly proportional to each other
(C) infinitely many (D) Cannot be determined
(B) inversely proportional to each other
(C) unrelated 38. If X1 and X2 are two independent normal variates with
(D) either directly (or) inversely proportional to each means 100 and 60 respectively and variances 16 and 9 re-
other depending on nature of pavement. spectively, then which of the following statements is true?
(A) X1 – X2 is a normal variate with mean 40 and
30. The traffic survey data plotted by means of desire lines is
standard deviation 7.
(A) Accident (B) Classified volume
(B) X1 – X2 is a normal variate with mean 40 and
(C) Origin and destination (D) Speed and delay
standard deviation 5.
31. The shape of stop sign as per IRC : 67 – 2001 is (C) X1 – X2 is a normal variate with mean 20 and
(A) Circular (B) Triangular standard deviation 1.
(C) Octagonal (D) Rectangular (D) X1 – X2 need not be a normal variate.
32. A thin walled spherical shell of diameter 500 mm and 39. The area bounded by the curve y = x3 and the line y = x
thickness 8 mm is subjected to an internal pressure of 6 in square units is _____
MPa. The average hoop stress (in N/mm2) is ______. (A) 0.25 (B) 0.5
33. A fine grained soil has 40% (by weight) silt content. (C) 0.75 (D) 1
The Liquid limit, plastic limit and shrinkage limit 40. Which of the following statements are true with refer-
of soil are given as 45%, 30% and 18% respectively. ence to the evaluation of a definite integral
Mock Test 4 | 4.45

Ι = ∫ f ( x )dx by numerical integration methods?


a

I. If [a, b] is divided into 5 subintervals, then we can


0.4 m dia. piles
use any of the Trapezoidal rule or Simpson’s rule 10 m Clay soil
to evaluate the definite integral I.
II. If y = f(x) represents a parabola, then the value of Cu = 25 KN/mV
φu = 00
1
I obtained by the Simpson’s rd rule and by
3
direct integration are same.
1.2 m c/c
III.  While evaluating by the Trapezoidal rule, the
accuracy can be increased by dividing [a, b] into
large number of sub intervals.
(A) All of I, II and III (B) Only I and II 1.2 m c/c
(C) Only II and III (D) Only I and III (A) 2861 KN (B) 2681 KN
4 1. Match List - I with List - II (C) 2176 KN (D) 1874 KN
List – I List – II 46. During sub surface investigation of the soil, the follow-
a. Coriolis effect 1. Rotation of earth ing record of number of blows were given, which was
b. Fumigation 2. Lapse rate and vertical
obtained from the standard penetration test conducted
temp. profile at certain depth.
c. Ozone layer 3. Inversion Penetration depth (cm) Number of blows
d. Max. mixing depth 4. Dobson 0 – 7.5 3
(mixing height)
7.5 – 15 3
a b c d a b c d 15 – 22.5 6
(A) 2 1 4 3 (B) 2 1 3 4
22.5 – 30 6
(C) 1 3 2 4 (D) 1 3 4 2
30.0 – 37.5 8
42. The tests on soil determines, following properties of a
37.5 – 45 7
the particular sample.
Degree of saturation = 85% Assuming water table at ground level, soil as fine sand
Water content = 12% and correction for overburden pressure as 0.926; the
Specific gravity = 2.65 corrected value of ‘N’ for the soil would be _____.
If the factor of safety (FOS) against the quick sand con- (A) 27 (B) 21
dition is adopted as 2.5; the hydraulic gradient provided (C) 20 (D) 15
will be _____. 47. A 1 hr rainfall of 10 cm has teturn period of 50 yr. The
(A) 3 (B) 2.52 1 hour of rainfall 10 cm (or) more will occur in each of
(C) 0.48 (D) 3.48 two successive year is
4 3. Two straight lines intersect at an angle of 600. The (A) 0.04 (B) 0.2
radius of a curve joining the two straight lines is 600 m. (C) 0.02 (D) 0.0004
The length of long chord and mid – ordinates in meters 4 8. A direct runoff hydrograph due to an isolate storm with
of the curve are an effective rainfall of 2 cm was trapezoidal in shape
(A) 80.4, 600 (B) 600, 80.4 as shown in figure. The hydrograph corresponds to a
(C) 600, 39.89 (D) 49.89, 300 catchment area (in sq. km) of
44. Match the following: 6 hrs
Group – I Group – II
P. Alidade 1. Chain surveying 90
cumecs
Q. Arrow 2. Leveling
R. Bubble tube 3. Plain table surveying
S. Stadia hair 4. Teodilite surveying
P Q R S P Q R S
(A) 3 2 1 4 (B) 2 4 3 1
(C) 1 2 4 3 (D) 3 1 2 4 70 hrs
4 5. What is ultimate capacity of the pile group shown in (A) 790.2 km 2
(B) 615.6 km2
figure assuming the group to fail as a single block (C) 599.4 km2 (D) 435.3 km2
4.46 | Mock Test 4

49. An outlet irrigates an area of 40 ha. The discharge (L/S) B C


required at this outlet to met the evaportransiration
requirement of 40 mm occurring uniformly in 40 days
neglecting other field losses is _______. 5m

50. At a reclamation site having soil strata as shown in


the figure, 4 m thick layer of fill material is to be laid E
A
instantaneously on the top surface. If the coefficient of D
volume compressibility, mv for clay is 2.2 × 10–4 m2/ 5m 5m
KN, the consolidation settlement of clay layer due to
56. From the given network, the Inference float and inde-
placing fill material will be _____.
pendent float for the activity 5 – 4 will be ______
3
3 4
4m γ bulk = 20 KN/ m 2 Fill material 2
2

2
1 6
5m Silty sand
γ sat = 19 KN/ m2
3 3
2 5
4
10 m Clay
γ sat = 18 KN/ m2
(A) Negative and Positive (B) Both positive
(C) Both negative (D) Both equals to zero
57.
(A) 211 mm (B) 189 mm
(C) 176 mm (D) 142 mm P = 400 kN
×
51. A homogeneous dam is 21.5 m high and has a free
board of 1.5 m. A flow net was constructed and the fol- 300 mm O
lowing results were observed.
No. of potential drops = 12
No. of flow channels = 3
A
The discharge/m length of the dam if the coefficient
of permeability of the dam material is 2 × 10–6 m/sec
On a short masonry column of circular cross section
is _____ × 10–5 cumecs/m
of diameter 300 mm a load of 400 kN acts axially at a
52. An isolated T – beam has effective span of 10 m. Actual radial distance of 100 mm from centre as shown in the
width of flange is 2 m, width of web is 250 mm; then figure. Strain developed at point A in N/mm2 is
effective flange width will be _____. (A) 9.43(compressive) (B) 9.43(Tensile)
(A) 1.36 m (B) 1.57 m (C) 20.75(compressive) (D) 20.75 (Tensile)
(C) 1.82 m (D) 2 m 58. A rod of length 1.2 m tapered from 100 mm. diameter
53. During CBR test, the load sustained by a remolded soil to 50 mm diameter. If a torque of 3 kN m is applied, the
specimen at 5 mm penetration is 50 kg. The CBR value angular rotation of free end (in radians) is.
of the soil will be (Take modulus of rigidity = 80 kN/mm2)
(A) 10% (B) 5% 5 9. For a Pelton wheel, head at the base of the nozzle is 80 m,
(C) 3.6% (D) 2.4% discharge through nozzle is 0.3 m3/s at a jet velocity of
38.2 m/s. Loss of power in the nozzle in kW) is
54. For a road with camber of 3% an design speed of 80
(A) 12.76 (B) 16.55
kmph, the min radius of a curve beyond which no super
(C) 18.49 (D) 20.36
elevation is needed is
(A) 1680 m (B) 948 m 60. For a surface of area A immersed in a liquid of specific
(C) 406 m (D) 280 m weight w, if x is the depth of centre of area and h is
the depth of centre of pressure from the liquid surface,
55. Determine the vertical displacement of joint D (in mm)
the total pressure acting on the surface is given by
if member BC of the pin – jointed frame as shown
below is long by an amount 4 × 10–3 from the original (A) wAx (B)
wAh
length 5 m. All the members of frame have same AE 2
wA
(C) wAx (D)
(x + h )
value [AE = 10 × 100 kN]
2
Mock Test 4 | 4.47

61. If velocity potential satisfies, the Laplace equation, it is 100 ISF 12


a case of
(A) unsteady, compressible and rotational flow
(B) steady, incompressible and irrotational flow.
(C) unsteady, incompressible and rotational flow.
(D) steady, incompressible and rotational flow.
62. A piping system consists of 3 pipes of lengths 1000 m, (plug weld) T
800 m and 600 m arranged in series. Diameters of the
20 mm
pipes are 750 mm, 600 mm and 450 mm respectively.
If the system is to be replaced by an equivalent pipe of
diameter 600 mm, length of the equivalent pipe (in m)
58 mm 10 mm
will be _________
63. A close coiled helical spring has to carrying a load of (A) 105 (B) 212
800 N. Mean diameter of the spring is to be 75 mm. If (C) 441 (D) 145
the allowable shear stress is 100 N/mm2 wire diameter  4 3
65. The stiffness matrix of a beam is given as k  .
required for the spring (in mm) is.  3 5
(A) 9.64 (B) 11.52 The flexibility matrix is _____.
(C) 13.47 (D) 15.26 k  5 −3 11  5 −3
(A)   (B)
64. The service load permitted on the connection shown 11  −3 4  k  −3 4 
for the fig. below in kN is _____. Assume field welding 1  5 −3  5 −3
and Fe 410 steel. (C)   (D)
11k  
11 × k  −3 4   −3 4 

Answer Keys
1. C 2. B 3. D 4. A 5. A 6. 6800 7. C 8. B 9. B 10. D
11. C 12. B 13. 3 14. C 15. –2 16. D 17. B 18. C 19. B 20. B
21. B 22. B 23. D 24. D 25. A 26. A 27. C 28. C 29. B 30. C
31. C 32.  93.25 to 94.25 33. A 34. B 35. C 36. 2 37. B 38. B 39. B
40. C 41. D 42. C 43. B 44. A 45. C 46. C 47. D 48. B
49. 0.925 lit/sec 50. C 51. 1 52. A 53. D 54. B 55. 2 mm 56. D 57. B
58.  0.02135 to 0.02145 59. B 60. A 61. B 62.  3655.5 to 3656.5 63. B 64. A
65. C

Hints and Explanations


1. In the conditional case (as opposed to the actual case) Let the total wage paid to the three be `T.
the 4 kg mixture will have 3 kg of x and 1 kg of y. Also 11 T T
the 12 kg mixture will have 3 kg of y and 9 kg of z. T = + + 320
20 4 5
\ Total quantities of x, y and z added are 3 kg, 4 kg ⇒ T = 3200
and 9 kg respectively, i.e., in the quantity added, (if it is 11
added), the ratio of x, y, z is 3 : 4: 9. In the final mixture T = 1760
20
also, it is 3 : 4 : 9. Therefore, the ratio in the present
mixture also, it has to be 3 : 4 : 9. Choice (C) \ Wage of Rohan = ` 1760. Choice (B)
3. E and F are ranked 2nd and 3rd. There is at least one
2. Part of the job completed by wrestler weaker than B, C and D. This indicates that
 1 1 A is the weakest among all. Hence, five wrestlers are
(i) Mohan = 3   =
 12  4 stronger than A. Choice (D)
4. Something that is hilarious, funny or ludicrous is said
 1 1 to be ‘risible’. The antonym of ‘risible’ is ‘grave’ or
(ii) Sohan = 3   =
 15  5 serious. Choice (A)
5. The blank is best filled by ‘bleached’, meaning, ‘make
 1 1  11
(iii) Rohan = 1 –  +  = the appearance white’. Other options fall short of rep-
 4 5  50
resenting the blank properly. Choice (A)
4.48 | Mock Test 4

6. Number of combinations of three dots which can be taken ∴ B is idempotent but not non – singular
is 36C3. But for some combinations, all the 3 dots may be Also, we know that every nilpotent matrix is a singular
collinear, and we cannot form a triangle. In the given ar- matrix. Choice (C)
rangement, by joining the dots we will have the following:
12. We know that if L[f(t)] = F(s), then
(1) 6 horizontal lines having 6 points each
d
(2) 6 vertical lines having 6 points each L−1 [ sF ( s) ] = ( f (t ) ) only if f(0) = 0. Choice (B)
(3) 2 diagonal lines having 6 points each dt
(4) 4 diagonal lines having 5 points each 13. Given PDE is
(5) 4 diagonal lines having 4 points each and ∂2 u ∂2 u ∂2 u ∂u ∂u
(6) 4 diagonal lines having 3 points each. 3 2 −6 + C 2 − 7x + 5x2 y + 3u = 0
∂x ∂x ∂y ∂y ∂x ∂y
Number of triangles which can be formed
 → (1)
= 36C3 – [6(6C3 + 6C3) + 2(6C3) + 4 (5C3 + 4C3 + 3C3)]
= 7140 – [14(20) + 4(15)] = 6800. Ans: 6800 Comparing (1) with the general second order linear
7. The correct answer is C. To weaken an argument, we partial differential equation, we have
should show the assumption in the argument to be false. A = 3, B = –6, C = C
The assumption in the given argument is that there is no For (1) not to be hyperbolic, we have
other cause for the near extinction of the cobra. If we B2 – 4AC ≯ 0
prove this wrong, it weakens the argument. Choice (C), ⇒ B2 – 4AC ≤ 0
by showing an alternative cause, does this. Others are ⇒ (–6)2 – 4 × 3 × C ≤ 0
either irrelevant or out of scope. Choice (C) ⇒ 36 – 12C ≤ 0
8. We use the verb in the simple present tense for a future ⇒ C – 3 ≤ 0
event that is part of a fixed time table. Hence, choice
⇒ C ≤ 3
(B) is apt. The reference is to a specific morning and
hence ‘mornings’ is incorrect. Choice (B) ∴ The greatest value of C such that (1) is not hyper-
9. Statement 1 ends with ‘… set the stage for industrial bolic is 3 Ans: 3
revolution’. So it is followed by (4) which talks of 14. F × G is solenoidal
‘industrial revolution.’ (2) and (3) do not talk of indus- ⇒ Div ( F × G ) = 0
tries. Though (5) talks of ‘industrial development’, the ⇒  G . curl F – F . Curl G = 0
word ‘also’ indicates it follows something else. The
sentences 4532 in that order tell us chronologically So, F × G is solenoidal when curl F = 0 and
the steps that followed the industrial revolution and the curl G = 0
growth of capitalism. Choice (B) i.e., Both F and G have to be irrotational. Choice (C)
10. Choice (A) is an assumption of the people / the writer. 1 5. Population mean = µ = 450 hours
Choice (B) is false. It is evident that the Chief Minister
failed in his duties so it will hamper his chances of Population standard deviation = σ = 30 hours
becoming the Prime Minister. ‘Will not’ in (B) rules Sample mean = x = 440 hours
it out. (C) is not an inference as it is openly stated that Sample size = n = 36
even a Dalit CM could not stop the killing of Dalits. x−µ
The last word of the paragraph ‘continue’ suggests that ∴ Test statistic = Z =
σ
in the past too Dalits were killed. Among the given sen-
n
tences none of them qualify as inferences. Choice (D)
440 − 450 −2
11. We know that, a matrix A is an involutory matrix, if = = × 36
A2 = I (OR) A–1 = A  30  6
 
∴ Every involutory matrix is non – singular 36 
Counter Example for (A): ∴ Z = –2 Ans: –2
4 6 16.
Let A =  
6 9
Clearly, AT = A but A = 0
∴ A is symmetric but not non – singular
Counter Example for (B):
3 3
Let B =  
 −2 −2 
Clearly, B2 = B but B = 0 Minimum at centre, Maximum at edges. Choice (D)
Mock Test 4 | 4.49

17. 35. KD = 0.4 × 0.434 = 0.1736


BOD5 = L[1 – (10)–KDt]
= 222[1 – (10)–5 × 0.1736]
1.5 m
= 192 mg/L
2.5 m
BOD remaining = 192 mg/L
Hence, BOD consumed = 222 – 192 = 30 mg/L
2m  Choice (C)
1m
36. Given g(x) = (x + 5) (x + 2) (x – 1) (x – 6) and f(x)
WT = ∫ g ( x )dx
⇒ g(x) = f1(x) = (x + 5) (x + 2) (x – 1) (x – 6)
1 f1(x) = 0 ⇒ (x + 5) (x + 2) (x – 1) (x – 6) = 0
Correction factor = 0.5 + 0.5 × = 0.75 Choice (B)
2 ∴ The critical values of f(x) are –5, –2, 1 and 6
19. If water table rises; Total pressure and pure water pres- Also, f1(x) > 0 for x < –5
sure increases but effective pressure decreases. f1(x) < 0 for –5 < x < –2
 Choice (B) f1(x) > 0 for –2 < x < 1
3 f1(x) < 0 for 1 < x < 6
21. Torsional reinforcement = × steel in shorter span
4 and f1(x) > 0 for x > 6
3 As f1(x) > 0 for x < –5, f1(x) < 0 for –5 < x < –2 and f1(x)
= × 812 = 609 mm2  Choice (B) = 0 for x = –5, f(x) has a local maximum at x = –5
4
Similarly, as f1(x) > 0 for –2 < x < 1,
32. For a thin spherical shell, hoopstress (circumferential
pd f1(x) < 0 for 1 < x < 6 and f1(x) = 0 for x = 1
stress) = f(x) has a local maximum at x = 1
4t
∴ The number of local maxima is 2. Ans: 2
where p = internal pressure.
d = diameter 3 7. Given that the system of linear equations
t = thickness. a1 x1 + b1 x2 + c1 x3 = 0
p = 6 MPa = 6 N/mm2 a2 x1 + b2 x2 + c2 x3 = 0 →(1)
d = 500 mm a3 x1 + b3 x2 + c3 x3 = 0
t = 8 mm has only a trivial solution.
6 × 500  a1 b1 c1 
\ Hoop stress =
4×8 ∴ Its coefficient matrix A =  a2 b2 c2  is non
 
= 93.75 N/mm2 Ans: 93.25 to 94.25  a3 b3 c3 
– singular.
3 3. % clay content in the soil = 100 – 40 = 60%
Consider the system of linear equations
Plasticity index = Liquid limit – Plastic limit
a1 x1 + b1 x2 + c1 x3 = 5
= 45 – 30 = 15
a2 x1 + b2 x2 + c2 x3 = 3 →(2)
Plasticity index, I P a3 x1 + b3 x2 + c3 x3 = 2
⇒ Activity number =
% clay fraction The augmented matrix of (2) is
15  a1 b1 c1 : 5 
= = 0.25  
60  A B  =  a2 b2 c2 : 3  .
The activity number is less than 0.75. So, the given soil  a3 b3 c3 : 2 
is said to be inactive soil. Choice (A) As A is non – singular, we have
34. Carbonate harness = 3.5 × 10–3 g ρ (A) = ρ ([A/B]) = 3(= The number of unknowns)
eq [if NCH is present sodium alkalikrity will be absent ∴ The system (2) has a unique solution. Choice (B)
i.e., NaHCO3 absent] 3 8. We know that, if X1 and X2 are two independent normal
=3.5 × 10–3 × 50 g/L as CaCO3 variates, then X1 – X2 is also a normal variate.
=175 mg/L as CaCO3 Mean of X1 = µ X1 = 100
Non – Carbonate hardness = Total hardness – Carbonate
hardness Mean of X2 = µ X 2 = 60
Total hardness = 5 × 50 mg/L as CaCO3 = 250 mg/L Variance of X1 = σ X1 2 = 16
as CaCO3
NCH = 250 – 175 = 75 mg/L as CaCO3. Choice (B) Variance of X2 = σ X 2 2 = 9
4.50 | Mock Test 4

∴ Mean of X1 – X2 = µ X − X = µ X − µ X Length of mid – ordinate = R[1 – cos(D/2)]


1 2 1 2
=600[1 – cos(60/2)]
= 100 – 60 = 40 =600 × 0.134 = 80.4 m Choice (B)
Standard deviation of X1 – X2
45. Qug = CuNc . Ab + Pb L .Cu
= σ X1 − X 2 = σ X1 + σ X 2 = 16 + 9 = 5
2 2
Choice (B) Pb = Perimeter of block = 4(1.2 + 0.4) = 6.4 m
Ab = area of block = (1.2 + 0.4)2 = 2.56 m2
39. The area bounded by the curve y = x3 and the line
⇒ Qug = (25 × 9 × 2.56) + (6.4 × 10 × 25)
y = x is the shaded region shown in the figure. Also,
the required area = 2 × The shaded region in the first    =2176 KN Choice (C)
quadrant. 4 6. The number of blows for first 15 cm is not taken into
consideration, only blows required for last 30 cm of
Y
penetration is taken.
A (1,1)
∴ Number of blows = 6 + 6 + 8 + 7 = 27
(i) Correction of over burden pressure
y=x corrected value, N1 = 0.926 × 27 = 25
(ii) Correction for dilatancy
 25 − 15 
0 N = 15 +  = 20
 2 
y=x ∴ The correct value for N is 20. Choice (C)
B (–1,–1) 4 7. Return period of rainfall T = 50 yrs
∴ Probability of occurrence once in 50 yrs.
1
1
P = = 0.02
∴ The required area = 2 ∫ ( x − x ) dx 50
3

x=0
Probability of occurrence in each of 2 successive year
1
x x 
2 4
= P2 = (0.02)2 = 0.0004 Choice (D)
2 − 
=
 2 4 0 4 8. Area of given DRH = Volume of direct runoff
1 
=
1 1  1
2  −  = square units Choice (B)
=  2 (70 + 6) × 60 × 60 × 90 
 2 4  2
= 12312000 m3
40. We know that [a, b] must be divided into even number Effective rainfall = 2 cm
of subintervals in order to apply the Simpson’s Rule. Volume of DRH
So, when [a, b] is divided into 5 subintervals, we can’t ∴ Catchment are  =
use the Simpson’s rule. Effective rainfall
∴ I is not true  12312000 
=
 
II and III are properties of the respective numerical 2 × 10 −2 
integration methods and are always true. Choice (C)
=615.6 km2 Choice (B)
42. G = 2.65, Sr = 85% and w = 12%
49. Volume of water required for evapotranspiration
WG
e= = 40 × 104 × 40 × 10–3 × 103
Sr = 16 × 105 lit
0.12 × 2.65 ∴ Discharge required at outlet
e= = 0.374
0.85 16 × 105
= = 0.925 lit/sec
critical hydraulic gradient, ic =
(G − 1) =
1.6
= 1.2
20 × 24 × 60 × 60
1+ e 1.374  Ans: 0.925 lit/ sec
i 5 0. mv = 2.2 × 10 m /KN
–4 2
FOS= c
i σ1i = 4(20) + 5(19 – 10) + 10(18 – 10) = 205 KN/m
1.2 1.2
∴ The hydraulic gradient provided = = ∆σ1 = 4 × 20 = 80 KN/m
FOS 2.5
σ f = σ i + ∆σ = 285 KN/m
⇒ i = 0.48 Choice (C)
4 3. Length of long chart T1 T2 = 2Rsin(D/2) ∆H = mv . H0 . ∆σ1 = 2.2 × 10–4 × 10 × 80
= 2 × 600 × sin(60/2) = 600 m ∴ ∆H = 176 mm Choice (C)
Mock Test 4 | 4.51

Nf Reactions
51. q(m3/sec/m) = K . H . Due to symmetry;
Nd
1
3 RA = RB = kN
=
2 × 10–6 × 20 × 2
12
ΣFX = 0
=
1 × 10–5 cumecs/m Ans: 1
HA = 0
52. For isolated T – beam;
l0 Force in member BC:
Effective flange width, bf = + bw Pass a section (1) – (1) through member BC.
l0
+4 Considering equilibrium of RHS
b Taking moment at point ‘D’
l0 = 10 m, b = 2m and bw = 250 mm 1
(FBC) × 5 – RE × 5 = 0 ⇒ FBC = + RE = kN
10000 2
⇒ bf = 10000 + 250 = 1.36 m Choice (A)
+4 Table
2000
Member K d1 k. d1
53. CBR(%) = BC 4 mm 2.0 mm
load sustained by specimen at 5mm penetration AB – 0 0
×100
load sustained by std aggregates at 5mm penetration BD – 0 0
50 CD – 0 0
= × 100 = 2.4% Choice (D)
2055 DE – 0 0
AD – 0 0
54. Taking camber as super elevation
3 v2 Σk δ1 = 2.0 mm
e= = 0.33 =
100 225 R ∴ S=D = 2.0 mm  Ans : 2 mm
(80)2 56.
0.33 = 2/6 9/9
225 R 3
3 4
2 2
R = 948.14 m Choice (B)
2
0/0 1 6 11/11
55. SD = Σk δ!
3 3
Where 2 5
4
k = force in members due to unit load applied where 3/3 7/7
deflection is desired. Activity:
δ! = deformation of a members due to lack of fit. 7/7 9/9
Since only in member BC, there is lack of fit. So find 5
2
7
force in member due to unit load in member BC only.
Forces in other members are not required. Inference float = Total float – Free float
Total float = (9 – 7) – 2 = 0
1 Free float = (9 – 7) – 2 = 0
B C ∴ Inference float = 0
Independent float = (9 – 7) – 2 = 0
∴ Both inference float and independent floats are
Sm zero. Choice (D)
57.
E
A
1 D ×P 150
HA
1 kN
X O X

RE
5m 5m
A
4.52 | Mock Test 4

The stress at a point is the sum of the compressive L 1000 800 600
P \ = + +
stress
A
and stress due to moment P × 100 (0.6) 5
(0.75) 5
(0.6) 5
(0.45)5
Area of cross section A = pR2 = 4213.99 + 10288.07 + 32,515.37
= p × (150)2 = 47,017.43
= 22500 p mm2 ⇒ L = 3656.08 m Ans: 3655.5 to 3656.5
P 400 × 103 63. W = 800 N
Direct stress =  = 5.6588 N/mm2
A 22500 π t = 100 N/mm2
Stress at A due to moment (compressive) 75
Mean radius = = 37.5 mm
2
PeR 400 × 103 × 100 × 150
= = 16WR
Ι  π t = πd 3 where d = wire diameter
  (300 )
4

64
16 × 800 × 37.5
= 15.0902 (Tensile) \ 100 =
πd 3
Resultant stress at A
⇒ d = 11.52 mm Choice (B)
= -15.0902 + 5.6588
= -9.4314
Lw ( KS ) f u
= 9.4314 (Tensile) Choice (B) 64. Design strength of side fillet weld =
3rmw
5 8. For a tapered rod angular rotation
(angle of twist) Lw = Effective length of fillet weld
2TL r12 + r1r2 + r22   = 50 + 50 = 100 mm
q= tt = Effective throat thickness
3πG r13 r23
  = k × s = 0.7 × 10 = 7
where T = torque applied
fu = 410 N/mm2
L = length
r1, r2 = radii. rmw = (for field welding) = 1.50
2 × 3 × 106 × 1200 50 2 + 50 × 25 + 252 100 × 0.7 × 10 × 410
\ q = × \ ( Pdw )side = = 110.46 kN
3π × 80 × 103 503 × 253 fillet 3 × 1.50
  = 0.02139 radian Ans: 0.02135 to 0.02145 Design strength of plug weld = ( Pdw ) plug
5 9. Discharge Q = 0.3 m3/s
π
× (20 ) × 410
2
Jet velocity v = 38.2 m/s
Power at the base of nozzle ( Pdw ) plug = 4
= wQH 3 × 1.5
= 9.81 × 0.3 × 80 ( Pdw ) plug = 49.577 kN
= 235.44 kN
1  wQ  2 \ Design strength of fillet weld
Kinetic energy of jet =  v Pdw = ( Pdw )sidefillet + ( Pdw ) plug = 110.46 + 49.577
2  g  =
1 9.81 × 0.3 Pdw = 160.037 kN
× (38.2)
2
= × 160.037
2 9.81 \ Service load carrying capacity =
= 218.89 kW 1.5
Loss of power in nozzle = 106.69 KN Choice (A)
=Power at base - Kinetic energy in nozzle
=235.44 - 218.89 65. [F] = inverse of stiffness matrix
=16.55 kW Choice (B) 1  5 − 3
   = k
6 2. Diameter of the equivalent pipe D = 600 mm = 0.6 m ( ad − bc)  − 3 4 
Using Dupit’s equation.
L L L L 1  5 −3
= 15 + 25 + 35 [F] =  −3 4   Choice (C)
D 5
D1 D2 D3 11 × k  
Mock Test 5 | 4.53

Mock Test 5
Number of Questions: 65 Total Marks: 100

Section – I: General Aptitude


Questions 1 to 5 carry One Mark each. 5. Although Jack volunteered to help me cook the dinner,
Directions for questions 1 and 2: Select the correct alterna- he threw a spanner in the works when it came to the
tive from the given choices. actual cooking.
(A) started laughing
1. A, B, C, D are four points on a plane. AB = 5, AC = 7
(B) left the place
and AD = 10. Which of the following is not a possible
(C) caused hindrance
value of BC + CD?
(D) wholeheartedly assisted
(A) 6.5 (B) 10.5
(C) 24 (D) 30 Questions 6 to 10 carry Two Marks each.
2. Seven persons – A, B, C, D, E, F and G – are sitting Directions for questions 6 and 8: Select the correct alterna-
in a row. A, B and C are women, D and E are children, tive from the given choices.
and F and G are men. No two children or no two men 6. If a, b, c, p, q, r are non-zero integers, ap = b, bq = c,
or no two women sit next to each other. B and A have cr = a and pqr = 15, the possible values of a + b + c
two persons sitting between them. F is two places away are _____.
from A. D has two persons sitting to his right. G has two (A) 1, –1 (B) 3, –3
persons sitting to his left. If the maximum number of (C) 1 (D) 3
people sit between the two children, then who sits to the 7. A dealer in travel goods sells four types of suitcases
right of F? – Standard, Deluxe, Super-deluxe and Premium. The
(A) D (B) C number of each type of suitcase sold during the four
(C) E (D) B quarters of 2015 are tabulated below. The costs of four
Directions for question 3: In the following question, deter- types are ` 700, ` 900, ` 1000 and ` 1200 per suitcase
mine the relationship between the pair of capitalised words respectively.
and then select the pair of words which has a similar rela- Type/ Super-
tionship to the capitalized words. Mark the number of that Standard Deluxe Premium
Quarter deluxe
pair as your answer. Q1 18,500 17,500 15,010 13,000
3. SICKLE : FARMER : : Q2 16,060 16,800 15,590 13,400
(A) Shears : Gardener Q3 19,800 18,900 16,040 12,500
(B) Painter : Brush
Q4 20,740 19,500 16,090 12,800
(C) Computer : Whizkid
(D) Workshop : Lathe Which suitcase accounts for the greatest part of the rev-
Directions for question 4: The following statement has enue for the dealer?
a part missing. Choose the best option from those given (A) Standard (B) Deluxe
below the statement to make up the missing part. (C) Super - deluxe (D) Premium
4. Not only was the tone and tenor of the interaction 8. Vikram, his brother, his daughter and his son are play-
especially positive, the two principals and their delega- ing doubles tennis match. Vikram’s son is diagonally
tions _____________. across the net from the tallest player’s sibling. Vikram’s
(A) managed to generate specific outcomes also on brother is directly across the net from Vikram’s daugh-
any number of issues ter. The shortest and the tallest players are on the same
(B) also manage to generate specific outcomes on a side of the net. Who is the shortest player?
number of issues (A) Vikram (B) Vikram’s brother
(C) also managed to generate specific outcomes on a (C) Vikram’s son (D) Vikram’s daughter
number of issues
Directions for question 9: In each of the following ques-
(D) will also manage to generate specific outcomes of
tions, statements 1 and 6 are respectively the first and the
any number of issues
last sentences of a paragraph and statements 2, 3, 4 and
Directions for question 5: In the following question, four 5 come in between them. Rearrange 2, 3, 4 and 5 in such
alternatives are given for the idiom/phrase printed in bold a way that they make a coherent paragraph together with
in the sentence. Choose the alternative which best expresses statements 1 and 6. Select the correct order from the given
the meaning of the idiom/phrase. choices and mark its number as your answer.
4.54 | Mock Test 5

9. 1. A lot of us do not realize that to carry out our day-to- personal choice, and depends greatly on how well
day functions we need a minimum amount of endur- we are able to pursue sound physical fitness pro-
ance strength and flexibility in our bodies. grammes that help maintain a strong body and mind.
2. Adding to this, 21st century seems to have many an (A) 5432 (B) 3254
unforeseen emergency in store for all of us; whether (C) 2345 (D) 5243
it be the 9/11 attack, outbreak of epidemics, or Directions for question 10: Select the correct alternative
strikes of terrorism – one never knows where and from the given choices.
when what might happen.
10. Over the years, there is a change in the attitude of disci-
3. Our current ways of life in general, are no longer
ples. While some are symbols of dedication and grace,
providing our bodies with sufficient exercise to
others want to become superstars overnight, and in the
maintain adequate levels of fitness.
process, defocus from their path to the extent of ques-
4. The realization of this dawns only when the neces-
tioning the guru.
sity occurs, and that could be too late.
Which of the following can be inferred from the above?
5. When we give in to the comforts of modern day
(A) Those disciples who question the guru cannot
living and inadvertently let our bodies deteriorate,
become superstars.
our abilities to carry out physical activities are so
(B) Earlier all the disciples were graceful and dedicated.
adversely affected that one may not derive the full
(C) Gurus are always right.
pleasure of living.
(D) For the disciples who are graceful and dedicated,
6. In such circumstances proving the quality and
becoming super stars overnight is not the priority.
most likely the longevity of our lives is a matter of

Section – II: Civil Engineering


Directions for questions 11 to 65: Select the correct alter- (C) Buckling of longitudinal bars
native form the given choices. (D) Bothe bending moment and shear
Questions 11 to 35 carry One Mark each. 17. Anchorage value of hook in RCC is _____ (φ = diame-
11. The order and degree of the partial differential equation ter of bar)
3 5
(A) 16φ (B) 12φ
∂2 z  ∂2 z   ∂z  ∂z (C) 8φ (D) 4φ
+ 5   − 4   + 3 = 4z respectively
∂x 2
 ∂x ∂y   ∂x  ∂y 18. The most active clay mineral in shrinkage and swelling
are _____. is _____.
(A) 2 and 1 (B) 2 and 3 (A) Kaolinite (B) Illite
(C) 2 and 5 (D) 1 and 5 (C) Hollysite (D) Montmorillonite
1 2. If f and g are any two scalar point functions that are 19. Following the usual notations, the flow value Nφ is
continuous and have continuous first partial derivatives given as _____.
and if G = grad g, then curl(f G ) is equal to _____. (A) tan2(45 + φ/2) (B) tan2(45 – φ/2)
(C) tan(45 + φ/2) (D) tan(45 – φ/2)
(A) f(curl G ) (B) grad f × G
20. For a soil, the void ratio is given as 0.68. If the specific
(C) g(grad f) (D) None gravity of soil is 2.75, the shrinkage limit of the soil
13. 8 books are placed at random in a shelf. The probability is _____
that three books B1, B2 and B3 are together is ______. (A) 24.7% (B) 2.7%
(C) 40.4% (D) 12.3%
14. If A is a 3 × 4 matrix with rank 3, then for any system
of linear equations AX = B has 21. The soil “loam” means,
(A) a unique solution, (A) Silt with little sand
(B) no solution (B) Sandy silt with little clay
(C) infinitely many solutions (C) Clayey silt exhibiting slight cohesion
(D) exactly three solutions (D) Mixture of sand, silt and clay sized particles in
approximately equal proportion.
15. If f(x) + f(6 – x) = 0 and f(x) is continuous in [0, 6], then
6 22. The organism, with exhibits very nearly the character-
the value of ∫ f ( x )dx is _____. istics of an ideal pathogenic indicator is
0
(A) Entamoeba histolytica
16. Lateral ties in RCC columns are provide to resist _____. (B) Escherichia coli
(A) Bending moment (C) Salmonella typhi
(B) Shear (D) Vibrio conma
Mock Test 5 | 4.55

23. Pathogens are usually removed by 32. A three hinged parabolic arch of span 4 m and rise 8 m
(A) Chemical precipitation carries a uniformly distributed load of 8 KN per unit
(B) Sedimentation over the whole span. The horizontal thrust at each sup-
(C) Activated sludge process port (in KN) is _____.
(D) Chlorination 33. As per 1S : 800, the maximum bending moment for
24. The reoxygenation coefficient K of stream is 0.5 at design of purlins can be taken as
200C. Its K value at 320C is likely to be _____. WL WL
(A) (B)
25. The load transfer to the lower layers of flexible pave- 8 4
ments is by WL WL
(C) (D)
(A) Bending action of layers 10 12
(B) Shear deformation 34. When length of side fillet weld is 400 times the effec-
(C) Grain to grain contact tive throat thickness, the design shear capacity of fillet
(D) Consolidation of sub grade weld is
26. Camber on highway pavement is provided to take (A) Decreased by 33% (B) increased by 33%
care of (C) Increased by 66% (D) Decreased by 66%
(A) Centrifugal force 35. State of stress at a point is as follows. sx = 900 MPa, sy
(B) Drainage = 300 MPa. Maximum shear stress = 500 MPa. Value
(C) Sight distance of maximum principal stress (in MPa) is
(D) Off tracking (A) 900 (B) 1000
27. The type of surveying in which curvature of the earth is (C) 1100 (D) 1200
taken into account is called Questions 36 to 65 carry Two Marks each.
(A) Geodetic surveying
36. The system of linear equations
(B) Plane surveying
2x + 3y – 4z = 0
(C) Preliminary surveying
4x + 5y + z = 0
(D) Topographical surveying
2x + 4y – 13z = 0
28. The ratio of actual evapotraspiration to potential evap- has
otranspiration is in the range of (A) only a trivial solution
(A) 0 to 0.4 (B) exactly one linearly independent solution.
(B) 0.6 to 0.9 (C) exactly two linearly independent solutions.
(C) 0 to 1.0 (D) exactly three linearly independent solutions.
(D) 1.0 to 2.0
37. The coefficient of x8 in the Taylor’s series expansion of
29. At a point in a strained body, normal stresses are zero f(x) = 1 + cos2(x2) about x = 0 is _____
and shear stresses are 100 MPa. Value of principal 2 3
stress are (A) 3! (B)
4!
(A) 50 MPa
5 7
(B) 100 MPa (C) (D)
(C) 150 MPa 6! 8!
(D) 20 MPa
38. Which of the following is a value of the function f(x, y)
30. In laminar flow through a pipe, the pressure drop per
x 2 y 2 + 8( x + y )
unit length of pipe is given by = at one of its local extreme points?
32µu 16µu xy
(A) (B) (A) 12 (B) 23
D D2
128µQ 128µQ (C) 17 (D) 10
(C) (D)
πD 4 πD 2 39. If y(x) satisfies the differential equation y11 + 8y1 +
31. A closed cylindrical vessel of radius R completely 7y = 49, then the value of Lt y(x) is _____.
x →∞
filled with a liquid of density r is rotated at an angular
40. For a set of 5 pairs of values of x and y,
velocity w about its vertical axis. The total fluid pres-
∑x = 20, ∑y = 325, ∑xy = 366 and ∑x2 = 120
sure force acting on the top is
If a straight line of the form y = a + bx is fitted to this
ρ 2 4 ρ 2 3
(A) ω πR (B) ω πR data using the method of least squares, then the value
4 4 of y at x = 10 is ______
ρ 2 2 ρ 2 (A) –25 (B) –75.10
(C) ω πR (D) ω πR
4 4 (C) 41.32 (D) 30.50
4.56 | Mock Test 5

41. An RC short column 400 mm × 600 mm is made of safe bearing capacity of the footing as per terezaghi’s
M30 concrete and has 6 no’s of 16 mm Fe500 steel theory is ______.
bars. The ultimate load carrying capacity of the column (A) 150.3 kpa
is _____, (B) 451 kpa
(A) 1478 KN (B) 3270 KN (C) 142.5 kpa
(C) 3837 KN (D) 4904 KN (D) 166.5 kpa
42. A pre stressing concrete beam 150 mm × 300 mm sup- 48. The composition of a certain MSW sample and specific
ports a load of 5 KN/m over a simply supported span of weight of its various components are as given below:
8 m. The beam has parabolic cable of zero eccentricity
at ends and 50 mm towards soffit at the central span. Percentage Specific weight
Component
The pre – stressing force required to balance external by weight (kg/m3)
load is _____. Food  50 300
(A) 800 KN (B) 850 KN Dirt and Ash  30 500
(C) 875 KN (D) 900 KN Plastic 210  65
43. Wood and yard waste  20 125
3 4
2 4 6
2
(A) 319 (B) 139
(C) 217 (D) 199
1 3 3 3
49. A setting tank is designed for a surface overflow rate of
4 40 m2/day m2. Assuming specific gravity of sediment
5
3
2 particles = 2.65. Density of water gw = 1000 kg/m3,
dynamic velocity of water = 0.00 NS/m2 and stoke’s
Identify the critical path, for the given network. law’s valid. The approximate minimum size of particles
(A) 1 – 2 – 3 – 4 – 5 – 6 which can be completely removed is _____.
(B) 1 – 3 – 4 – 5 – 6
(C) 1 – 2 – 3 – 5 – 6 50. Match
(D) All the above List – I List – II
4 4. A retaining wall is shown below. a. Evapora transpiration 1. Penman method
The active earth pressure at 5 m. depth is ______. b. Infiltration 2. Snyder’s method

φ = 24° C = 0, γ = 16 KN/m3 c. Synthetic unit hydrograph 3. Museingham method


3m
d. Channel routing 4. Horton’s method

φ = 30° C = 10 KN/m2, γ = 2 KN/m3 a b c d


2m (A) 1 3 4 2
(B) 1 4 2 3
(C) 3 4 1 2
(A) 17.8 kpa (B) 29.3 kpa
(C) 40.8 kpa (D) 229.4 kpa (D) 4 2 1 3
4 5. Two soil samples A and B, each weighing 1 kg and hav- 51. A road is being designed for a speed of 110 km/hr on
ing water contents of 50% and 75% are mixed together. a horizontal curve with a super elevation of 8%. If the
The resulting water content of the mixture is _____. coefficient of side friction is 0.1, the minimum radius
(A) 63.8% (B) 62.5% of curve 9 in m) required for safe vehicular movement
(C) 61.7% (D) 60.8% is ____.
46. A clay sample originally 30 mm thick at a void ratio 52. Match the following:
of 1.10 was subjected to a compressive load. After
P. Resistance to impact 1. Hardness
some period of time the clay sample was measured as
25 mm. The void ratio is _____. Q. Resistance of wear 2. Strength
(A) 0.385 (B) 1.45 R. Resistance to weathering action 3. Toughness
(C) 0.68 (D) 0.75 S. Resistance to crushing 4. Soundness

47. A continuous footing of width 2.5 m rests at 1.5 m (A) P – 1, Q – 3, R – 4, S – 2


below the ground surface in clay. The unconfined com- (B) P – 3, Q – 1, R – 4, S – 2
pressive strength of the clay is 150 kpa and unit weight (C) P – 4, Q – 1, R – 3, S – 2
of soild is 16 KN/m3. For using factor of safety of 3, the (D) P – 3, Q – 4, R – 2, S – 1
Mock Test 5 | 4.57

53. In a cylindrical bituminous mix VMA = 15% and 61. A cylindrical vessel 3 m long and 1 m diameter is sub-
Vv = 4.5%. The magnitude of VFB is jected to an internal pressure of 1.5 N/mm2. The hoop
(A) 24 (B) 54 stress developed was 50 N/mm2. If Young’s modulus
(C) 40 (D) 70 and Poisson’s ratio are 2 × 105 N/mm2 and 0.3 respec-
tively change in volume (in cm3) is _____.
54. The speed density (v – k) relationship on a single lane
road with uni directional flow is v = 70 – 0.7 k, where 62. A cylindrical vessel of 1.5 m internal diameter and 4 m
v is km/hr and k is in veh/km. The capacity of road (in length has 10 mm shell thickness. If it is subjected to a
veh/hr) is _____. fluid pressure of 2 MPa, maximum shear stress induced
(in N/mm2) is
55. The width of expansion joint gap is 2.5 cm in a cement (A) 25
concrete pavement. The spacing between expansion (B) 37.5
joint for a maximum rise in temperature of 250C is (C) 50
(Assuming coefficient of thermal expansion of con- (D) 62.5
crete as 10 × 10–6 per degree C)
63.
(A) 5 m B
(B) 50 m
(C) 100 m
(D) 25 m
56. In a leveling work, sum of back sight (B S) and fore-
sight (F S) have been found to be 3.985 m and 5.725 m 120°
respectively. If R.L of the starting station is 100.00 m, 120° A
the RL (in m) of the last station is ______ O

57. The chainage of the intersection point of two straight is 120°


1585.6 m and angle of intersection is 1500. If the radius
of a circular curve is 600.0 m, the tangent distance and
length of the curve respectively are C
(A) 418.88 and 1466.08
(B) 218.38 and 1648.49 At a stressed point in a two dimensional stress system,
(C) 314 m and 160.79 strain were measured to 3 directions OA, OB and OC,
(D) 418.88 and 218.38 120° to each other. The observed strains are
eA = 300 × 10-6, eB = -225 × 10-6 and
58. A hydraulic turbine is to develop 1010 kW while run- eC = 100 × 10-6.
ning at 120 rpm under a head of 12 m. Overall effi- Value of strain at an angle 90° clockwise to OA (in
ciency of the turbine at the best operating point is 90%. micron) is _______.
For the prediction of performance a 1 : 10 scale model
is tested under a head of 7 m. Power output (in kW) of 6 4. The rigid jointed plane frame ABC is as shown below.
the model turbine for similar conditions of prototype is. The deflection at center of AB is _______ (in mm)
10 KN/m
59. A Kaplan turbine works under the following conditions
Head = 20 m
Speed = 150 rpm. A B
Hydraulic efficiency = 95%
Outer diameter of runner = 4.5 m.
Hub diameter = 2 m E I = 1 KN - m2
1m
Velocity of flow = 10.2 m/s
Runner inlet vane angle (in degree) at outer periphery
of the runner is
(A) 160.7
C
(B) 155.2
(C) 150.4 1m
(D) 144.3
60. An inward flow reaction turbine has an external diam- 65. The design bending strength of laterally supported
eter of 1 m. Speed of the runner is 210 rpm and guide beam of ISLB 350 @ 486 N/m, when the design shear
blades make an angle of 10° to the wheel tangent. force (V) is less than the design shear strength is _____.
If velocity of flow at inlet is 2.2 m/s, the runner vane (in KN – m). (Assume plastic section, ZPZ = 850 103
a ________. mm3 Zez = 750 103 mm3 and steel of grade Fe410]
4.58 | Mock Test 5

Answer Keys
1. D 2. D 3. A 4. C 5. C 6. B 7. B 8. B 9. A 10. D
11. B 12. B 13.  0.106 to 0.108 14. C 15. 0 16. C 17. A 18. D 19. A
20. A 21. B 22. B 23. D 24. 0.604 25. C 26. B 27. A 28. C 29. B
30. C 31. A 32. 2 33. D 34. A 35. C 36. B 37. A 38. A 39. 7
40. B 41. B 42. A 43. A 44. A 45. C 46. B 47. D 48. B
49. 5.14 × 10–9 m 50. B 51. 0.071 52. B 53. D 54.  1750 veh/hr 55. B
56. 98.26 57. C 58.  4.49 to 5.55 59. A 60.  56.0 to 56.1
61.  1119.15 to 1119.25 62. B 63.  183 to 184 64.  13.00 to 13.05
65. 190 – 195 KN – m

Hints and Explanations


1. AB = 5, AC = 7 \ 2 ≤ BC ≤ 12 is successful at a very young age, not necessarily in
BC = 7, AD = 10 \ 3 ≤ CD ≤ 17 computers. A ‘lathe’ is a machine that shapes pieces
\ 5 ≤ BC + CD ≤ 29 of wood or metal by holding and turning them against
Among the choices, 30 is not a possible value. a fixed cutting tool and a ‘workshop’ is a place where
 Choice (D) things are repaired or made. Choice (A)
2. Let us take down the data as given below: 4. Since we have ‘was’ in the beginning of the sentence
(i) Seven persons – A, B, C, D, E, F and G - sit in a it should be ‘managed’ (past tense) in the second half.
row. So we look at choices A and C. ‘A number of’ is better
(ii) 3 women : A, B and C than ‘any number of’ since the latter suggests a very
2 children : D and E high number. Choice (C)
2 men : F and G 5. The idiom ‘throw a spanner in the works’ means ‘to
(iii) No two persons of the same gender (or no two cause hindrance’. It means to do something that pre-
children) sit next to each other. vents a plan or activity from succeeding. Choice (C)
B A 6. ap = b, bq = c, cr = a  .... (1)
(iv) ––
A B and pqr = 15  …. (2)
F A ap = b ⇒ crp = b ⇒ bqrp = b
(v) – \ Either b = 1, 0, – 1 or pqr = 1. As pqr ≠ 1, b = 1
A F
or – 1
(vi) D _ _ (2 persons sitting to the right of D) (∵ a, b, c are nonzero integers). If b = 1, then c = 1
(vii) _ _ G (G has two persons sitting on his left). and a= 1
The positions of G and D are located as shown: If b = – 1, (as p, q, r are odd integers), it follows that c
     G   D = –1 and a = – 1
– – – – – – – \ a + b + c = 3 or – 3. Choice (B)
     (m)  (c)
Here, m → Man, w → Woman and c → Child. 7. The total sales (number), the cost of each type of suit-
For the maximum number of people to sit between the case and the total revenue from the four types of suit-
two children, the other child has to be seated at the cases are tabulated below.
extreme left end. Also, other conditions are used to get Type Number Cost Revenue
the following arrangement: S 75,100  700 5,25,70,000
E  C  G    A    D  F    B D 72,700  900 6,54,30,000
SD 62,730 1000 6,27,30,000
---  ---  ---   ---  ---  ---  ---
P 51,700 1200 6,20,40,000
(1) (w)  (m) (w) (c) (m) (w)
x(f)     x(f) The greatest revenue is from Deluxe. Choice (B)
Hence, B sits to the right of F. Choice (D) 8. Let Vikram, his brother, his son, and his daughter be V,
3. A ‘sickle’ is used by the farmer to cut the weeds i.e. B, S and D.
for weeding. In the same way ‘shears’ are used by the If S is diagonally opposite to D, then S is the taller
‘gardener’ to cut the bushes. (Shears are large pair of player. The arrangement is as follows:
scissors). Hence option A is the correct answer. Option
S B
B can be ruled out as the order is reversed, it should
V D
be (Brush : Painter). A ‘whizzkid’ is a person who
Mock Test 5 | 4.59

And now, B is the shortest player (∵ shortest and tallest 3!× 6! 3


players are on same side) \ Required probability = = = 0.1071
8! 28
If S is diagonally opposite to V, then D and B cannot be
directly opposite.  Ans: 0.106 to 0.108
If S is diagonally opposite to B, then V is the taller 14. Given that A is a 3 × 4 matrix with rank 3.
player. The arrangement is as follows: \ Any system of linear equations AX = B will consist
of 4 unknowns
S D Also, as r(A) = 3, we have
V B r([A/B]) = 3
\  r (A) = r ([A/B]) = 3 < 4 (The number of unknowns),
And now, B is the shortest player. Choice (B) the system of equations AX = B has infinitely many
9. Statement 1 says to carry out our day-to-day activities solutions. Choice (C)
a minimum amount of endurance and flexibility in our 1 5. We know that, if f(x) = –f(2a – x), then
bodies is needed. If we look at the statements, ‘5’ is the 2a
most appropriate one to follow 1. ‘5’ emphasizes on the ∫ f ( x )dx = 0
fact that our physical activities are adversely affected, 0

which is a continuation of the idea expressed in ‘1’. The Now f(x) + f(6 – x) = 0 ⇒ f(x) = –f(6 – x)
words “the realization of this ….” refers to the reason \ a = 3
for our abilities to be adversely affected. Hence ‘4’ is 6
a better statement to follow ‘5’. Further in ‘3’ we find
emphasis on the current ways of life. Thus ‘543’ is the
Hence ∫ f ( x)dx = 0
0
Ans: 0

logical order which is further followed by ‘2’. 16. Lateral ties in columns are provided, to avoid buckling
 Choice (A) of longitudinal (main) steel. Choice (C)
10. According to the passage a disciple becomes defocused 17. 180 bend is called Hook
0

to the extent of questioning the guru once they want Anchorage value for 1800 bend is 16f Choice (A)
to become superstars overnight. But the statement does
not explicitly indicate whether they would be success- WG
20. e =
ful or not. Hence, (A) cannot be inferred. The passage Sγ
describes the attitude of present disciples but not the
At shrinkage limit, Sg = 1
past ones. Hence, (B) cannot be inferred. The passage
did not discuss about the gurus, hence, (C) cannot be W × 2.75
0.68 =
inferred. The passage demarks the disciples as those 1
who are dedicated and graceful and the other as those ∴ W = 24.7% Choice (A)
who want to become super stars overnight. From this it
can be understood that one group is different from the 24. KR(T0C) = KR(200C)[1.016]T – 20C
other. Hence, (D) can be inferred. Choice (D) = 0.5 [1.016]32 – 20
= 0.604 Ans: 0.604
11. Order = 2 and degree = 3. Choice (B)
29. px =0, py = 0, t = 100 MPa
12. Given G = Grad g
2
Now curl(f G ) = Grad f × G + f(curl G ) px + p y  px − p y 
p1,2 = ±  + τ2
= Grad f × G + f(curl (grad g)) 2  2 
= Grad f × . G . + f( 0 )
=0 ± 0 + τ2 = t
(∵ curl (Grad g) = 0 )
=
100 MPa Choice (B)
= Grad f × G . Choice (B)
13. Total number of ways of placing 8 books in a shelf = 8! ∆P 32µu
30. =
As three books B1, B2 and B3 has to be together, take the L D2
three books as a single entity.
Q
The number of ways of arranging the three books Substituting u =
among themselves = 3!  πd 2 
The number of ways of arranging the remaining 5  4 
books and this entity (The three books together) = 6!
\ The number of ways of arranging the 8 books such ∆P 128µQ
=  Choice (C)
that the three books are together = 3! × 6! L πD 4
4.60 | Mock Test 5

2 The number of variables = n = 3


32. Horizontal thrust at each support is wl So, the number of linearly independent solutions is n –
8h
r = 3 – 2 = 1 Choice (B)
8KN/m × (4)2 m 2 8 × 16
∴ H = = 3 7. We have f(x) = 1 + cos (x )
2 2
8 × 8m 8×8
 1 + cos 2 x 2 
H = 2 KN =1+  
 Ans: 2  2
3 3. As per 1S : purlins are assumed as continuous beams 3 1
= + cos 2x2
in design. So maximum bending moment to be taken 2 2
WL
3 1  (2 x ) (2 x ) (2 x ) 
2 2 2 4 2 6
as . Choice (D)
12 = + 1 − + − + ....∞ 
2 2 2! 4! 6! 
34. LJ = 400 tt  
If LJ > 150 tt ⇒ a Reduction factor need to be applied
x4 x8 x12
for design shear capacity of fillet weld. = 2−2 + 23 − 25 + ......∞
2! 4! 6!
Reduction factor (β LW )
23 2
0.20 LJ \ The coefficient of x 8 = =  Choice (A)
(β LW ) = 1.20 – 150tt
4! 3!
x 2 y 2 + 8( x + y ) 8 8
0.20 × 400tt 38. Given f(x,y) = = xy + +
=
1.20 – xy x y
150tt 8 8
\ 
fx = y – 2 and fy = x – 2
(β LW ) = 0.66 x y
So strength reduced by 33% Choice (A) 8
fx = 0 ⇒ y – 2 = 0
3 5. Maximum principal stress x
⇒ x2y = 8 → (1)
 σx + σ y 
=  + ( τ max ) 8
 2  and fy = 0 ⇒ x – 2 = 0
y
 900 + 300  ⇒ xy2 = 8 → (2)
= 
  + 500
2 From (1) and (2), we have
= 1100 MPa Choice (C) x2y = xy2 ⇒ x = y
36. Given system of linear equations is \ From (1), x2 × x = 8  ⇒  x = 2
⇒  y = 2
2x + 3y – 4z = 0 \ The stationary point of f(x, y) is (2, 2)
4x + 5y + z = 0    → (1)
2x + 4y – 13z = 0 16 16
Now r = fxx = 3
; s = fxy = 1 and t = fyy = 3
It can be written in matrix form as x y
AX = 0 At (2, 2), rt – s2 = 3 > 0 and r = 2> 0
 2 3 −4  \ f(x, y) has a local minimum at (2, 2) and the value
Where A =  4 5 1  of f (x, y) at (2, 2) is 12. Choice (A)
 
 2 4 −13 39. Given differential equation is
y11 + 8y1 + 7y = 49 → (1)
R2 → R2 – 2R1 and R3 → R3 – R1
Its general solution is y = yc + yp  → (2)
 2 3 −4  To find yc:
~ 0 −1 9 
 The homogenous differential equation corresponding
 
0 1 −9  to (1) is
y11 + 8y1 + 7y = 0
R3 → R3 + R2 Its auxiliary equation is
 2 3 −4  D2 + 8D + 7 = 0
\ A 0 −1 9 
 ⇒ (D + 1) (D + 7) = 0
  ⇒ D = –1; D = –7
0 0 0  \ The complementary function of (1) is
\ r (A) = r = 2 yc = c1e–x + c2e–7x  → (3)
Mock Test 5 | 4.61

To find yp: 44. At 5m;


1 1 1 σ V = 3 × 16 + 2 × 20 = 88 KPa
yp = P.I = X = 2 49 = × 49
f ( D) ( D + 8 D + 7) 7 Pa = Ka2 σ V – 2C2 Ka2

yp = 7  → (4) Ka2 = = 0.33


∴ Pa = 0.33 × 88 – 2(10) 0.33
Substituting the values of yc and yp in (2)
We get the general solution of (1) as = 17.80 KPa Choice (A)
y(x) = c1e–x + c2e–7x + 7 45. For sample A;
Now Lt y(x) = Lt [c1e–x + c2e–7x + 7] = 7  Ans: 7 W
x →∞ x →∞ W= w
Ws
40. Given n = 5, ∑x = 20, ∑y = 325, ∑xy = 366 and ∑x2
= 120 Ww = 0.5 Ws
The normal equations in the process of fitting a straight Ww + Ws = 1kg
line y = a + bx to the above data are 0.5 Ws + Ws = 1
∑ y = na + b∑x Ws = 0.667 kg and Ww = 0.333 kg
and ∑xy = a∑x + b∑x2 Similarly, for sample B;
i.e., 325 = 5a + 20b Ws = 0.571 kg and Ww = 0.43 kg
366 = 20a + 120b Total water weight; Ww = 0.333 + 0.43 = 0.763 kg
Solving these equations for a and b, we have Total solids weight Wg = 0.667 + 0.571 = 1.237 kg
a = 158.4 and b = –23.35
0.763
\  The straight line that fits to the given data is ∴ Water content (wg) = × 100
y = 158.4 – 23.35 x 1.237
\ y at x = 10 is = 61.68% Choice (C)
y(10) = 158.4 – 23.35 × 10 = –75.1 Choice (B) ∆e ∆H
46. =
π 1 + e0 H 0
4 1. ASC = 6 × × 162 = 1206 m2
4 ∆e 30 − 25
=
AC = Ag – ASC 1 + 1.1 30
= (400 × 600) – 1206 ∆e = 0.35
= 238794 mm2 ef = e + ∆e
Pu = 0.4 fck AC + 0.67 fy ASC
  = 1.1 + 0.35
= 0.4(30)(1206) + 0.67(500)(1206) ∴ ef = 1.45 Choice (B)
= 3269838 N
∴ Pu = 3270 KN Choice (B) 150
47. Cu = = 75 kPa
2
wl 2
42. P.e = Qs = 1 (CNc) + r.D
8
F
5 × (8000)2 1
P(50) =   = (75 × 5.7) + 16(1.5) = 166.5 kPa Choice (D)
8 3
P = 8,00,000N
∴ P = 800 KN Choice (A) 100 50 30 20 20
48. = + + + (Harmonic mean)
x 300 500 65 125
43.
2/2 5/5
100
11/11 = 0.72
3 4 x
2 4 6
2 100
x= = 138.8 kg/m3 ⋍ 139 kg/m3. Choice (B)
0/0 1 3 3 3 0.72

4 49. Vo = Vs
3 5

5/5
2 8/8
Vo =
(GS − 1) rw d 2
18µ
Total flat for 1 – 3 and 3 – 5 is not zero. Choice (A)
4.62 | Mock Test 5

40m3 40 ∆
Now Vo = = Tangent distance = R tan = 600 × tan(150)
day.m 2
24 × 60 × 60 2
= 160.79m Choice (C)
(2.65 − 1) × 9.81 × 103 × d 2
= 5 8. Prototype -
18 × 0.01
Pp = 1010 kW
Solving we get Np = 120 rpm
d = 5.14 × 10–9 m Ans: 5.14 × 10–9 m
Hp = 12 m
V2 V2 Model -
51. e + f = (or) e + f =
gR 127R Hm = 7 m
Scale ratio = 1 : 10
(60)2 Equating the head coefficient
e+f=
127 × 128  H   H 
 2 2  =  2 2 
(60)2 N D p N D m
e= – 0.15
127 × 128 N  H
2
 Dp 
  = 0.071 Ans: 0.071 ⇒  m  = m ×  
 N p  H p  Dm 
VB  15 − 4.5  2
53. VFB = =  × 100 = 70% Choice (D)
⇒  m  = × 10 2
N 7
VV  15 
 120  12
54. C = VK ⇒ Nm = 916.5 rpm
  = 70k – 0.7 kz Equating power coefficient
dc
= 70 – 1.4k = 0  P   P 
dk  3 5  =  3 5 
N D p N D m
K = 50
3 5
C = 70 × 50 – 0.7(50)2 P  N  D 
  = 1750 veh/hr Ans: 1750 veh/hr ⇒  m  =  m  ×  m 
 Pp   N p   D p 
2.5 3 5
55. σ1 = = 1.25 cm = 1.25 × 10–2 m Pm  916.5   1 
2 ⇒ =  × 
1010  120   10 
t2 – t1 = 250C
⇒ Pm = 4.4996 kW Ans: 4.49 to 5.55
coefficient of thermal expansion α = 10 × 10–6/C
σ1 = L α (t2 – t1) 59. Head H = 20 m
Speed N = 150 rpm
σ1
∴ spacing of expansion joint L = Hydraulic efficiency hh = 95%
α (t2 − t1 ) Outer diameter D = 4.4 m
1.25 × 10 −2 Hub diameter d = 2 m
= = 50 metres Choice (B) Hydraulic efficiency = 95 %
10 × 10 −6 × 25
Vr2
56. Using rise and fall method
∑FS > ∑ B.S
Fall = ∑F.S – ∑B.S = R. L of first station – R. L of last
station
= 5.725 – 3.985
= 1.74 m
R.L (last station = R.L (first station) – fall Vr1 V1
= 100 – 1.74 = 98.26 Ans: 98.26
θ Vf1
πR
5 7. Length of the curve = ∆ Vw1
180 0 U1
π × 600 At the outer periphery of the runner,
=× 30 (D = 180 – 150 = 300)
0

180 Velocity of flow,
=
314 m V f1 = 10.2 m/s
Mock Test 5 | 4.63

Blade velocity at inlet 61. L = 3 m = 3000 mm


πDN π × 4.4 × 150 d = 1 m = 1000 mm
u1 = = = 34.5575 m/s p = 1.5 N/mm2
60 60
E = 2 × 105 N/mm2
Hydraulic efficiency m = 0.3
Vw1 u1 Hoop stress f1 = 50 N/mm2
hh = where Vw = velocity of whirl
gH 1
Longitudinal stress
Vw1 × 34.5575 f1
f2 = = 25 N/mm 2
\ 0.95 = 2
9.81 × 20
⇒ Vw1 = 5.3936 m/s Diametral strain
δd f1 f
From inlet velocity triangle, e1 = = −µ 2
d E E
V f1
tan(180 - q) =
 1
u1 − Vw1   = ( f1 − µf 2 )
E
10.2 42.5
= = 0.3497   =(50 - 0.3 × 25) =
34.5575 − 5.3936 E
⇒ 180 - q = 19.277° Longitudinal strain
⇒ q = 160.7° Choice (A) δL f 2 f
60. e2 = = −µ 1
L E E
1
  = ( f2 − µf1 )
E
1 10
  = (25 − 0.3 × 50 ) =
E E
Volumetric strain
V δV 2 × 42.5 10 95
Vr Vf1 = 2e1 + e2 = + =
α θ V E E E
U1 Change in volume
V πd 2 L
D1 = 1.2 m. dV = × 95 = × 95
E 4E
N = 210 rpm.
πD1 N π × 1 × 210 π × (1000 ) × 3000 × 95
2

u1 = =    =
60 60 4 × 2 × 105
  = 10.9956 m/s    = 1119,192.4 mm3
V f1 = 2.2 m/s    = 1119.19 cm3. Ans: 1119.15 to 1119.25
Guide blade angle a = 10° 62. p = 2 MPa = 2 N/mm2
Let q be the runner vane angle at inlet. d = 1.5 m = 1500 mm
From velocity triangle, L = 4 m = 4000 m
V f1 t = 10 mm
tan θ = pd 2 × 1500
V1 cos α − u1 Hoop stress f1 = =
2t 2 × 10
V f1 =
150 N/mm2
But = tan α = tan10
V1 cos α pd f1 150
Longitudinal stress f2 = = =
V f1 2.2 4t 2 2
⇒ V1­cosa = = = 12.4768
tan α tan10 =
75 N/mm2
2.2 f1 − f 2 150 − 75
\ tanq = = 1.4853 Maximum shear stress =
12.4768 − 10.9956 2 2
⇒ q = 56.04870 Ans: 56.0 to 56.1 =
37.5 N/mm2 Choice (B)
4.64 | Mock Test 5

63. Treating OA as x-axis, 90° clockwise to OA is y-axis 64. The above frame can be simplified as
eA = ex = 300 micron.
eB = -225 micron.
eC = 100 micron.

ex + e y ex − e y 1
eB = + cos 240 + γ xy sin 240
2 2 2
300 + e y 300 − e y  −15.5 
⇒ -225 = + ×
2 2  2 
1 Since there is only vertical load in column BC.
+ γ xy ( −0.866)  →(1)
2 5WL4
∴ Deflection at center of AB =
ex + e y ex − e y 1 384 ∈Ι
eC = + cos 480 + γ xy sin 480
2 2 2 5 × (10)(1) 4
= = 13.02 mm Ans: 13.00 to 13.05
384 × 10
300 + e y 300 − e y
⇒ 100 = + (−0.5) 65. For Laterally supported beam;
2 2
Design bending capacity;
1
γ xy (0.866)  →(2) Md = β p Z pz
fy
[For plastic section β p = 1.0]
2
rmo
Adding (1) and (2) 250
   = 1.0 × 850 × 100 ×
-225 + 100 = (300 + ey) -
(300 − e )
y
1.1
2    = 193.18 × 106 N – mm
Md = 193.18 KN – m
300 3
-125 = + ey fy 750 × 103 × 250
2 2 Md < 1.2 Ze = 1.2 ×
rmo 1.0
3 =204.54 KN – m
⇒ e y = −125 − 150
2 ∴ Hence OK
⇒ ey = 183.33 micron Ans: 183 to 184 Md = 193 KN – m Ans: 190 – 195 KN – m

You might also like